Superpage
Skin nontumor

Authors: Maryam Aghighi, M.D., Aadil Ahmed, M.D., Heba Ahmed Abdelkader, M.D., Ohoud Aljarbou, M.D., Sepideh Nikki Asadbeigi, M.D., Jaya Ruth Asirvatham, M.D., M. Suzanne Bloomquist, M.D., Candice E. Brem, M.D., Chau M. Bui, M.D., Erin M. Carlquist, M.D., Osward Y. Carrasquillo, M.D., M.P.H., Gerardo Cazzato, M.D., Ph.D., Chico J. Collie, M.B.B.S., Mary-Katharine Collins, M.D., Carina Dehner, M.D., Ph.D., Louis P. Dehner, M.D., Ha Kirsten Do, Favia Dubyk, M.D., M.S., Juanita Duran, M.D., Amira Elbendary, M.B.B.Ch., M.Sc., Ashley N. Elsensohn, M.D., M.P.H., Hillary Rose Elwood, M.D., Michael Fitz-Henley, M.B.B.S., D.M., Cale Michael Max Fletcher, B.Med., M.P.H., Matthew Franklin, M.D., Maxwell A. Fung, M.D., Jerad M. Gardner, M.D., Silvija P. Gottesman, M.D., MacKenzie Griffith, B.A., Narina Grove, M.D., M.A., Ruta Gupta, M.D., Christopher S. Hale, M.D., Mowafak Hamodat, M.B.Ch.B., M.Sc., Jonathan D. Ho, M.B.B.S., D.Sc., Eric W. Hossler, M.D., Chelsea Huang, M.D., Giuseppe Ingravallo, M.D., Ph.D., Julie M. Jorns, M.D., Viktoryia Kozlouskaya, M.D., Ph.D., Mahyar Khazaeli, M.D., Randie H. Kim, M.D., Ph.D., Robert E. LeBlanc, M.D., Aofei Li, M.D., Constanza Lobos, M.D., Thai Yen Ly, M.D., Maria E. Mazzei, M.D., Jennifer M. McNiff, M.D., Alicia McNish, M.B.B.S., Mariel Molina Nunez, M.D., Shabnam Momtahen, M.D., Kiran Motaparthi, M.D., Patricia Muñoz-Hernández, M.D., Priya Nagarajan, M.D., Ph.D., Michael Occidental, M.D., Ifeoma U. Perkins, M.D., Nat Pernick, M.D., Jose A. Plaza, M.D., Syeda Qasim, M.D., Shyam Raghavan, M.D., Mazaher Ramezani, M.D. , Nemanja Rodic, M.D., Ph.D., Pedro Rodríguez-Jiménez, M.D., Ph.D., Bethany R. Rohr, M.D., Gabriela Rosa, M.D., Cecilia Rosales, M.D., Simon F. Roy, M.D., Jasmine Saleh, M.D., M.P.H., Elie Saliba, M.D., Jorge L. Sánchez, M.D., Omar P. Sangueza, M.D., Casey Schukow, D.O., Shahin Shahsavari, B.A., Sara C. Shalin, M.D., Ph.D., Michi Shinohara, M.D., Hanna Siatecka, M.D., Gülçin (Güler) Şimşek, M.D., Raj Singh, B.S., Andrzej Slominski, M.D., Ph.D., Bruce R. Smoller, M.D., Stephen Somach, M.D., Abha Soni, D.O., M.P.H., Jodi Speiser, M.D., Lauren N. Stuart, M.D., M.B.A., Pranvera Sulejmani, M.S., Liye Suo, M.D., Ph.D., Mariantonieta Tirado, M.D., Joel Tjarks, M.D., Noel Turner, M.D., V. Claire Vaughan, M.D., Noreen M. Walsh, M.D., Jing Wang, M.D., Ph.D., Janelle Welch, M.B.B.S., Joshua Wisell, M.D., Bicong (Crane) Wu, M.D., Jiejun Wu, M.D., Ph.D., Hillary Zalaznick, M.D., Pooria Zare, M.D., M.P.H., Natalia Zhovta, M.D.
Resident / Fellow Advisory Boards: Josephine K. Dermawan, M.D., Ph.D., Caroline I.M. Underwood, M.D.
Editorial Board Members: Hillary Rose Elwood, M.D., Jonathan D. Ho, M.B.B.S., D.Sc., Viktoryia Kozlouskaya, M.D., Ph.D., Robert E. LeBlanc, M.D., Kiran Motaparthi, M.D., Bethany R. Rohr, M.D., Sara C. Shalin, M.D., Ph.D., Brandon Umphress, M.D.
Deputy Editors-in-Chief: Jonathan D. Ho, M.B.B.S., D.Sc., Debra L. Zynger, M.D.
Editor-in-Chief: Debra L. Zynger, M.D.

Copyright: 2002-2024, PathologyOutlines.com, Inc.

DermPath related: Jobs, Fellowships, Conferences, Cases, CME, Board Review, What's New

Related chapters: Skin-Melanocytic tumor, Skin-Nonmelanocytic tumor

Editorial Board oversight: Bethany R. Rohr, M.D. (last reviewed February 2024)
Page views in 2024 to date: 21

Abscess
Definition / general
  • Latin: abscessus
  • Skin abscess represents a localized collection of pus that generally develops in response to infection or to the presence of other foreign materials under the skin; formed by tissue disintegration and surrounded by an inflamed area
Essential features
  • Destruction of tissue
  • Addition of fibrin and neutrophils
Terminology
  • Skin abscesses are often referred to as boils
Epidemiology
  • Develops in patients of all age groups
Etiology
  • Typically caused by either an inflammatory reaction to an infectious process (bacteria or parasite) or, less commonly, to a foreign substance within the body (a needle or a splinter, for example)
  • Abscesses may develop because of obstructed oil (sebaceous) or sweat glands, inflammation of hair follicles, or from minor breaks and punctures of the skin; abscesses may also develop after a surgical procedure
  • Most common bacterial organism is Staphylococcus aureus, although various other organisms can also lead to abscess formation
Clinical features
  • An abscess is typically painful, and it appears as a warm, swollen area
  • Hospitalizations for skin abscesses may be increasing (Eur J Clin Microbiol Infect Dis 2011;31:93)
  • Skin surrounding an abscess typically appears pink / red
  • Middle is filled with pus and debris
Treatment
  • Antibiotics alone will not typically cure a skin abscess
  • In general, abscesses must be opened and drained to start healing process
Case reports
Clinical images

Images hosted on other servers:
Missing Image Missing Image

Abscess

Microscopic (histologic) description
  • Cavity usually located on the dermis containing inflammatory infiltrate with abundant neutrophils and necrotic debris
  • Cavity is usually surrounded by inflammatory infiltrate
  • There is also subepidermal edema
Positive stains
  • Gram positive or negative if of bacterial origin

Acanthosis nigricans
Definition / general
  • Brown, velvety and verrucous plaques in axillae, back of neck and other skin folds, associated with visceral malignancies, endocrine diseases and congenital disorders
Etiology
  • Cutaneous manifestation of a diverse group of diseases
  • May occur as inherited disorder, with Down syndrome or after ingestion of drugs
  • 80% are "benign" type, either autosomal dominant or associated with tissue resistance to insulin, including diabetes, obesity and Cushing's disease
  • 20% are associated with GI or other internal malignancies
  • Usually age 40+ years
Clinical features
  • Brown, velvety, and verrucous plaques in axillae, back of neck and other skin folds
  • Oral mucosa (lips and tongue) affected in 25% of cases; rare involvement of esophagus
  • Hyperkeratotic lesions may develop on the palms, soles and knuckles
Clinical images

Contributed by Mark R. Wick, M.D.

Breast



Images hosted on other servers:

Hyperpigmented, brownish, velvety lesions

Right neck

Microscopic (histologic) description
  • Orthokeratotic hyperkeratosis (not actually acanthosis) and papillomatosis of stratum spinosum
  • Hyperpigmentation of basal cell layer, but no melanocytic hyperplasia
  • Usually no dermal inflammation
Microscopic (histologic) images

Contributed by Mark R. Wick, M.D.

Breast



Images hosted on other servers:

Papillomatosis and hyperkeratosis, H&E

Differential diagnosis

Acne keloidalis (pending)
[Pending]

Acne vulgaris
Definition / general
  • Common skin disease of seborrhea (scaly red skin), comedones (blackheads and whiteheads), papules (pinheads), pustules (pimples), nodules (large papules) and possibly scarring
  • Typically affects skin with densest sebaceous follicles: face, upper chest, back; uncommon in scalp for unknown reasons
Terminology
  • Acne vulgaris is common acne, typically with comedones
  • Acne rosacea is a different disorder
Etiology
  • Due to hormonal variations and alterations in hair follicle maturation
  • Begins with abnormal follicular keratinization with retention of keratinous material in the follicle
  • Then increased sebum production
  • Then appearance of gram positive, anaerobic, diphtheroid Propionibacterium acnes
  • May become inflammatory
Diagrams / tables

Images hosted on other servers:

Stages of acne

Clinical features
  • Usually teenagers of either sex, but often more severe in males
  • Worsens with drugs (steroids, testosterone, contraceptives), oils / tars, heavy clothing and tropical climates
  • Non-inflammatory acne is due to open and closed comedones
  • Open comedones: small follicular papules with central, black keratin plug (due to oxidation of melanin)
  • Closed comedones: follicular papules with a deeper plug below the surface
  • Follicles may rupture and become inflamed
  • Diverse lesions such as comedones, papules, pustules, cysts, sinuses and scars
Treatment
Clinical images

Images hosted on other servers:

Various images

Microscopic (histologic) description
  • 3 major components are comedones, inflammatory lesions and scars
  • Comedones (open or closed) are cyst-like cavities filled with compact mass of keratinous material and numerous bacteria
  • Closed comedo has 1 - 2 hairs trapped in lumen and atrophic, sebaceous acini
  • Open comedo has 10 - 15 hairs in lumen and sebaceous acini are atrophic or absent
  • Epithelial lining of comedones is usually thin

Acrokeratosis paraneoplastica (Bazex syndrome) (pending)
[Pending]

Actinic (solar) purpura (pending)
[Pending]

Actinic granuloma
Definition / general
  • Erythematous or brown annular infiltrate in face, neck or dorsa of hands of elderly or heavily sun exposed individuals
  • May be a variant of granuloma annulare
Terminology
  • Also called O’Brien actinic granuloma or Miescher granuloma
Case reports
Clinical images

Images hosted on other servers:

Neck lesion

Microscopic (histologic) description
  • Dermis in region of rim has histiocytes and foreign body giant cells engulfing elastotic fibers (elastoclasis)
  • Also variable lymphocytes, plasma cells and eosinophils
  • Usually no necrobiosis, no increased dermal mucin
Microscopic (histologic) images

Images hosted on other servers:

Granulomatous infiltrate of multinucleated giant cells in upper reticular dermis

Elastophagocytosis
and loss of elastic
fibers in center of lesion
(orcein staining)

Differential diagnosis
  • Granuloma annulare and necrobiosis lipoidica usually have necrobiosis, increased dermal mucin

Actinomycosis
Definition / general
Essential features
  • Causative agent Actinomyces israelli
  • Treated with high dose penicillin
  • Splendore-Hoeppli phenomenon is a histologic feature
Terminology
  • Most common causative agent is Actinomyces israelii
  • Sulfur granules: small yellow granules found within the abscesses formed by Actinomyces infection (Ophthal Plast Reconstr Surg 1992;8:237)
  • Lumpy jaw syndrome: large abscesses located on the head and neck, usually following dental disease and mandibular osteomyelitis (BMJ Case Rep 2015;2015:bcr2014206557 )
  • Splendore-Hoeppli phenomenon (asteroid bodies): pink rim at the periphery of the colony, due to immunoglobulin and cell debris, which occurs around colonies of fungi, bacteria and parasites (J Oral Maxillofac Pathol 2018;22:161)
ICD coding
  • ICD-10: A42.9 - actinomycosis, unspecified
Epidemiology
Sites
  • Cervicofacial (post dental infection), skin (posttraumatic injury creating an anaerobic environment), pelvic (post intrauterine device placement), abdominal (status post ruptured appendix or bowel perforation) and pulmonary (smokers with poor dental hygiene, aspiration of infective material) (Infect Drug Resist 2014;7:183)
Pathophysiology
  • Predisposing factors include poor oral hygiene, trauma, male gender, diabetes mellitus, immunosuppression, alcoholism and malnutrition (Infect Drug Resist 2014;7:183)
  • Filamentous bacteria are a normal commensal inhabitant of the oral and buccal cavities, gastrointestinal tract and female genitalia
  • Etiology of infection in skin is commonly linked with a traumatic injury, human bite or a perforating injury, which creates an anaerobic environment for Actinomyces israelii to grow in (J Clin Diagn Res 2014;8:YD03)
Etiology
  • Caused by Actinomyces israelii in humans and Actinomyces bovis in animals
Clinical features
  • Localized pain, swelling and draining fistulas
Diagnosis
  • Culture on chocolate agar media at 37 °C
  • Gram stain is more sensitive than culture, especially if patient is on antibiotics
  • Filamentous bacteria of actinomycosis can be identified on H&E and are highlighted by Gram stain (Infect Drug Resist 2014;7:183)
  • Polymerase chain reaction (PCR) and nucleic acid probes are being developed for faster and more accurate identification
Case reports
Treatment
  • High dose penicillin is necessary to penetrate areas of fibrosis and suppuration / granules (Clin Infect Dis 2004;38:444)
  • Drainage of abscesses or radical excision of sinus tracts
Clinical images

Contributed by Dhiraj B. Nikumbh, M.B.B.S., M.D.
Missing Image

Multiple sinuses in foot



Images hosted on other servers:
Missing Image

Indurated lesion

Missing Image

Lesion over back

Missing Image

Left thigh lesion

Missing Image

Molar tooth appearance

Gross description
  • Firm, swollen region on the skin with multiple draining abscesses and fistula tracts
  • Pus draining yellow sulphur granules
Gross images

Images hosted on other servers:
Missing Image

Outer / inner surface and cross section of excised tissue

Microscopic (histologic) description
  • Lymphocytes, neutrophils, giant cells and fibroblasts
  • Bacterial colonies (sulphur granules) found at the center of the inflammatory reaction, composed of basophilic radiating filaments
Microscopic (histologic) images

Contributed by Kuroli Eniko, M.D., Yale Rosen, M.D. and Haitham Alfalah, M.D. (source: Wikipedia)
Suppurative and granulomatous inflammation

Suppurative and granulomatous inflammation

Dense sheets of neutrophils

Dense sheets of neutrophils

Colony of Actinomyces Colony of Actinomyces

Colony of Actinomyces


Thin gram positive filamentous organisms Thin gram positive filamentous organisms Thin gram positive filamentous organisms

Thin gram positive filamentous organisms

Grocott stain

Grocott stain

Gram stain

Gram stain

Cytology description
  • Mixed inflammatory infiltrate, foreign body multinucleated giant cells and clumps of filamentous organisms
Cytology images

Contributed by Dhiraj B. Nikumbh, M.B.B.S., M.D.
Missing Image Missing Image Missing Image Missing Image

Mixed inflammatory infiltrate with clumps of fibrillar organisms



Images hosted on other servers:
Missing Image

Gram stain

Positive stains
Negative stains
Sample pathology report
  • Skin, biopsy:
    • Primary cutaneous actinomycosis
Differential diagnosis
  • Nocardiosis:
    • Branching filamentous bacteria are partially acid fast positive
    • Histologic appearance of Nocardia is similar to other actinomycetes family members; culture and biochemical testing is necessary for diagnosis / identification
    • Fite-Faraco stain is helpful since positive in nocardiosis
  • Tuberculosis:
    • Characteristic histologic findings include caseating granulomas
    • Detected by Ziehl-Neelsen stain and Mycobacterium culture
  • Sporotrichosis:
    • Characteristic histologic findings include mixed acute and chronic inflammatory response
    • Organisms "sporothrix asteroids" can be detected via PAS, GMS stains
  • Paracoccidioidomycosis:
    • Characteristic histologic findings include mixed acute and chronic inflammatory response
    • Organisms referred to as captain wheel on morphology can be detected via PAS, GMS stains
Board review style question #1

What is the name of the unique histologic pattern that takes place in a cutaneous actinomycosis infections?

  1. Asteroid body reaction
  2. Flame figures
  3. Gout
  4. Splendore-Hoeppli phenomenon
Board review style answer #1
D. Splendore-Hoeppli phenomenon

Comment Here

Reference: Actinomycosis
Board review style question #2
What type of ​granules are found within the abscesses formed by Actinomyces infection?

  1. Metachromatic granules
  2. Polyphosphate granules
  3. Polysaccharide granules
  4. Sulfur granules
Board review style answer #2
D. Sulfur granules

Comment Here

Reference: Actinomycosis

Acute cutaneous lupus erythematosus (pending)
[Pending]

Acute febrile neutrophilic dermatosis (Sweet syndrome)
Definition / general
  • Also called acute febrile neutrophilic dermatosis
  • Abrupt onset of tender or painful erythematous plaques and nodules on the face and extremities and less commonly on the trunk, in association with fever (usually), malaise and a neutrophil leukocytosis
  • Associated with AML, less often with solid malignancies
  • Often females, any age but rare in childhood
  • Unknown etiology, but may represent immunological hypersensitivity reaction
Treatment
  • Most cases respond to oral corticosteroids
  • Thalidomide was successful in one patient who failed to respond to metronidazole, dapsone and methotrexate; IV immunoglobulin was used in a child with concurrent immunodeficiency
  • Also response to 5-azacytidine in patient with myelodysplastic syndrome
  • A case of histiocytoid Sweet's syndrome was responsive to dapsone; dapsone has also been used with systemic corticosteroids in a patient who was HIV positive
Microscopic (histologic) description
  • Intense neutrophilic dermal infiltrate in reticular dermis, may be perivascular, diffuse and surround sweat glands; edema with marked leukocytoclasia; marked papillary edema
  • Occasional presence of dermal papillary microabscesses can result in confusion with dermatitis herpetiformis
  • Epidermis is normal, occasionally slight spongiosis, vesiculation with spongioform pustule
  • Necrotic keratinocytes may be present
  • Variable eosinophils, lymphocytes and histiocytes
  • Blood vessels are dilated and show endothelial swelling
Microscopic (histologic) images

Contributed by Angel Fernandez-Flores, M.D., Ph.D.

Differential diagnosis
  • Behçet disease may be associated with lesions similar to Sweet syndrome
  • Gram stain and PAS exclude infection
  • Granuloma faciale: fibrinoid necrosis is minimal but eosinophils are prominent
  • Late lesions of erythema elavatum diutinum and granuloma faciale show fibrosis, not seen in Sweet Syndrome
  • Pyoderma gangrenosum: has ulcer, no leukorrhexis
  • Rheumatoid neutrophilic dermatitis, neutrophil rich variant of anaplastic large cell lymphoma
  • Presence of fibrinoid vascular changes distinguishes necrotizing vasculitis such as leukocytoclastic vasculitis, erythema elevatum diutinum and granuloma faciale from Sweet syndrome
Additional references

Acute generalized exanthematous pustulosis
Definition / general
  • Pustular drug eruption
  • May have superficial desquamation
Essential features
  • Pustular drug eruption
  • Most commonly associated with beta lactams and macrolides but also calcium channel blockers, acetaminophen, nonsteroidal anti-inflammatory drugs, proton pump inhibitors, anticonvulsants (carbamazepine), cetirizine, antimalarials, antifungals and more rarely, enterovirus or mercury exposure (Int J Dermatol 2017;56:405)
  • Short interval from exposure to eruption, < 4 days
  • Quick improvement after withdrawal of offending agent
Terminology
  • Acute generalized exanthematous pustulosis, AGEP, toxic pustuloderma
ICD coding
  • ICD-10: L27.0 - Generalized skin eruption due to drugs taken internally
Epidemiology
Pathophysiology
Diagrams / tables

Images hosted on other servers:

Proposed mechanism

Etiology

Clinical features
  • Acute onset of nonfollicular based monomorphic sterile pustules on a background of edematous erythema with superficial desquamation in areas of confluence (J Am Acad Dermatol 2015;73:843)
  • Spreads from the face and intertriginous areas
  • Mucous membrane involvement may occur
  • Fever, pruritus or burning sensation
  • Resolves in 1 - 2 weeks
  • 17% of cases have visceral involvement
  • 1 - 2% mortality
Diagnosis
  • Histological findings are nonspecific and a definitive diagnosis should only be rendered in the appropriate clinical context and after consideration of other entities with the same histological appearance
  • Eruption should occur shortly after initiation of a new drug or, less commonly, with viral infection and should also convalesce after discontinuation of the drug
  • Patch testing can be useful in confirming the patient's sensitivity to the drug in question
Laboratory
  • Neutrophilia and leukocytosis
  • Abnormal renal function and hepatic function tests may occur
  • Immunofluorescence is negative (as opposed to IgA pemphigus and pemphigus foliaceus)
  • 50 - 60% of cases have a positive patch test to offending agent
Case reports
Treatment
  • Discontinue drug (J Immunol Res 2017;2017:1503709)
  • Topical or systemic steroids (cautiously as steroids may provoke a pustular psoriasis flare)
  • May pursue patch testing, especially in cases in which multiple drugs are suspected
Clinical images

Images hosted on other servers:

Erythematous macules

Vesicles and desquamation

Sterile
micropustules
on a background
of macular erythema

Pruritic nonfollicular
based pustules
over edematous
erythema


Facial edema

Pustules occur in intertriginous areas such as the axilla

Eruption resolves with superficial desquamation

Microscopic (histologic) description
  • Subcorneal pustules
  • Spongiosis
  • Histologically similar to pustular psoriasis but may have rare dermal eosinophils
  • Scattered epidermal neutrophils
  • Papillary dermal edema
  • Perivascular mixed infiltrate, often with eosinophils and neutrophils
  • Necrotic keratinocytes are sometimes seen
Microscopic (histologic) images

Contributed by University of Colorado Department of Pathology and Joshua Wisell, M.D.

Subcorneal neutrophilic pustules, spongiosis, scattered dermal eosinophils


Superficial epidermal pustulation, spongiosis, scattered dermal eosinophils

Immunofluorescence description
  • Negative
Negative stains
  • Special stains may be used to exclude mimickers (e.g. a Gram stain may be used to investigate the possibility of bullous impetigo)
Differential diagnosis
Board review style question #1
A skin biopsy is received from a patient staying in the intensive care unit and undergoing antimicrobial therapy for bacterial pneumonia. On physical exam, there are innumerable minute monomorphic pustules over the trunk on a background of intense erythema with desquamation in the skin folds. The biopsy shows subcorneal clefting with neutrophilic pustules, spongiosis and scattered dermal eosinophils. Which of the following is most likely given the biopsy and clinical context?

  1. Acute generalized exanthematous pustulosis
  2. Cellulitis
  3. Folliculitis
  4. Sneddon-Wilkinson disease
  5. Staphylococcal scalded skin disease
Board review style answer #1
A. Acute generalized exanthematous pustulosis

Comment Here

Reference: Acute generalized exanthematous pustulosis
Board review style question #2

This skin biopsy is taken from the trunk of a 50 year old man. Which of the following is the most likely diagnosis?

  1. Acute generalized exanthematous pustulosis
  2. Bullous pemphigoid
  3. Eczema
  4. Pemphigus vulgaris
  5. Sweet syndrome
Board review style answer #2
A. Acute generalized exanthematous pustulosis

Comment Here

Reference: Acute generalized exanthematous pustulosis

Algal infection-Prototheca
Definition / general
  • Not fungi but similar features
  • First recognized as human pathogen in 1964; 2 species associated with human disease: P. wickerhamii and P. zopfii
  • Aerobic, achlorophyllous, algae-like, unicellular organisms in water, sewage and soil
  • < 100 cases of human infection reported, usually via trauma or contaminated water, involving face or exposed extremities with erythematous plaques, nodules or superficial ulcers
  • Rarely infects toenails of patients with diabetes (Mycopathologia 2011;172:207)
  • Primary cell or spherule is called the theca; species reproduce by internal septation, forming sporangia which contain up to 20 endospores
  • P. wickerhamii: cells are rounder than the oval / cylindrical shapes of P. zopfii
  • Diagnosis: corn meal agar
Case reports
  • 34 year old woman with index finger pain and hobbies of scuba diving and maintaining an aquarium (Arch Pathol Lab Med 2001;125:450)
  • 73 year old man with nodules on leg secondary to splinter injury while chopping firewood (Case #225)
Treatment
  • Amphotericin B if disseminated, surgical excision if focal cutaneous or subcutaneous
Clinical images

Images hosted on other servers:

Lesion on foot

Microscopic (histologic) description
  • Necrotizing granulomas of subcutis containing spherical organisms with central basophilia, internal septation and double layer cell walls
  • Also multiple endospores 2 - 4 microns
Microscopic (histologic) images

Images hosted on other servers:

Various images

Internal septation

H&E, GMS and PAS

Positive stains
Additional references

Allergic contact dermatitis
Definition / general
  • T cell mediated, delayed (type IV) hypersensitivity reaction to allergens from skin contact
  • Most common occupational skin disease
    • Healthcare workers, hairdressers / beauticians, machinists, chemical industry
  • Most common allergen: nickel
    • Other common allergens: latex, fragrances, cosmetic products, soaps, topical antibiotics, poison ivy
  • Reference: Contact Dermatitis 2019;80:77
Essential features
  • Spongiotic dermatitis from a skin allergen
ICD coding
  • ICD-10: L25.9 - contact dermatitis (occupational) NOS
Epidemiology
  • Any age / gender
Sites
  • Any
Pathophysiology
  • Hapten binding phase (initial step)
    • Contact allergen (hapten) penetrates skin barrier
    • Hapten is bound to a skin protein carrier
  • Sensitization (afferent) phase
    • Antigen presenting cells (including Langerhans cells) present the hapten protein complex to priming T cells
    • Generates hapten specific memory / effector T cells
  • Elicitation (efferent phase) (Allergy 2009;64:1699)
    • Repeat exposure to hapten elicits hapten specific memory / effector T cells
      • Elicits an inflammatory / cytokine response
      • Leads to tissue damage - clinical manifestation of allergic contact dermatitis (ACD), 48 - 72 hours after repeat exposure
Etiology
  • Common culprits include nickel, poison ivy, dyes / fragrances
Clinical features
  • Eczematous scaly lesions
    • Severe cases can be vesicular / bullous
  • Chronic / repeated exposure can lead to lichenification
  • Mostly confined to site of direct contact with allergen
    • Transfer of allergen from primary contact site to other areas can occur
  • Reference: J Cutan Pathol 2016;43:498
Diagnosis
  • Can be established on clinical history and presentation alone
  • Histologic features are nonspecific
  • Gold standard for definitive diagnosis: patch testing
  • Reference: J Clin Aesthet Dermatol 2010;3:36
Case reports
Treatment
  • Allergen avoidance and symptomatic treatment
Clinical images

Images hosted on other servers:

Linear demarcation due to textile dyes

Due to hair dye

Microscopic (histologic) description
  • Spongiotic dermatitis in various stages (acute, subacute and chronic)
    • Acute:
      • Epidermal edema
      • With or without prominent intercellular bridges or vesiculation
      • Normal basket weave orthorkeratosis
    • Subacute:
      • Acanthosis and epidermal edema
      • With or without hyperkeratosis or parakeratosis
    • Chronic:
      • Marked acanthosis
      • Hyperkeratosis with or without parakeratosis
      • Minimal epidermal edema
    • Langerhans cell microabscesses can be associated with ACD but are a nonspecific finding
      • Langerhans cells have abundant cytoplasm and horseshoe shaped nuclei
    • Reference: J Cutan Pathol 2016;43:498
Microscopic (histologic) images

Contributed by Gabriela Rosa, M.D.
Severe acute spongiotic dermatitis

Severe acute spongiotic dermatitis

Early acute spongiotic dermatitis

Early acute spongiotic dermatitis

Subacute spongiotic dermatitis

Subacute spongiotic dermatitis

Chronic spongiotic dermatitis

Chronic spongiotic dermatitis

Langherhans cell microabscess

Langherhans cell microabscess

Immunofluorescence description
  • IHC not necessary for diagnosis
Sample pathology report
  • Skin, arm, punch biopsy:
    • Acute spongiotic dermatitis (see comment)
    • Comment: Possibilities could include an eczematous dermatitis, such as allergic contact dermatitis, given the presence of a Langerhans cell microabscess. There is literature regarding Langerhans cell microabscesses in association with allergic contact dermatisis, although this association is nonspecific.
Differential diagnosis
  • Atopic dermatitis:
    • Spongiotic changes similar to ACD
    • Clinical presentation and patch testing will help differentiate from ACD
  • Dermatophytosis:
    • Spongiotic changes similar to ACD
    • Changes to the stratum corneum can be a clue to a dermatophyte infection
      • Neutrophils in the stratum corneum
      • Sandwich sign - orthokeratosis between parakeratosis and hyphae
    • Clinical presentation, potassium hydroxide (KOH) prep or fungal stains (PAS, GMS) will help differentiate from ACD
  • Irritant contact dermatitis:
    • Dermatitis from direct contact with irritant chemical or substance (nonimmune mechanism)
    • Spongiotic changes similar to ACD, with or without keratinocyte necrosis
      • With or without neutrophils in the epidermis (neutrophilic spongiosis)
  • Mycosis fungoides (MF):
    • Early patch stage mycosis fungoides can show spongiotic changes similar to ACD
    • To differentiate from ACD (the following favors MF)
      • Degree of lymphocyte exocytosis
      • Lymphocyte tagging basement membrane
      • Pautrier microabscess (lymphocyte collections in epidermis)
      • Unusual to have numerous eosinophils
      • Unusual to have severe spongiotic changes
    • Clinical presentation and disease course will help differentiate from ACD
    • T cell receptor gene rearrangement analysis can help differentiate MF from other entities
  • Early / urticarial phase of bullous pemphigoid:
    • Can show spongiotic changes
    • Eosinophils tagging basement membrane with or without eosinophilic spongiosis can be a clue to bullous pemphigoid
    • Clinical presentation and direct immunofluorescence will help differentiate from ACD
Board review style question #1
What is the gold standard for establishing a diagnosis of allergic contact dermatitis?

  1. Endoscopy
  2. Patch testing
  3. Serum histamine levels
  4. Skin biopsy
Board review style answer #1
B. Patch testing. Commercial patch testing involves applying a set of the most common allergens to the skin and assessing for reactions at 48 and 72- 96 hours respectively (as determined by the North American Contact Dermatitis Group criteria). Skin biopsy could be helpful in establishing the injury pattern of a rash and could raise suspicion for allergic contact dermatitis but a skin biopsy by itself would not be diagnostic. Serum histamine levels and endoscopy play no role in the diagnosis of allergic contact dermatitis.

Comment Here

Reference: Allergic contact dermatitis
Board review style question #2

What pattern of injury is classically seen in allergic contact dermatitis (shown above)?

  1. Granulomatous dermatitis
  2. Interface dermatitis
  3. Sclerosing dermatitis
  4. Spongiotic dermatitis
Board review style answer #2
D. Spongiotic dermatitis. The classic histologic features of allergic contact dermatitis are those of a spongiotic dermatitis. In chronic longstanding allergic contact dermatitis, there can also be psoriasiform changes with marked acanthosis and lesser degrees of spongiosis. Sclerosing dermatitis is a dermal process that would be seen in lichen sclerosus, morphea / scleroderma and other sclerosing lesions but not typically in ACD. Interface dermatitis is seen in autoimmune connective tissue disease, drug reactions, erythema multiforme and other interface processes. Irritant contact dermatitis can show a combination of both spongiotic and interface changes. Granulomatous dermatitis is seen in granuloma annulare, granulomatous drug reactions, sarcoidosis and infections, among other entities.

Comment Here

Reference: Allergic contact dermatitis

Alopecia areata
Definition / general
Essential features
  • Originates from loss of the immune privilege of the hair follicle, which allows for a lymphocyte mediated autoimmune attack of the hair follicle bulb (Expert Rev Clin Immunol 2015;11:1335)
  • A characteristic clinical finding is exclamation point hairs: 2 - 3 mm hairs that appear to float on the scalp because of shaft narrowing and hypopigmentation near the scalp surface that appear at the advancing margin of the patches (Postepy Dermatol Alergol 2014;31:113)
  • Increased miniaturized hairs and characteristic "swarm of bees" peribulbar mononuclear cell inflammation affecting terminal anagen and catagen hairs seen in acute phase (Histopathology 2010;56:24, Arch Dermatol 2003;139:1555)
  • Numerous miniaturized (vellus) hairs and mild peribulbar mononuclear cell inflammation involving nanogen hairs seen in chronic phase (Arch Dermatol 2003;139:1555)
Epidemiology
Sites
  • Scalp and hair bearing skin
  • Any site with hair can be affected
Pathophysiology
  • Originates from loss of the immune privilege of the hair follicle, which allows for a lymphocyte mediated autoimmune attack of the hair follicle bulb (Expert Rev Clin Immunol 2015;11:1335)
  • Lymphocytes are comprised of CD8+ NK group 2D positive (NKG2D+) T cells that release proinflammatory cytokines and chemokines that reject the hair (Clin Dev Immunol 2013;2013:348546)
  • Exact mechanism is not yet understood although a genome wide association study provided evidence that both acquired and innate immunity contribute to the pathogenesis of alopecia areata (Nature 2010;466:113)
  • Onset or recurrence of hair loss is sometimes triggered by viral infection, trauma, hormonal changes or emotional / physical stressors (Nat Rev Dis Primers 2017;3:17011)
Clinical features
  • Classic alopecia areata
  • Subtypes of alopecia areata (Chin Med Sci J 2017;32:44)
    • Alopecia totalis (5% of cases)
      • All or the majority of scalp hair is lost
    • Alopecia universalis (less than 1% of cases)
      • All or the majority of hair on the scalp and body is lost
    • Ophiasis
      • Pattern of alopecia areata affecting the occipital and lateral scalp in which a bald area encircles the scalp
    • Diffuse alopecia areata (alopecia areata incognita)
      • Sudden, diffuse loss of hair characterized by the gray color of the persisting hair (turning white overnight) and has a positive hair pull test
    • Alopecia areata of the nails (affects 10 - 50% of patients with alopecia areata)
      • Regular pitting and ridging of the nails
Diagnosis
  • Scalp biopsy followed by histologic examination
  • Trichoscopy (J Dermatol 2018;45:692)
    • Findings include yellow dots, short vellus hairs, black dots, broken hairs, exclamation mark hairs, tapered hair, upright regrowing hairs, pigtail (circle) hairs, and Pohl-Pinkus constrictions
Prognostic factors
  • Treatment may help halt further hair loss or initiate hair regrowth (Can Fam Physician 2015;61:757)
  • Prognostic factors associated with poor outcomes in individuals with alopecia areata (Can Fam Physician 2015;61:751)
    • Extensive loss
    • Ophiasis variant
    • Nail changes
    • Early age of onset
    • Family history
    • Concomitant autoimmune diseases (e.g. atopy, Hashimoto thyroiditis)
Case reports
Treatment
Clinical images

Images hosted on other servers:

Patchy hair loss

Trichoscopy

Regrowth after treatment

Microscopic (histologic) description
  • Hair cycle consists of three phases: anagen (the growth phase), catagen (the regressing phase) and telogen (the resting phase) (J Investig Dermatol Symp Proc 2003;8:56)
  • Anagen:
    • Terminal anagen hair extends from its bulb in the subcutaneous tissue to its point of emergence from the epidermis through the follicular infundibulum (J Investig Dermatol Symp Proc 2003;8:56)
    • Has fully developed inner and outer root sheaths with no signs of apoptosis in the outer root sheath
  • Catagen:
    • Hair shaft tracts upward and the lower follicle disappears, leaving an angiofibrotic strand or streamer (stela) (J Investig Dermatol Symp Proc 2003;8:56)
    • Has thickening of the basal membrane and apoptotic bodies in outer root sheath
  • Telogen:
    • Club shaped root is situated at the level of the bulge at the insertion of the arrector pili muscle (J Investig Dermatol Symp Proc 2003;8:56)
    • Has wrinkling of the inner root sheath (flamethrower appearance)
Microscopic (histologic) images

Contributed by Jodi Speiser, M.D.
Vertical section of scalp biopsy

Vertical section of scalp biopsy

Perifollicular lymphocytes

Perifollicular lymphocytes

Peribulbar / intrabulbar lymphocytic infiltration

Peribulbar / intrabulbar
lymphocytic infiltration

Immunofluorescence description
Positive stains
Electron microscopy description
  • 4 patterns (J Dermatol 2013;40:1045)
    • Long tapering structure with no accumulation of scales
    • Club shaped hair root with fine scales
    • Proximal accumulation of scales
    • Sharp tapering of the proximal end of hair
Sample pathology report
  • Skin, left posterior scalp, punch biopsy:
    • Nonscarring alopecia (see comment)
    • Comment: Histological sections show a reduction in number of hair follicles with evidence of miniaturization. There is bulbar and periadnexal lymphocytic inflammation. Immunohistochemical stain for CD3 demonstrates lymphocytes are predominantly T cells. There is a catagen / telogen shift. PAS stain shows normal basement membrane thickness and EVG stain shows preservation of elastic fibers. These findings are consistent with alopecia areata.
Differential diagnosis
  • Syphilitic alopecia:
    • Characterized by superficial and deep (peribulbar) inflammation with plasma cells (in contrast to alopecia areata and psoriatic alopecia, which have peribulbar lymphocytic infiltrates with occasional eosinophils)
    • Confirmatory serology is recommended
    • Endothelial cell predominance
    • May be indistinguishable based on histopathology alone
  • Telogen effluvium:
    • Normal number of total hair follicles but with catagen / telogen shift
    • No miniaturization of hair follicles
    • Minimal inflammation is present
    • Synchronization is typical of alopecia areata but not telogen effluvium
    • While alopecia areata can have catagen / telogen counts of 50% or more, this is not observed in telogen effluvium
  • Psoriatic alopecia:
    • Normal or decreased number of total hair follicles but with catagen / telogen shift
    • Extensive follicular miniturization
    • Atrophy of sebaceous glands
    • Epidermal changes of seborrheic dermatitis or psoriasis
    • May show mild, focal peribulbar lymphocytic infiltrates with occasional eosinophils
    • Premature desquamation of the inner root sheath
  • Trichotillomania::
    • May have the follwing histologic features:
      • Incomplete, disrupted follicular anatomy and increased catagen / teolgen hairs without inflammation or miniaturization
      • Trichomalacia
      • Melanin in collapsed fibrous root sheaths
      • Fractured hair shafts
      • Perifollicular and interfollicular hemorrhage
    • Has pigment casts but these are nonspecific and can be observed in alopecia areata, traction alopecia and syphilis
    • Clinical history is especially important
  • Acute traction alopecia:
    • May have the follwing histologic features:
      • Normal follicular count with catagen / telogen shift
      • Trichomalacia and pigment casts present
      • Clinical history is espcially important - tight hair pulling or braiding
      • Absence of inflammation
      • Often sharp reduction in terminal hairs but with preservation of sebaceous lobules
  • Androgenetic alopecia:
    • Reduced follicular count at dermal / subcutaneous junction
    • Prominent sebaceous glands, minimal inflammation
    • Anisotrichosis and miniaturization with minimal inflammation
    • Mild increase in catagen / telogen hairs
    • Solar elastosis present in more advanced cases
Board review style question #1

Which of the following is true about the disorder shown in the image above?

  1. Increased number of terminal catagen and telogen hair
  2. Not associated with autoimmune disorders
  3. No hair follicular miniaturization
  4. Predominantly affects young Caucasian women
Board review style answer #1
A. Increased number of terminal catagen and telogen hair

Comment Here

Reference: Alopecia areata
Board review style question #2
In which of the following alopecias are exclamation point hairs seen?

  1. Alopecia areata
  2. Androgen alopecia
  3. Cicatricial alopecia
  4. Syphilitic alopecia
Board review style answer #2
A. Alopecia areata

Comment Here

Reference: Alopecia areata

Alopecia mucinosa
Epidemiology
  • Coexisting lymphoma associated with very poor prognosis
Clinical features
  • Also called follicular mucinosis
  • Edematous and erythematous plaques of alopecia on head and neck
  • Children: benign, self limited
  • Adults: associated with cutaneous T cell lymphoma, Sezary syndrome, Hodgkin lymphoma, acute myeloblastic leukemia, chronic lymphocytic lymphoma and squamous cell carcinoma of the tongue
  • Nodular or plaquelike lesion

  • Patterns:
    • Infiltrated plaque, solitary or multiple, associated with alopecia when in scalp or beard area
    • Group of of follicular papules, either localized or extensively distributed on trunk and proximal limbs in addition to scalp and face
    • Acneiform lesion with comedones, mucinorrhea (discharge of mucinous fluid from follicular ostia) and severe pruritis

  • Clinical course: either spontaneous regression, chronic relapsing but benign course over many years or associated with lymphoma
Case reports
Microscopic (histologic) description
  • Follicular infundibulum keratinocytes and outer root sheath are separated by pools of mucin
  • Mixed infiltrate of lymphocytes, histiocytes and conspicuous eosinophils
  • Marked follicular dilation with cyst formation and perifollicular scarring
  • Both the dermis and and affected epithelium are typically infiltrated by lymphocytes, histiocytes and eosinophils
  • In cases associated with lymphoma, atypical lymphocytes, convoluted lymphocytes, large transformed cells and mitotic figures may be seen
Microscopic (histologic) images

Contributed by Mowafak Hamodat, M.B.Ch.B., M.Sc.

Various images



Images hosted on other servers:

Superficial and deep,
perivascular and perifollicular,
infiltrate of lymphocytes and
numerous eosinophils

Positive stains
  • Alcian blue
Differential diagnosis
  • Coexisting mycosis fungicides: has atypical or cerebriform lymphocytes, bandlike infiltrate in upper dermis, no / minimal eosinophils; obtain multiple biopsies as needed
  • Note: TCR gene rearrangement present in 50% of patients whether associated with tumor or not

Alopecia mucinosa
Definition / general
  • Alopecia mucinosa is an inflammatory hair loss condition, characterized by the accumulation of mucin in sebaceous glands and around hair follicles (i.e., pilosebaceous units)
  • This inflammatory disorder may lead to subsequent follicular degeneration and may occur secondary to other diseases (e.g., lymphoma)
Essential features
  • Hair loss condition in which mucin accumulates in sebaceous glands and hair follicles (i.e., pilosebaceous units), causing subsequent degeneration and hair loss
  • It most frequently affects the head and neck regions, including the face and scalp
  • More commonly affects children and young adults and may appear clinically as edematous or erythematous papules, plaques or nodules of alopecia in hair bearing skin areas
  • Can either be a primary (i.e., idiopathic) or secondary condition and is typically benign and self limited, with more favorable outcomes in younger patients
  • Prognosis may be poor in patients with disease due to lymphoma or of older age
Terminology
  • Pinkus follicular mucinosis, follicular mucinosis
ICD coding
  • ICD-10: L65.2 - alopecia mucinosa
  • ICD-11: E70.5Y - scarring alopecia due to other specific cause
Epidemiology
Sites
  • Lesions on the face, neck and scalp are the most common but other body parts may also be affected
  • Affected skin areas will appear as pruritic, pink-white papules and plaques, along with subsequent hair loss (Indian Dermatol Online J 2013;4:333)
Pathophysiology
Etiology
Diagrams / tables
Not relevant to this topic
Clinical features
  • Edematous and erythematous alopecia papules, plaques or nodules of alopecia on the head, scalp, face or neck
  • Benign, self limited disease may be favored in children (i.e., spontaneous resolution within 2 - 24 months); secondary etiologies may be more common in adults (Indian Dermatol Online J 2013;4:333)
  • Patterns
    • Infiltrating solitary or multiple plaques associated with hair loss in the scalp or beard areas
    • Group of follicular papules, either localized or extensively distributed on trunk and proximal limbs in addition to scalp and face
    • Acneiform lesions with comedones, mucorrhea (discharge of mucinous fluid from follicular ostia) or severe pruritus
  • May spontaneously regress within a few years or chronically relapse but demonstrates a benign course over many years
  • Lesions present as an acneiform eruption, characterized by cysts or comedones, refractory to traditional acne therapies (Clin Exp Dermatol 2018;43:921, Am J Dermatopathol 2013;35:792)
  • May be observed in the setting of other nonlymphoproliferative conditions, such as basaloid follicular hamartoma, squamous cell carcinoma, seborrheic keratosis, prurigo, acne vulgaris, medication induced skin toxicity / vasculitis, polymorphous light eruption, insect / tick bite and Demodex infiltration (An Bras Dermatol 2022;97:45, Am J Dermatopathol 2014;36:705)
Diagnosis
  • Diagnosis is made via clinicohistopathologic correlation
Laboratory
Radiology description
Not relevant to this topic
Radiology images
Not relevant to this topic
Prognostic factors
Case reports
Treatment
Clinical images

Images hosted on other servers:
On back of child On back of child

On back of child

Hypopigmented patch on eyebrow

Hypopigmented patch on eyebrow

Gross description
Gross images
Not relevant for this topic
Frozen section description
Not relevant for this topic
Frozen section images
Not relevant for this topic
Microscopic (histologic) description
  • Follicular infundibulum keratinocytes and outer root sheath are separated by pools of mucin
  • Mucin may appear pale and basophilic, displacing collagen fibers and reside within the pilosebaceous unit
    • Dehydrated mucin may also appear as beads on string
  • There may be a mixed dermal infiltrate of lymphocytes, histiocytes and conspicuous eosinophils
  • Marked follicular dilation with cyst formation and perifollicular scarring may also be seen
  • In cases associated with lymphoma, folliculotropism by lymphocytes with cerebriform nuclei or large transformed cells may be seen
  • Reference: Cureus 2019;11:e4746, JAAD Case Rep 2023;34:83, J Am Acad Dermatol 2019;80:1524
Microscopic (histologic) images

Contributed by Mowafak Hamodat, M.B.Ch.B., M.Sc.
Alopecia mucinosis

Mucin seeming basophilic in appearance

Alopecia mucinosis

Separation of layers produced by pools of mucin

Mucin accumulation within the dermis

Mucin accumulation within the dermis

Perifollicular mucin deposition

Perifollicular mucin deposition

Virtual slides
No virtual slides found
Cytology description
Not relevant to this topic
Cytology images
Not relevant to this topic
Immunofluorescence description
  • Although not routinely used today, prior direct immunofluorescence studies demonstrated complement (C3) and fibrin / fibrinogen deposition within areas of follicular degeneration (J Am Acad Dermatol 1984;10:760)
Immunofluorescence images
Not relevant to this topic
Positive stains
Negative stains
N/A
Electron microscopy description
  • Although not routinely used today, prior electron microscopy studies demonstrated dilated cystic organelles and vesicles within affected root sheath cells accompanied by amorphous collections of fine granular materials
  • Extracellular hyaluronic acid accumulation may be seen (Acta Derm Venereol 1976;56:163)
Electron microscopy images
Not relevant to this topic
Molecular / cytogenetics description
Molecular / cytogenetics images

Images hosted on other servers:
Monoclonal products via PCR

Monoclonal products via PCR

TCR rearrangement via PCR

TCR rearrangement via PCR

Videos

Follicular mucinosis with Antonina Kalmykova & Phillip McKee

Sample pathology report
  • Skin (scalp), punch biopsy:
    • Alopecia mucinosa (see comment)
    • Comment: Histologic sections demonstrate loss of hair follicles. Residual follicles show mucinous degeneration. A mixed dermal infiltrate composed of lymphohistiocytic cells is also noted. Significant epidermotropism or folliculotropism is not seen. The etiology of this condition is not completely known, however it is associated with aberrant immune system activity and can be seen in the setting of inflammatory or lymphoproliferative conditions.
Differential diagnosis
Board review style question #1

A 10 year old boy presents with painless bumps on his cheek for the past 2 months. Patient denies any systemic symptoms. Coalescing erythematous papules are present on physical exam. A skin punch biopsy is obtained with findings shown above. Which of the following is most likely true?

  1. Alcian blue is likely to be positive
  2. Granulomatous infiltration must be present to consider leprosy
  3. Negative TCR gene rearrangement effectively rules out malignancy
  4. Spontaneous resolution is uncommon this age group
  5. Staphylococcus aureus is not implicated in the pathophysiology of this condition
Board review style answer #1
A. Alcian blue is likely to be positive. The most likely diagnosis in this patient is alopecia mucinosis (previously known as follicular mucinosis). Although the pathogenesis of this condition is not completely understood, it is hypothesized to be a result of cellular changes leading to mucin deposition due to aberrant follicular keratinocyte activity. This condition frequently presents as erythematous papules or plaques with subsequent hair loss. In alopecia mucinosis, mucin stains (like Alcian blue) are likely to stain positive. Answer E is incorrect because histology reveals pilosebaceous unit and peribulbar mucin deposition with a mixed inflammatory infiltrate that may be reacting to local antigens like Staphylococcus aureus. Answer B is incorrect because although granulomatous infiltration is suggestive of leprosy in patients within endemic regions, it may not always be present. Answer C is incorrect because the same can be said for TCR gene rearrangement and malignancy (i.e, lymphoma). Answer D is incorrect because as opposed to older patients, younger patients more frequently experience a benign disease course with spontaneous resolution within 2 years and have a more favorable prognosis.

Comment Here

Reference: Alopecia mucinosa
Board review style question #2
Where is the most common location that alopecia mucinosa presents?

  1. Extremities
  2. Hands and feet
  3. Head and neck
  4. Trunk
  5. Buttocks
Board review style answer #2
C. Head and neck. The most common location for alopecia mucinosa to present is the head and neck region (including the face) of children and young adults. More generalized distributions involving other regions may be seen in patients with chronic disease. While alopecia mucinosa as been historically referred to as a histologic manifestation of folliculotropic mycosis fungoides (MF), it is not specific to MF.

Comment Here

Reference: Alopecia mucinosa

Alpha-1-antitrypsin deficiency panniculitis
Clinical features
  • Patients have low serum alpha-1-antitrypsin
  • Cutaneous lesions are diffuse indurated erythematous plaques or nodules that drain serosangineous fluid, on extensor surfaces of arms, legs, thighs, chest, back and abdomen
  • Also pulmonary effusion, embolism edema and anasarca
Case reports
Clinical images

Images hosted on other servers:

Ulcer on upper extremity

Microscopic (histologic) description
  • Early: diffuse neutrophilic infiltrate through dermis into subcutaneous septa, followed by destruction of collagen, reduction of inflammatory cells, detachment of lobules of fat from fibrous septa in a background of neutrophils
  • Draining lesions have channels lined by squamous epithelium
  • Late: extensive scarring of dermis and subcutis
Positive stains
  • Immunoglobulin IgM and C3 may be found in blood vessel walls
Differential diagnosis
  • Bacterial cellulitis: infectious etiology

Amyloidosis
Clinical features
  • In primary localized cutaneous amyloidosis, by definition, there is no systemic or other dermatologic disorder
  • Either macular (Central / South America, India, Middle East), lichenoid (China), biphasic or nodular
  • Macular: moderately pruritic dark brown papules in reticulated or rippled pattern, symmetric over upper back and arms
  • Lichenoid: intensely pruritic, discrete, firm, hyperkeratotic plaques and papules of anterior shins and extensor forearms, with amyloid deposits in papillary dermis accompanied by hyperkeratosis, papillomatosis and epidermal hyperplasia
  • Biphasic: lichenoid and macular
  • To differentiate between amyloid AA and other types, use potassium permanganate reaction; amyloid AA does not stain with Congo Red after potassium permanganate, but others do (Am J Pathol 1979;97:43)
Case reports
Clinical images

Images hosted on other servers:

Facial involvement

Microscopic (histologic) description
  • Primary amyloidoisis: masses of eosinophilic, amorphous, fissured material in dermis and subcutaneous tissue
  • Macular amyloidosis: focal / small amounts of eosinophilic faceted deposits in papillary dermis; also pigment incontinence
  • Lichenoid amyloidosis: hyperkeratosis, acanthosis, basal hydropic degeneration; small eosinophilic globules in papillary dermis; also mild chronic inflammatory cell infiltrate, pigment incontinence
Microscopic (histologic) images

Images hosted on other servers:

Various images

Congo Red staining without (left) and with (right) polarization microscopy

Congo Red staining (left) and Thioflavin-T staining with fluorescent microscopy (right)

Positive stains
  • Congo Red shows apple-green birefringence under polarized light
  • Methyl or cresyl violet
  • Thioflavin T with examination using fluorescence microscopy
Electron microscopy description
  • Straight non branching amyloid filaments with a diameter of 6 - 10 nm showing a hollow core on cross section
  • Infra red microscopy reveals a beta pleated antiparallel configuration
Electron microscopy images

Images hosted on other servers:

Mesh-like pattern consisting of 6 - 10 nanometer nonbranching fibrils diagnostic of amyloid


Androgenic alopecia
Definition / general
Essential features
Terminology
Epidemiology
Sites
  • Scalp
Pathophysiology
Clinical features
  • Polygenetic condition with varying severity, rate of progression, age of onset and affected scalp sites (BMJ 1998;317:865)
  • Men: hair loss typically involves the temporal and vertex region while sparing the occipital region; the hairline also recedes to form a characteristic “M” shape (BMJ 1998;317:865)
  • Women: diffuse thinning of the frontal / parietal scalp with retention of the frontal hairline (Int J Womens Dermatol 2018;4:203)
  • Has been associated with several other medical conditions:
  • Hair pull test is normal (Int J Womens Health 2013;5:541)
  • Male pattern baldness (Norwood-Hamilton classification) (Endotext: Male Androgenetic Alopecia [Accessed 8 July 2020])
    • Type I: minimal bitemporal recession of hair
    • Type II: extension of type I
    • Type III: hair loss in tonsure area and recession of hair from the forehead
    • Type IV - VI: extension of type III
    • Type VII: most severe pattern of hair loss, with a confluence of the balding areas; hair preserved only around the back and the sides of the head
  • Female pattern baldness (Ludwig classification) (Int J Womens Dermatol 2018;4:203)
    • Type I: mild (hair loss on the front and top of the scalp with relative preservation of the frontal hairline)
    • Type II: moderate
    • Type III: severe
Diagnosis
  • Scalp biopsy
Case reports
Treatment
Clinical images

Images hosted on other servers:

Hair shaft diameter variability

Microscopic (histologic) description
Microscopic (histologic) images

Contributed by Jodi Speiser, M.D.

Vertical section of scalp biopsy

Positive stains
Sample pathology report
  • Skin, left scalp, punch biopsy:
    • Nonscarring alopecia (see comment)
    • Comment: Histological sections show a decrease in hair follicles with a mild periadnexal lymphocytic infiltrate around the upper third of the hair follicle. No peribulbar inflammation is present in the sections examined. There are increased vellus hairs present. Some sections show prominent sebaceous glands. PAS stain shows normal basement membrane thickness and EVG stain shows preservation of elastic fibers. These findings are consistent with androgenic alopecia.
Differential diagnosis
  • Telogen effluvium:
    • Increased telogen hairs without follicular miniaturization
  • Alopecia areata:
    • Uniformly miniaturized follicles with less variation in hair size
    • Lower terminal to vellus and anagen to catagen ratios compared to androgenic alopecia
    • Peribulbar lymphocytic infiltrate
  • Traumatic alopecia (traction alopecia / trichotillomania):
    • Melanin pigment casts and trichomalacia with increased catagen hairs and no follicular miniaturization
  • Cicatricial alopecia:
    • Follicular scars with absent elastic fibers
  • Fibrosing alopecia in a pattern distribution:
    • Lichenoid inflammatory infiltrate in the upper follicle region, peripilar casts and loss of follicular openings
  • Syphilis:
    • Superficial and deep perivascular and perifollicular lymphoplasmacytic infiltrate and sometimes necrotizing pustular folliculitis
Board review style question #1

Which of the following is true about androgenic alopecia?

  1. No association with several other medical conditions
  2. Positive hair pull test
  3. Progressive miniaturization with varying size of the hair follicles
  4. Scarring alopecia
Board review style answer #1
C. Progressive miniaturization with varying size of the hair follicles

Comment Here

Reference: Androgenic alopecia
Board review style question #2
Which of the following features is observed in androgenic alopecia?

  1. Increased number of terminal hair follicles
  2. No fibrous tracts
  3. No hair miniaturization
  4. Reduced terminal to vellus ratio of 4:1 or less
Board review style answer #2
D. Reduced terminal to vellus ratio of 4:1 or less

Comment Here

Reference: Androgenic alopecia

Anetoderma
Definition / general
  • Benign disorder of focal loss of dermal elastic tissue, causing focal flaccid or saclike skin
  • Either primary (idiopathic) arising in normal skin, or secondary arising at site of previous skin lesions
Terminology
  • Greek: Anetos means slack
Epidemiology
  • Acquired purely cutaneous disorder
  • Most common in young adults, onset most often in second decade
  • Affects upper trunk and proximal arms; may be widespread
  • Affect both sexes equally; rare familial variant
Clinical features
  • Erythematous macules and urticarial plaques, progressing to flaccid, easily reduced papules
  • Also fine, diffuse wrinkling
  • May be accompanied by herniation into subcutaneous fat
Case reports
Clinical images

Images hosted on other servers:

Lesion of lower leg

Lesions of shoulders and upper back

Microscopic (histologic) description
  • No abnormality on H&E
  • Elastic stains show no / marked reduction in elastic fibers of papillary and mid-reticular dermis
  • Early lesions: variable perivascular and periadnexal mixed inflammatory infiltrate
Microscopic (histologic) images

Images hosted on other servers:

Faintly eosinophilic separated collagen bundles in upper dermis

Minimal dermal perivascular chronic inflammatory cells infiltrate


Anthrax
Definition / general
  • "Coal" in Greek; refers to black coloration of skin eschar
Epidemiology
  • 2,000 cases per year worldwide - 95% are skin infections
Etiology
  • An uncommon, cutaneous infection due to endospores of Bacillus anthracis, a common soil organism, which typically causes illnesses in animals
  • Exposure is through breathing, eating and exposed cuts in skin
Clinical features
  • Causes cutaneous, pulmonary or gastrointestinal symptoms
  • Cutaneous anthrax is marked by a boil-like lesion that eventually forms an ulcer with a black center (eschar)
  • Culture: nonhemolytic, nonmotile and ground glass colonies that retain their shape when manipulated; grows readily on sheep red blood cell agar (no special conditions needed)
  • Gram stain: gram positive, spore forming rods
Diagnosis
  • Blood, skin, spinal fluid or respiratory secretions are collected for testing
  • Organisms are gram positive
  • PCR and Immunofluorescence useful for confirmation
  • Xray and CT scans recommended to confirm mediastinal widening / pleural effusions in inhalation anthrax
Case reports
Treatment
  • The cutaneous form of anthrax responds well to several antibiotics; with treatment, complete recovery is usual
  • Antibiotics reduce mortality from 20% to less than 1%
Microscopic (histologic) description
  • Eschar shows coagulative necrosis of superficial epidermis and dermis, with prominent edema of underlying viable dermis, frequent focal hemorrhages, intense, reactive appearing mononuclear inflammatory infiltrates around small vessels and some adnexal structures
  • Neutrophils only around necrotizing sebaceous glands
  • Sharp demarcation between superficial, necrotic and deeper edematous viable tissue (at periphery); occasional islands of regenerating epidermis under necrotic layer of eschar
  • Vessels with degenerated endothelial cells and focal thrombi
  • No abscess
  • No granulation tissue
Positive stains

Argyria
Definition / general
  • Permanent discoloration of skin due to dietary, medicinal or industrial exposure to silver compounds
  • Silver deposition in skin and mucous membranes, liver, spleen, adrenal, muscle and brain
  • Silver deposits cause pigmentation of basement membranes of adnexal structures
Clinical features
  • Initially present in gingiva as slate blue line due to metallic silver and silver sulfide
  • Cutaneous manifestations follow a sun-exposed distribution
  • Nails can also be involved
  • Laser treatment may be useful (Int J Dermatol 2011;50:751)
Clinical images

Images hosted on other servers:

Various images

Microscopic (histologic) description
  • Silver granules in vascular and adnexal basement membranes and adjacent to dermal elastic fibers
  • Granules are brown-black
Microscopic (histologic) images

Images hosted on other servers:

Tiny, gray-brown-black granules in dermis and concentrated near eccrine glands

Electron microscopy description
  • Membrane bound granules within macrophages, lysosomes or free within dermis

Arsenic
Definition / general
  • Arsenic is a well water contaminant, used in industrial, mining, agricultural (pesticide) and medicinal (chemotherapy) substances (Toxicol Sci 2011;123:305)
  • Often causes hyperkeratotic lesions of skin called arsenical keratoses
  • Risk factor for Bowen disease, squamous cell carcinoma, basal cell carcinoma and carcinomas of lung, bladder and kidney
  • Skin related problems are rare in U.S.
Clinical features
  • Acute arsenical dermatitis or long term sequelae as a diffuse erythematous papular or pustular bullous dermatosis that can progress to exfoliative dermatitis
  • "Rain drops on a dusty road": hyperpigmented macules with small foci of hypopigmentation and darker hyperpigmentation in trunk, areola and flexural
  • Transverse white nail striations
  • Palmar and plantar keratoses 2+ years after exposure; may transform to Bowen’s disease, squamous cell carcinoma and superficial basal cell carcinoma
Microscopic (histologic) description
Microscopic (histologic) images

Images hosted on other servers:

Left: normal;
right: hyperkeratotic
skin due to arsenic


Arthropod bites
Definition / general
  • Seen in all age groups in various cutaneous sites
  • Caused by fleas, bed bugs, biting flies, mosquitoes, ants, ticks and spiders
  • Most bites are clinically self limited and not biopsied
  • Bites from brown recluse and black widow spiders have the greatest potential morbidity, causing necrosis requiring operative debridement and occasionally amputation
Essential features
  • Variable clinical presentation from erythematous papule to nodular lesion with induration, ulceration and vesicles
  • Often self limited; extensive or persistent cutaneous reactions may be treated with intralesional steroids
  • Typically manifests microscopically with a mixed infiltrate of lymphocytes, histiocytes and eosinophils
  • Exuberant cases with dense deep lymphocyte rich infiltrate may mimic a low grade lymphoma
  • Spider bites often show necrosis and vasculitis
Sites
  • Variable, relating to site of exposure, including:
    • Flea bites: ankles
    • Mosquito bites: exposed skin
    • Chigger bites: socks and belt lines
Clinical features
  • Varied clinical presentation ranging from small clusters of erythematous papules to large deep violaceous nodules with induration, ulceration or vesicles
  • May resemble lymphoma or ulcerated carcinoma
  • A solitary punctum may be found at the site where insect mouth parts contact the skin
  • Spider bites may have necrosis with nonviable tissue present at the ulcer base
Treatment
  • Most bites are self limited
  • Antihistamines may provide symptomatic relief
  • Severe mosquito reactions ("Skeeter syndrome") may require prednisone (Am Fam Physician 2013;88:841)
Clinical images

Images hosted on other servers:

Various images

Microscopic (histologic) description
  • Typically wedge shaped superficial and deep mixed inflammatory infiltrate composed of lymphocytes, histiocytes, eosinophils and sometimes neutrophils
  • Scattered eosinophils in interstitial areas (away from vessels), especially in deep dermis, are a useful clue
  • May also have a prominent granulomatous component
  • Dermal edema commonly seen; vessels may be prominent
  • Variable epidermal features including spongiosis, acanthosis, parakeratosis; these are most prominent in center of lesion where mouth parts penetrate the epidermis
  • Excoriated lesions may have prominent parakeratosis with scale crust formation, epidermal erosion or overt ulceration
  • May have lymphoid germinal centers resembling lymphoma, with destruction of adnexae, particularly sweat glands (J Cutan Pathol 2009;36:26)
  • Mouth parts may be identified in center of lesion, particularly in tick bites
  • May have secondary vasculitis
  • Spider bites: often show marked necrosis with extensive suppurative neutrophilic dermal inflammation, often extending into underlying subcutaneous tissue; small and large vessel vasculitis is also common
  • Fire ant stings:
    • Acute phase: may be characterized by an urticarial reaction
    • Later stages: mixed inflammatory infiltrate consisting of eosinophils and neutrophilic pustules
Microscopic (histologic) images

Images hosted on other servers:

Wedge shaped and perivascular inflammatory infiltrate of lymphocytes and eosinophils

Differential diagnosis
  • Allergic contact dermatitis: may occasionally have increased eosinophils but infiltrate is usually mostly in superficial not deep dermis; often has more broad epidermal changes including spongiosis, parakeratosis and Langerhans cell microabscesses; necrosis and vasculitis are uncommon
  • Eosinophilic folliculitis: perifollicular infiltrate with numerous eosinophils; may be associated with HIV and thus diagnosis should be made with caution; clinical correlation very important
  • Dermal hypersensitivity reaction (e.g. drug eruption): can have very similar appearance to arthropod bite reaction; clinical correlation is best way to separate; presence of epidermal changes or wedge shaped infiltrate may favor arthropod bite
  • Lymphomatoid papulosis: scattered or numerous CD30+ atypical lymphocytes
  • Well's syndrome (eosinophilic cellulitis): can have very similar appearance; superficial and deep infiltrate with eosinophils that may be so numerous that they degranulate, and free granules coat degenerated collagen bundles ("flame figures")
    • Well's syndrome is a diagnosis of exclusion - must rule out arthropod bite reaction (may have flame figures) and drug eruption clinically

Atopic dermatitis
Definition / general
  • Eczema (or atopic dermatitis) is a general term that refers to a group of chronic and relapsing skin conditions that cause inflamed skin
  • Signs and symptoms of atopic dermatitis include rash, pruritus and xerosis
Essential features
  • Essential (required features) (J Am Acad Dermatol 2014;70:338)
    • Pruritis and xerosis
    • Age specific patterns with chronic or relapsing history
  • Microscopic features are variable depending on the chronicity of the lesion (acute, subacute and chronic)
    • Acute lesions show more marked spongiosis
    • Subacute lesions show irregular acanthosis along with spongiosis
    • Chronic lesions show hyperkeratosis, moderate to marked psoriasiform hyperplasia and variably mild spongiosis (Patterson: Weedon's Skin Pathology, 5th Edition, 2020)
Terminology
  • Eczema: atopic dermatitis
  • Dyshidrotic eczema: pompholyx, dyshidrosis
ICD coding
  • ICD-10
    • L20.9 - atopic dermatitis, unspecified
    • L30.1 - dyshidrosis (pompholyx)
Epidemiology
Sites
  • Typical sites according to age group
    • Infants (birth to 6 months): face, scalp, trunk and extensor aspects of extremities; sparing of diaper area
    • Children (6 months to 12 years): flexural areas, including antecubital fossa, popliteal fossa, wrists, ankles and the nape of neck; dorsum of feet, hands and periorbital folds
    • Adolescents and adults (12 years and beyond): flexural areas, hands, upper trunk and forehead
    • Adults: head and neck, hands and feet (dyshidrotic eczema)
  • References: J Investig Allergol Clin Immunol 2017;27:78, Pediatr Dermatol 2022;39:345, JAAD Int 2022:11:1
Pathophysiology
  • Complex interactions of genetic predispositions, environmental triggers and immune dysregulation resulting in subsequent compromised skin barrier
  • Mutation in filaggrin gene, a vital gene for skin cell maturity; causes a haphazard organization of the skin cells resulting in a disrupted epidermal barrier, which leads to increased water loss and decreased protection from harmful substances
  • Reference: Br J Dermatol 2019;180:464
Etiology
Clinical features
  • Universal features include pruritis and xerosis
  • Clinical presentation is variable according to the phase of disease, age of patient and changes during the disease course
  • Atopic dermatitis may be categorized depending on the phase of the disease
    • Acute: ill defined, oozing, crusted, eroded papules or vesicles on erythematous plaques
      • Lesions on dark skinned individuals may appear as hypopigmentation or have follicular accentuation
    • Subacute: thickened, scaly or excoriated plaques
    • Chronic: thickened, hyperpigmented plaques, fibrotic papules or hypopigmentation resulting from extensive irritation and rubbing; lichenified skin from scratching results in accentuated skin fold lines
  • Reference: J Am Acad Dermatol 2014;70:338
Diagnosis
  • Diagnosis can be established clinically from history and physical examination alone with adequate childhood and family history of allergies, asthma and skin disease (J Am Acad Dermatol 2014;70:338)
    • Pruritis and xerosis with typical inflammatory skin lesions and excoriation in an age specific distribution
    • Diagnosis is often straightforward in infants, children and adults with a childhood onset
  • Adults with new onset dermatitis without childhood history of eczema must exclude other conditions with inflamed scaly rashes (J Investig Allergol Clin Immunol 2017;27:78)
    • Skin biopsy may be required in adults with a de novo onset of pruritis rash
    • Skin biopsy may also be performed in adult patients presenting with recalcitrant, eczematous dermatitis to rule out secondary causes
    • Skin biopsy will not help in differentiating atopic dermatitis from allergic contact dermatitis or id reaction
Laboratory
  • Elevated serum immunoglobulin E (IgE) level is seen in 80% of patients (J Am Acad Dermatol 2014;70:338)
    • Serum IgE is not routinely obtained; can be helpful for uncertain diagnosis
  • Skin patch testing is gold standard for identifying allergens (J Investig Allergol Clin Immunol 2017;27:78)
    • Mainly helpful in adults with de novo atopic dermatitis with a history of possible allergen exposure to determine if the patient has allergic contact dermatitis rather than eczema
    • Helpful in cases of refractory chronic atopic dermatitis unresponsive to treatment, rash with atypical pattern according to age and rash with a changing pattern
Prognostic factors
Case reports
Treatment
  • Restore and maintain epidermal barrier: emollient, oil baths, topical moisturizers
  • Identification and avoidance of potential allergens and triggers
  • Anti-inflammatory treatment: topical steroids (to include wet wraps), calcineurin inhibitors, oral steroids, oral antihistamines and cyclosporine (rapid onset); more recent targeted and effective therapies include subcutaneous IL4R inhibitors, subcutaneous IL13R inhibitors and oral JAK inhibitors
  • Ultraviolet (UV) therapy has been proven to be helpful in patients
  • Treatment of secondary skin infections
  • Reference: Postepy Dermatol Alergol 2020;37:1
Clinical images

Contributed by Chelsea Huang, M.D. and Ashley N. Elsensohn, M.D., M.P.H.
Pruritic papules with hypopigmentation

Pruritic papules with hypopigmentation

Acute onset pruritic papules

Acute onset pruritic papules

Acute pruritic rash

Acute pruritic rash

Relapsing dry itchy rash

Relapsing dry itchy rash

Microscopic (histologic) description
  • Atopic dermatitis falls under the spongiotic dermatosis pattern histologically; spongiotic dermatitis is a microscopic terminology while atopic dermatitis (eczema) is a clinical diagnosis
  • Microscopically, atopic dermatitis is broken down into 3 stages
    • Acute eczema
      • Retained basket weave orthokeratosis
      • Spongiosis featuring intercellular edema with widening of the intercellular spaces showing accentuation of intercellular bridges of the squamous epithelium
      • Intradermal microvesicles or less commonly, subepidermal microvesicles
      • Vesicles filled with proteinaceous fluid containing lymphocytes, histiocytes and occasional eosinophils
      • Variable dermal edema with highly variable mixed inflammatory cell infiltrates composed of lymphocytes, histiocytes and occasionally eosinophils or neutrophils
    • Subacute eczema (most frequently encountered type of eczema for pathologists)
      • Irregular acanthosis and parakeratosis
      • Mild to moderate spongiosis with focal spongiotic vesiculation
      • Exocytosis of inflammatory cells
      • Associated superficial dermal perivascular lymphohistiocytic inflammatory infiltrate
      • Swelling of endothelial cells
      • Papillary dermal edema
    • Chronic eczema
      • Significant epidermal acanthosis
      • May show a psoriasiform pattern with hyperkeratosis, hypergranulosis and minimal parakeratosis
      • Fibrosis of the papillary dermis is another feature of chronicity
      • Mild to no spongiosis
  • References: Adv Anat Pathol 2008;15:76, Clin Lab Med 2017;37:673
Microscopic (histologic) images

Contributed by Chelsea Huang, M.D. and Ashley N. Elsensohn, M.D., M.P.H.
Acute/subacute spongiotic dermatitis

Acute / subacute spongiotic dermatitis

Spongiosis and hyperkeratosis

Spongiosis and hyperkeratosis

Chronic psoriasiform/spongiotic dermatitis Chronic psoriasiform/spongiotic dermatitis

Chronic psoriasiform / spongiotic dermatitis

Spongiotic dermatitis with excoriation

Spongiotic dermatitis with excoriation

Spongiosis with intraepithelial microvesicle

Spongiosis with intraepithelial microvesicle

Virtual slides

Images hosted on other servers:
Acute spongiotic dermatitis

Acute spongiotic dermatitis

Subacute spongiotic dermatitis

Subacute spongiotic dermatitis

Immunofluorescence description
  • Direct and indirect immunofluorescence are negative
Negative stains
Videos

Eczema / atopic contact dermatitis, etc.: spongiotic dermatitis pattern

Spongiotic dermatitis

Sample pathology report
  • Skin, left upper back, punch biopsy:
    • Subacute spongiotic dermatitis with secondary excoriation (see comment)
    • PAS negative for fungal elements (controls adequate)
    • Comment: The patient's clinical history and photos were reviewed and multiple levels were examined. Sections show mild to moderate epidermal acanthosis with spongiosis and serum scale crust. There is a superficial perivascular lymphohistiocytic infiltrate with scattered neutrophils and rare eosinophils. Moderate papillary dermal edema is also noted. The histologic differential diagnosis includes atopic dermatitis, allergic / irritant contact dermatitis and id reaction. Clinical correlation is recommended.
Differential diagnosis
  • Contact dermatitis (irritant or contact):
    • Variable spongiosis
    • Langerhans cell microabscesses and eosinophils in the epidermis (eosinophilic spongiosis) may be clues to favor allergic contact dermatitis over other spongiotic processes
    • Perivascular infiltrate and exocytosis of neutrophils and lymphocytes
    • Vasodilatation and edema in dermis
  • Eczematous drug eruption:
    • Variable spongiosis
    • Eosinophils in dermis
    • Marked vascular wall thickening
    • Vacuolar interface changes
    • Often lymphocytic exocytosis, dyskeratotic keratinocytes, parakeratosis
  • Id reaction:
    • Variable spongiosis
    • Superficial perivascular infiltrate with exocytosis of neutrophils and lymphocytes
  • Tinea:
    • Variable spongiosis
    • PAS or GMS special stains positive for fungal organisms
  • Scabies:
    • Variable spongiosis
    • Eosinophilic spongiosis
    • Stratum corneum with mite, scybala or eggs
  • Reference: Am J Clin Dermatol 2021;22:349
Board review style question #1

A 30 year old woman presented with a generalized skin eruption for 4 days with associated pruritis. What is the best histologic diagnosis from the punch biopsy taken?

  1. Atopic dermatitis
  2. Dermatitis herpetiformis
  3. Early guttate psoriasis
  4. Erythema multiforme
Board review style answer #1
A. Atopic dermatitis. Atopic dermatitis demonstrates variable spongiosis. Acute lesions show retained basket weave orthokeratosis with more marked spongiosis. As lesions become more chronic, they begin to show less spongiosis and more acanthosis. Answer C is incorrect because although guttate psoriasis may show some spongiosis, it is characterized by a mound of parakeratosis with associated neutrophilic abscess (so called open faced jelly sandwich). Guttate psoriasis also shows mild acanthosis, papillary dermal edema, capillary dilatation and a sparse lymphocyte predominant dermal infiltrate. Answer D is incorrect because due to the acute nature of erythema multiforme (EM), it can retain basket weave orthokeratosis in the stratum corneum; however, EM would show other features such as dyskeratotic keratinocytes in all layers of the epidermis with confluent epidermal necrosis and associated basal layer vacuolar interface pattern, rather than a spongiotic reaction pattern. The dermis can show sparse lymphocytic infiltrate. Answer B is incorrect because dermatitis herpetiformis may feature subepidermal blistering with dense neutrophilic infiltrate and associated eosinophils. In early lesions, the presence of dermal papillary neutrophilic microabscesses is a characteristic feature.

Comment Here

Reference: Atopic dermatitis
Board review style question #2
Which of the following choices is in the histologic differential diagnosis for spongiotic dermatitis?

  1. Allergic contact dermatitis, id reaction
  2. Darier disease, Grover disease
  3. Psoriasis, pityriasis rubra pilaris
  4. Subacute lupus erythematosus, erythema multiforme
Board review style answer #2
A. Allergic contact dermatitis, id reaction. Spongiotic dermatitis can be seen in irritant dermatitis, allergic contact dermatitis, eczematous dermatitis, id reaction and arthropod reaction. Answer D is incorrect because subacute lupus erythematosus and erythema multiforme show a vacuolar interface pattern. Answer C is incorrect because psoriasis and pityriasis rubra pilaris show a psoriasiform pattern. Answer B is incorrect because Darier disease and Grover disease show a pattern of focal acantholytic dyskeratosis.

Comment Here

Reference: Atopic dermatitis

Atrophic papulosis
Definition / general
  • Rare vasculopathy with benign skin limited form and frequently lethal systemic variant with intestinal involvement
Essential features
  • All ages, mostly in young adults
  • Typically affects proximal extremities and trunk
  • Papules with porcelain white centers and erythematous peripheral rims
  • Wedge shaped dermal necrosis with mucin deposition
  • Vessels with endothelial swelling, luminal obliteration, thrombosis
Terminology
  • Köhlmeier-Degos disease
  • Degos disease
  • Malignant atrophic papulosis
  • Lethal intestinocutaneous syndrome
ICD coding
  • ICD-10: L90.9 - atrophic disorders of skin
Epidemiology
Sites
  • More commonly affects proximal extremities and trunk (Clin Exp Gastroenterol 2015;141:147)
  • Less commonly affects genital, buccal and conjunctival sites
  • GI tract involvement, specifically small intestine
  • CNS, hepatorenal and cardiopulmonary system involvement
Pathophysiology
  • High interferon alpha expression and accumulation of late stage complements (C5b9) in diseased vessels (Pediatrics 2018;S481:S484)
Clinical features
  • Early disease: up to 5 mm in diameter, skin colored papules on proximal extremities and trunk
  • Established disease: atrophic papules with porcelain white centers and erythematous peripheral rims, surrounded by telangiectasia
  • Late disease: atrophic scars
  • May be associated with factor V Leiden or parvovirus B19 infection or autoimmune connective tissue diseases, including systemic lupus erythematosus, antiphospholipid syndrome, dermatomyositis or systemic sclerosis (JAMA Dermatol 2014;150:96)
Diagnosis
  • Biopsy from skin, intestines or other involved organs
  • Complete blood count
  • Stool guaiac test
  • Antinuclear antibody titer
  • Factor V Leiden, protein C and S, antithrombin III and homocysteine levels
  • Anticardiolipin antibody titer
  • Antiphospholipid antibody titer
  • Endoscopy
  • Laparotomy (Orphanet J Rare Dis 2013;8:10)
Laboratory
  • In association with systemic lupus erythematosus: antinuclear antibody titer, anti-dsDNA, Coombs test, complement protein C3, C4, CH50 or immune complex assays (C1q or C3d)
  • May be differentiated from other associated autoimmune disorders by antiphospholipid and anticardiolipin antibodies, rheumatoid factor, anti-Mi2, antitopoisomerase I and anti-Saccharomyces cerevisiae antibodies (Ann Dermatol 2017;29:215)
Prognostic factors
  • Benign atrophic papulosis
    • Normal life expectancy
    • No systemic involvement
    • Represents 70% of cases of atrophic papulosis
    • Retrospective diagnosis that requires longterm followup
  • Malignant atrophic papulosis
    • Onset of systemic disease approximately 1 year after cutaneous findings
    • Average 2 year survival in untreated patients with systemic disease
    • Peritonitis secondary to intestinal perforation is the leading cause of death (Orphanet J Rare Dis 2013;8:10)
    • Cerebral infarction is a less common cause of death
  • Patients with underlying autoimmune connective tissue diseases have a better prognosis
Case reports
Treatment
  • Monotherapy or combination therapy
Clinical images

Contributed by Kiran Motaparthi, M.D.
Malignant atrophic papulosis

Malignant atrophic papulosis

Microscopic (histologic) description
  • Early disease:
    • Superficial and deep perivascular and periadnexal lymphocytic infiltrate
  • Established disease:
    • Features of lichen sclerosus: epidermal atrophy, hyperkeratosis and edema with homogenization of the superficial dermis
    • Lymphocytic vasculitis
  • Late disease (classic features):
    • Thrombotic vasculopathy
    • Wedge shaped dermal necrosis with mucin deposition (Cutis 1985;35:131)
Microscopic (histologic) images

Contributed by Kiran Motaparthi, M.D.
Atrophic papulosis in the context of systemic lupus erythematosus Atrophic papulosis in the context of systemic lupus erythematosus Atrophic papulosis in the context of systemic lupus erythematosus Atrophic papulosis in the context of systemic lupus erythematosus

Atrophic papulosis in the context of systemic lupus erythematosus


Benign atrophic papulosis Benign atrophic papulosis Benign atrophic papulosis

Benign atrophic papulosis

Immunofluorescence description
Positive stains
Electron microscopy description
Sample pathology report
  • Right upper arm, biopsy:
    • Atrophic papulosis (Degos disease) (see comment)
    • Comment: There is a wedge shaped scar, along with a perivascular lymphocytic infiltrate. Dermal vessels have hyaline walls and contain fibrin thrombi. There is also overlying epidermal atrophy and hyperkeratosis.
Differential diagnosis
Additional references
Board review style question #1

A 35 year old woman presents with white, atrophic papules on her proximal extremities and trunk, malar rash, Raynaud phenomenon, malaise, arthritis and anemia. Laboratory evaluation is significant for thrombocytopenia, leukopenia and high titer ANA and anti-dsDNA. Skin biopsy was performed, shown above. Which of the following statements is correct?

  1. A poor prognosis is anticipated, with significant neurologic or gastrointestinal involvement
  2. Late histopathologic findings include dermal necrosis and scar
  3. The cutaneous findings are most likely idiopathic
  4. The cutaneous findings represent extragenital lichen sclerosus
  5. The cutaneous findings represent lupus erythematosus
Board review style answer #1
B. Late histopathologic findings include dermal necrosis and scar. The diagnosis is malignant atrophic papulosis.

Comment Here

Reference: Atrophic papulosis
Board review style question #2
A 30 year old man presents with porcelain white papules on his left arm and abdominal discomfort. Skin biopsy showed vacuolar interface dermatitis, hyperkeratosis, perivascular lymphocytic infiltrates, thrombi, mucin and wedge shaped necrosis. Which of the following is the best diagnosis?

  1. Dermatomyositis
  2. Leukocytoclastic vasculitis
  3. Livedoid vasculopathy
  4. Malignant atrophic papulosis
  5. Systemic lupus erythematosus
Board review style answer #2
D. Malignant atrophic papulosis

Comment Here

Reference: Atrophic papulosis

Atrophoderma (pending)

Bacillary angiomatosis
Definition / general
  • Reactive vascular proliferation due to infection by Bartonella henselae or Bartonella quintana
  • Most commonly affects immunocompromised individuals (especially those with HIV)
Essential features
  • Lobular proliferation of capillaries with ectatic vessels lined by prominent endothelial cells
  • Neutrophils, lymphocytes and histiocytes are frequently present
  • Purplish grey bacterial colonies may be seen
Epidemiology
  • May affect adults and children
  • Most commonly affects immunocompromised individuals
Sites
  • Can occur in any cutaneous site
  • Rarely occurs in mucosa or internal organs
Etiology
  • B. Henselae is acquired via cat scratch or bite in 2/3 of cases
Clinical features
  • Red, smooth papules and nodules which are widely distributed
  • May mimic Kaposi sarcoma and pyogenic granuloma clinically
Laboratory
  • PCR and IHC may be helpful in identifying organisms
Case reports
Treatment
  • Antibiotics - typically doxycycline
Clinical images

Images hosted on other servers:

Papules on face, chest, abdomen and back; mass lesion on face

Exophytic lesion on neck

Small, cherry angiomata-like lesions and nodules

Microscopic (histologic) description
  • Lobular proliferation of capillaries with ectatic vessels lined by prominent endothelial cells in edematous stroma
  • Neutrophils, lymphocytes and histiocytes are frequently present
  • Purplish grey bacterial colonies may be seen, especially near neutrophils
  • Peripheral collarette may be seen (low power histologic image mimics pyogenic granuloma)
  • Warthin-Starry special stain will highlight Bartonella organisms
Microscopic (histologic) images

Contributed by Tammie Ferringer, M.D.

Bacillary angiomatosis profile



Contributed by Nathan Lee, M.D.

Bacillary angiomatosis



Contributed by Mark R. Wick, M.D.

Breast skin

Steiner stain, breast

Positive stains
Differential diagnosis
  • Kaposi sarcoma: spindled vascular proliferation with slit-like vascular spaces, plasma cells, HHV8 positive, lacks purplish grey bacterial colonies
  • Pyogenic granuloma: lobular proliferation of capillaries, but typically somewhat less inflamed (unless ulcerated) and no purplish grey bacterial colonies

Behçet disease
Definition / general
  • Autoinflammatory multisystemic disease
  • Involvement includes mucocutaneous, ocular, cardiovascular, renal, gastrointestinal, pulmonary, urologic, central nervous system, joints, blood vessels and lungs
Essential features
  • Recurrent oral and genital ulcers in 90% of cases
  • Variable vessel vasculitis and thrombosis
  • Affects mainly Mediterranean basin and far east Asia
  • Variable cutaneous manifestations and pathology
Terminology
  • Behçet disease, Behçet syndrome, Adamantiades-Behçet syndrome
ICD coding
  • ICD-9: 136.1 - Behçet syndrome
  • ICD-10: M35.2 - Behçet disease
Epidemiology
  • Distinct geographic distribution pattern along the ancient Silk Road
  • Prevalence of 0.64 - 660/100,000 (Clin Rheumatol 2020;39:3449)
  • M:F = 0.36 - 4.9
  • Lowest male ratios reported in Western Europe; significant male predominance in Arab populations (Clin Rheumatol 2020;39:3449)
  • Erythema nodosum, arthritis / arthralgias and central nervous system symptoms more frequent in females
  • Males show a lower mean age at onset, a higher frequency of pseudofollicular lesions and panuveitis (Clin Rheumatol 2020;39:3449)
Sites
  • Mucocutaneous, ocular, cardiovascular, renal, gastrointestinal, pulmonary, urologic, central nervous system, joint, blood vessel and lung involvement
Clinical features
  • Oral ulcerations (95% of cases) on lips, gingival, buccal mucosa and tongue, leaves no scars
  • Genital ulcers (90% of cases) on scrotum, major and minor labia, forms scars when healed
  • Acne-like papulopustular lesions on face, upper trunk and extremities
  • Erythema nodosum-like lesions, healing with hyperpigmentation
  • Recurrent asymmetric mono or oligoarthritis, arthralgia, usually involving lower extremities, resolves with no deformity or erosion
  • Ocular involvement (50% of cases), mainly posterior or panuveitis and retinal vasculitis
  • Superficial and deep venous thrombosis
  • Gastrointestinal involvement ranging from mild abdominal pain to emergency complication such as hemorrhage or perforation
  • Neurologic involvement (5% of cases) such as severe headache, cranial nerve palsy, ataxia and hemiparesis
  • Variable vessel vasculitis with contemporary involvement of arteries and veins and tendency for aneurysm formation
  • Reference: Rheumatology (Oxford) 2020;59:iii101
Diagnosis
  • International criteria for Behçet disease - point score system: scoring ≥ 4 indicates Behçet diagnosis (J Eur Acad Dermatol Venereol 2014;28:338)
    • Ocular lesions: 2 points
    • Genital aphthosis: 2 points
    • Oral aphthosis: 2 points
    • Skin lesions: 1 point
    • Neurological manifestations: 1 point
    • Vascular manifestations: 1 point
    • Positive pathergy test: 1 point
Prognostic factors
Case reports
Treatment
  • Treatment is tailored to each patient's clinical manifestations
  • Corticosteroids are useful in controlling acute manifestations
  • Cytotoxic medications are indicated in patients with ocular, central nervous system and vascular disease
  • European League Against Rheumatism (EULAR) has recently released guidelines for the management of Behçet disease (Ann Rheum Dis 2008;67:1656)
  • Otezla® (apremilast) is the only FDA approved treatment for Behçet
  • Colchicine is also commonly used
  • Reference: (Clin Exp Rheumatol 2018;36:13)
Clinical images

Images hosted on other servers:
Erythematous papules and pustules resembling acne

Erythematous papules and pustules resembling acne

Erythematous plantar maculae

Erythematous plantar maculae

Microscopic (histologic) description
  • Oral and genital ulcers: not specific; mixed dermal inflammatory infiltrate perivascular and at the base of the ulcer
  • Erythema nodosum-like lesions: septal panniculitis; vasculitis (lymphocytic or neutrophilic) and necrobiosis may be found
  • Pathergy reaction: perivascular infiltrate of mononuclear cells; mast cell infiltrate and neutrophilic vasculitis may be found
  • Papulopustular lesions: spongiosis, basal keratinocyte vacuolization, intraepidermal pustules, suppurative folliculitis
  • Thrombophlebitis: thrombi in vessel lumens, perivascular inflammatory infiltrate
  • Leukocytoclastic vasculitis
  • Reference: Patholog Res Int 2012;2012:209316
Microscopic (histologic) images

Contributed by Amira Elbendary, M.B.B.Ch., M.Sc.
Acne-like lesion Acne-like lesion

Acne-like lesion

Vasculitis Vasculitis

Vasculitis

Erythema nodosum-like lesion Erythema nodosum-like lesion

Erythema nodosum-like lesion

Sample pathology report
  • Right leg, erythematous subcutaneous nodule, incisional biopsy:
    • Erythema nodosum-like lesion (given the history, compatible with Behçet disease) (see comment)
    • Comment: Septal panniculitis with unremarkable epidermis. Dermis shows moderate perivascular lymphocytic infiltrate. The subcutaneous fat demonstrates widening of the fibrous septa with edema and infiltration of lymphocytes and histiocytes. Spillover of infiltrate to the adjacent fat lobules is present. Small focus of lymphocytic vasculitis is noted.
Differential diagnosis
  • Localized chronic fibrosing vasculitis:
    • Histologically resembles late stage erythema elevatum diutinum (EED) but has a different clinical presentation
    • Erythemaotous nodules on lower limbs
    • Dense concentric and lamellar fibrosis, foci of neutrophilic dust, sparse infiltrate of histiocytes, neutrophils, eosinophils and lymphocytes
    • No granulomas present; no involvement of subcutaneous fat
    • Tissue cultures for micro-organisms are negative
    • Evaluations for systemic vasculitides and connective tissue disorders are negative
  • Chronic granulomatous disease (CGD) (Clin Immunol 2020;214:108381):
    • A primary immunodeficiency
    • Genetic mutations in any of 4 genes within the nicotinamide adenine dinucleotide phosphate oxidase complex
    • Impaired oxidative burst after phagocytosis
    • Clinical features: abscess formation and granulomatous lesions in skin and mucous membranes, lungs, liver and lymph nodes
    • Recurrent oral ulceration without genital or skin lesions has been reported
    • Positive nitroblue tetrazolium (NBT) test
  • Periodic fever, aphthous stomatitis, pharyngitis and cervical adenitis (PFAPA) (Clin Immunol 2020;214:108381):
    • A noninherited periodic fever syndrome
    • Abnormal IL1β signaling
    • Intermittent episodes of high fever associated with at least 1 of the features mentioned in its name
    • Fever lasts 3 - 7 days with periods of health in between attacks
    • None will fulfill the criteria for Behçet disease
  • Inflammatory bowel disease:
    • Marked overlap between the clinical features of inflammatory bowel disease and Behçet syndrome, including oral ulceration, similar skin manifestations, arthritis and uveitis
    • Lower lip thickening, tags and cobblestoning within the oral mucosa would suggest Crohn’s disease rather than Behçet disease
    • Perianal abscesses, fistula, strictures occur more commonly in Crohn’s disease
    • Presence of noncaseating granulomas and absence of vasculitis in small veins and venules
Board review style question #1
A 25 year old male with a history of recurrent orogenital ulcers presented with erythematous nodules that ulcerate and blurred vision and pain in the right eye. What is the best next step?

  1. ANA, pANCA, cANCA
  2. Biopsy from an oral ulcer
  3. Perform skin biopsy from the edge of the ulcer
  4. Upper and lower endoscopy
  5. Urgent ophthalmology consultation
Board review style answer #1
E. Urgent ophthalmology consultation. The patient has Behçet disease.

Comment Here

Reference: Behçet disease
Board review style question #2

A 19 year old male patient presented with recalcitrant papulopustular lesions on the face, shoulders, back and occasionally on lower limbs. He started to suffer from recurrent orogenital ulcers. What pathology finding in a skin biopsy from a pustule on the back would suggest further evaluation for Behçet disease?

  1. Follicular plug
  2. Folliculitis
  3. Leukocytoclastic vasculitis
  4. Perifolliculitis
  5. Superficial perivascular lymphocytic infiltrate
Board review style answer #2
C. Leukocytoclastic vasculitis

Comment Here

Reference: Behçet disease

Benign lichenoid keratosis (pending)
[Pending]

Body & pubic louse
Definition / general
  • Defined as infestation caused by body louse (Pediculus humanus corporis)
  • Skin lesions due to direct bite, hypersensitivity reaction and itching related excoriation
  • Louse is also vector for other diseases such as epidemic typhus, trench fever and relapsing fever
  • Pubic louse: often a sexually transmitted disease but can also be transmitted by sharing of towels or bedding (Wikipedia: Crab Louse [Accessed 28 August 2020])
Terminology
  • Pediculosis, Pediculus humanus corporis
Epidemiology
  • Found worldwide and affects people of all races and social classes (CDC)
  • Highest in endemic areas, young, elderly, immunocompromised, nursing home residents and resource poor, overcrowded populations (Rev Chil Pediatr 2014;85:312)
Sites
  • Lice live and lay eggs in clothing and bedding
  • Usually do not live on human body, but are found only when they feed
Etiology
  • Body lice prefer lower temperature and live and lay eggs in clothing and bedding
  • Lice are bloodsucking insects, feeding ~ 5 times a day by biting host
  • Small amount of louse saliva is injected into host during feeding, causing mild hypersensitivity reaction and associated pruritus
  • Lice are vector for epidemic typhus (Rickettsia prowazeki), trench fever (Bartonella quintana) and relapsing fever (Borrelia recurrentis) (Clin Microbiol Infect 2012;18:332, Clin Lab Med 2015;35:847)
Clinical features
  • Intense itching and irritation of the body
  • Extensive excoriation, serous crusting and scaling on the body
  • Identification: Body lice are about the size of sesame seeds and are found on clothing and bedding, while nits are found in seams of clothing or rarely attached to body hair
  • Can be associated with secondary infections such as epidemic typhus, trench fever and relapsing fever (Infect Genet Evol 2014;23:209)
Diagnosis
  • Thorough physical examination
  • Dermatoscopy is a non invasive technique important for the diagnosis of skin infestation diseases (Clin Dermatol 2014;32:315)
  • Histology examination is useful to find the diagnostic organism and to establish dermal hypersensitivity reactions associated with insect bites
Case reports
Treatment
  • Improving personal hygiene
  • Systemic medication: oral ivermectin (Dermatol Online J 2016:16;22)
  • Topical treatment: Lindane shampoo, benzyl alcohol 5% and permethrin
  • Prevention: Improving personal hygiene, regular washing of clothes and avoiding direct physical contact with those who carry body lice
Clinical images

Images hosted on other servers:

Body louse

Scalp infection and scraping lesions

Pubic louse

Human infestations of pubic louse

Microscopic (histologic) description
  • Skin lesion: perivascular lymphohistiocytic infiltrate mixed with variable numbers of eosinophils, arranged in a wedge shape that tapers towards deep dermis, rarely intradermal hemorrhage
  • Body lice: 3 - 4 mm long, 3 pairs of legs, elongated body and thin head with narrow mouth parts
  • Nits (eggs): small white structures, 1 mm long, teardrop shaped, translucent with apical operculum
Differential diagnosis

Botryomycosis
Definition / general
Terminology
  • The disease was originally discovered by Otto Bollinger in 1870, and its name was coined by Sebastiano Rivolta in 1884
  • The name refers to its grape-like granules (Greek: botryo = grapes) and the mistakenly implied fungal etiology (Greek: mykes = fungus)
  • In 1919 the bacterial origin of the infection was discovered
Etiology
  • Staphylococcus aureus is usually the causative organism, however, it can also be caused by Pseudomona aeroginosa or several other species of bacteria
  • The anatomic structure resembles actinomycosis and mycetoma, and its granules resemble the sulfur granules of actinomycosis
  • It is usually described in individuals with impaired immunity or with an underlying disease such as diabetes mellitus, cystic fibrosis or HIV infection
Case reports
Treatment
Microscopic (histologic) description
  • The lesion is similar to that of actinomycosis and mycetoma, and its granules resemble the sulfur granules of actinomycosis
Microscopic (histologic) images

Contributed by Angel Fernandez-Flores, M.D., Ph.D.


Brown recluse spider
Definition / general
Essential features
  • Bite and venom can cause extensive necrosis, hemorrhage, blistering and ulceration
  • Misdiagnosis of methicillin resistant Staphylococcus aureus (MRSA) as spider bites has been reported and may have fatal consequences
  • No standard treatment to date
  • Usually associated with eosinophils but other histologic features vary: neutrophilic vasculitis with hemorrhage, arterial wall necrosis or eschar covered ulceration and subcutaneous necrosis
Terminology
  • Also called loxoscelism (Loxosceles reclusa), necrotizing arachnidism
Epidemiology
  • Mainly populates the southern central states and some South American countries
  • In 2011, 1487 Loxosceles bites were reported in the US, mainly in Southeastern states (Clin Toxicol (Phila) 2012;50:911)
Sites
  • Any body site can be affected, especially exposed skin
  • Cutaneous manifestations occur in around 80% cases around the site of bite, predominantly in the lower limbs (Rev Inst Med Trop Sao Paulo 1989;31:403)
Etiology
  • Bite induced injury
  • Hemotoxic and proinflammatory venom
Clinical features
  • Initial symptoms include burning stinging, pain, erythema and edema
  • Local envenomation associated skin lesions: extensive necrosis, hemorrhage, blistering and ulceration (Semin Cutan Med Surg 2014;33:123)
  • May take 2 months to heal (N Engl J Med 2013;369:e6)
  • Secondary bacterial infections and osteomyelitis have also been reported (Eplasty 2008;8:e45)
  • A chronic pyoderma gangrenosum-like reaction has also been reported (South Med J 1990;83:243)
  • Systemic loxoscelism is a rare complication after envenomation (J Clin Immunol 2014;34:544)
    • Symptoms include severe hemolysis, acute kidney injury, secondary hemophagocytic lymphohistiocytosis
Diagnosis
  • Misdiagnosis may have fatal consequences, especially misdiagnosis of methicillin resistant Staphylococcus aureus (MRSA) as spider bites (N Engl J Med 2006;355:666)
    • NOT RECLUSE: a mnemonic to avoid misdiagnoses (JAMA Dermatol 2017;153:377)
    • Diagnosis can be made by swabbing the lesion and using a specific enzyme linked immunosorbent assay (ELISA) to detect the venom
Case reports
  • 6 year old boy with massive hemolysis, shock and acute renal failure after loxosceles envenomation (Pediatrics 2014;134:e1464)
  • 10 year old boy developed necrotic rash, secondary hemophagocytic lymphohistiocytosis from a brown recluse spider bite (J Clin Immunol 2014;34:544)
  • 22 year old woman with severe haemolytic anaemia with erythrophagocytosis following the bite of a brown recluse spider (Br J Haematol 2014;167:1)
  • 23 year old man developed dermonecrosis and acute kidney injury following the bite of brown recluse spider (Indian J Nephrol 2014;24:246)
  • 60 year old woman with a necrotic lesion of the left breast (JAAPA 2014;27:32)
  • 63 year old woman with bilaterally decreased vision after a bite from a brown recluse spider (Cutan Ocul Toxicol 2016;35:168)
Treatment
  • No standard treatment to date
  • First aid treatment: wash the bite area with soap and water; apply a cold pad to reduce inflammation; seek professional medical treatment (CDC website - Venomous Spiders)
  • Topical treatment: pain control, anti inflammatory medications; topical antibiotics for secondary bacterial infections; hyperbaric oxygen therapy for nonhealing wounds (Adv Skin Wound Care 2016;29:560)
  • Surgical interventions including debridement of the envenomation site and possibly skin grafting (J Foot Ankle Surg 2014;53:320)
  • Therapeutic plasma exchange for severe systemic loxoscelism associated hemolysis and acute kidney failure (J Med Toxicol 2015;11:364)
Clinical images

Images hosted on other servers:
Missing Image

Brown recluse

Missing Image

Dorsal view of the brown recluse spider

Missing Image

Necrotic rash from
a brown recluse
spider bite

Missing Image Missing Image

Brown recluse spider bites

Microscopic (histologic) description
  • Varies from neutrophilic vasculitis with hemorrhage to arterial wall necrosis to eschar covered ulceration and subcutaneous necrosis
  • Usually eosinophils in the inflammatory infiltrate
Differential diagnosis
Board review style question #1
    Which description of brown recluse spider bites is FALSE:

  1. Cutaneous manifestations occur in around 80% cases around the site of bite, predominantly in the lower limbs
  2. The gold standard for treatment is antibiotics and debridement
  3. Microscopic features vary from neutrophilic vasculitis with hemorrhage to arterial wall necrosis to eschar covered ulceration and subcutaneous necrosis
  4. Misdiagnosis of methicillin resistant Staphylococcus aureus (MRSA) as spider bites may have fatal consequences
  5. Loxosceles reclusa, the brown recluse spider, is one of two medically significant venomous spiders in the North America.
Board review style answer #1
B. There is no standard treatment for a bite from a brown recluse spider.

Comment Here

Reference: Brown recluse spider

Bullous lupus erythematosus (pending)
[Pending]

Bullous pemphigoid
Definition / general
  • Most common autoimmune blistering skin disorder
  • Characterized by autoantibodies against hemidesmosomal antigens, bullous pemphigoid antigens 1 and 2
  • Patients present with tense bullae on an erythematous base and pruritus
Essential features
  • Subepidermal blister with eosinophils, often accompanied by eosinophilic spongiosis
  • Early lesions can appear urticarial or eczematous
  • Direct immunofluorescence (DIF): linear C3 > IgG along the basement membrane zone, n-serrated pattern
  • DIF on salt split skin: immunoreactants deposited on the blister roof (40%) or on both roof and floor (60%)
  • Indirect immunofluoresence: can be performed on human skin or monkey esophagus; immunoreactants localize to blister roof (95%) on salt split analysis
  • Treatment: corticosteroids, tetracyclines, cytotoxic steroid sparing agents and rituximab, IVIG infusions
ICD coding
  • ICD-10: L12.0 - bullous pemphigoid
Epidemiology
  • Incidence rate: 7.63 per 100,000 person years (Br J Dermatol 2021;184:68)
    • Increases with age
  • 80% of subepidermal immunobullous dermatoses
  • M = F
  • Elderly > children and infants
Sites
  • Inner and anterior thighs, groin, flexor surfaces of upper extremities, lower abdomen
  • Childhood bullous pemphigoid: vulvar localization is most common
Pathophysiology
  • Development of IgG autoantibodies against hemidesmosomal proteins BPAG1 and BPAG2
Etiology
  • Autoimmune disease associated with bullous pemphigoid antigen 180 (BP180, BPAG2) and bullous pemphigoid antigen 230 (BP230, BPAG1, BPAG1e) (Am J Clin Dermatol 2017;18:513)
  • Associated with medication use:
    • Anti-PD1 immunotherapy
    • Diuretics, ACE inhibitors, antibiotics, D penicillamine
  • Pediatric cases associated with viral illness and immunization (BMC Pediatr 2017;17:60)
Clinical features
  • Prodrome phase: mild to severe pruritus and eczematous, papular or urticarial lesions
  • Bullous stage: tense bullae containing serous fluid or hemorrhage
  • Unusual to have mucosal involvement (seen in oral cavity in 10 - 30% of cases) (Lancet 2013;381:320)
Diagnosis
  • Requires histological evaluation in addition to confirmation by direct immunofluorescence studies or detection of circulating autoantibodies
Laboratory
Prognostic factors
Case reports
  • 61 year old man with tender blisters on his feet, upper legs and central upper back, diagnosed with dyshidrosiform bullous pemphigoid (Cureus 2020;12:e6630)
  • 72 year old woman with metastatic non small cell lung cancer develops nivolumab induced bullous pemphigoid (Oncologist 2018;23:1119)
  • 84 year old woman with clear cell renal cell carcinoma and widespread paraneoplastic bullous pemphigoid (Urol Case Rep 2020;30:101119)
  • 88 year old woman with newly diagnosed melanoma and associated bullous pemphigoid (Melanoma Res 2017;27:65)
Treatment
  • First line: topical or systemic corticosteroids
  • Steroid sparing agents (mycophenolate mofetil, methotrexate)
  • Tetracycline
  • IVIG in steroid resistant disease
  • Rituximab or IVIG for refractory disease (not resistant)
Clinical images

Images hosted on other servers:

Tense bullae

Ruptured bullae

Microscopic (histologic) description
  • Subepidermal blister
  • Superficial perivascular mixed inflammatory infiltrate with eosinophils in the dermis and blister cavity
  • Eosinophils may line up along the dermal epidermal junction and extend into the epidermis (eosinophilic spongiosis) (Am J Clin Dermatol 2017;18:513)
  • Urticarial stage:
    • Eosinophil rich perivascular and interstitial infiltrate in the papillary dermis
    • Eosinophilic spongiosis with pseudovacuolar interface change
  • Rare eosinophilic flame figures
  • Uncommon presentations include neutrophil rich and cell poor infiltrates
Microscopic (histologic) images

Contributed by Michael Occidental, M.D. and Randie H. Kim, M.D., Ph.D.
Subepidermal blister with eosinophils Subepidermal blister with eosinophils Subepidermal blister with eosinophils

Subepidermal blister with eosinophils

Eosinophil microabscesses in papillary dermis

Eosinophil microabscesses in papillary dermis


Eosinophilic spongiosis Eosinophilic spongiosis Eosinophilic spongiosis

Eosinophilic spongiosis

Eosinophilic spongiosis with adjacent subepidermal blister Eosinophilic spongiosis with adjacent subepidermal blister

Eosinophilic spongiosis with adjacent subepidermal blister

Virtual slides

Images hosted on other servers:

Eosinophil poor BP

Clinical images, virtual slide, direct immunofluorescence

Positive stains
  • Collagen IV immunohistochemistry:
  • Can be helpful in differentiating subepidermal blistering disorders with level of split below lamina densa (EBA, antilaminin gamma1/p200 pemphigoid, p105 pemphigoid) from BP and other disorders with split within lamina lucida or above lamina densa
    • If split above lamina densa - dermal staining pattern
    • If split below lamina densa - epidermal staining pattern
Immunofluorescence description
  • Linear C3 > IgG and C3 deposition along the basement membrane zone of epithelium, hair follicles and eccrine glands, n-serrated pattern (Br J Dermatol 2013;169:100)
    • Only C3 may be present in early stage lesions
  • Salt split skin analysis:
    • Direct immunofluoresence: localization of immunoreactants to either roof only (40%) or roof and blister (60%)
    • Indirect immunofluorescence: localization of immunoreactants to the roof of the blister (95%)
Immunofluorescence images

Contributed by Randie H. Kim, M.D., Ph.D.
Linear C3 deposition along the basement membrane Linear C3 deposition along the basement membrane

Linear C3 deposition along the basement membrane

Positive IgG with n-serrated pattern (arrow)

Positive IgG with n-serrated pattern (arrow)

Salt Split skin with epidermal reactivity for IgG on roof of blister cavity (star)

Epidermal reactivity for IgG on roof of blister cavity (star)

Videos

Bullous pemphigoid
Dr. Phillip McKee

Bullous pemphigoid
Dr. Christine Ko

Sample pathology report
  • Skin, left medial thigh, shave biopsy:
    • Bullous pemphigoid (see comment)
    • Comment: There is a subepidermal vesicle beneath which there is a mixed cell infiltrate including eosinophils. The associated specimen submitted for immunofluoresence studies shows linear C3 and IgG in an n-serrated pattern along the basement membrane.
  • Skin, left abdomen, shave biopsy:
    • Subepidermal blister with eosinophils (see comment)
    • Comment: There is a subepidermal vesicle beneath which there is a mixed cell infiltrate including eosinophils. The periphery of the lesion shows areas of eosinophilic spongiosis. The differential diagnosis includes bullous pemphigoid, cicatricial pemphigoid, linear IgA disease, a bullous drug eruption and a severe arthropod bite reaction. Additional material for immunofluorescence studies is recommended.
Differential diagnosis
  • Epidermolysis bullosa acquisita:
  • Bullous lupus erythematosus:
    • Neutrophil rich infiltrate, variable interface changes
    • Dermal mucin deposition
    • Eosinophils not present
    • DIF: immunoreactive on dermal side of salt split skin (multiple reactants or IgG and C3), u-serrated pattern
    • Positive lupus serology
    • Positive epidermal nuclear Ig deposition(in vivo ANA) (J Rheumatol 1983;10:733)
  • Dermatitis herpetiformis:
    • Papillary neutrophilic microabscesses
    • Immunofluorescence = granular IgA in dermal papillae
  • Linear IgA bullous disease:
    • Neutrophil rich infiltrate
    • Immunofluorescence = linear IgA along the basement membrane zone
  • Porphyria cutanea tarda:
    • Pauci-inflammatory
    • Festooning dermal papillae
    • Hyalinized superficial blood vessels (positive PAS deposits)
    • Caterpillar bodies (positive for PAS and collagen IV)
    • Direct immunofluoresence: multiple immunoreactants at the dermo-epidermal junction and within superficial vessels (granular deposition)
Board review style question #1

An 87 year old man presents with erythema and blistering of his chest and upper extremities. Examination shows tense bullae on an erythematous base. A skin biopsy is obtained which reveals subepidermal blister with a mixed inflammatory infiltrate with numerous eosinophils. Which of the following is true for the direct immunofluorescence findings regarding the diagnosis?

  1. Direct immunofluorescence would show granular IgA deposition at the tips of the dermal papillae
  2. Direct immunofluorescence would show IgG and C3 reactivity in the epidermis in a fishnet-like pattern in the intercellular squamous region
  3. Direct immunofluorescence would show linear IgA reactivity along the basement membrane
  4. Direct immunofluorescence would show linear IgG and C3 reactivity at the basement membrane
Board review style answer #1
D. Direct immunofluorescence would show linear IgG and C3 reactivity at the basement membrane

Comment Here

Reference: Bullous pemphigoid
Board review style question #2
What is / are the antigen(s) targeted in bullous pemphigoid?

  1. BPAG1 and BPAG2
  2. Collagen VII
  3. Desmoglein 1 and Desmoglein 3
  4. Envoplakin and periplakin
Board review style answer #2
A. BPAG1 and BPAG2

Comment Here

Reference: Bullous pemphigoid

Calcinosis cutis (pending)
[Pending]

Calciphylaxis
Definition / general
  • Rare, life threatening condition of progressive cutaneous necrosis due to small and medium sized vessel calcification
  • Also called calcifying panniculitis or vascular calcification-cutaneous necrosis syndrome
Etiology
  • Due to primary, secondary or tertiary hyperparathyroidism and associated elevated calcium and phosphate levels, which exceed their solubility and deposit in vessels, causing progressive vascular compromise in skin and subcutaneous fat and less often muscle
Clinical features
  • Lesions are bilateral and symmetric, pruritic and frequently painful / tender
  • Often lower extremity, with preservation of pulses
  • Less often affects the breast, abdomen and penis
  • Mortality rate up to 60%
Case reports
Treatment
  • Surgical removal of autonomous parathyroid gland (if due to primary hyperparathyroidism)
  • Also hyperbaric oxygen, antihistamines, corticosteroids
Clinical images

Images hosted on other servers:

Necrosis resembling a burn injury

Calciphylaxis on
abdomen of patient
with end stage
renal disease

Microscopic (histologic) description
  • Involves intima and media of large and small blood vessels
  • Deposits are associated with thrombi
  • Intimal fibroblastic proliferation with luminal narrowing has also been described
  • Also hemorrhage within the subcutaneous fat, fat necrosis accompanied by lobular lymphohistiocytic infiltrate
  • Interstitial calcification is rare
  • Pseudoxanthoma elasticum like changes may occur
Microscopic (histologic) images

Contributed by Sepideh N. Asadbeigi, M.D.

Calcium deposits

Calcium deposits in deep dermis

Calciphylaxis
involving small
blood vessels with
associated thrombi

Subcutaneous fat with vascular calcium deposits

Differential diagnosis
  • Metastatic calcification: due to hyperparathyroidism or chronic renal failure
  • Dystrophic calcinosis cutis: deposits in areas of previously abnormal skin
  • Tumoral calcinosis: no known cause
  • Idiopathic calcinosis of scrotum
  • Subepidermal calcified nodule: childhood papule, small, fleshy, firm, on face

Candida
Definition / general
  • Candida albicans is a part of the normal human skin flora
  • Cutaneous candidiasis is a superficial infection of skin and mucous membranes and the most common Candidal infection
Terminology
  • Cutaneous candidal infections include:
    • Oral candidasis
    • Candidal intertrigo (affects body folds): acute (wet and red), subacute (red +/- maceration), or chronic (red and dry)
    • Candidal diaper dermatitis
    • Candidal vulvovaginitis
    • Candidal balanitis
    • Candidal nail infection: chronic paronychia, onycholysis
Epidemiology
  • In U.S., Candida species are a common cause of intertrigo in both elderly and diabetic patients
  • Candida species colonize the oropharynx in 30 - 55% of healthy young adults, and are commonly found in normal fecal flora
  • 3 out of 4 women will have at least one Candidal vulvoganitis during their lifetime
  • For patients with systemic infections, Candida species is now the 4th most common pathogen from blood cultures
  • More than 90% of HIV population who are not on highly active antiretroviral therapy will develop oropharyngeal candidiasis and 10% develop esophageal candidasis
  • Internationally, Candida species have replaced Cryptococcus species as the most common fungal pathogens affecting immunocompromised hosts
Clinical features
  • Predisposing factors for opportunistic infection with C. albicans include:
    • Infancy or elderly
    • Warm climate
    • Occlusive clothing, poor personal hygiene, dental plates
    • Immune deficiencies (low levels of immunoglobulins, HIV, cancer)
    • Broad spectrum antibiotic treatment
    • High dose estrogen contraceptive pills or pregnancy
    • Chemotherapy or immunosuppressive medications such as systemic steroids
    • Locally applied topical steroids
    • Diabetes mellitus, obesity, Cushing syndrome and other endocrine conditions
    • Iron deficiency
    • Malnutrition
    • Underlying dermatological disease like psoriasis, lichen planus, irritant contact dermatitis
    • Mortality is relative low for cutaneous Candidal infection in healthy patients; however, the mortality rate is up to 30 - 40% in disseminated / systemic candidasis in immunosuppressed patients

  • The characteristic skin manifestation is red and white patches on mucosal surfaces (leukoplakia)
  • In skin folds, it results in moist fissuring with a superfical erythema patch with satellite papulopustules
Diagnosis
  • KOH preparation and skin scraping is the easiest and most cost effective method for diagnosing cutaneous candidiasis
  • Culture from intact pustule or skin biopsy tissue can support the diagnosis
Case reports
Treatment
  • Note: please confirm accuracy of medications below before use

  • Oral candidiasis:
    • Nystatin oral suspension x 10 - 14 days or until 48 - 72 hours after resolution of symptoms
    • Dosage for preterm infants is 0.5 mL (50,000 U) to each side of mouth 4 times / day; for infants is 1 mL (100,000 U) to each side of the mouth 4 times/d; for adults 4 - 6 mL (100,000 U) PO swish and swallow qid

  • Candidal intertrigo:
    • Keep the skin dry, with the addition of topical nystatin powder, clotrimazole, or miconazole twice daily, often in conjunction with a midpotency corticosteroid
    • Extensive infection may require the addition of fluconazole (100 mg PO qd for 1 - 2 wk) or itraconazole (100 mg PO qd for 1 - 2 wk)

  • Acute intertrigo:
    • Can use Domeboro solution, Castellani paint or vinegar/water (1 tbsp vinegar per quart room-temperature water) to apply twice per day for 5 - 10 minutes for 3 - 5 days as needed
    • Dry the area with a hair dryer (low heat)
    • Can also apply triamcinolone-nystatin cream twice daily

  • Subacute intertrigo:
    • Can use benzoyl peroxide wash to cleanse the area instead of application of vinegar or Castellani paint
    • A topical anticandidal cream of choice is applied twice per day, with or without a mild hydrocortisone cream

  • Chronic intertrigo:
    • Can use zinc-talc shake lotion once or twice daily, and the hydrocortisone cream / antifungal mixture may be applied at night
    • Local hyperhidrosis may be treated with antiperspirants (ie, Arid Extra Dry Unscented, Dry Idea) on a long-term basis

  • Candidal diaper dermatitis:
    • Goal is to minimize the time the diaper area is exposed to hot and humid conditions; air drying, frequent diaper changes and generous use of baby powders and zinc oxide paste are adequate preventive measures
    • Apply topical nystatin, amphotericin B, miconazole or clotrimazole to affected areas twice daily x 7 days

  • Candidal vulvovaginitis:
    • Topical antifungal agents (Micatin, Monistat-Derm), or clotrimazole (Lotrimin, Mycelex) creams twice daily x 7 days or intravaginal appliator QHS x 7 days are curative
    • One-time oral therapy with fluconazole (150 mg) or itraconazole (600 mg) is effective and may be a more attractive alternative to some patients, but it is more costly

  • Candidal balanitis:
    • Topical therapy is effective in most patients
    • Evaluate asymptomatic sexual partners and treat them if they are infected to prevent recurrence
    • For persistent lesions beyond the genitalia, consider the possibility of underlying diabetes or other diseases

  • Candidal paronychia:
    • Topical treatment is usually not effective but should be tried for chronic candidal paronychia
    • Drying solutions or antifungal solutions are used
    • Oral therapy with either itraconazole (pulse dosing with 200 mg bid for 1 wk of each of 3 consecutive months) or terbinafine (250 mg qd for 3 months) is recommended

Clinical images

Contributed by Mark R. Wick, M.D.

Breast skin

Microscopic (histologic) images

Contributed by Mark R. Wick, M.D. and Yale Rosen, M.D.

Breast skin

PAS, breast skin

Lung: GMS staining

Cytology description
  • Periodic Acid-Schiff (PAS) stain reveals nonseptated hyphae, which distinguishes Candida from tinea
Positive stains
  • GMS, PAS

Carbuncle
Definition / general
  • Deep form of bacterial folliculitis involving several hair follicles, often due to Staphylococcus aureus, which can be cultured from skin
Terminology
  • Also known as boils
  • Furuncle: small abscess exuding purulent material, involving skin and subcutis in areas with hair follicles
  • Carbuncle: aggregate of connected furuncles, involves multiple hair follicles
Epidemiology
  • Most individuals with carbuncles are otherwise healthy and have good personal hygiene, but they are among the 10 - 20% of population that are Staphylococcus aureus carriers on surface of skin
  • Common in obese patients
  • Carbuncles may occur in patients with immune deficiency, anemia, diabetes or iron deficiency
Sites
  • Staphylococcus aureus is most commonly found in intertriginous regions (where two skin areas may touch or rub together) of nostrils, armpits, groin and intergluteal cleft
Etiology
  • Staphylococcus aureus is present in patients in intertriginous regions, but may be transferred to other sites by scratching
  • Tiny nicks or scratches in the skin can inoculate the bacteria into the wall of a hair follicle, a weak point in the skin
  • Once innoculated, bacteria cause a boil, which runs its usual infectious course in about 10 days
Clinical features
  • Painful localized bacterial infection of skin and subcutis, usually with several openings through which pus is discharged
  • Tender, swollen red papules or pustules
Treatment
  • Antiseptic cleanser, like povidone-iodine or chlorhexidine is used daily for several weeks to clean carbuncle
  • Topical antibiotics include erythromycin BID and clindamycin BID to affected area until clear

  • Note: verify current drug dosages before use (eMedicine)

  • Oral antibiotics include:
    • Cephalexin (adult) 1 - 4 g/day divided BID or QID x 10 - 14 days; (children) 25 - 50 mg/kg/day PO divided q6H for mild to moderate infection; 50 - 100 mg/kg/day divided q6H for severe infection; not to exceed 4g/day for any age
    • Dicloxacillin (adult or children > 40 kg) 125 - 500 mg q6h x 10 - 14 days; (children < 40 kg): 12.5 - 50 mg/kg/d PO divided q6h
    • Erythromycin (adult) 500 mg BID or 330mg q8hr x 10 - 14 days
    • Clindamycin (adult) 150 - 450 mg q6 - 8h x 10 - 14 days; (children) 10 - 30 mg/kg/day q6 - 8h
    • Minocycline (adult) 100 mg BID x 10 - 14 days
    • Rifampin (adult) 600 mg qD x 10 days; (children) 15 mg/kg/day divided BID x 10 days
    • Ciprofloxacin (adult): 250 - 750 mg BID x 10 - 14 days

  • In recurrent cases, obtain nasal culture of patient and family members to look for Staphylococcus aureus colonization
  • If patient is Staphylococcus aureus nasal carrier, consider applying topical antibiotic like mupirocin ointment to nostrils twice daily for 5 days (Wikipedia)
  • If family members are nasal carriers, consider mupirocin ointment or rifampin 600 mg daily for 10 days
  • Consider checking nasal swab for MRSA or culture for sensitivity for treatment failure

  • Prevention:
    • Good hygiene including bathing, hand-washing, keeping nails short and clean
    • Avoid close shaving until lesions are completely resolved
    • Change disposable razors daily or clean electric razor heads daily
    • Wear loose-fitting clothes (tight-fitting clothes may cause occlusion); change clothes daily

  • In obese patients, weight reduction will be beneficial
Clinical images

Images hosted on other servers:

Various images

Microscopic (histologic) description
  • Inflammatory cells, predominantly neutrophils, within the wall and ostia of the hair follicle, creating a follicular-based pustule
  • Inflammation can be either limited to the superficial follicle, primarily involving the infundibulum, or can affect both the superficial and deep aspects of the follicle
  • Deep folliculitis can arise from the chronic lesions of superficial folliculitis or from lesions that are manipulated or scratched; may cause scarring
Differential diagnosis

Central centrifugal cicatricial alopecia
Definition / general
  • Central centrifugal cicatricial alopecia (CCCA) is a lymphocytic scarring alopecia that predominantly affects women of African descent
Essential features
  • A form of lymphocytic scarring alopecia primarily affecting females of African descent (StatPearls: Central Centrifugal Cicatricial Alopecia [Accessed 7 December 2022])
  • Hair loss starts on the scalp vertex and progresses outward in a centrifugal and symmetric pattern
  • Premature desquamation of the inner root sheath may play an important role (Clin Cosmet Investig Dermatol 2016;9:175)
  • Clinicopathologic correlation is required for definitive diagnosis as histologic features overlap lichen planopilaris and frontal fibrosing alopecia
  • Microscopic features of active disease include variably dense perifollicular lymphocytic infiltrate, concentric, lamellated perifollicular fibrosis (onion skin-like), premature desquamation of the inner root sheath and asymmetric thinning of follicular epithelium
  • Microscopic features of more advanced disease include follicular drop out, fibrous tracts, minimal inflammation, compound follicles and naked hair shafts
Terminology
  • Central centrifugal scarring alopecia (CCSA)
  • Follicular degeneration syndrome (historical)
  • Hot comb alopecia (historical)
ICD coding
  • ICD-10: L66.8 - other cicatricial alopecia
  • ICD-11: ED70.5Y - scarring alopecia due to other specified cause
Epidemiology
Sites
Pathophysiology
  • Incompletely understood
  • Likely results from a combination of external insults to hair and genetic susceptibility
  • Premature desquamation of the inner root sheath is thought to play an important role (StatPearls: Central Centrifugal Cicatricial Alopecia [Accessed 7 December 2022])
  • Some families have an autosomal dominant genetic defect in the inner root sheath (J Am Acad Dermatol 2014;70:679)
  • Mutations in the PADI3 (peptidyl arginine deiminase type III) gene were recently described in ~30% of a small cohort of women with central centrifugal cicatricial alopecia (n = 16) (N Engl J Med 2019;380:833)
  • Follicular destruction occurs as a result of the inflammatory response
  • Resultant follicular scarring develops at the site of inflammatory response
Etiology
Clinical features
Diagnosis
Prognostic factors
Case reports
Treatment
Clinical images

Contributed by Jonathan D. Ho, M.B.B.S., D.Sc.

CCCA affecting vertex

Advanced CCCA

Microscopic (histologic) description
  • Features are best appreciated on transversely bisected, 4 mm punch biopsies taken from the active edge (fringe) of the process
  • Features may be consistent or compatible with central centrifugal cicatricial alopecia but clinicopathologic correlation is essential, as pathologic features overlap with other lymphocytic scarring alopecias
  • Premature desquamation of the inner root sheath (IRS) (Clin Cosmet Investig Dermatol 2016;9:175)
    • Loss of the inner root sheath below the level of the isthmus
    • The level at which sebaceous glands appear indicates the isthmus; thus loss of the inner root sheath in sections prior to their appearance or at the level of the subcutis or eccrine coils indicate premature desquamation (PDIRS)
    • May be seen in uninflamed follicles
    • Commonly present in but not specific for central centrifugal cicatricial alopecia
  • Asymmetric thinning of follicular epithelium
  • Follicular density and drop out are inversely proportional to chronicity of alopecia (recall that normal baseline follicular density in persons of African descent may be as low as 21 - 22 hairs/4 mm punch) (Arch Dermatol 1999;135:656)
  • No major shift out of anagen
  • May exhibit follicular miniaturization (Am J Dermatopathol 2014;36:859)
  • Decrease in sebaceous gland number
  • Variably dense perifollicular lymphocytic infiltrate affecting the isthmus and lower infundibulum
  • Concentric, lamellated, often myxoid perifollicular fibrosis (onion skin-like)
  • In cases with progressive fibrosis, lymphocytes may appear to back away from follicle
  • Basal layer vacuolation of follicular or interfollicular epithelium is not a feature of central centrifugal cicatricial alopecia
  • Marked interfollicular fibrosis is not typical of central centrifugal cicatricial alopecia
  • Follicular destruction may be present with resultant naked hair shafts with or without a foreign body granulomatous reaction (late stage)
  • While follicular destruction may result in a more mixed inflammatory cell infiltrate with occasional neutrophils, plasma cells and eosinophils, a heavy mixed inflammatory cell infiltrate should raise the suspicion for folliculitis decalvans
  • Compound follicles in groups of 2 - 3 hairs may be seen; compound follicles with ≥ 4 hairs should raise suspicion for a neutrophilic scarring alopecia
  • In advanced disease, complete loss / marked reduction in follicular density with follicular dropout and deep fibrous tracts
  • Multiple dilated eccrine glands (Am J Dermatopathol 2019;41:871)
  • Overlying epidermis is unremarkable
  • Features of a second, concomitant alopecia (e.g., traction alopecia) may be present (J Cutan Pathol 2016;43:483)
Microscopic (histologic) images

Contributed by Jonathan D. Ho, M.B.B.S., D.Sc.

Premature desquamation of the inner root sheath

Inflammation, fibrosis and epithelial thinning

Perifollicular fibrosis

Deep fibrous tracts

Inflammation, fibrosis and dilated eccrine glands

Dilated eccrine gland

Positive stains
  • Verhoeff-van Gieson (VVG) stain reveals tree trunk-like fibrosis if concomitant vertical sections available (Skinmed 2020;18:365)
  • PAS stain is useful to exclude tinea capitis; spores in the infundibula or stratum corneum represent Malassezia sp. and are not currently thought to be pathogenic
Sample pathology report
  • Skin, scalp, punch biopsy:
    • Lymphocytic scarring alopecia consistent with central centrifugal cicatricial alopecia (see comment)
    • Comment: Transverse sections reveal 17 hairs, in anagen and terminal in size. Sebaceous glands are reduced in number. There is a mild to moderately dense perifollicular lymphocytic infiltrate at the level of the isthmus and concentric perifollicular fibrosis. Premature desquamation of the inner root sheath, deep fibrous tracts and naked hair shafts are present at deeper levels. Compound follicles in groups of 2 - 3 hairs are identified. The overlying epidermis is unremarkable. PAS stain is negative for fungal elements. It should be noted that lichen planopilaris may exhibit overlapping histopathologic features; however, given the clinical setting (patient of African descent with alopecia at the vertex), central centrifugal cicatricial alopecia is favored.
Differential diagnosis
  • Lichen planopilaris (LPP) and frontal fibrosing alopecia (FFA):
    • May be impossible to differentiate based on pathologic findings alone (J Cutan Pathol 2020;47:128)
    • Significant basal layer vacuolation involving follicular epithelium or interface / lichenoid dermatitis affecting the overlying interfollicular epidermis favor lichen planopilaris / frontal fibrosing alopecia (Int J Dermatol 2006;45:375)
    • Narrow wedge shaped fibrosis rather than broad tree trunk fibrosis on vertical sections (VVG stain) favors lichen planopilaris over central centrifugal cicatricial alopecia
    • Clinicopathologic correlation required for accurate diagnosis
      • Clinical setting is crucial:
        • Black patients with vertex predominant alopecia = central centrifugal cicatricial alopecia
        • Scattered patches of scarring alopecia in a footprints in the snow / sand appearance = lichen planopilaris
        • Scarring alopecia primarily affecting frontal hairline, eyebrows, sideburns with or without limbs = frontal fibrosing alopecia
  • Fibrosing alopecia in a pattern distribution (FAPD):
    • Combination of lymphocytic scarring alopecia and pattern hair loss
    • Inflammation should primarily affect miniaturized hairs or both miniaturized and terminal hairs
    • Interface change involving the follicular epithelium of miniaturized hairs favors fibrosing alopecia in a pattern distribution over other lymphocytic scarring alopecias
  • Discoid lupus erythematosus:
    • When active, has the densest inflammation of the lymphocytic scarring alopecias
    • Typically exhibits dense superficial and deep perivascular, perifollicular and perieccrine lymphocytic infiltrate with or without scattered plasma cells
    • Perifollicular fibrosis, miniaturization or total obliteration of follicles may be seen
    • Overlying epidermis demonstrates features of interface dermatitis (active or old), often with follicular hyperkeratosis (plugging) and PAS positive thickening of the basement membrane zone
    • Direct immunofluorescence demonstrating a positive lupus band test may be useful in distinguishing burnt out discoid lupus erythematosus from other late stage scarring alopecias
Board review style question #1

A 56 year old woman of African descent presents with hair loss involving the vertex of the scalp. She has a history of tight hair styles and weaves and complains of an itching sensation. A punch biopsy demonstrates the features shown in the associated photomicrograph. What is the most likely diagnosis?

  1. Central centrifugal cicatricial alopecia
  2. Female pattern hair loss
  3. Frontal fibrosing alopecia
  4. Lichen planopilaris
  5. Traction alopecia
Board review style answer #1
A. Central centrifugal cicatricial alopecia. The biopsy demonstrates perifollicular lymphocytes, concentric (onion skin-like) fibrosis, asymmetry of the follicular epithelium and absence of sebaceous glands consistent with a lymphocytic scarring alopecia. While there are overlapping features among this group, in a female patient of African descent with hair loss affecting the vertex, the most likely diagnosis is central centrifugal cicatricial alopecia (CCCA).

Comment Here

Reference: Central centrifugal cicatricial alopecia
Board review style question #2

A 45 year old woman of African descent with alopecia affecting the crown has a biopsy consistent with central centrifugal cicatricial alopecia. The associated photomicrograph is taken at the level of the eccrine coils / subcutis. What finding is shown in the image?

  1. Asymmetry of follicular epithelium
  2. Deep fibrous tract
  3. Dilated eccrine coil
  4. Onion skin-like perifollicular fibrosis
  5. Premature desquamation of the inner root sheath
Board review style answer #2
E. Premature desquamation of the inner root sheath. At this deep level (eccrine coils and fat), there should be a well formed inner root sheath but instead the image demonstrates an absence of this structure. While all the answer options are seen in central centrifugal cicatricial alopecia, premature desquamation of the inner root sheath is demonstrated in the photomicrograph.

Comment Here

Reference: Central centrifugal cicatricial alopecia

Chondrodermatitis nodularis helicis
Definition / general
  • Idiopathic, nonneoplastic ulcerative lesion of auricle
  • Unilateral, painful dome shaped nodule; may be due to chronic trauma
Terminology
  • Also called Winkler disease
Epidemiology
  • Usually men ages 40+
Clinical features
  • Dome shaped nodule, 0.3 to 1.8 cm, with crusty scale covering central area of ulceration
Treatment
Clinical images

Images hosted on other servers:

Chondrodermatitis nodularis

Microscopic (histologic) description
  • Central ulceration of epidermis with adjacent acanthosis, hyperkeratosis, parakeratosis and pseudoepitheliomatous hyperplasia
  • Base of ulcer has granulation tissue that usually involves perichondrium and cartilage
  • May have foci of fibrinoid necrosis
  • Vascular proliferation may resemble a glomus tumor
  • No dermal adnexa at site of lesion
Differential diagnosis
  • Clinically resembles carcinoma

Chondrodermatitis nodularis helicis
Definition / general
Essential features
Terminology
ICD coding
  • ICD-11: AA12 - chondrodermatitis nodularis
Epidemiology
Sites
Pathophysiology
Etiology
  • Mechanism is not entirely clear but it is believed to be caused by chronic, excessive pressure; commonly attributed to sleeping in lateral decubitus position but also associated with hearing aids, headphones, etc. (Dermatol Online J 2021;27:13030, J Eur Acad Dermatol Venereol 2021;35:e506, J Dtsch Dermatol Ges 2011;9:287)
  • Inciting injury may also be due to solar damage or cold exposure
  • ~15% of patients with chondrodermatitis nodularis chronica helicis (CNCH) may have an underlying autoimmune or connective tissue disease, including but not limited to polymyalgia rheumatica, psoriasis, rheumatoid arthritis, CREST (calcinosis, Raynaud phenomenon, esophageal dysmotility, sclerodactyly and telangiectasia) syndrome, vitiligo, chronic dermatitis (Eur J Dermatol 2022;32:347)
Diagrams / tables
N/A
Clinical features
  • Intensely painful papule on the superior helix, measuring 4 - 5 mm on average (up to 20 mm maximum dimension) (J Dtsch Dermatol Ges 2011;9:287)
  • Skin colored, smooth appearance with a central crater or crusted surface
  • Chronic changes include prominent central ulceration and raised edges
  • Some cases lack nodule formation and instead present as a keratotic plaque with surrounding erythema (StatPearls: Chondrodermatitis Nodularis Helicis [Accessed 29 February 2024])
  • Clinically suspicious for malignancy, particularly squamous cell carcinoma and basal cell carcinoma
  • Dermoscopic appearance: radially arranged, thick, white lines converging to a central rounded yellow-brown clod (Arch Dermatol Res 2018;310:551)
  • Underlying autoimmune and connective tissue diseases present in up to 15% of cases (Eur J Dermatol 2022;32:347)
Diagnosis
  • Diagnosis can be made clinically, dermoscopically or with histological assessment
  • WHO essential criteria: cup shaped crater with necrotic debris and fibrin extending to involve the superficial cartilage
Laboratory
Not applicable
Radiology description
Not applicable
Radiology images
Not applicable
Prognostic factors
Case reports
  • 12 year old man with tender, ulcerating nodule on antihelix after prolonged headphone use (Pediatr Dermatol 2019;36:388)
  • 15 year old man with painful nodule on right antihelix following prolonged headphone use (Dermatol Online J 2021;27:13030)
  • 60 year old woman with a painful, red papule on antihelix after prolonged cell phone use (BMJ Case Rep 2024;17:e258101)
  • 61 year old man with painful, red papule on crus of right ear after Mohs surgery and radiation therapy for adjacent squamous cell carcinoma (Cutis 2023;111:E5)
  • 68 year old man with painful, red papule on bridge of nose from BiPAP mask use (chondrodermatitis nodularis nasi) (J Cutan Pathol 2020;47:1046)
Treatment
  • Randomized or blinded trials have not been performed but a systematic review identified surgery as the most effective treatment option (Dermatol Surg 2016;42:1125)
  • Surgery
    • Highest cure rate (82%) (Dermatol Surg 2016;42:1125)
    • Variations in surgical approach: wedge excision, removal of prominent areas, punch excision, curettage followed by cauterization, cartilage removal alone
  • Topical nitroglycerin therapy
    • Curative in 51% cases (Dermatol Surg 2016;42:1125)
    • Can be used in combination with pressure relieving devices with improved success
  • Pressure relieving devices
    • Curative in 37% (Dermatol Surg 2016;42:1125)
    • Nonadherence is a large factor to low cure rate, particularly pressure relieving devices to wear during sleep (neck pain, discomfort)
  • Other therapies
    • CO2 laser, photodynamic therapy, platelet rich plasma, procaine injections
Clinical images

Images hosted on other servers:
Dermoscopic appearance Dermoscopic appearance

Dermoscopic appearance

Superior helix

Superior helix

Antihelix

Antihelix

Gross description
  • Well circumscribed skin colored papule arising in the helix
  • Bears central ulceration with crusting, plus surrounding erythema
Gross images

Contributed by Cale Michael Max Fletcher, B.Med., M.P.H. and Ruta Gupta, M.D.
Excision of antihelix Excision of antihelix

Excision of antihelix

Excision of helix

Excision of helix

Frozen section description
Not applicable
Frozen section images
Not applicable
Microscopic (histologic) description
  • Essential criteria, as per WHO: cup shaped crater with necrotic debris and fibrin extending to involve the superficial cartilage
  • Ulcerating nodule filled with necrotic debris, fibrin, variable inflammatory cells
  • Hyperkeratotic and parakeratotic crusting of the epidermis with adjacent acanthosis containing acute inflammatory cells
  • Inverted funnel shaped defect in dermis, with necrobiosis of underlying collagen; the dermal collagen appears homogenous and eosinophilic with admixed fibrin
  • Superficial cartilage erosion characterized by loss of basophilia, fibrosis and chondrocyte dropout
  • Reference: J Clin Pathol 1959;12:179
Microscopic (histologic) images

Contributed by Cale Michael Max Fletcher, B.Med., M.P.H. and Ruta Gupta, M.D.
Punch biopsy, helix

Punch biopsy, helix

Cartilage erosion

Cartilage erosion

Sinus tract with fibrin

Sinus tract with fibrin

Funnel shaped ulcer

Funnel shaped ulcer

Ulcer with adjacent acanthosis

Ulcer with adjacent acanthosis


Degenerate collagen and fibrin

Degenerate collagen and fibrin

Ulceration and parakeratotic crust

Ulceration and parakeratotic crust

Homogenized dermis and inflammation

Homogenized dermis and inflammation

Adjacent acanthosis

Adjacent acanthosis

Absence of squamous atypia

Absence of squamous atypia

Virtual slides

Images hosted on other servers:
64M, painful ear nodule

64 year old man, painful ear nodule

76M, painful ear nodule

76 year old man, painful ear nodule

Cytology description
Not applicable
Cytology images
Not applicable
Immunofluorescence description
Not applicable
Immunofluorescence images
Not applicable
Positive stains
Not applicable
Negative stains
Not applicable
Electron microscopy description
Not applicable
Electron microscopy images
Not applicable
Molecular / cytogenetics description
Not applicable
Molecular / cytogenetics images
Not applicable
Videos

Chondrodermatitis nodularis helicis by Dr. Jerad Gardner

Sample pathology report
  • Left helix, punch biopsy:
    • Chondrodermatitis nodularis chronica helicis (see comment)
    • Comment: Punch biopsy of squamous epithelium to underlying cartilage shows a funnel shaped ulcer with adjacent acanthosis. Skin surface bears a parakeratotic crust. The underlying dermis appears homogenized and eosinophilic with fibrin deposition extending to involve the superficial cartilage. Chondrocyte dropout and loss of basophilia is seen. There is an accompanying population of mixed inflammatory cells and ectatic vessels.
    • No squamous atypia is seen. There are no features of malignancy.
    • The features are those of chondrodermatitis nodularis chronica helicis.
Differential diagnosis
  • Keratinocyte proliferative lesions:
    • Squamous cell carcinoma:
      • Macroscopic: resembles CNCH (ulcerated or keratotic nodule)
      • Microscopic: squamous atypia with nuclear hyperchromasia, pleomorphism, mitoses and incomplete maturation and paradoxical keratinization
      • Adjacent epidermis may show actinic and Bowenoid actinic keratosis
    • Basal cell carcinoma:
      • Macroscopic: cream colored nodule, which may be ulcerated
      • Microscopic
        • Nodules of basaloid appearing cells with scanty cytoplasm and angulated, hyperchromatic nuclei
        • Mitoses and apoptosis are easily found
        • There is palisading of nuclei at the periphery of the nests and retraction artifact
  • Inflammatory:
    • Granuloma annulare:
      • Interstitial granulomatous inflammation centered around necrobiotic collagen, with mucin deposition between collagen fibers
    • Gouty tophi:
      • Nodular aggregates of acellular, amorphous pale eosinophilic material with granulomatous inflammation
  • Painful nodular tumors, which occur rarely on the pinna (J Dtsch Dermatol Ges 2011;9:287):
    • Eccrine spiradenoma:
      • Circumscribed basaloid tumor within the dermis
      • Composed of tubules and ductules containing basement membrane-like material
      • 2 cell populations are seen (central pale cells and more peripheral basaloid cells)
    • Glomus tumor:
      • Well circumscribed mass comprising glomus cells, vessels and smooth muscle cells
    • Leiomyoma:
      • Well circumscribed soft tissue mass composed of intersecting fascicles of bland smooth muscle
    • Neuroma:
      • Circumscribed, unencapsulated spindle cell proliferation comprising axons, Schwann cells, endoneurial and perineurial cells
Board review style question #1

The painful lesion shown in the image above is found on the helix of a 78 year old man. What is the diagnosis?

  1. Basal cell carcinoma
  2. Chondrodermatitis nodularis chronica helicis
  3. Gouty tophus
  4. Squamous cell carcinoma
Board review style answer #1
B. Chondrodermatitis nodularis chronica helicis. No neoplastic proliferation is present. Inflammatory changes, including a funnelshaped ulcer and fibrinous sinus tract, can be seen extending to the underlying cartilage. Answers A and D are incorrect because they require the presence of a neoplastic population of epithelial cells. Answer C is incorrect because it requires the presence of an amorphous, eosinophilic nodule with associated granulomatous inflammation.

Comment Here

Reference: Chondrodermatitis nodularis helicis
Board review style question #2
Which of the following is true regarding chondrodermatitis nodularis chronica helicis?

  1. Most cases occur in patients with an autoimmune or connective tissue disease
  2. Most commonly found on the helix and antihelix of older men
  3. Neoplastic lesion characterized by necrobiotic dermal collagen and erosion of superficial cartilage
  4. Precipitating injury is believed to be microbial infection
Board review style answer #2
B. Most commonly found on the helix and antihelix of older men. The helix and antihelix are the most common sites for this lesion, however, any cartilage containing, pressure prone area may be susceptible. Answer C is incorrect because chondrodermatitis nodularis chronica helicis is a nonneoplastic lesion. Answer A is incorrect because ~15% of cases occur in patients with an autoimmune or connective tissue disease (e.g., polymyalgia rheumatica). Answer D is incorrect because the precipitating injury is believed to be solar / actinic damage or cold exposure.

Comment Here

Reference: Chondrodermatitis nodularis helicis

Chromoblastomycosis
Definition / general
  • Chronic deep cutaneous fungal infection usually affecting the limbs at the inoculation site

  • Causative agents include several fungi found in soil, wood and decaying plant material:
    • Phialophora verrucosa
    • Fonsecaea pedrosi
    • (most common pathogen, accounts for > 90% of the cases in South America)
    • Fonsecaea compacta
    • Cladosporium carrionii
    • Rhinocladiella aquaspersa (Ramichloridium cerophilum)
Epidemiology
  • First case of chromoblastomycosis was reported by Pedroso in Brazil in 1911
  • Incidence of chromoblastomycosis is greatest in tropical and subtropical regions, including Madagascar, Brazil, Gabon, Colombia, Venezuela, Cuba, the Dominican Republic and Mexico
  • Up to 70% of cases occur in males
  • Barefooted farmers account for almost 75% of patients with chromoblastomycosis
Clinical features
  • Common clinical presentation is verrucous lesion over extremities of adult men engaged in outdoor work (Indian J Pathol Microbiol 2010;53:666)
  • The disease is slowly progressive - the average time between the initial lesion and the clinical diagnosis is 15 years
  • Lesion starts as a small firm red / gray bump on the traumatized skin on the foot or hand
  • Grows slowly (2 mm/year) to form raised hyperkeratotic (crusted, warty-looking) plaque
  • The affected limp can be enlarged (elephantiasis)
  • Can develop satellites lesions (new lesions near primary lesion)
  • Rarely, squamous cell carcinoma develops within longstanding chromoblastomycosis (An Bras Dermatol 2010;85:267)

  • Diagnosis: culture at 25 - 30 degrees Celsius grows olive-green to black fungal colonies after 1 - 2 weeks
Treatment
  • Treatment is difficult and long
  • A Mexican study showed 30% cure rate, 60% improvement, and 10% failed therapy
  • Smaller lesions are best treated with surgical excision or cryotherapy (multiple treatments may span up to 10 years)
  • Extensive lesions can be treated with itraconazole (up to several years), or terbinafine (up to several months)
  • Often dramatic improvement at first, but difficult to cure
  • Some studies suggest a multidrug approach is more effective: itraconazole + flucytosine; flucytosine + amphotericin B
  • Recalcitrant lesions can be treated with a combination of itraconazole and cryotherapy or local hyperthermia or CO2 laser vaporization
Clinical images

Images hosted on other servers:

Hyperkeratotic plaque

Cultures are
typically olive
black with a
suede-like surface

Microscopic (histologic) description
  • Clinical suspicious is important to alert pathologists to check for sclerotic bodies, which may be rare
  • The classical histopathological hallmarks are pseudoepitheliomatous hyperplasia with intraepidermal abscess and pigmented fungal sclerotic bodies (Medlar bodies or copper bodies)
  • Fungi appear in clusters that reproduce by equatorial septation rather than budding
  • Fungal stains show fungi within macrophages and rarely in factor XIIIa positive dedrocytes or Langerhan cells
Microscopic (histologic) images

Images hosted on other servers:

Sclerotic bodies: H&E and KOH prep

H&E stain of
sclerotic bodies in
an intraepidermal
abscess

Skin biopsy specimen

Cladosporium carrionii (left) and Fonsecaea pedrosoi (right), the 2 most common pathogenic fungi causing chromoblastomycosis

Differential diagnosis

Chronic cutaneous lupus
Definition / general
  • Lupus erythematosus is a multisystem autoimmune disease that involves the skin and internal organs
  • Cutaneous lupus erythematosus is categorized as acute, subacute and chronic
Essential features
  • Most common sites for discoid lupus erythematous are face, scalp and conchal bowl
  • Discoid lupus erythematous presents with erythematous and scaly plaques with hyperpigmentation; scalp involvement may lead to scarring, dyspigmentation and alopecia
  • Microscopic features of discoid lupus erythematous include epidermal atrophy, effacement of rete ridges, hyperparakeratosis, follicular keratotic plugging and basement membrane thickening with vacuolar interface change and keratinocyte apoptosis
Terminology
  • Lupus erythematous (LE)
  • Systemic lupus erythematous (SLE)
  • Discoid lupus erythematous (DLE)
  • Tumid lupus erythematosus
  • Chilblain lupus erythematosus
  • Lupus panniculitis
  • Lupus profundus
ICD coding
  • ICD-10: L93.0 - Discoid lupus erythematosus
Epidemiology
  • Mostly young women between 20 and 30
  • More common in persons of African descent than in Caucasians
Sites
  • Discoid lupus erythematosus (DLE): face, scalp, conchal bowl
  • Other variants of chronic cutaneous lupus erythematosus
    • Tumid lupus erythematosus: upper trunk, lateral face
    • Chilblain lupus erythematosus: acral
    • Lupus panniculitis: upper extremities, upper trunk
Pathophysiology
  • In patients with defects in innate immunity and opsonization, UV radiation causes apoptosis; subsequent phagocytosis leads to recognition of self antigens and autoimmunity
  • Activated immune reaction is a result of interaction between hereditary predisposition and environmental or infectious agents
  • MHC II may identify self antigens which show cross reactivity with pathogens
  • HLA-B8, HLA-DR3, HLA-DR2, HLA-A1, HLA-B15, HLA-DRw6 are strongly associated with lupus (Arthritis Res Ther 2015;17:182)
Etiology
Clinical features
  • Discoid lupus erythematosus: erythematous and scaly plaques with hyperpigmentation; scalp involvement often results in scarring, dyspigmentation and alopecia (Postepy Dermatol Alergol 2019;36:739)
  • Tumid lupus erythematosus: erythematous plaques without scale or dyspigmentation
  • Chilblain lupus erythematosus: violaceous patches or plaques incited or exacerbated by cold exposure
  • Lupus panniculitis and profundus: tender subcutaneous nodules with or without epidermal or pigmentary changes
Diagnosis
  • Clinical findings
  • Blood draw
  • Skin biopsy
Laboratory
  • Localized discoid lupus erythematosus: 5% of cases associated with systemic lupus erythematosus (Int J Womens Dermatol 2017;3:S62)
  • Generalized discoid lupus erythematosus: 30% of cases associated with systemic lupus erythematosus
  • Tumid lupus erythematosus: rarely associated with systemic lupus erythematosus (J Clin Rheumatol 2008;14:338)
  • In association with systemic disease: urinalysis, serum creatinine for renal disease, CBC / differential for anemia, leukopenia, lymphopenia, thrombocytopenia (Dtsch Arztebl Int 2015;112:423)
Prognostic factors
  • Discoid lupus erythematosus is associated with low mortality but significant impact on quality of life due to dysesthesia, pruritus, scarring alopecia and dyspigmentation
  • Early treatment helps to reduce scarring and atrophy (J Am Acad Dermatol 2011;64:849)
Case reports
Treatment
Clinical images

Contributed by Kiran Motaparthi, M.D.

Discoid lupus erythematosus, nose

Discoid lupus erythematosus, scalp and ear

Tumid lupus erythematosus, chest

Chilblains lupus in patient with systemic lupus erythematosus

Lupus profundus



Images hosted on other servers:

Malar rash of face

Microscopic (histologic) description
  • Discoid lupus erythematosus
    • Epidermal atrophy or hypertrophy, attenuation (flattening) of rete ridges
    • Hyperkeratosis and follicular keratotic plugging
    • May have basement membrane thickening
    • PAS highlighting the basement membrane zone
    • Vacuolar degeneration with keratinocyte apoptosis in the basilar layer
      • Cell poor infiltrate in the papillary dermis in most cases
      • Lichenoid (band-like) infiltrate composed of lymphocytes in particularly hypertrophic cases
    • Dilated blood vessels, edema and pigmentary incontinence in dermis
    • Dense superficial and deep lymphocytic infiltrate around the adnexae and vessels
      • Also increased plasma cells
    • With or without increased mucin in dermis
    • Pilosebaceous unit damage and broad zones of dermal fibrosis (Int J Womens Dermatol 2017;3:S62)
  • Tumid lupus erythematous
    • Increased mucin in dermis
    • Lymphocytic infiltrate in superficial and deep dermis, around the adnexae and vessels
    • There is typically little to no involvement of the epidermis
  • Chilblain lupus erythematosus
    • Vacuolar interface change
    • Papillary dermal edema
    • Lymphocytic infiltrate in superficial and deep dermis and surrounding eccrine glands
    • Variable vasculitic or vasculopathic changes (Br J Dermatol 2020;183:729)
  • Lupus panniculitis and profundus
    • Lupus profundus
      • Hyperkeratosis and acanthosis
      • Vacuolar interface change
      • With or without increased mucin in dermis
      • Lymphocytic infiltrate in superficial and deep dermis, around the adnexae and vessels
    • Lupus panniculitis and profundus
      • Lymphocytes, plasma cells, neutrophils with karyorrhectic debris and rare eosinophils in subcutaneous fat lobules and septa
      • While typically lymphoplasmacytic, neutrophils and even rare eosinophils may be present
      • Hyaline fat necrosis
Microscopic (histologic) images

Contributed by Kiran Motaparthi, M.D.

Discoid lupus erythematosus, late

Discoid lupus erythematosus, late


Tumid lupus erythematosus

Chilblains lupus


Chilblains lupus

Lupus panniculitis

Lupus panniculitis

Virtual slides

Images hosted on other servers:

Discoid lupus erythematous

Lupus erythematosus profundus

Immunofluorescence description
  • Positive lupus band test: IgM is most common, while IgG is most specific and C3 may also be paired with IgM (PLoS One 2013;8:e70983)
    • Granular IgM is continuous in sun exposed skin and interrupted in sun protected skin
  • In patients with systemic lupus, lupus band is identified in lesional and nonlesional skin, while in patients with cutaneous lupus but no systemic involvement, immunoreactant deposition is only identified in lesional skin
    • However, a positive lupus band can be observed in sun exposed skin of healthy patients; specificity is increased in sun protected skin
  • Possibility of in vivo antinuclear antibody in patients who have systemic lupus erythematosus (Ann Rheum Dis 1990;49:163)
  • IgM also highlights cytoid bodies in cutaneous lupus erythematosus (J Cutan Pathol 2007;34:481)
Positive stains
Electron microscopy description
Sample pathology report
  • Scalp, biopsy:
    • Discoid lupus erythematosus (see comment)
    • Comment: There is epidermal atrophy and follicular ostia are dilated and contain keratotic plugs. There is vacuolar degeneration in the basilar layer along with basement membrane thickening. There is a superficial and deep perivascular and periadnexal lymphocytic infiltrate with plasma cells. Dermal mucin is increased.
  • Central chest, biopsy:
    • Tumid lupus erythematosus (see comment)
    • Comment: There is a superficial and deep perivascular and periadnexal lymphocytic infiltrate with plasma cells. Dermal mucin is increased.
  • Shoulder, biopsy:
    • Lupus profundus (see comment)
    • Comment: There is epidermal hyperkeratosis and acanthosis. There is vacuolar degeneration in the basilar layer. Dermal mucin is increased. There is a superficial and deep perivascular, periadnexal and subcutaneous lymphocytic infiltrate with plasma cells. There is hyaline fat necrosis with karyorrhexis.
Differential diagnosis
  • Discoid lupus erythematosus:
    • Lymphocytic infiltrate of Jessner:
      • Variant of tumid lupus erythematosus
      • Erythematous annular plaques that heal without scarring or dyspigmentation
      • Superficial and deep perivascular and periadnexal lymphocytic infiltrate with plasma cells
      • Absence of epidermal changes and mucin in dermis is differentiating feature from discoid lupus erythematosus
  • Lichen planus variants:
    • Hypertrophic lichen planus:
      • Violaceus, hyperkeratotic and lichenified plaques on extensor surfaces
      • Parakeratosis, hypergranulosis and pseudoepitheliomatous hyperplasia
      • Lichenoid interface dermatitis limited to base of rete ridges
      • Differentiated from hypertrophic (verrucous) discoid lupus erythematosus by presence of eosinophils and absence of periadnexal and deep perivascular inflammation
    • Lichen planopilaris:
      • Perifollicular scale, erythema and scarring alopecia
      • Lichenoid interface tissue reaction affects infundibulum and isthmus, with or without involvement of the epidermis
      • Differentiated from discoid lupus erythematosus by absence of deep and perieccrine inflammation and mucin
    • Lupus erythematosus and lichen planus overlap:
      • Differentiated from discoid lupus erythematosus based on combination of histopathologic and immunopathologic features
      • Epidermal changes typical of lichen planus along with deep and periadnexal inflammation
      • Immunofluorescence findings of lupus erythematosus and lichen planus
  • Rosacea:
    • Presents with erythematous papules and pustules and involvement of nasolabial folds
    • No pigmentary changes, scarring or atrophy
    • Lymphohistiocytic perifolliculitis, with variable perifollicular and intrafollicular neutrophils
  • Lupus panniculitis:
  • Chilblain lupus erythematosus:
    • Perniosis:
      • Painful erythematous or edematous nodules on acral skin following exposure to cold
      • Clinical context variable: chronic cutaneous lupus erythematosus (CCLE) without systemic disease, chilblain lupus erythematosus in the context of systemic lupus erythematosus, idiopathic perniosis and COVID-19
      • Vacuolar interface change is common
      • Marked subepidermal edema
      • Superficial and deep perivascular and perieccrine lymphocytic infiltrate
      • Vasculitis variable
      • Interstitial fibrin and increased mucin favor chilblain lupus erythematosus over idiopathic perniosis (Am J Dermatopathol 2018;40:265)
  • Tumid lupus erythematosus:
    • Polymorphous light eruption:
      • Pruritic papules, vesicles and plaques with photodistribution
      • Marked papillary dermal edema
      • Superficial and deep perivascular lymphocytic infiltrate
      • Distinguished from tumid lupus by marked edema, absence of mucin and lack of significant periadnexal inflammation
Board review style question #1

A 37 year old woman presents with erythematous plaques without dyspigmentation or scale on her upper trunk. The biopsy demonstrates a perivascular and periadnexal lymphocytic infiltrate without epidermal changes. Which of the following statements is correct?

  1. Edema is typically prominent
  2. The histopathologic findings include prominent keratinocyte necrosis in the basal layer of the epidermis and follicular epithelium
  3. These findings represent polymorphous light eruption
  4. These findings represent tumid lupus erythematous
  5. This condition is strongly associated with systemic lupus erythematosus
Board review style answer #1
D. The cutaneous findings represent tumid lupus erythematosus

Comment Here

Reference: Chronic cutaneous lupus
Board review style question #2
A 32 year old woman presents with hyperpigmented plaques on the conchal bowl and a scarring alopecia. Skin biopsy revealed an atrophic epidermis, basement membrane thickening with vacuolar interface change and increased mucin in dermis. There is a superficial and deep and perieccrine lymphocytic infiltrate. Which of the following is the best diagnosis?

  1. Discoid lupus erythematosus
  2. Lichen planopilaris
  3. Lymphocytic Infiltrate of Jessner
  4. Polymorphous light eruption
  5. Rosacea
Board review style answer #2
A. Discoid lupus erythematosus

Comment Here

Reference: Chronic cutaneous lupus

Chronic radiation dermatitis
Clinical features
  • Radiation causes various benign vascular proliferations including benign lymphangiomatous papules (the lymphatic counterpart of telangiectases due to destruction/obstruction of lymphatic drainage), atypical vascular lesions mimicking benign lymphangioendothelioma, patch stage Kaposi sarcoma and well differentiated angiosarcoma (involves subcutaneous tissue, marked cytologic atypia, piling up of endothelial cells, Ki67+)
  • Occurs 3 - 20 years after radiation exposure
  • May be acute or chronic and follows therapeutic or accidental overexposure
  • Often women treated for breast cancer
  • Redness, swelling, hair loss and blistering
  • Morbiliform, papular, annular and bullous lesions
  • An erythema multiforme-like dermatosis is rare complication of radiotherapy
  • Chronic radiation may present with acne vulgaris, ringworm, atrophy and scaling, variable hypo and hyperpigmentation, telangiectasias and often alopecia

  • Description: papules, small vesicles or erythematous plaques on irradiated field
Case reports
Microscopic (histologic) description
  • Epidermis may be necrotic and accompanied by both spongiosis and intracellular edema
  • Hydropic degeneration of basal layer of epidermis and sometimes subepidermal vesiculation
  • Dermis is edematous and may show fibrin deposition; also dermal macrophages, eosinophils, plasma cells and lymphocytes
  • In early stages, vascular thrombosis is a feature
  • Chronic radiation dermatitis: epidermis shows hyperkeratosis and may show foci of parakeratosis, acanthosis or atrophy with attenuation of ridge pattern; may be spongiosis or basal cell liquefactive changes, cytologic atypia and dyskeratosis; dense fibrosis and elastosis in dermis, with fibrinous excaudate; blood vessels often thickened and fibrointimal hyperplasia is present; telangiectatic vessels may be present; loss of appendages, particularly hair follicles; associated with epidermal dysplasia, squamous or basal cell carcinoma
  • Bizarre fibroblasts, with abundant polydendritic basophilic cytoplasm and large hyperchromatic or vesicular nuclei may suggest a neoplastic process; also seen in chronic lichen simplex, pressure ulcer and pleomorphic fibroma
  • Post UV-B radiation: damaged keratinocytes (sunburn cells), intercellular edema and exocytosis; dermal changes include endothelial cell swelling and perivenular edema with a predominantly mononuclear intradermal chronic inflammatory cell infiltrate; also elastosis in fair skinned individuals
  • Post UV-A radiation: keratinocyte swelling, vacuolation accompanied by intercellular edema and diminished numbers of Langerhans cells, but no sunburn cells; dermis has mixed infiltrate of neutrophils, lymphocytes and occasionally basophils and eosinophils; also endothelial swelling
  • Benign lymphangiomatous papules/plaques: superficial dermal involvement by irregularly dilated vascular spaces in branching and anastomosing pattern; vessels have thin walls and lymphatic appearance, with single layer of discontinuous flattened endothelial cells; also numerous small stromal papillary formations lined by endothelial cells that project into lumina
  • Atypical vascular proliferations mimicking benign lymphangioma or patch stage Kaposi sarcoma: poorly circumscribed and focally infiltrating, irregular jagged vascular spaces involving entire dermis; lined by inconspicuous endothelial cells; variable dissection of dermal collagen bundles
Positive stains
  • CD31 stains vessels
Negative stains
Differential diagnosis
  • Acute GVHD

Cicatricial pemphigoid
Definition / general
  • Rare autoimmune bullous disease with mucosal lesions, including conjunctiva; skin involvement in 30%
Terminology
  • Also called benign mucous membrane pemphigoid
  • Cicatrix means scar
Epidemiology
  • Usually elderly, F:M = 2:1
Etiology
  • Due to circulating autoantibodies against basement membrane proteins including BP180
  • Complication of D-penicillamine therapy
  • Occurs after acute severe ocular inflammation in patients with Stevens-Johnson syndrome
Clinical features
  • Affects mucosa of mouth and eyes; lesions heal with scarring
  • May cause blindness, supraglottic stenosis or airway obstruction
  • Cutaneous cases present with tense blisters and erosions, often on head, neck or sites of trauma
  • May have low titer of circulating antibodies

  • Anti-epiligrin subtype: uncommon, primarily affects mucous membranes but also skin; involves autoantibodies against subunit of epiligrin (laminin 5, laminin 332); associated with higher incidence of solid cancers and mortality from treatment with systemic immunosuppressive drugs (Dermatol Online J 2008;14:3)
Case reports
Treatment
  • Suppress blister formation; topical or systemic steroids, possibly immunosuppressants (eMedicine #1, #2)
Clinical images

Images hosted on other servers:

Various bullae

Microscopic (histologic) description
  • Similar to bullous pemphigoid with subepidermal blisters
  • Subepidermal vesicle contains edema fluid, fibrin and variable inflammatory cells
  • Perivascular lymphohistiocytic infiltrate, plasma cells and neutrophils
  • Fewer eosinophils than generalized bullous pemphigoid
  • Conjunctival squamous metaplasia with foci of hyperkeratosis and parakeratosis, accompanied by goblet cell depletion; conjunctival vesicles or bulla are rare
Microscopic (histologic) images

Images hosted on other servers:

Vacuolar change and subepidermal bullae with perivascular infiltration in dermis; indirect immunofluorescence on NaCl split skin shows antibodies on dermal side

Linear IgG deposition

Positive stains
  • Direct immunofluorescence shows linear C3 and IgG staining along basement membrane zone
  • Salt-split skin indirect immunofluorescence shows C3 and IgG binding to dermal side of split
Electron microscopy description
  • Antibodies in lamina lucida
Differential diagnosis
  • Bullous pemphigoid:
    • May present with mucosal involvement but salt-split skin testing shows C3 and IgG binding to epidermal side of split
  • Epidermolysis bullosa acquisita:
    • Blisters form shortly after birth due to pressure, rubbing or trauma
  • Ocular involvement should not be confused with drug induced pemphigoid (pseudoocular cicatricial pemphigoid), which is self limiting and usually develops after long term use of glaucoma medication

Coccidioidomycosis
Definition / general
Essential features
Terminology
ICD coding
  • ICD-10: B38.9 - coccidioidomycosis, unspecified
Epidemiology
  • Major systemic mycosis
  • Relatively more common in Mexico, Central and South America (J Am Acad Dermatol 2006;55:929)
  • Endemic in the U.S. southwestern deserts
  • Most important risk factor is dust exposure in endemic areas
Sites
Pathophysiology
  • Coccidioides species exist in two phases (J Am Acad Dermatol 2006;55:929)
    • Mycelial phase
    • Spherule phase
  • Mycelia are found in the soil and made of septate and ramified hyphae; as the environment dries, mycelia reproduce thick walled spores named arthroconidia
  • Infection occurs with inhalation or rare inoculation of arthroconidia
  • Within the host, spherules are preferentially produced, which then develop internal endospores; the endospores are released and spread to nearby or distant tissues
  • May be classified into primary and secondary disease; 40% of patients present with primary disease which generally affects the lungs
  • Primary involvement of the skin is quite uncommon; acquired by direct inoculation of the fungus by means of splinters and abrasions
  • 5 - 10% of cases present with secondary disease which can affect the lungs and become a chronic process
  • Disseminated disease commonly involves the skin, bones, joints, nervous system pericardium, peritoneum, skeletal muscle and meninges
Etiology
  • Caused by the dimorphic, soil borne ascomycete fungi Coccidioides immitis (typically in California) and Coccidioides posadasii (typically outside of California); clinical differences between the two species have not been observed (J Am Acad Dermatol 2006;55:929)
Clinical features
  • Cutaneous manifestations are categorized as reactive and organism specific (J Am Acad Dermatol 2006;55:929)
  • Reactive manifestations do not contain visible microorganisms and may exhibit features of erythema nodosum, Sweet syndrome and interstitial granulomatous dermatitis
  • Rarely, patients may exhibit features of erythema multiforme, acute generalized exanthema, secondary to hypersensitivity to systemic infection
  • Organism specific manifestations present with lesions that contain the organism and include secondary cutaneous disease or primary cutaneous disease
  • Secondary cutaneous disease occurs in patients with disseminated disease
  • Clinical appearance is heterogeneous, including papules, nodules, gummas, pustular acneiform lesions, ulcerated and verrucous plaques, scars, abscesses and fistulae (Am J Dermatopathol 2018;40:e41)
  • Primary cutaneous infection results from direct traumatic inoculation of the organism into the skin by an external source and typically manifests as a painless, indurated or verrucous nodule with or without ulceration, typically on an extremity
  • Secondary nodules may arise in a linear lymphatic sporotrichoid distribution (Clin Exp Dermatol 2010;35:e42)
Laboratory
Prognostic factors
  • Immunosuppression is a poor prognostic factor
Case reports
Clinical images

Images hosted on other servers:

Facial infection

Ulcerated should lesion

Red plaque on finger

Disseminated disease

Microscopic (histologic) description
  • Primary cutaneous lesions are typically characterized by coexistence of pseudoepitheliomatous hyperplasia of epidermis and adjacent or admixed acute suppurative inflammation (Am J Dermatopathol 2014;36:531, J Am Acad Dermatol 2006;55:929, Am J Dermatopathol 2018;40:e41)
  • Variable infiltrate of neutrophils, eosinophils, histiocytes, multinucleated giant cells, plasma cells and rarely lymphocytes are present
  • Organisms are typically rare and may require multiple sections to identify
    • Measure up to 100 μm in diameter, with an eosinophilic wall, containing multiple basophilic or pale endospores (up to 5 μm)
  • Spherules are usually present in association with histiocytic neutrophilic infiltrate or rarely within the hyperplastic squamous epithelium
  • Budding is extremely rare
Microscopic (histologic) images

Contributed by Priya Nagarajan, M.D., Ph.D.

Epidermal hyperplasia, inflammation

Large yeast forms

PAS positive yeasts

Positive stains
Negative stains
Sample pathology report
  • Left lower leg, skin shave biopsy:
    • Rare large yeast forms with broad based budding, in association with epidermal hyperplasia and acute inflammation, consistent with cutaneous coccidioidomycosis
Differential diagnosis
Board review style question #1
Which of the following histologic features is typical of cutaneous coccidiodomycosis>

  1. Irregular epidermal hyperplasia, mixed inflammatory infiltrate and rare large spherical fungal forms with eosinophilic wall and pale center
  2. Lobulated, endophytic epidermal hyperplasia, with large pink intracytoplasmic inclusions that compress the nucleus against the cell membrane
  3. Multinucleated histiocytes with angulated nuclei that fit against each other and basophilic chromatin that marginates against the nuclear membrane
  4. Subcorneal cleft with scant inflammatory infiltrate and variable acantholysis
Board review style answer #1
A. Irregular epidermal hyperplasia, mixed inflammatory infiltrate and rare large spherical fungal forms with eosinophilic wall and pale center. B, C and D are characteristic histologic features of molluscum, herpes and staphylococcal scalded skin syndrome.

Comment here

Reference: Coccidioidomycosis
Board review style question #2
Which of the following is the most common primary site of infection in coccidioidomycosis?



  1. Brain
  2. Lung
  3. Mucocutaneous junction
  4. Skin
Board review style answer #2
B. Lung

Comment here

Reference: Coccidioidomycosis

Colloid milium (pending)
[Pending]

Coma blister (pending)

Common terms & patterns
Definition / general
  • This topic represents an overview of common terms and microscopic findings of the most frequently seen reaction pattern in dermatopathology
Terminology
  • Acantholysis:
    • Separation of cell - cell connections (desmosomes) between keratinocytes leading to single, rounded keratinocytes
    • Seen in pemphigus variants and related disorders
  • Acanthosis:
    • Thickening of epidermis
    • Elongated rete ridges usually extend into dermis
    • Can be regular (all rete pegs at roughly the same level) or irregular (rete pegs at different levels of the papillary dermis)
  • Anagen:
    • Hair during growth phase
  • Anaplasia:
    • Atypical nuclei (abnormal in size and shape) and pleomorphism (variation in nuclear characteristics)
  • Asteroid body:
    • Collections of eosinophilic material in sporotrichosis or star shaped inclusion in sarcoidal giant cells / other granulomatous processes
  • Atrophy:
    • Epidermal:
      • Decreased thickness due to reduced numbers of keratinocytes
      • Epidermis may appear flat at its base with loss of rete ridge pattern
      • May occur after inflammation / certain topical treatments
    • Dermal:
      • Thinning of dermal layer due to loss of collagen / elastin
      • May occur after steroids and atrophodermas
  • Banana bodies:
    • Yellow-green, banana shaped fibers in the dermis; can be due to exogenous or endogenous causes
  • Basal vacuolar degeneration:
    • Also known as liquefactive / hydropic degeneration
    • Small vacuoles resembling bubbles in the basal layer
    • Often associated with individually necrotic keratinocytes
    • May or may not have an associated band of infiltrate obscuring the dermoepidermal junction
    • Associated with vacuolar / lichenoid interface dermatitis (e.g., cutaneous lupus, erythema multiforme, lichen planus)
  • Basement membrane:
    • Thin membranous layer of extracellular matrix which is separating the epidermis from the dermis
  • Birefringence:
    • Optical capacity of a material with a refractive index that depends on polarization and propagation of direction of light
  • Blister:
    • Fluid filled skin lesions
    • May result from separation of keratinocytes (intraepidermal blisters) or loss of adhesion between the epidermis and the underlying dermis (subepidermal blisters)
    • Examples of diseases commonly associated with blisters include autoimmune bullous dermatoses, interface dermatitis, spongiotic dermatitis and some viral and bacterial infections
  • Bullae:
    • Fluid filled lesion > 1 cm (= large blister)
  • Calcinosis:
    • Cutaneous deposition of insoluble calcium in response to damage, metabolic derangements or undetermined etiology
  • Catagen:
    • Hair during regressive phase
  • Caterpillar bodies:
    • Eosinophilic bodies present within the roof of the blister formed in porphyric bullous eruptions, such as porphyria cutanea tarda
  • Civatte / colloid bodies:
    • Globular residue of apoptotic keratinocytes
    • Typically pink in color
    • Referred to as Civatte bodies when present within the epidermis and as colloid bodies when in the papillary dermis
    • Classically described in lichen planus but may be seen in all causes of interface / lichenoid dermatitis
  • Comedo:
    • Dilated follicular infundibulum filled with keratin, lipid (not visible with standard stains) and often basophilic debris
    • Bacterial colonies may be present
    • Classical lesion in acne vulgaris
  • Cornoid lamellae:
    • Layers of parakeratosis often in a ~45° angle overlying an area of diminished granular layer with dyskeratotic cells present at the base
    • Classically seen in porokeratosis variants
  • Cowdry bodies:
    • Cowdry A: intranuclear inclusion in herpes simplex virus
    • Cowdry B: intranuclear inclusion in cytomegalovirus, adenovirus, poliovirus
  • Crust:
    • Hyperkeratosis (often parakeratosis) with inspissated serum with or without Inflammatory cells (= wet scale)
  • Cyst:
    • Encapsulated cavity or sac lined by true epithelium
  • DEJ:
    • Dermoepidermal junction
  • Dermal papilla (hair):
    • Layer that is located at the base of the hair follicle
  • Desmosome:
    • Adhesion molecule between two cells
  • Diffuse infiltrate:
    • sheets of inflammatory ells diffusely infiltrating dermis
  • Donovan body:
    • Rod shaped, oval organisms within histiocytes in granuloma inguinale
    • Characteristic bipolar staining pattern gives safety pin appearance
    • Highlighted by Warthin-Starry and Giemsa stains
  • Dutcher body:
    • Spherical, intracytoplasmic immunoglobulin inclusions that appear intranuclear due to folding into the nucleus of plasma cells
    • Seen in plasmacytoma
  • Dyskeratosis:
    • Abnormal, premature keratinization of keratinocytes below granular cell layer
    • Often have brightly eosinophilic cytoplasm with or without a pyknotic nucleus
  • Effacement:
    • Synonymous with consumption of the epidermis
    • Refers to thinning of the epidermis / loss of rete ridges
    • Commonly seen in melanoma
  • Elastolysis:
    • Defect in elastic fibers leading to atrophy, skin laxity
  • Elastorrhexis:
    • Fragmentation of elastic fibers; systematic elastorrhexis seen in pseudoxanthoma elasticum
  • Elastosis:
    • Degenerative changes in elastic fibers (increase in abnormal elastin) leading to loss of elastic quality of the dermis
  • Epidermolysis:
    • Alteration of granular layer with perinuclear clear spaces, swollen and irregular keratohyalin granules, increased thickness of granular layer
    • Different from acantholysis
    • Classically seen in epidermolytic ichthyosis and keratodermas but may be seen in epidermal nevi, isolated keratoses and incidentally
  • Epidermotropism:
    • Atypical individual cells present in epidermis without significant spongiosis
    • Most commonly refers to atypical lymphocytes in cutaneous T cell lymphomas but can be seen in other malignant neoplasms, including melanoma and metastatic deposits
  • Erosion:
    • Discontinuity of skin causing partial loss of epidermis
  • Excoriation:
    • Deep scratch with abrupt loss of epidermis without full thickness loss at an ulcer
    • Excoriation may result in partial or full thickness epidermal loss (erosion versus ulcer)
    • Often self induced
  • Exocytosis:
    • Inflammatory cells migrating into the epidermis (commonly seen in spongiotic dermatitis)
    • Lymphocytes, neutrophils and eosinophils most commonly seen
  • Festooning:
    • Undulating pattern of the dermal papillae
    • Commonly seen underneath bullae / blisters (e.g. in porphyria cutanea tarda / subepidermal autoimmune bullous diseases)
  • Fibrinoid necrosis:
    • Necrosis associated with vascular damage
  • Fibrosis:
    • Increased amount of fibroblasts forming a scar or skin thickening
  • Flame figure:
    • Deposition of eosinophil granules from degranulated eosinophils and nuclear debris on dermal collagen fibers with or without surrounding histiocytes
    • Classically seen in Wells syndrome (eosinophilic cellulitis) but may be seen in a variety of eosinophil rich diseases
  • Foam cell:
    • Histiocyte whose cytoplasm is replaced by variably sized lipid vacuoles
    • Commonly seen in xanthoma variants, lipidized dermatofibromas and some histiocytoses
  • Follicular plugging:
    • Process of abundant hyperkeratotic material plugging up follicular ostia
  • Globi:
    • Amphophilic collection of mycobacteria, classically seen in leprosy
  • Granuloma:
    • Collection of macrophages forming a cluster / round structure in response to certain types of inflammation (foreign material, microorganisms, etc.)
  • Grenz zone:
    • Thin layer of uninvolved dermis directly beneath the epidermis, separating it from an inflammatory cell infiltrate or tumor
    • Grenz zones classically seen in granuloma faciale, leukemia cutis, dermatofibromas, lepromatous leprosy and solar elastosis
  • Hemidesmosome:
    • Adhesion molecule between cell and basal lamina
  • Henderson-Patterson body:
    • Cytoplasmic viral inclusion bodies often filling cup shaped cavity space in molluscum contagiosum (molluscum bodies)
  • Horn (cutaneous horn):
    • Massive vertically oriented hyperkeratosis (parakeratotic, orthokeratotic or mixed) most commonly overlying a keratinocytic neoplasm
    • Importance lies with the lesion which underlies the horn which may be malignant in a significant minority
  • Horn cyst:
    • Hyperkeratotic epithelial structure presumably due to abrupt complete keratinization
  • Hypergranulosis:
    • Thickening of the stratum corneum
  • Hyperkeratosis:
    • Thickened cornified layer (stratum corneum)
    • Keratin may be abnormal
    • Either orthokeratotic (typically compact keratin with no nuclei) or parakeratotic (nuclei in cornified layer)
  • Inferior follicle:
    • Inferior segment of the follicle from the bulge to the base of the follicle
  • Infundibulum:
    • Upper segment of the hair follicle
  • Interface dermatitis:
    • Pattern of inflammatory dermatitis characterized by basal layer vacuolation with or without a band of inflammation obscuring the dermoepidermal junction
    • When band-like lymphocytic infitlrate present, referred to as lichenoid interface or lichenoid dermatitis
  • Isthmus:
    • Segment between the sebaceous gland duct opening and the hair bulge
  • Kamino bodies:
    • Eosinophilic bodies associated with spitz nevi
  • Koilocyte:
    • Keratinocyte that underwent structural changes due to viral infection
  • Lentiginous:
    • Linear pattern of melanocytic proliferation within epidermal basal cell layer
  • Leukocytoclasis:
    • Vascular karyorrhexis of degenerating neutrophils is a feature of neutrophilic vasculitis (also called leukocytoclastic vasculitis)
  • Leukocytoclastic vasculitis:
    • Inflammatory reaction of blood vessels including fibrinoid changes within the vessel walls
  • Lichenification:
    • Thick, rough skin with prominent skin markings usually due to repeated rubbing
    • Associated with hyperkeratosis
  • Lichenoid dermatitis:
    • Inflammatory pattern characterized by basal vacuolar change and a band of inflammatory cells at the dermo - epidermal junction
  • Lichenoid infiltrate:
    • Band-like lymphocytic infiltrate at the dermoepidermal junction
  • Macule:
    • Flat lesion less than 1.0 cm
  • Medlar body:
    • Also known as sclerotic bodies
    • Brown, round fungal cell in between yeast and hyphal state with the appearance of copper pennies
    • Diagnostic for chromoblastomycosis
  • Michaelis-Gutmann body:
    • Round, calcified structures located within the cytoplasm of histiocytes in malakoplakia
    • Stain with Perls stain for iron and von Kossa for calcium
  • Mikulicz cells:
    • Large foamy histiocytes containing rods of klebsiella rhinosclermomatis in rhinoscleroma
  • Munro microabscess:
    • Neutrophilic collections in the stratum corneum
    • Classically seen in psoriasis
  • Necrobiosis:
    • Altered / necrotic collagen
    • Seen in granuloma annulare and other necrobiotic lesions
  • Nevus:
    • Often used to refer to a benign tumor of melanocytes but may also refer to a hamartomatous lesion of any component of the skin (epidermis, dermis, blood vessels etc.)
  • Nodular inflammation:
    • Inflammatory cells forming nodular collections
  • Nodule:
    • Solid lesion (> 1 cm) whose vertical dimension is greater than its horizontal
  • Oncholysis:
    • Separation of the nail plate from the underlying nail bed
    • Associated with psoriasis, trauma, onychomycosis, thyrotoxicosis and phototoxic drug ingestion
  • Owl eye inclusion:
    • Term used to describe a cells infected by cytomegalovirus
  • Pagetoid spread:
    • Single malignant cells involving the epidermis
  • Panniculitis:
    • Lobular:
      • Inflammation of fat involving predominantly the fat lobules (e.g., erythema induratum, pancreatic panniculitis, etc.)
    • Septal:
      • Inflammation of fat involving predominantly the septae in between the fat lobules (e.g., erythema nodosum)
  • Papillary dermal edema:
    • Extracellular accumulation of fluid leading to thinning of the dermis
  • Papillary mesenchymal body:
    • Structures associated with follicular differentiation; can serve as helpful clue in follicular type adnexal neoplasms, such as trichoepithelioma / trichoblastoma
  • Papillomatosis:
    • Finger-like undulation of the epidermis with upward expansion of dermal papillae
    • May have associated epidermal hyperplasia
  • Papule:
    • Raised lesion < 1 cm
  • Parakeratosis:
    • Retention of nuclei in the stratum corneum
    • Normal for mucous membranes without stratum granulosum
  • Parasitized macrophages:
    • Macrophages infected by parasites present within their cytoplasm (e.g., in leishmaniasis)
  • Patch:
    • Flat lesion > 1 cm
  • Pautrier microabscess:
    • Collections of atypial lymphocytes involving the epidermis; seen in mycosis fungoides
  • Periadnexal inflammation:
    • inflammatory cells predominantly surrounding the adnexal structures
  • Perivascular inflammation:
    • Inflammatory cells predominantly surrounding vasculature
  • Plaque:
    • Raised, relatively flat topped lesion > 1 cm
  • Poikiloderma:
    • Combination of atrophy, telangiectasia and pigmentary changes
  • Pseudoepitheliomatous hyperplasia:
    • Exaggerated acanthosis of the epidermis mimicking squamous cell carcinoma
    • Commonly seen in the epidermis overlying prior procedure sites, chronic inflammation, granular cell tumor, blastomycosis
  • Psoriasiform epidermal hyperplasia:
    • Acanthosis of the epidermis with regular elongation the rete ridges
  • Purpura:
    • Extravasation of red blood cells into the skin or mucous membranes with or without inflammation
  • Pustule:
    • Intraepidermal or subepidermal vesicle or bullae filled with neutrophils
  • Scale:
    • Dry, horny, plate-like excrescence corresponding to hyperkeratosis
    • Usually due to imperfect cornification
  • Schaumann body:
    • Laminated calcified structure
    • Seen in sarcoidosis but rare in skin lesions
  • Sclerosis:
    • Excess connective tissue with decreased amount of fibroblasts
  • Shadow cells:
    • Cells without a nucleus due to prior cell death
  • Sinus:
    • Tract connecting cavities to each other or to the surface
  • Spongiform pustule of Kogoj:
    • Collection of neutrophils within the stratum spinosum
    • Associated with spongiosis at the periphery (e.g. seen in psoriasis)
    • Seen in psoriasis, including pustular variants
  • Spongiosis:
    • Intraepidermal edema causing splaying apart of keratinocytes in stratum spinosum (resembling a sponge)
    • Intercellular adhesion proteins (desmosomes) often visible
    • Vesicles due to shearing of desmosomes
  • Squamous eddies:
    • Whorled profiles composed of keratinocytes (e.g. seen in inverted follicular keratoses)
  • Storiform:
    • Growth pattern consisting of spindle cells arranged in a cartwheel-like pattern
    • Classically seen in dermatofibrosarcoma protuberans (DFSP)
  • Telogen:
    • Hair during resting phase
  • Ulceration:
    • Discontinuity of skin causing complete loss of epidermis and possible loss of dermis with accompanying surface fibrin deposition or exudate with or without granulation tissue response
  • Verocay body:
    • Structure composed of 2 nuclear palisades with central eosinophilic cytoplasmic processes
    • Classically seen in schwannoma
  • Vesicle:
    • Fluid filled lesion < 1 cm
  • Von Hansemann cells:
    • Large macrophages containing Michaelis-Gutmann bodies seen in malakoplakia
  • Wheal:
    • Itchy, transient, elevated area with variable blanching and erythema
    • Due to dermal edema
Granulomatous dermatitis
  • Similar pattern seen in various clinical settings
    • Sarcoidosis
    • Necrobiosis lipoidica (associated with diabetes mellitus)
    • Granuloma annulare (localized, generalized, etc.)
    • Palisaded neutrophilic and granulomatous dermatitis
    • Granulomatous reaction to foreign material

Contributed by Carina Dehner, M.D., Ph.D.
Perivascular and interstitial inflammation

Perivascular and interstitial inflammation

Infections
  • Bacterial
    • Impetigo:
      • Nonbullous or bullous forms
      • Gram positive cocci are the trigger
    • Staphylococcus scalded skin syndrome:
      • Sterile subcorneal split of epidermis
      • Driven by toxins from certain Staphylococcus aureus strains
    • Rhinoscleroma:
      • Diffuse inflammatory infiltrate underlying submucosa, Mikulicz cells present
      • Trigger: Klebsiella pneumoniae
    • Erythrasma:
      • Erythematous intertriginous patches
      • Triggered by Corynebacterium minutissimum infection
  • Viral
    • Herpes virus:
      • Multinucleated keratinocytes with chromatin margination and molding
      • Mucosal sites (HSV1 / 2) or dermatome distribution (varicella zoster virus)
    • Epstein-Barr virus (EBV):
      • Infectious mononucleosis, oral hairy leukoplakia, Gianotti-Crosti syndrome
      • EBV virus can be detected by in situ hybridization
    • Cytomegalovirus:
      • Large, intranuclear inclusions with halo
    • Molluscum contagiosum:
      • Molluscum bodies
      • Trigger: poxvirus
  • Spirochetes
    • Syphilis:
      • Psoriasiform hyperplasia, mixed pattern dermatitis
      • Treponema pallidum
  • Fungal
    • Dermatophytosis:
      • Neutrophils in stratum corneum sandwich sign: parakeratosis or compact orthokeratosis underlying basket weave type stratum corneum
    • Majocchi granuloma:
      • T. rubrum
      • Fungi surrounding degenerate hair shafts
    • Pityriasis (tinea) versicolor:
      • Malassezia
      • Need to see spores and hyphae in stratum corneum
    • Blastomycosis:
      • May induce pseudoepitheliomatous hyperplasia
  • Arthropods / parasites
    • Demodex:
      • Commonly seen, mostly incidental
      • If exuberant, can induce pathologic changes leading to dry eye, chalazion, etc.
    • Scabies:
      • Potassium hydroxide preparation (KOH prep) can be done on bed site
      • Itchy for years

Contributed by Carina Dehner, M.D., Ph.D.
Eggshells of scabies

Eggshells of scabies

Mixed pattern dermatitis

Mixed pattern dermatitis

Lichenoid dermatitis
  • Lichenoid dermatitis, with or without interface
    • Band-like inflammation and basal vacuolar degeneration (lichen planus, etc.)
  • Interface changes without lichenoid inflammation (discoid lupus, erythema multiforme)
    • Basal vacuolar degeneration and dyskeratotic keratinocytes only (erythema multiforme)
    • Basal vacuolar degeneration and dyskeratotic keratinocytes at the dermoepidermal junction + superficial and deep perivascular, periadnexal inflammation and increased dermal mucin (discoid lupus erythematosus)

Contributed by Carina Dehner, M.D., Ph.D.
Lichenoid interface dermatitis

Lichenoid interface dermatitis

Panniculitides
  • Septal panniculitis
    • Erythema nodosum
  • Lobular panniculitis: diverse group associated with rheumatoid arthritis, gout (Arch Dermatol 1998;134:501)
    • Pancreatic panniculitis: lobular fat necrosis with saponification and ghost cells
    • Erythema induratum: prominent fat necrosis, histiocytes, giant cells, granulomas, vasculitis
    • Lupus panniculitis: may show lymphoid follicles
    • Sclerosing panniculitis (lipodermatosclerosis)

Contributed by Carina Dehner, M.D., Ph.D.
Saponification and ghost cells

Saponification and ghost cells

Perivascular dermatitis
  • Wide spectrum of diagnoses
    • Superficial: dermal hypersensitivity with or without eosinophils, urticaria
    • Superficial and deep: arthropod bite, connective tissue diseases
Psoriasiform dermatitis
  • Psoriasiform hyperplasia (e.g. psoriasis, lichen simplex chronicus, pityriasis rubra pilaris, etc.)

Contributed by Carina Dehner, M.D., Ph.D.
Psoriasiform hyperplasia

Psoriasiform hyperplasia

Alternating parakeratosis and orthokeratosis

Alternating parakeratosis and orthokeratosis

Sclerosing dermatitis
  • Localized scleroderma (morphea)
  • Guttate morphea
  • Systemic sclerosis

Contributed by Carina Dehner, M.D., Ph.D.
Sclerosis of dermal collagen

Sclerosis of dermal collagen

Spongiotic dermatitis
  • Spongiosis: accumulation of fluid leading to intercellular edema in the epidermis
  • Acute: wet scale + extreme spongiosis (= intraepidermal vesicles) + eosinophils in the inflammatory infiltrate
  • Subacute: hyperkeratosis with little parakeratosis + mild acanthosis + mild spongiosis
  • Chronic: mild hyperkeratosis + marked acanthosis + minor spongiosis

Contributed by Carina Dehner, M.D., Ph.D.
Spongiosis

Spongiosis

Vasculopathic / vasculitic dermatitis
  • Leukocytoclastic vasculitis: associated with infections, drugs, autoimmune diseases, etc.
    • Granuloma faciale: form of leukocytoclastic vasculitis, Grenz zone
    • Erythema elevatum diutinum
  • Occlusive vasculopathy: fibrin thrombi filling venules and capillaries

Contributed by Carina Dehner, M.D., Ph.D.
Fibrinoid material within vessels

Fibrinoid material within vessels

Vesiculobullous dermatitis
  • Subcorneal split:
    • Pemphigus foliaceus:
      • Intercellular IgG, C3 on direct immunofluorescence (DIF)
      • Sparing of basal layer
    • Acute generalized exanthematous pustulosis (AGEP):
      • Neutrophils, eosinophils in dermal infiltrate
      • Negative DIF
  • Suprabasal split:
    • Pemphigus vegetans:
      • Neutrophilic abscesses
      • Intercellular IgG, C3 on DIF
    • Pemphigus vulgaris:
      • Eosinophils, neutrophils, tombstone appearance
      • Intercellular IgG, C3 on DIF
    • Darier disease:
      • Acantholysis and dyskeratosis
      • Negative DIF
  • Intraepidermal split:
    • Bullous arthropod bite:
      • Spongiotic edema, eosinophils
      • Negative DIF
  • Subepidermal split:
    • Dermatitis herpetiformis:
      • Neutrophils in the dermal papillae
      • Granular IgA on DIF
    • Porphyria cutanea tarda:
      • Cell poor split
      • IgG around superficial dermal vessels on DIF
    • Bullous pemphigoid:
      • Eosinophil rich
      • Linear IgG and C3 at the dermoepidermal junction on DIF
    • Linear IgA bullous dermatosis:
      • Neutrophil rich
      • Linear IgA at the dermoepidermal junction on DIF

Contributed by Carina Dehner, M.D., Ph.D.
Subcorneal acantholysis

Subcorneal acantholysis

Acantholytic blister with tombstone

Acantholytic blister with tombstone

Board review style question #1

What is a Majocchi granuloma?

  1. Deeper fungal infection in which fungi surround degenerate hair shafts
  2. Necrotizing granuloma seen in infectious diseases, such as tuberculosis
  3. Nonnecrotizing granuloma seen in diseases such as sarcoidosis
  4. Palisaded histiocytic granulomatous process with central fibrin, seen in rheumatoid nodule
  5. Palisaded histiocytic granulomatous process with central mucin, seen in granuloma annulare
Board review style answer #1
A. Majocchi granuloma is a deep fungal infection involving hair follicles caused by dermatophytes, such as T. rubrum

Comment Here

Reference: Common terms & patterns

Corynebacteria (pending)

Cryptococcus (pending)
[Pending]

Cutaneous larva migrans
Definition / general
  • Localized curvilinear and pruritic lesions collectively identified clinically as cutaneous larva migrans
  • Such infestations, most commonly due to hookworms, occur because humans are dead end hosts of penetrating nematode larvae
Etiology
  • Ova of helminths are deposited in sand and soil in warm and shady areas, where larvae hatch
  • Activities such as walking barefoot on the beach, playing in the sand and crawling under houses allow larvae to penetrate human skin
  • Hookworms such as Ancylostoma braziliense (dog and cat hookworms) and Ancylostoma caninum (dog hookworms) are most common causes of cutaneous larva migrans
  • Many other penetrating nematode larvae cause cutaneous larva migrans, such as:
    • Uncinaria stenocephala (hookworm of European dogs)
    • Bunostomum phlebotomum (hookworm of cattle)
    • Guinea worm (dracunculiasis)
    • Liver flukes (fascioliasis)
    • Genus Gnathostoma (gnathostomiasis)
    • Hookworms
    • African eye worm (loiasis)
    • Paragonimus westermani (paragonimiasis)
    • Spirometrosis (sparganosis)
    • Strongyloides stercoralis (strongyloidiasis)
  • Rarely fly larvae (maggots) can cause myiasis, which can also cause migratory dermatologic lesions
Epidemiology
  • Seen in U.S. travelers returning home after holidays
  • Endemic in subtropical areas, such as southeast U.S., Carribbean, Africa, Central America and Southeast Asia
Sites
  • Exposed distal portions of upper / lower limbs and buttocks are most commonly affected anatomic sites
Pathophysiology
  • Localized dermal inflammatory reaction including eosinophilic infiltrate and edema
  • Sometimes larvae move deeper into subcutaneous fat
Etiology
  • Helminth infestation
Clinical features
  • Most common finding is localized pruritis; however, some helminths can cause secondary systemic symptoms, as larvae enter blood vessels and migrate to intestinal mucosa (example, Strongyloidiasis)
  • Helminths may also cause visceral larva migrans, such as due to Toxocara canis, Toxocara cati, Ascaris lumbricoides, whereby larvae enter via tubular GI and secondarily cause disseminated visceral and cutaneous findings
Diagnosis
  • Clinicopathologic findings
Laboratory
  • Peripheral eosinophilia
  • Skin biopsy from advancing point of the lesion can show parts of parasite(s)
Treatment
  • Avoid direct skin contact with contaminated soil
  • Topical glucocorticoid application under occlusion for symptomatic relief
  • Antihelmintic agents such as thiabendazole, ivermectin, albendazole for topical or systemic treatment
Clinical images

Images hosted on other servers:

Various images

Gross description
  • Lesions show surrounding erythema and have serpiginous shape and tunnel-like contour
Microscopic (histologic) description
  • Curvilinear eosinophilic larvae within the epidermis
Microscopic (histologic) images

Images hosted on other servers:

Various images


Cutaneous vasculitis
Definition / general
  • Vasculitides are inflammatory processes that can affect capillaries, venules and small, medium and large sized vessels (Curr Opin Rheumatol 2019;31:46, An Bras Dermatol 2020;95:355)
  • Cutaneous vasculitis can present in several forms: a component of systemic vasculitides, a limited expression of systemic vasculitis or a single organ vasculitis (Arthritis Rheumatol 2018;70:171)
  • Many classifications have been proposed; however, the most accepted one is the Chapel Hill consensus conference which integrates the clinical presentation, histopathology and lab findings; it also classifies vasculitis based on the size of the affected vessel in small, medium and large vessel vasculitis (Arthritis Rheum 2013;65:1)
Essential features
  • Vasculitis represents inflammation of the capillaries, postcapillary venules and small, medium and large sized vessels
  • Variable clinical presentation (purpura, erythema, papules, ulcer, etc.)
  • Majority of cutaneous vasculitides are self limited and have an excellent prognosis
  • Etiology of cutaneous vasculitis is multifactorial but generally presents as a result of an immune hypersensitivity reaction, associated with direct damage of the vessel wall and inflammation
  • Clinical presentation, skin biopsy and direct immunofluorescence are crucial to establish a definitive diagnosis
  • Treatment varies depending on the nature of the disease and includes symptomatic treatment (self limited conditions), steroids or immunomodulators
Terminology
  • Vasculitides
ICD coding
  • ICD-10:
    • D89.1 - cryoglobulinemia
    • L95 - vasculitis limited to skin, not elsewhere classified
    • L95.1 - erythema elevatum diutinum
    • M30.1 - polyarteritis with lung involvement (Churg-Strauss)
    • M31.7 - microscopic polyangiitis
    • M31.9 - necrotizing vasculopathy, unspecified
    • M35.2 - Behçet disease
  • ICD-11:
    • 4A44.A1 - granulomatosis with polyangiitis
Epidemiology
  • Cutaneous vasculitides affect both genders equally and patients of all ages
  • Presence of an underlying systemic vasculitis, connective tissue disease or malignancy is much more common in adults than in children (Mayo Clin Proc 2014;89:1515)
  • Overall annual incidence of cutaneous vasculitis is ~38.6 per million (J Rheumatol 1998;25:920)
Sites
  • Systemic vasculitides can involve cranial or peripheral nerves, gastrointestinal tract, retina, aorta, kidneys, oral mucosa and skin, among other organs (Circulation 2021;143:267)
  • Most types of vasculitis affect the lower extremities (Pathologe 2020;41:355)
Pathophysiology / etiology
  • Small vessel vasculitis:
  • Medium vessel vasculitis:
  • Large vessel vasculitis:
    • Giant cell arteritis:
    • Takayasu arteritis:
      • Unknown pathogenesis; is thought to be a combination of immune mediated process and genetic predisposition
      • Granulomatous inflammation of the media and adventitia (Heart 2018;104:558)
    • Behçet disease:
      • Affects arteries and veins
      • Recurrent oral or genital aphthous ulcers accompanied by cutaneous, ocular, articular, gastrointestinal or central nervous system inflammatory lesions
      • Leukocytoclastic vasculitis of small vessels affecting mostly venules
    • Cogan syndrome:
      • Arteritis and aortitis
      • May present with aortic aneurysm or mitral valvulitis
      • Vasculitis of small arteries in the panniculus and dermosubcutaneous junction
    • Vasculitis associated with systemic diseases (systemic lupus erythematosus, rheumatoid arthritis, sarcoidosis):
      • Vasculitis of small or medium sized vessels
      • Presents as a component of the systemic condition
      • Present with cutaneous leukocytoclastic vasculitis
    • Vasculitis associated with probable etiology:
      • Caused by drugs, infections (sepsis) or autoimmune conditions such as Sjögren syndrome
      • Present with cutaneous leukocytoclastic vasculitis
    • Nodular vasculitis:
      • Vasculitis with lobular panniculitis of mostly postcapillary venules
    • Erythema elevatum diutinum:
      • Neutrophilic dermatosis with chronic localized fibrosing leukocytoclastic vasculitis
    • Granuloma faciale:
      • Chronic, progressive, inflammatory dermatosis
      • Characterized by asymptomatic papules, nodules or plaques of red-brown to violaceous color
      • Perivascular inflammatory infiltrates in the dermis with leukocytoclasis
      • Necrotizing vasculitis is rare (StatPearls: Granuloma Faciale [Accessed 31 January 2023])
    • Palisaded neutrophilic and granulomatous dermatitis:
    • Hypergammaglobulinemic purpura of Waldenström:
Clinical features
  • Skin is the most frequently involved organ in vasculitides and is usually the first organ to reveal signs of systemic disease; skin involvement by vasculitides may lead to significant morbidity and mortality (Z Rheumatol 2013;72:436, Curr Opin Rheumatol 2022;34:25)
  • Small vessel vasculitis:
    • Cutaneous leukocytoclastic vasculitis:
      • Presents as petechiae, confluent purpura, urticarial hives or deep ulcers and nodules
      • Mainly affects the lower extremities and back (Intern Emerg Med 2021;16:831)
    • Immunoglobulin A vasculitis (Henoch-Schönlein purpura):
    • Cryoglobulinemic vasculitis:
      • Most patients are asymptomatic
      • Symptomatic patients present with ulcers, gangrene, urticaria, livedo reticularis, purpura, retinal hemorrhages and neurologic disturbances (J Autoimmun 2019;105:102313)
    • Hypocomplementemic urticarial vasculitis (anti-C1q vasculitis):
      • Leukocytoclastic vasculitis, severe angioedema, laryngeal edema, pulmonary involvement, arthritis, arthralgia, glomerulonephritis and uveitis (Curr Rheumatol Rep 2009;11:410)
    • Antineutrophil cytoplasmic antibody (ANCA) associated vasculitis:
      • Rhinosinusitis, arthralgia, dyspnea, cough, hemoptysis, purpura, renal involvement, neurologic dysfunction (Eur J Intern Med 2020;74:18)
    • Microscopic polyangiitis:
    • Eosinophilic granulomatosis with polyangiitis (Churg-Strauss syndrome):
      • Constitutional symptoms, maxillary sinusitis, asthma, arthritis, peripheral neuropathy, focal or crescentic glomerulonephritis (less frequent), purpura mainly in lower extremities (Semin Respir Crit Care Med 2018;39:471)
    • Granulomatosis with polyangiitis (Wegener granulomatosis):
  • Medium vessel vasculitis:
    • Polyarteritis nodosa:
      • Constitutional symptoms, myalgia, arthralgia, gastrointestinal and skin involvement (Curr Opin Pediatr 2022;34:229)
      • Cutaneous polyarteritis nodosa is a variant of polyarteritis nodosa confined to the skin, muscle and peripheral nerves in the absence of systemic involvement (Clin Exp Dermatol 1983;8:47)
    • Kawasaki disease:
      • Oral mucosa erythema and cracked lips, strawberry tongue, conjunctivitis, polymorphous rash, extremity changes, lymphadenopathy (Curr Rheumatol Rep 2020;22:75)
  • Large vessel vasculitis:
    • Giant cell arteritis:
      • New onset headaches, scalp tenderness, jaw claudication, fever, fatigue, polymyalgia and vision loss (Neurol Clin 2019;37:335)
    • Takayasu arteritis:
      • Arterial stenosis, aneurysms and occlusion involving the ascending and descending aorta, renovascular hypertension, lower extremity claudication, arthralgia, myalgia and skin nodules (Heart 2018;104:558, Dermatologica 1990;181:266)
    • Behçet disease:
      • Recurrent aphthous stomatitis, genital aphthous ulcers, erythema nodosum-like lesions, papulopustular lesions, ocular manifestations, thrombophlebitis, pericarditis, myocarditis, arthralgia, fibromyalgia, neurologic and gastrointestinal manifestations (Clin Dermatol 2017;35:421)
    • Cogan syndrome:
    • Vasculitis in systemic lupus erythematosus:
      • Petechiae, palpable purpura, papulonodular lesions, livedo reticularis and ulcers are associated with myocarditis, serositis and Raynaud phenomenon (Clin Dermatol 2004;22:148)
    • Rheumatoid arthritis associated vasculitis:
      • Purpura, ulcers, gangrene, mononeuritis multiplex, episcleritis, scleritis, myocarditis, pericarditis, lung, kidney, gastrointestinal and central nervous system involvement (Curr Opin Rheumatol 2015;27:63)
    • Sarcoid vasculitis:
      • Constitutional symptoms, peripheral adenopathy, rash, scleritis, iridocyclitis, musculoskeletal and pulmonary parenchymal involvement (Semin Arthritis Rheum 2000;30:33)
Diagnosis
  • Proper diagnosis requires a combination of clinical history and physical examination to assess organ involvement (systemic disease)
  • Obtain a complete blood count, complete metabolic panel and urinalysis
  • Specific tests depending on the etiology include infectious disease serology (hepatitis C, B, HIV), immunoglobulin levels, C3 and C4 complement levels, antinuclear antibody, rheumatoid factor, antineutrophilic cytoplasmic antibodies, cryoglobulins and anti-CPP, amongst others
  • Skin biopsy remains the cardinal step in confirming diagnosis of cutaneous vasculitis; direct immunofluorescence contributes to an accurate diagnosis and differentiates it from pseudovasculitic disorders (Am J Clin Pathol 2005;124:S84, Am J Dermatopathol 2005;27:504, Curr Opin Rheumatol 2022;34:25, Am J Clin Dermatol 2008;9:71)
  • In cases of systemic vasculitis, a kidney or nerve biopsy can provide additional diagnostic elements
  • Complementary laboratory exams, including workup for infectious, autoimmune disorders or malignancy, may be needed (An Bras Dermatol 2020;95:355)
Laboratory
Prognostic factors
Case reports
  • 4 year old boy presented with pruritic, nonblanching maculopapular rash in association with COVID-19 infection (BMJ Case Rep 2021;14:e239910)
  • 7 year old sisters presented sequentially with erythematous nodules in trunk and lower extremities associated with pain (Mod Rheumatol 2018;28:1049)
  • 16 year old girl presented with fever, myalgia, diarrhea and an erythematous maculopapular rash in abdomen and arms (BMC Gastroenterol 2020;20:352)
  • 17 year old girl without significant past medical history presented with arthralgia, facial and lower extremity edema (Lupus 2014;23:1426)
  • 78 year old women presented with recalcitrant skin ulcer in her right ankle (J Dermatol 2018;45:1009)
Treatment
Clinical images

Contributed by Jose A. Plaza, M.D.

Polyarteritis arteritis nodosa

Leukocytoclastic vasculitis

Urticarial vasculitis

Microscopic (histologic) description
  • Cutaneous vasculitis:
    • Small vessel cutaneous vasculitis affects arterioles, capillaries and postcapillary venules in the superficial and mid dermis
    • Medium vessel cutaneous vasculitis involves small arteries and veins within the deep dermis and subcutis
    • Timing and proper sampling is key to yield a definitive diagnosis
    • Ideally, a skin biopsy should be made within 24 - 48 hours of appearance
    • Classic histopathologic feature is leukocytoclastic vasculitis:
      • Swelling of the vessel wall (primarily postcapillary venules)
      • Transmural infiltration of neutrophils
      • Degranulation and fragmentation of neutrophils creating nuclear dust
      • Fibrinoid necrosis and extravasation of red blood cells
      • Signs of endothelial damage
      • When lesions have more than 48 - 72 hours are predominantly infiltrated by mononuclear cells instead of neutrophils (Bolognia: Dermatology, 4th Edition, 2017)
  • Small vessel vasculitis:
    • Cutaneous leukocytoclastic vasculitis:
      • Fibrinoid necrosis / intraluminal fibrin deposition
      • Disruption of the vessel wall with red cell extravasation
      • Leukocytoclasia or nuclear dust
      • Eccrine gland necrosis
      • Granulomas may or may not be present depending on the underlying entity (J Autoimmun 2022;127:102783)
      • Direct immunofluorescencecan show deposits of IgM, IgG, IgA and C3 along the dermoepidermal junction and the superficial vasculature (Clin Dermatol 2022;40:639)
    • IgM / IgG vasculitis:
      • Leukocytoclastic vasculitis with IgM / IgG deposition
    • IgA vasculitis (Henoch-Schönlein purpura):
      • Leukocytoclasia and IgA deposition
    • Cryoglobulinemic vasculitis:
      • Leukocytoclastic vasculitis, vessels obstruction and ischemia (type I)
      • Mixed cryoglobulinemia (type II and III) is due to immunocomplex deposition
      • Cold urticaria, palpable purpura, livedo reticularis and Raynaud phenomenon are part of the cutaneous manifestations
      • Diffuse proliferative glomerulonephritis (J Autoimmun 2019;105:102313)
    • Hypocomplementemic urticarial vasculitis:
      • Leukocytoclasia and anti-C1q antibodies deposition
    • Microscopic polyangiitis:
    • Eosinophilic granulomatosis with polyangiitis:
    • Granulomatosis with polyangiitis:
    • Urticarial vasculitis:
      • Most often affects postcapillary venules in the dermis
      • Leukocytoclastic vasculitis with endothelial swelling, karyorrhexis of neutrophils, fibrin deposition in the vessel wall, luminal occlusion and extravasation of erythrocytes in the dermis (Immunol Allergy Clin North Am 2014;34:141)
      • Perivascular infiltrate comprised mainly of neutrophils, eosinophils or lymphocytes (Int J Womens Dermatol 2021;7:290)
      • Inflammatory infiltrate tends to shift with time, from predominantly neutrophils to primarily lymphocytic (Am J Dermatopathol 2020;42:399)
  • Medium vessel vasculitis:
    • Polyarteritis nodosa (PAN):
    • Kawasaki disease:
      • Acute process is expressed as nongranulomatous, necrotizing arteritis with neutrophilic infiltration
      • Subacute process presents with asynchronous infiltration of lymphocytes, plasma cells and eosinophils
      • Chronic process is comprised of a luminal myofibroblast proliferation associated with chronic inflammation
      • Progressive arterial stenosis (Circulation 2017;135:e927)
  • Large vessel vasculitis:
    • Large vessel vasculitis:
      • Affects the aorta and branches of major vessels
      • Segmental granulomatous involvement with prominent inflammation of the intima and media
      • Fibrosis of the adventitia (Vasc Med 2019;24:79)
    • Giant cell arteritis:
    • Takayasu arteritis:
      • Granulomatous inflammation of the media and adventitia
      • Infiltration of lymphocytes, macrophages, plasma cells and giant cells
      • Fibrosis with scarring leading to stenosis (Heart 2018;104:558)
    • Behçet disease:
      • Fibrinoid necrosis
      • Endothelial swelling
      • Significant neutrophilic and mononuclear infiltration around dermal vessels
      • Mucocutaneous lesions show leukocytoclasia, lymphocytic infiltration, immunoglobulin and complement deposition with subsequent liquefactive degeneration at the at the dermoepidermal junction, ending in ulcer formation (Autoimmun Rev 2018;17:567)
      • Can present with folliculitis or papulopustular lesions associated with vasculitis (An Bras Dermatol 2017;92:452, Yonsei Med J 2007;48:573)
    • Cogan syndrome:
      • Histopathologic examination of cochlear and corneal tissue show lymphocytic and plasma cell infiltration (Autoimmun Rev 2013;12:396)
    • Vasculitis in systemic lupus erythematosus:
    • Rheumatoid arthritis associated vasculitis:
      • Vessel wall necrosis
      • Leukocytoclasia
      • Mononuclear and neutrophilic infiltration of small to medium sized vessels
      • Disruption of the internal and external elastic lamina (Curr Rheumatol Rep 2010;12:414)
    • Sarcoid vasculitis:
      • Well defined noncaseating granulomas
      • Multinucleated giant cells
      • Leukocytoclasia
      • Dermal fibrosis
      • Epidermal changes include a lichenoid / interface inflammatory pattern, apoptotic keratinocytes
      • Asteroid and Schaumann bodies can be present (Australas J Dermatol 2010;51:198)
    • Erythema elevatum diutinum:
      • Early lesions show leukocytoclastic vasculitis in the upper to mid dermis composed by a neutrophilic infiltrate with some eosinophils (Am J Dermatopathol 2005;27:397)
      • Progression of the lesions show involvement of the papillary and periadnexal dermis
      • Mature lesions exhibit granulation tissue, fibrosis, mixed inflammation and intracellular lipoidosis (J Am Acad Dermatol 1993;29:363)
      • Direct immunofluorescence demonstrates IgG, IgM, C3 and fibrinogen deposits in the vessel wall within the upper dermis (J Am Acad Dermatol 1992;26:38)
    • Granuloma faciale:
    • Palisaded neutrophilic and granulomatous dermatitis:
Microscopic (histologic) images

Contributed by Jose A. Plaza, M.D. and Medical College of Wisconsin Dermatology Department

Polyarteritis arteritis nodosa

Urticarial vasculitis

Urticarial vasculitis


Leukocytoclastic vasculitis

Leukocytoclastic vasculitis


Leukocytoclastic
vasculitis
involving vessels
in the papillary

Leukocytoclastic vasculitis

Leukocytoclastic vasculitis

Immunofluorescence description
  • Predominantly IgA1 deposits in the postcapillary venules (Henoch-Schönlein purpura) (Pediatr Dermatol 2008;25:630, Arthritis Rheumatol 2018;70:171)
  • Depending on the age of the lesions, skin biopsies of leukocytoclastic vasculitis can show deposition of IgG, IgA, IgM or complement C3 (Am J Dermatopathol 2006;28:486)
  • Direct immunofluorescence negative cases may reflect sampling error or consumption of immunocomplex
  • Most direct immunofluorescence patterns in leukocytoclastic vasculitis are nonspecific and a negative DIF does not preclude the presence of cutaneous vasculitis (J Cutan Pathol 2018;45:16)
Immunofluorescence images

Contributed by Alexander J. Gallan M.D. and Medical College of Wisconsin Dermatology Department

Glomerulus with mesangial IgA and C3 deposition

Deposits of IgM, IgA and C3



Contributed by Children's Wisconsin Pathology Department

Urticarial vasculitis with deposits of IgA, C3 and fibrin

Sample pathology report
  • Skin, right leg, punch biopsy:
    • Leukocytoclastic vasculitis (see comment)
    • Comment: The specimen shows superficial and deep perivascular inflammation comprised predominantly of neutrophils, with marked cellular debris, extravasation of red blood cells, vessel wall expansion and fibrinoid necrosis.
Differential diagnosis
  • Thrombocytopenic purpura:
  • Pigmented purpuric dermatosis:
    • Multiple petechial hemorrhages expressed as red to purple macules
    • Histopathology shows dilated vessels, hemosiderin deposits, red cell extravasation, perivascular lymphohistiocytic infiltration and reaction patterns that can be lichenoid, spongiotic or granulomatous (J Clin Med 2021;10:2283)
    • No vasculitis is seen
  • Senile purpura:
    • Elderly
    • Macules and patches in sun exposed areas
    • Degeneration of vessel wall and supporting stroma (Ann Dermatol 2019;31:472)
  • Embolism and infections:
  • Drug induced vasospasm:
  • Vascular trauma:
    • Hypothenar hammer syndrome is typical in males (often athletes) and is a consequence of recurrent trauma resulting in an aneurysm or thrombosis of the ulnar artery
    • Angiography confirms the diagnosis
    • Histologically recurrent trauma can cause intimal and media hyperplasia and fibrosis (fibromuscular dysplasia) (J Emerg Med 2019;56:105, Am J Dermatopathol 2007;29:44)
  • Calciphylaxis:
    • Fibrointimal hyperplasia
    • Comorbidities: end stage renal disease
    • Chronic, painful, nonhealing wounds
    • Calcification of small to medium size vessels with intimal hyperplasia (Am J Dermatopathol 2013;35:582)
Board review style question #1
Which of the following is true regarding cutaneous vasculitis?

  1. Both Behçet and Schamberg disease present with fibrinoid necrosis of the vessel wall
  2. Microscopic polyangiitis with renal involvement is expressed as rapidly progressive glomerulonephritis
  3. Most cutaneous vasculitides are chronic and recalcitrant
  4. Only small vessel vasculitis present with mucocutaneous manifestations
Board review style answer #1
B. Microscopic polyangiitis with renal involvement is expressed as rapidly progressive glomerulonephritis. Renal involvement by microscopic polyangiitis is characterized by a necrotizing crescentic glomerulonephritis with few or no deposits on immunofluorescence microscopy (Biomed Res Int 2015;2015:402826).

Comment Here

Reference: Cutaneous vasculitis
Board review style question #2


What type of immune deposits can be identified by direct immunofluorescence in cases of leukocytoclastic vasculitis?

  1. C3 and IgA only
  2. IgE
  3. IgG and C3 only
  4. IgG and IgM only
  5. IgG, IgM, IgA and C3
Board review style answer #2
E. IgG, IgM, IgA and C3. Leukocytoclastic vasculitis can show deposits of IgM, IgG, IgA and C3 by immunofluorescence microscopy.

Comment Here

Reference: Cutaneous vasculitis

Cysticercosis
Definition / general
  • Tissue infection caused by ingestion of larval cysts of the cestode Taenia solium (cysticercus cellulose) (CDC: Cysticercosis [Accessed 27 August 2018])
  • Acquired by swallowing food, water or feces contaminated by T. solium eggs
  • In cystecicercosis, the human represents an intermediate host and the parasite develops cysticerci in various organs
  • Taeniasis: infection caused by the adult tapeworm in the human intestine, which occurs from ingestion of larvae in undercooked pork
  • Cysticerci: larval forms of tapeworms found within a fluid filled cyst
Epidemiology
  • 50 - 100 million people infected worldwide (eMedicine: Cysticercosis [Accessed 27 August 2018])
  • Endemic areas include: Central and South America, India, China Southeast Asia, Africa and Eastern Europe
  • In the United States, infection is most common in rural areas and among Latin American immigrants
Sites
  • Cysticerci develop in the nervous system, heart, skeletal muscle, eyes and subcutaneous tissue
  • Cases in breast are rare; 8 cases were identified by FNAC over a 15 year period in India (Acta Cytol 1996;40:653)
Diagrams / tables

Images hosted on other servers:
Missing Image

Life cycle

Cysticercus

Etiology
  • Accidental ingestion of eggs or gravid proglottids of Taenia solium by human host via infected food, water or feces
Clinical features
  • Cysticercosis of the skin is rare
  • It presents as a palpable, subcutaneous nodule
  • In the breast, may present as a freely mobile cystic mass
Diagnosis
  • Thorough history, skin biopsy, serology (serum or CSF) and imaging can aid in the proper diagnosis
Radiology description
  • CT scan can reveal hyperdense lesions in subcutaneous tissue with or without calcification
  • Ultrasound can reveal cystic lesions
Radiology images

Images hosted on other servers:
Missing Image

Computed tomography

Cysts

Mammogram, tubular structure

Case reports
Treatment
  • Praziquantel and albendazole are effective
  • Surgery is appropriate for large, solitary lesions which would otherwise require prolonged antiparasitic therapy
Clinical images

Images hosted on other servers:
Missing Image

Gigantic cysticercosis pseudo tumour

Missing Image

Swelling over neck

Missing Image

Swellings over back

Gross description
  • Circumscribed, white to tan, cystic nodules containing a clear fluid
    • Viable cysts are translucent, through which a single scolex may be visible (2 - 3 mm nodule)
    • As the cyst begins to degenerate, the fluid becomes dense and opaque
    • In the later stages, only a calcified nodule may be present
  • Cyst sizes vary; commonly 1 mm - 2 cm
  • Larval forms identified within the cyst cavity
Gross images

AFIP images

Breast cyst



Images hosted on other servers:
Missing Image

Cysticercosis pseudotumor

Missing Image

Well circumscribed nodule

Missing Image

Specimen from trunk

Breast: tubular, worm-like structure

Microscopic (histologic) description
  • Cystic cavity contains the the larval form: scolex with hooklets and 2 pairs of suckers
    • The larval form, composed of duct-like invaginations, is lined by a double layered, eosinophilic membrane
    • Scolex is single and invaginated; contains a rostellum, 4 suckers and 22 - 23 bifrefringent hooklets (may persist for a long time)
    • Body wall exhibits a myxoid matrix and calcareous bodies (calcified concretions)
  • Cysticerci may remain viable for years
  • Colloidal stage: first stage of involution of cysticerci; transparent vesicular fluid is replaced by a turbid, viscous fluid and the scolex shows signs of hyaline degeneration
  • Granular stage: cysticercus is no longer viable; cyst wall thickens and the scolex is transformed into coarse mineralized granules
  • Host inflammatory reaction is usually not present if the larva is viable
  • Finally, a granulomatous reaction develops characterized by histiocytes, epitheloid cells and foreign body giant cells, leading to fibrosis of the supporting stroma and calcification of the parasitic debris
Microscopic (histologic) images

Contributed by Thiriveni Balajji, M.D., M. Kavitha, M.D., Case #458 and Case #115
Missing Image Missing Image Missing Image

Subcutaneous cysticercosis with neurocysticercosis


CNS

Eyelid

Cytology description
  • Fibrillary stroma with interspersed nuclei and a honeycomb pattern
  • Parts of parasite may be identifiable (Acta Cytol 1996;40:653)
  • Background usually consists of a mixed inflammatory infiltrate (Acta Cytol 1989;33:659)
  • Granulomas may be seen
Cytology images

Images hosted on other servers:

Breast: larval fragment

Bladder: wall of cysticercus

Wall of cysticercus

Epithelioid cell granulomas


Wall of cysticercosis cellulosae

Inflammatory infiltrate

Fibrillary stroma

Positive stains

Cytophagic histiocytic panniculitis
Definition / general
Terminology
  • Panniculitis associated with hemophagocytic syndrome
Etiology
  • Associated with CMV, EBV, HIV, bacteria, fungi and parasites
  • Occurs post-bone marrow transplant, with SLE, as adverse reaction to interferon alpha therapy
  • Associated with T cell lymphoma
  • Presents with spiking fever, erythematous subcutaneous skin nodules, anemia, leukopenia
  • May be fatal with multisystemic involvement
Clinical features
  • Erythematosus to violaceous or hemorrhagic nodules, often in lower limbs and trunk
  • Ulceration with localized or generalized edema
Microscopic (histologic) description
  • Lobular or partially septal distribution of florid lobular panniculitis, with large histiocytes containing eosinophilic cytoplasm; also neutrophils and lymphocytes
  • Red cell extravasation typically present with frank hemorrhage
  • Erythrophagocytosis, phagocytosis of lymphocytes or nuclear debris
  • Enlarged histiocytes may be described as "bean bag cell"
  • Usually no giant cells / granulomas
Microscopic (histologic) images

Images hosted on other servers:

11 year old girl with perforin gene mutation

Differential diagnosis
  • Angiocentric lymphoma
  • Cutaneous Rosai Dorfman disease
  • Subcutaneous T cell panniculitis lymphoma

Demodex
Definition / general
  • Demodex mites are commensal organisms that are occasionally implicated in inflammatory skin disease
Essential features
  • Commensal mite found in folliculosebaceous units
  • Increased incidence with age
  • Pathogenic with overgrowth or immune reaction causing rosacea or folliculitis
Terminology
  • Demodex mites, demodicosis, demodicidosis, Demodex folliculorum, Demodex brevis, face mite, follicle mite
ICD coding
  • ICD-10:
    • B88.0 - other acariasis (inclusion term dermatitis due to Demodex species)
    • L71.8 - other rosacea
    • L71.9 - rosacea, unspecified
    • L73.8 - other specified follicular disorder
    • L73.9 - follicular disorder, unspecified
Epidemiology
Sites
Pathophysiology
  • Unclear
  • 4 proposed mechanisms for how Demodex cause disease (J Am Acad Dermatol 2009;60:453):
    • Blockage of hair follicle and sebaceous ducts
    • Humoral or cell mediated immune reaction to mite components
    • Foreign body granulomatous reaction to mite’s exoskeleton
    • Vector role for bacteria
  • Wolbachia within Demodex may incite inflammation (Cutis 2005;76:294)
  • Upregulation of TNF alpha and IL1β in papulopustular rosacea (Clin Dermatol 2014;32:739)
  • Papulopustular reaction associated with HLA type (Cutis 2005;76:294, Clin Dermatol 2014;32:739)
    • Lack of HLA-A2 phenotype
    • HLA-Cw2 phenotype
Etiology
  • Unclear why it is pathogenic in some (Am J Dermatopathol 1996;18:589)
    • Demodex mites in 42% of 388 follicles with inflammation
    • Demodex mites in 10% of noninflamed follicles
    • 83% of follicles with Demodex with inflammation
Clinical features
Diagnosis
Case reports
Treatment
Clinical images

Images hosted on other servers:

Rosacea due to demodicosis

Granulomatous rosacea associated with Demodex

Microscopic (histologic) description
Microscopic (histologic) images

Contributed by Bethany R. Rohr, M.D., Eric W. Hossler, M.D. and Bobbi Pritt, M.D.

Inflamed follicle

Infundibulum

Nevus

Demodex mite found in rosacea papule


Demodex scraping

MIB stain

Dual MART - MIB stain

Positive stains
  • None required
  • Incidental finding of MIB positivity noted by author Eric W. Hossler, M.D. (see figure 7)
Videos

These face mites really grow on you

Demodex mite

Sample pathology report
  • Skin, face:
    • Suppurative folliculitis with Demodex (see comment)
    • Comment: In the correct clinical setting, these findings are consistent with Demodex folliculitis. Similar findings can be seen in rosacea or other acneiform process.
Differential diagnosis
  • Acne:
    • Usually teens
    • Comedones
    • Pustules, cysts, nodules
  • Bacterial folliculitis:
    • Swab or tissue culture
  • Fungal folliculitis:
    • Often in setting of tinea corporis
    • Fungal hyphae and spores within involved hair follicle
    • Tissue culture
  • Pityrosporum folliculitis (Cutis 2021;107:E40):
    • Caused by Malassezia furfur
    • Pruritic monomorphic folliculocentric pustules
    • KOH shows short hyphae and spores
    • Biopsy shows hyphae and spores in inflamed follicular infundibula
  • Eosinophilic folliculitis (Cutis 2021;107:E40):
    • Pruritic facial papules and pustules
    • 3 variants: Ofuji disease, immunosuppression associated and infancy associated
    • Peripheral eosinophilia
    • Histopathology shows follicular spongiosis with exocytosis of eosinophils
  • Miliaria (Cutis 2021;107:E40):
    • Due to occlusion of eccrine glands
    • 2 - 4 mm erythematous vesicular papules or pustules in area of occlusion
    • Not folliculocentric
Board review style question #1

Demodex mites are commensal arachnids that most commonly live in which anatomic location(s)?

  1. Arms
  2. Buttocks
  3. Face
  4. Feet and Hands
  5. Legs
Board review style answer #1
C. Face

Comment Here

Reference: Demodex
Board review style question #2
Demodex mites have been implicated in which of the following conditions?

  1. Atopic dermatitis
  2. Psoriasis
  3. Rosacea
  4. Seborrheic dermatitis
  5. Vitiligo
Board review style answer #2
C. Rosacea

Comment Here

Reference: Demodex
Board review style question #3

On histology, Demodex mites are usually identified in what?

  1. Dermis
  2. Eccrine coil
  3. Epidermis
  4. Folliculosebaceous unit
  5. Pilar muscle
Board review style answer #3
D. Folliculosebaceous unit

Comment Here

Reference: Demodex

Dermal hypersensitivity reaction
Definition / general
Essential features
  • Sparse to moderately dense superficial and mid perivascular and interstitial lymphocytic infiltrate with eosinophils
  • Epidermal changes may be absent, minimal (spongiotic or vacuolar interface) or secondary (excoriation)
Terminology
  • Comparable or equivalent histologic terms: cutaneous hypersensitivity reaction / response, dermal hypersensitivity response, urticarial hypersensitivity reaction
  • For idiopathic pruritic skin conditions, some clinicians include dermal hypersensitivity reaction in their clinical differential diagnosis
  • Similar or equivalent terms: itchy red bump disease, papular dermatitis, subacute prurigo and urticarial dermatitis
  • Note: papular urticaria is a synonym for arthropod bite reactions; drug induced hypersensitivity syndrome (DIHS) is a specific type of drug reaction
ICD coding
  • ICD-10:
    • L30.8 - other specified dermatitis
    • L30.9 - dermatitis, unspecified
Epidemiology
Sites
Pathophysiology
Etiology
  • Idiopathic until or unless confirmed to be other entities based on clinical correlation (see Differential diagnosis)
Clinical features
Diagnosis
  • Histologic reaction pattern that must be confirmed microscopically but the final diagnosis requires clinical correlation
  • Deeper tissue sections to rule out other diagnoses may be needed; e.g., folliculitis, spongiotic dermatitis, scabies may become apparent only in deeper sections
  • Exceptionally, may be a paraneoplastic hypersensitivity reaction (J Am Acad Dermatol 2010;63:e13, Clin Exp Dermatol 2016;41:322)
Laboratory
  • No laboratory requests are mandated but screening for evidence of specific causes may be prudent (see Differential diagnosis)
  • Ancillary testing may include a second biopsy for direct immunofluorescence (DIF) testing, scabies prep, complete blood count (CBC) or other serologic studies (J Am Acad Dermatol 2014;70:263)
Prognostic factors
  • Idiopathic cases have chronic course with potentially unsatisfactory responses to treatment (see Treatment)
  • No reported impact on life expectancy in idiopathic cases
Case reports
Treatment
Clinical images

Contributed by Maxwell A. Fung, M.D.

Crusted papules coalescing into plaques

Papules and lichenified plaques

Multiple papules on the leg

Urticarial papules and plaques

Microscopic (histologic) description
  • Superficial and mid dermal perivascular and interstitial infiltrate involving the superficial (papillary) and mid (upper reticular) dermis; may extend to deep reticular dermis
  • Predominantly lymphocytic infiltrate with eosinophils; with or without neutrophils
  • Varying intensity of infiltrate, typically sparse to moderately dense
  • Epidermal alteration, if present, may consist of minimal parakeratosis, spongiosis, vacuolar interface changes or secondary crust / excoriation
  • References: J Am Acad Dermatol 2002;47:898, Arch Dermatol 2006;142:29, J Am Acad Dermatol 2014;70:263
Microscopic (histologic) images

Contributed by Maxwell A. Fung, M.D.

Itchy red bump disease

Atopic dermatitis

Urticarial dermatitis

Videos

Perivascular dermatitis, invisible dermatoses and sclerosing disorders

Sample pathology report
  • Example #1 (circa 2001, University of Connecticut)
    • Clinical information: rule out hypersensitivity reaction
    • Specimen / site: left upper arm
    • Diagnosis: dermal hypersensitivity reaction (see comment)
    • Comment: Deeper sections have been performed. The histological differential diagnosis includes a lesion of urticaria. There is a sparse superficial and mid dermal perivascular infiltrate composed of lymphocytes, histiocytes, neutrophils and eosinophils.

  • Example #2 (circa 1998, Cornell University Medical Center)
    • Superficial and deep perivascular dermatitis, consistent with a dermal hypersensitivity reaction, biopsy:
      • T-01000; T-02471; M-40006
      • ICD-9: V72.6 and 693.89
      • Comment: A reaction to an internal antigen source seems likely because of the deep inflammation; however, slight spongiosis is noted and chronic allergic contact dermatitis cannot be completely ruled out histologically.

  • Example #3 (circa 2020, University of California Davis)
    • Skin, right posterior forearm, punch biopsy:
      • Sparse dermatitis with eosinophils and mild spongiosis (see comment)
      • Comment: The histologic differential diagnosis includes a drug reaction or spongiotic / eczematous dermatitis, as well as a sparse arthropod bite or other hypersensitivity reaction. There is focal epidermal necrosis consistent with external trauma. Overt fungi on H&E is not visible; a fungal stain may be obtained upon request if necessary.

  • Example #4 (circa 2020, University of California Davis)
    • Skin, mid back, punch biopsy:
      • Sparse perivascular and interstitial dermatitis with eosinophils (see comment)
      • Comment: The histologic differential diagnosis includes a drug or other hypersensitivity reaction, nondiagnostic spongiotic / eczematous dermatitis or as a diagnosis of exclusion, an idiopathic eruption (e.g., itchy red bump disease, urticarial dermatitis). Some eosinophils are present near the dermal epidermal junction, so urticarial pemphigoid could also be considered (but not specifically favored). Deeper sections fail to reveal additional findings.
Differential diagnosis
  • Differential diagnosis of dermal hypersensitivity reaction is broad; clinical correlation is required in all instances
  • Drug eruption:
    • Subtle vacuolar interface changes or spongiosis are often present
  • Arthropod bite reaction (papular urticaria):
    • Infiltrates tend to be deeper and denser with abundant eosinophils; may involve subcutis; may show centrally positioned intraepidermal vesicle or crusted erosion / ulcer (punctum)
    • Clinically, the lesions should be exclusively papules, erythematous and usually edematous / urticarial and tend to occur either as a single or localized cluster of papules on the extremities, ideally with a patient confirmed mosquito, flea, mite or other arthropod exposure
  • Atopic dermatitis:
    • Biopsies of atopic dermatitis usually present as spongiotic dermatitis so more prominent spongiosis, such as spongiotic vesicles and confluent or diffuse parakeratosis, would support this possibility
    • Clinically, patients present with eczematous skin plaques or papules and a personal or history of atopy; diagnostic criteria (Hanifin-Rajka) for atopic dermatitis may be applied by the clinician
  • Contact dermatitis:
    • Biopsies of contact dermatitis usually present as spongiotic dermatitis, so more prominent spongiosis, such as spongiotic vesicles and confluent or diffuse parakeratosis, would support this possibility
    • Clinically, allergens or irritants may be confirmed by clinical history or patch testing
  • Spongiotic / eczematous dermatitis, NOS:
    • Dermal hypersensitivity reaction with more prominent spongiosis, such as spongiotic vesicles and confluent or diffuse parakeratosis, would support this possibility
    • Clinical correlation is required to confirm a specific type of spongiotic / eczematous dermatitis (e.g., atopic, contact, nummular)
  • Urticaria:
    • Dermal hypersensitivity reaction with no epidermal changes at all; dermal edema and neutrophils typically present in addition to the lymphocytes and eosinophils
  • Pruritic urticarial papules and plaques of pregnancy (PUPPP):
    • Dermal hypersensitivity reaction, sometimes with more prominent spongiosis and marked papillary dermal edema
  • Associated with malignancy or other systemic conditions:
  • Urticarial vasculitis:
    • In addition to clinical correlation, evidence of vasculitis from another biopsy or a positive direct immunofluorescence (DIF) result would typically be required
  • Bullous pemphigoid:
    • Infiltrate of eosinophils centered upon the dermoepidermal junction and within the epidermis (eosinophilic spongiosis) would be typical of the urticarial or prebullous phase of pemphigoid
    • Positive DIF result from perilesional skin would be required
  • Scabies:
    • Crust and prominent eosinophils are typical
    • Deeper sections, multiple biopsies or scabies prep may be required to identify diagnostic findings (mite, eggs, feces [scybala])
  • Wells syndrome (eosinophilic cellulitis) and other eosinophil predominant infiltrates:
    • Dermal hypersensitivity reaction must contain eosinophils but the infiltrate is predominately lymphocytic (Dermatol Clin 2012;30:667)
    • Flame figures typify Wells syndrome but may be present occasionally in any eosinophil rich dermal infiltrate
  • Viral exanthem:
    • Vacuolar interface dermatitis, often with extravasated erythrocytes and exocytosis of lymphocytes, often accompanies sparse dermal infiltrates
    • Eosinophils are not specifically characteristic of a viral exanthem but would not exclude this possibility
Board review style question #1

Which of the following histologic findings is required for the diagnosis of dermal hypersensitivity reaction?

  1. Eosinophils
  2. Granulomas
  3. Neutrophils
  4. Plasma cells
  5. Spongiosis
Board review style answer #1
A. Eosinophils. Required histologic features of dermal hypersensitivity reaction include a perivascular and interstitial lymphocytic infiltrate with eosinophils. Neutrophils and spongiosis are variable but not required findings. Granulomas and plasma cells are not features of dermal hypersensitivity reaction.

Comment Here

Reference: Dermal hypersensitivity reaction
Board review style question #2
A punch biopsy of a pruritic eruption of 5 years duration involving the trunk of a 64 year old man is reported as a dermal hypersensitivity reaction. The pathologist commented that deeper sections were negative. The microscopic description documents scant parakeratosis and spongiosis. Which of the following may be considered in the differential diagnosis?

  1. Folliculitis
  2. Sarcoid
  3. Syphilis
  4. Urticaria
  5. Urticarial dermatitis
Board review style answer #2
E. Urticarial dermatitis. The negative deeper sections are against folliculitis. Sarcoid would require granulomas which are not a feature of dermal hypersensitivity reaction. Syphilis typically shows a psoriasiform lichenoid pattern with plasma cells. The parakeratosis and spongiosis excludes urticaria.

Comment Here

Reference: Dermal hypersensitivity reaction

Dermatitis herpetiformis
Definition / general
  • Acquired autoimmune subepidermal blistering disease due to gluten exposure and characterized by neutrophilic microabscesses and IgA deposits in the dermal papillae
Essential features
  • Most cases are triggered by gluten ingestion and subsequent intolerance
  • Anti tissue transglutaminase antibodies cross react with epidermal transglutaminase and leads to cutaneous deposits of IgA (J Exp Med 2002;195:747)
  • Subepidermal vesicles and blisters with papillary neutrophilic microabscesses that may contain eosinophils
  • Periphery of blister: neutrophilic interface inflammatory infiltrate and neutrophilic microabscesses in the dermal papillae
  • Direct immunofluorescence: granular or fibrillar IgA deposition along the basement membrane and in the dermal papillae
Terminology
ICD coding
  • ICD-10: L13.0 - dermatitis herpetiformis
  • ICD11: EB44 - dermatitis herpetiformis
  • SNOMED: 111196000 - dermatitis herpetiformis
Epidemiology
  • Most commonly occurs in second to fourth decade
  • Mainly seen in Caucasians of Northern European descent (An Bras Dermatol 2014;89:865)
  • M > F
  • Commonly seen in association with gluten sensitive enteropathies
Sites
  • Common locations: elbows, knees, buttocks and back, posterior shoulder, neck
  • May rarely involve oral mucosa
Pathophysiology
  • Most cases are triggered by gluten ingestion (specifically, gliadin protein)
  • Causes antibody formation against gliadin and autoantibody formation against transglutaminases (An Bras Dermatol 2014;89:865)
  • Anti tissue transglutaminase antibodies cross react with epidermal transglutaminase (TG3), leading to cutaneous deposits of IgA (J Exp Med 2002;195:747)
Etiology
  • Autoantibody formation to tissue transglutaminase following ingestion of gluten
Diagrams / tables

Images hosted on other servers:

Dermatitis
herpetiformis
physiopathogenesis

Clinical features
  • Pinhead sized and grouped vesicles on an erythematous base
  • May appear as erosions due to intense scratching
  • Associated with gluten sensitive enteropathies (e.g. celiac disease)
    • Both celiac disease and dermatitis herpetiformis have strong genetic association with HLA DQ2 and HLA DQ8 on chromosome 6 (J Invest Dermatol 2000;115:990)
Diagnosis
  • Enzyme linked immunoassay (ELISA)
    • Circulating IgA antibodies to tissue transglutaminase
  • Histologic evaluation and direct immunofluorescence
    • Biopsy should include samples of perilesional skin (routine stains, DIF) and lesional skin (H&E) (Clin Cosmet Investig Dermatol 2015;8:257)
    • Specimens submitted for direct immunofluorescence (DIF) can come from normal appearing perilesional skin
Laboratory
  • ELISA: circulating IgA antibodies to tissue transglutaminase
Prognostic factors
  • Strong association with HLA DQ2 and HLA DQ8 (Tissue Antigens 1997;49:29)
    • Same HLA haplotypes associated with celiac disease
  • Gluten free diet may cause prolonged remission in some patients or lower daily dapsone requirement in others
Case reports
Treatment
  • Gluten free diet (GFD): first line treatment
    • Bridge therapy with dapsone: can take up to 2 years for rash to disappear with gluten free diet alone (Dermatol Ther 2003;16:214)
  • Dapsone
    • Used in combination with gluten free diet to reduce symptoms
    • Rash returns 2 - 3 days after discontinuation of dapsone if used as the sole therapy
Clinical images

Images hosted on other servers:

Grouped papules,
vesicles and
hypopigmentation

Microscopic (histologic) description
  • In early lesions, superficial perivascular lymphocytic and neutrophilic infiltrate
  • Subepidermal vesicles or blisters with papillary neutrophilic microabscesses that may contain eosinophils
  • Vacuolar interface changes and nuclear dust may be seen
  • Dermal infiltrate contains lymphocytes, histiocytes and abundant neutrophils
  • Periphery of blister: neutrophilic interface inflammatory infiltrate and neutrophilic abscesses in the dermal papillae (An Bras Dermatol 2014;89:865)
  • Occasionally, findings may be nonspecific with just perivascular lymphocytes (requiring DIF for confirmation) (Am J Dermatopathol 2002;24:305)
  • Acantholysis may also be present (Front Immunol 2019;10:1290)
Microscopic (histologic) images

Contributed by Michael Occidental, M.D. and Randie H. Kim, M.D., Ph.D.

Blister

Skin directly adjacent to blister

Perilesional skin

Blister and perilesional skin

Neutrophilic microabscesses in perilesional skin

Virtual slides

Images hosted on other servers:

Dermatitis herpetiformis

Neutrophilic infiltration of dermal papillae

Immunofluorescence description
  • Direct immunofluorescence (An Bras Dermatol 2014;89:865)
    • Most commonly granular IgA deposits in the dermal papillae
      • Granular C3 deposits pattern in dermal papillae may be present
      • Rare but characteristic fibrillar IgA deposits along the basement membrane zone (J Cutan Pathol 2010;37:475)
Immunofluorescence images

Contributed by Randie H. Kim, M.D., Ph.D.

IgA on direct immunofluorescence

Videos

Dermatitis herpetiformis

Sample pathology report
  • Skin, shoulder, left, shave biopsy:
    • Dermatitis herpetiformis (see comment)
    • Comment: There is a subepidermal blister with associated lymphocytes, histiocytes and abundant neutrophils. The periphery of the lesion shows prominent vacuolar interface changes and collections of neutrophils at the tips of the dermal papillae. The associated immunofluorescence sample displays granular IgA and C3 deposits in the dermal papillae. The findings are consistent with dermatitis herpetiformis.
Differential diagnosis
  • Histopathological findings of neutrophil rich subepidermal autoimmune blistering diseases may be identical and include linear IgA dermatosis, bullous systemic lupus erythematosus, neutrophil rich bullous pemphigoid, inflammatory epidermolysis bullosa acquisita and p200 pemphigoid
  • Linear IgA dermatosis:
    • Homogenous band of IgA at the dermal epidermal junction
    • No anti endomysial or anti tissue transglutaminase IgA antibodies
    • Not gluten sensitive
  • Bullous pemphigoid:
    • Large tense blisters on flexor surfaces, trunk, intertriginous regions and mucosa
    • No neutrophil microabscesses in classical bullous pemphigoid
      • Neutrophil rich variants however may look very similar; rely on DIF to distinguish
  • Bullous systemic lupus erythematosus:
    • Autoantibodies to type VII collagen on direct immunofluorescence (u-serrated pattern)
    • Sublamina densa blistering disease that may have a linear u-serrated pattern on direct immunofluorescence
    • DIF may show granular or linear deposition of IgM, IgG or IgA and C3 along the basement membrane zone (full house staining)
  • Epidermolysis bullosa acquisita:
Board review style question #1

A 20 year old man presents with chronic diarrhea, weight loss and vesicular skin lesions on his upper extremities. The man was referred for an esophagogastroduodenoscopy and colonoscopy, which showed villous blunting and increased intraepithelial lymphocytes within the duodenum. The skin lesions were concurrently biopsied (shown in above image). What do you expect to see on direct immunofluorescence?

  1. Granular IgA with or without C3 deposits on the tips of the dermal papillae
  2. Intracellular IgG and C3 deposition in the epidermis (chicken wire pattern)
  3. Linear IgA deposition along the basement membrane
  4. Linear IgG and C3 deposition along the basement membrane
Board review style answer #1
A. Granular IgA with or without C3 deposits on the tips of the dermal papillae

Comment Here

Reference: Dermatitis herpetiformis
Board review style question #2
Antibodies to what protein are formed in dermatitis herpetiformis?

  1. BPAG1 and BPAG2
  2. Desmoglein 1 and 3
  3. Desmocollin 1
  4. Epidermal transglutaminase (TG3)
Board review style answer #2
D. Epidermal transglutaminase (TG3)

Comment Here

Reference: Dermatitis herpetiformis

Dermatomyositis
Definition / general
  • See also Muscle chapter
  • Autoimmune inflammatory disease of skeletal muscle and skin, usually affects women
  • Symmetric proximal muscle weakness and skin lesions
  • Affects face, dorsal hands and feet, particularly knuckles
  • 20% of cases lack muscle involvement (Arch Dermatol 2010;146:26)
  • 15% have coexisting adenocarcinoma of stomach, breast, ovary, lung or colon, with remission of dermatomyositis following tumor resection; high risk of nasopharyngeal carcinoma in Asian patients (Ann Acad Med Singapore 2010;39:843)
  • Increased risk of thyroid disease, particularly hypothyroidism, especially in patients with interstitial lung disease

  • Polymyositis: similar muscle changes without skin changes
Clinical features
  • Poorly demarcated, scaly, erythematous patches
  • Also heliotrope erythema of upper eyelids and extensor joint surfaces
Treatment
Clinical images

Images hosted on other servers:

Purple plaques on knees

Heliotrope rash and Gottron papules

Various images

Microscopic (histologic) description
  • Chronic nonspecific dermatitis or interface dermatitis resembling systemic lupus erythematosus
  • Often atrophic epidermis with prominent vacuolar interface change
  • Sparse perivascular lymphocytic infiltrate with markedly increased dermal mucin
  • Muscles show myositis with myofiber necrosis, fragmentation and phagocytosis; late myofiber atrophy, fibrosis and fatty change
Microscopic (histologic) images

Contributed by Hillary Rose Elwood, M.D.

Subtle vacuolar interface change and increased dermal mucin highlighted by Alcian blue stain



Images hosted on other servers:

Erythrodermic dermatomyositis

Interface dermatitis
with vacuolar
alteration at dermal
epidermal junction

Positive stains
  • C5-9 (by immunofluorescence)
Negative stains
  • IgG, IgA and IgM by immunofluorescence

Dermatophytes / tinea
Definition / general
  • Due to pathogens restricted to the stratum corneum, with little or no tissue reaction
  • Scalp and beard lesions may have superimposed bacterial folliculitis / perifolliculitis
  • Superfical fungal infections may also be found on neoplastic skin lesions
  • Usually caused by dermatophytes; in children, usually due to Trichophyton, Microsporum and Epidermophyton (Am Fam Physician 2008;77:1415)

  • Kerion celsi: superimposed bacterial folliculitis on tinea of scalp
  • Majocchi granuloma: nodular granulomatous perifolliculitis; inflammation of dermis and subcutis by dermatophytes, usually Trichophyton rubrum
  • Sycosis barbae: tinera barbae with superimposed bacterial follicultis
  • Tinea barbae: infection of beard area of adult men
  • Tinea capitis: infection causing hairless patches of skin in scalp, usually in children
  • Tinea corporis: infection of trunk of children and adults, associated with excessive heat and humidity; scaly red annular plaques (ringworm)
  • Tinea cruris: "jock itch", infection of inguinal area of obese men during warm weather
  • Tinea pedis: "athletes foot", infection causing diffuse erythema and scaling, initially in web spaces, often with bacterial superinfection
Clinical images

Images hosted on other servers:

Kerion celsi, Majocchi granuloma and Tinea capitis

Tinea corporis:
well demarcated
margins and
diffuse erythema

Microscopic (histologic) description
  • Cellulitis, abscesses, pseudoepitheliomatous hyperplasia, fungal spores and hyphae in horny cell layer and near hair shafts
  • Spores, hyphae and neutrophils usually are present in stratum corneum or hair shafts
  • Variable intercellular epidermal edema, dermal inflammation
Cytology images

Images hosted on other servers:

Microsporum audouinii

Positive stains
Additional references

Dissecting cellulitis (pending)
[Pending]

Ecthyma (pending)

Ecthyma gangrenosum (pending)

Encapsulated fat necrosis
Definition / general
  • Encapsulated fat necrosis is a degenerative process characterized by nodules of necrotic adipose tissue in a fibrous or connective tissue capsule
Essential features
  • Nonneoplastic inflammatory reaction that is commonly secondary to adipose tissue injury
  • Trauma interrupts local blood supply to affected adipose tissue, causing necrosis and local inflammation and resulting in fibrosis and encapsulation
  • Primarily affects lower extremities or other body regions subject to repeated trauma
  • Late lesions may become calcified
Terminology
  • Well circumscribed fat necrosis
  • Nodular cystic fat necrosis
  • Mobile encapsulated lipoma
  • Nodular fat necrosis
  • Encapsulated necrosis
  • Posttraumatic fat degeneration
  • Reference: Br J Plast Surg 2001;54:643
ICD coding
  • ICD-10: L98.9 - disorder of the skin and subcutaneous tissue, unspecified
Epidemiology
Sites
Pathophysiology
  • Trauma or interrupted blood flow leads to (Med Ultrason 2020;22:397)
    • Infarction of adipose tissue
    • Subsequent inflammation involving local proinflammatory cytokine release
    • Increased macrophage activity after initial insult that creates fibrotic strands separating injured necrotic tissue from surrounding healthy tissue
  • The overall fibrocellular response leads to the formation of a fibrous capsule and a cleavage plane between the lesion and surrounding adipose (Br J Plast Surg 2001;54:643)
  • Late lesions may develop dystrophic calcification, which can be seen on imaging (Br J Plast Surg 2001;54:643)
Etiology
  • Lesions may arise secondary to trauma or interrupted blood supply
Clinical features
Diagnosis
  • Diagnosis made using suggestive clinical (i.e., recent history of local trauma) and microscopic findings (Skeletal Radiol 2013;42:1465)
  • Radiographic imaging may show calcifications in chronic lesions but does not rule out malignancy
  • Microscopy (i.e., biopsy) required to confirm diagnosis (Skeletal Radiol 2013;42:1465)
Laboratory
  • Complete blood count, urinalysis, liver function tests, electrocardiogram and stool examinations are likely to be normal (J Dermatol 1998;25:616)
Radiology description
  • MRI (Skeletal Radiol 2013;42:1465)
    • Lesions often demonstrate heterogeneous, high signal intensity compared to muscle tissue but may appear isointense compared to healthy adipose tissue
    • Fat suppression visible on T2 weighted images
    • Fibrous capsule may demonstrate contrast enhancement
  • PET: heterogenous mass with focal fat encapsulation (Skeletal Radiol 2013;42:1465)
Radiology images

Images hosted on other servers:
Encapsulated abdominal mass

Encapsulated abdominal mass

MRI

MRI

CT

CT

Prognostic factors
Case reports
Treatment
Clinical images

Images hosted on other servers:
Subcutaneous nodule of right thigh

Subcutaneous nodule of right thigh

Mobile nodule with unusual mobility

Mobile nodule with unusual mobility

Gross description
Gross images

Images hosted on other servers:
Subcutaneous encapsulated fat necrosis

Subcutaneous encapsulated fat necrosis

Cut section of the gross lesion

Cut section of the gross lesion

Microscopic (histologic) description
Microscopic (histologic) images

Contributed by Aadil Ahmed, M.D.
Fibrous capsule surrounding adipose tissue Fibrous capsule surrounding adipose tissue

Fibrous capsule surrounding
adipose tissue

Histiocyte within adipose tissue

Histiocyte within adipose tissue

Focal lipomembranous change

Focal lipomembranous change

Virtual slides

Images hosted on other servers:
Subcutaneous fat necrosis of the skin

Subcutaneous fat necrosis of the skin

Positive stains
Videos

Posttraumatic fat necrosis (encapsulated subcutaneous fat necrosis) by Dr. Jerad Gardner

Sample pathology report
  • Left shin nodule, wide local excision:
    • Encapsulated fat necrosis (see comment)
    • Comment: Microscopic examination reveals necrosis of mature adipose tissue encapsulated by fibrous strands of dense collagen. Affected adipocytes display eosinophilic cytoplasm. A mixed, scattered inflammatory infiltrate is present within the necrosis, including macrophages, lymphocytes and plasma cells. No evidence of nuclear atypia, malignancy or invasion is identified.
Differential diagnosis
  • Lipoma:
    • Proliferation of mature adipose tissue without any significant necrosis
    • Paucicellular fibrous septa may be present
  • Liposarcoma:
    • Multilobulated with infiltrative lipoblasts on histology
  • Angiolipoma:
    • Composed of mature adipocytes and branching capillaries / vessels, which often contain fibrin thrombi
  • Membranous fat necrosis:
    • Nonencapsulated variant of fat necrosis with cystic foci that are lined by eosinophilic membrane that projects within the cyst, creating a pseudopapilla or arabesque appearance
    • Positive periodic acid-Schiff stain
  • Pancreatic panniculitis:
    • Necrotic adipocytes with amorphous or granular blue-gray substance, consistent with the presence of ghost cells
    • Infiltrate is predominantly neutrophilic
  • Amyloidoma:
    • Homogeneous, pale pink deposition that will be positive for Congo red stain
Board review style question #1

A 36 year old woman presents with a gradually enlarging, painless mobile nodule on the right anterior shin. She sustained an injury to the same area 10 years ago. The team decides to proceed with surgical excision of the lesion. Histology is shown in the image. What is the most likely diagnosis?

  1. Encapsulated fat necrosis
  2. Lipoma
  3. Liposarcoma
  4. Pancreatic panniculitis
Board review style answer #1
A. Encapsulated fat necrosis demonstrates mature adipose tissue with central necrosis within a fibrous capsule. The capsule may become calcified over time. This lesion can be preceded by trauma to the area. Answer B is incorrect because lipoma would demonstrate mature adipocytes without necrosis due to intact blood supply. Larger lipomas can exhibit focal necrosis but without any capsule formation. Answer C is incorrect because liposarcoma would demonstrate multilobulated adipose tissue with infiltrative lipoblasts. Answer D is incorrect because pancreatic panniculitis would exhibit neutrophilic infiltrate with characteristic ghost cells.

Comment Here

Reference: Encapsulated fat necrosis
Board review style question #2
A 50 year old man presented for excision of a 1 cm, painless subcutaneous nodule of the left lower extremity. Upon microscopic review of the specimen, the lesion was diagnosed as encapsuled fat necrosis. Which stain would confirm the presence of a lipomembranous necrosis process?

  1. Giemsa
  2. Oil red O
  3. Periodic acid-Schiff
  4. Von Kossa
Board review style answer #2
C. Periodic acid-Schiff. PAS is specific to lipomembranous necrosis. Answer A is incorrect because Giemsa is commonly used to stain peripheral blood and bone marrow aspirate smears. Answer B is incorrect because oil red O is a hydrophobic, fat soluble dye that highlights fat droplets in lipid storage disease, steatosis or for detection of fat embolism. Answer D is incorrect because this stain is used to detect mineral deposits in tissues, such as calcium.

Comment Here

Reference: Encapsulated fat necrosis

Eosinophilic cellulitis (Wells syndrome)
Definition / general
  • Idiopathic inflammatory dermatitis with eosinophilic infiltration
  • Also known as eosinophilic cellulitis
Essential features
  • Idiopathic condition typically characterized by recurrent pruritic to painful plaques that often clinically resemble cellulitis
  • Histopathology characterized by a prominent eosinophilic infiltrate with "flame figures"
Epidemiology
  • Equal prevalence in men and women
  • Most cases sporadic
  • Higher prevalence in adults than children
Etiology
  • Unknown
  • Some cases may be secondary to hypersensitivity reaction to insect bites, medications, infections, vaccinations, malignant tumors or myeloproliferative disorders (J Clin Aesthet Dermatol 2011;4:55)
Clinical features
  • Clinical picture is variable but most often recurrent bouts of edematous nodules and plaques, often preceded by prodromal itching or pain
  • Usually a limited course over weeks to month but often recur
  • May resemble cellulitis clinically but not warm or tender (Postepy Dermatol Alergol 2014;31:322) and does not improve with antibiotics (J Clin Aesthet Dermatol 2011;4:55)
  • Erythematous papules, plaques or nodules that may be painful or pruritic (Postepy Dermatol Alergol 2014;31:322)
  • Sometimes annular configuration, can blister
  • May be single or multiple
  • Peripheral blood eosinophilia common (found in 67% of cases) (Can J Plast Surg 2012;20:91)
  • Leukocytosis (found in 41% of cases)
Treatment
  • Oral or topical corticosteroids most commonly, often with dramatic improvement
  • Antihistamines and immunomodulatory agents also used for refractory or unresponsive cases
Clinical images

Images hosted on other servers:

Swollen plaques on right forearm

Diffuse erythematous plaque

Resembling bacterial cellulitis

Microscopic (histologic) description
  • Diffuse dermal infiltrate of perivascular and interstitial eosinophils throughout superficial and deep dermis, often with extension into subcutis
  • Admixed histiocytes and lymphocytes
  • Old lesions may show granulomas
  • Flame figures often present (deposition of eosinophil basic protein on collagen bundles)
  • Can have subepidermal edema, sometimes with blister formation
  • May have eosinophilic spongiosis
  • No vasculitis
Microscopic (histologic) images

Contributed by Favia Dubyk, M.S., M.D.

Nodular collections of eosinophils



Images hosted on other servers:

Diffuse dermal eosinophilic infiltrate

High power view of eosinophils and flame figures

Multiple flame figures

Interstitial infiltrate
with eosinophils
magnification 100x

Numerous eosinophils magnification 200x

Differential diagnosis
  • Bacterial cellulitis: dermal to subcutaneous infiltrate of neutrophils, often with superficial dermal edema; typically lacks flame figures and neutrophils predominate over eosinophils
  • Churg-Strauss syndrome: may have similar eosinophilic granulomas, vasculitis not always present in skin biopsy, clinical correlation helpful
  • Eosinophilic annular erythema: has features of Wells syndrome as well as dermal mucin, vacuolar interface change
  • Flame figures are rarely seen in allergic contact dermatitis, arthropod bite reaction, bullous pemphigoid, dermatophyte infection, drug reaction, herpes gestationis; correlation with clinical findings is needed
  • Parasite infestation: can appear identical to Wells syndrome; in areas with endemic parasite infections, lesions resembling Wells syndrome are more often related to an underlying parasite infection
  • Board review style question #1
    Which of the following statements is true of Wells syndrome?

    1. Flame figures are pathognomonic
    2. Granulomatous inflammation always absent
    3. Lesions can clinically resemble cellulitis
    4. Peripheral blood eosinophilia is required for diagnosis
    5. Vasculitis usually present
    Board review style answer #1
    C. Often clinically resembles cellulitis, hence it's other name eosinophilic cellulitis. Answer A is incorrect because while flame figures are a hallmark of the disease, these can sometimes be seen in a number of other settings such parasite infestation, arthropod bite, drug reaction, allergic contact dermatitis and bullous pemphigoid. Answer B is incorrect because older lesions can have a granulomatous component, sometimes with prominent giant cells. Answer D is incorrect because this is seen in up to 67% of patients but not required for diagnosis. Answer E is inocrrect because vasculitis is typically absent although there may be extravasated erythrocytes.

    Comment Here

    Reference: Wells syndrome

    Eosinophilic dermatosis of hematologic malignancy (pending)
    [Pending]

    Eosinophilic fasciitis
    Definition / general
    • Swelling, tenderness and stiffness of an extremity, often lower forearm
    • May be associated with carpal tunnel syndrome
    • Inflammatory arthritis, contractures and subclinical myositis have been documented
    • Esophageal dysmotility, pericardial and pleural effusion, lung and kidney involvement
    • The association with serious hematological abnormalities has led to the suggestion that all patients with this disease
    • Should have a bone marrow examination to exclude myeodysplasia
    Case reports
    Treatment
    • Corticosteroids therapy
    Microscopic (histologic) description
    • Mild chronic inflammatory cell infiltrate consisting of lymphocytes, plasam cells, histiocytes, and variable numbers of eosinophils may be present in the deep reticular dermis, which is also fibrosed with atrophy of sweat glands
    • Occasionally the dermal changes may not be distinguished from Morphea
    • The most dramatic changes are found in the superficial fascia which is markdly thickened, fibrosed and sclerotic
    • A chronic inflammatory cell infiltrate is present both within the fascia and around blood vessels
    • Tissue eosinophilia is focal and often transitory
    • Its absence in no way preclude the diagnosis
    • Lymphoid follicles with germinal centre are also occasionally evident
    Microscopic (histologic) images

    Images hosted on other servers:

    Lymphoplasmacytic infiltrate of fascia and skeletal muscle

    Positive stains
    • Immunofluorescence has revealed deposition of IgM at the dermo-epidermal junction, immunoglobulin and complement around blood vessels in the deep dermis and IgG and complement in the deep fascia and skeletal muscle
    Differential diagnosis

    Eosinophilic pustular folliculitis
    Definition / general
    • Associated with HIV in U.S.
    • Sudden onset of disseminated pustules on trunk
    Terminology
    • Also called Ofuji disease, eosinophilic folliculitis
    Epidemiology
    • Typically recurrent disease and spontaneous resolution within months to several years
    • Associated with HIV, lymphoma (non-Hodgkin and Hodgkin), myelodysplastic syndrome, eosinophilic cellulitis and nevoid basal cell carcinoma syndrome
    Clinical features
    • Three variants:
      • Classic (Ofuji disease)
      • Immunosuppression / HIV associated
      • Infancy associated
    • Clinically resembles acne
    • No known cause
    Clinical images

    Images hosted on other servers:

    Widespread hyperpigmented patches

    Various images

    Microscopic (histologic) description
    • Eosinophilic infiltration of hair follicle; follicular epithelium is spongiotic, adjacent dermis has lymphocytic and eosinophilic infiltrate
    • Superficial dermal perivascular mononuclear and eosinophilic infiltrate
    • Follicular mucinosis may rarely be seen
    • Healing is associated with postinflammatory hyperpigmentation
    Microscopic (histologic) images

    Images hosted on other servers:

    Eosinophilic folliculitis


    Epidermodysplasia verruciformis
    Definition / general
    • Disorder of multiple flat warts on face or distal extremities, often disseminated throughout the body
    Etiology
    Clinical features
    • 30 - 50% progress to invasive squamous cell carcinoma, mean age 31 years
    • Equivocal cases are best distinguished by histology and viral testing, in context of clinical setting
    • Flat, skin colored, red or hyperpigmented papules resembling flat warts or scaly brown pityriasis versicolor-like macules and patches
    • Also seborrheic keratosis-like
    Treatment
    • Excision or locally destructive methods
    Clinical images

    Images hosted on other servers:

    Papillomatous and macular lesions

    Hypopigmented papules and plaques

    Microscopic (histologic) description
    • Classic features are mild to moderate acanthosis and hyperkeratosis
    • Large cells with blue-gray cytoplasm, perinuclear halos, often dysplastic changes, particularly post kidney transplantation (J Cutan Pathol 2002;29:480)
    • More commonly has irregular granular layer with rare perinuclear halos
    Microscopic (histologic) images

    Contributed by @JMGardnerMD on Twitter
    Epidermodysplasia verruciformis

    Epidermodysplasia verruciformis



    Images hosted on other servers:

    Various images

    Characteristic enlarged keratinocytes with blue-gray pallor; no significant keratinocytic atypia, pleomorphism or mitotic figures


    Epidermolysis bullosa
    Definition / general
    • Rare genetic blistering disorder (incidence of 8 - 19 per million) with cleavage in dermis, lower epidermis or at dermoepidermal junction
    • Classified as EB simplex, junctional EB, dystrophic EB and Kindler syndrome, based on level of tissue separation within the cutaneous basement membrane zone (Orphanet J Rare Dis 2010;5:12)
    • Also classified and subclassified based on clinical, electron microscopic, immunohistochemical and genotypic features (J Am Acad Dermatol 2008;58:931, Orphanet J Rare Dis 2010; 5: 12)

    • EB simplex: intraepidermal tissue separation; degeneration of basal cell layer causes clinical bullae; usually due to dominant mutations in keratin 14 or keratin 5 (J Clin Invest 2009;119:1784)
    • Junctional: blisters within lamina lucida, skin appears normal
    • Dystrophic: blisters are sub-lamina densa; dominant or recessive
    • Kindler syndrome: blisters at multiple levels (intra-lamina lucida and sub-lamina densa)
    Etiology
    • Defects in keratin K5 / K14 filament network architecture cause basal keratinocytes to become fragile and account for their trauma-induced rupture
    Diagrams / tables

    Images hosted on other servers:

    Normal epidermis and components

    Level of disruption in EB subtypes

    Clinical features
    • Blisters form shortly after birth due to pressure, rubbing or trauma
    • Blisters cause scarring or milia on dorsum of hands, elbows and knees and oromucosal lesions
    • EB Simplex-Dowling Meara: associated with marked morbidity; may cause death during early infancy; exhibits intact vesicles or small blisters in a grouped or arcuate (curved) configuration
    • Junctional EB: characterized by enamel hypoplasia, with pitting of some or all of the tooth surfaces
    • Junctional EB-Herlitz subtype: severe symptoms; presents at birth and involves all skin surfaces with exuberant granulation tissue of skin and possibly upper airway
    • Recessive dystrophic EB: subtypes are severe generalized (formerly Hallopeau-Siemens), non-Hallopeau-Siemens and inverse; each arises at birth; severe generalized is "one of the most devastating multiorgan genetically transmitted diseases of mankind"; has generalized blistering at birth, progressive and often mutilating scarring of skin, corneal blisters or scarring, profound growth retardation, multifactorial anemia, failure to thrive, esophageal strictures and debilitating hand and foot deformities

    • Diagnosis: usually requires focused lab testing, such as transmission EM (gold standard), immunofluorescence antigen mapping or immunohistochemical staining with EB specific monoclonal antibodies
    Clinical images

    Images hosted on other servers:

    Typical noninflammatory blister

    EB simplex blistered foot of infant

    EB simplex, generalized, with Dowling-Meara variant

    Junctional EB: marked pitting of enamel

    Junctional EB-Herlitz
    subtype: exuberant
    granulation tissue
    on child's neck


    Dominant dystrophic
    EB-milia arising within
    erythematous patch

    Generalized dominant dystrophic EB

    Generalized dominant
    dystrophic EB:
    hypertrophic scarring

    Generalized dominant
    dystrophic EB: dystrophy
    of all 20 nails


    Recessive dystrophic EB

    Severe generalized recessive dystrophic EB

    Congenital absence of
    skin in neonate with Bart
    syndrome (not specific
    for any EB subtype)

    Microscopic (histologic) description
    • Subepidermal blister with variable inflammation
    • Superficial dermis is fibrotic (type IV collagen positive)
    Microscopic (histologic) images

    Images hosted on other servers:

    EB simplex: intact stratum corneum and upper epidermis, with vesicle formation in the lower epidermis at the basal layer caused by degeneration of individual epidermal cells

    Dystrophic EB: pretibial lesion

    Positive stains
    Electron microscopy description
    Electron microscopy images

    Images hosted on other servers:

    EB simplex: epidermal basal keratinocytes


    Epidermolysis bullosa acquisita
    Definition / general
    • Rare, noncongenital, autoimmune, chronic listering disease of skin and mucus membranes (eMedicine)
    Etiology
    • Usually IgG autoantibodies against NC1 (noncollagenous domain of type VII collagen), major component of anchoring fibrils that connect basement membrane to dermal structures; also antibodies to central triple helical (collagenous) domain of type VII collagen and IgA antibodies instead of IgG
    Clinical features
    • Occurs at any age, usually affects elderly
    • Blisters, scars and milia at trauma prone areas
    • Some patients have generalized inflammatory skin blister phenotype
    • Chronic disorder with partial remissions and exacerbations
    • Causes significant morbidity but death due to disease is rare
    Treatment
    • Corticosteroids and immunosuppressants but relatively resistant to treatment
    Microscopic (histologic) description
    Immunofluorescence images

    Images hosted on other servers:

    Direct immunofluorescence on perilesional skin: linear band of IgG along dermal-epidermal junction

    Indirect immunofluorescence on salt-split normal human skin substrate using serum from affected patient: IgG autoantibodies on dermal side of basement membrane

    Positive stains
    • Direct immunofluorescence on perilesional skin shows linear band of IgG along dermal-epidermal junction
    • Indirect immunofluorescence on salt-split normal human skin substrate using serum from affected patient shows IgG autoantibodies on dermal side of basement membrane
    Videos

    Epidermolysis bullosa acquisita

    Differential diagnosis
    • Bullous pemphigoid: mild dermal infiltrate including eosinophils; reactivity on epidermal side in NaCl split skin

    Epidermolytic hyperkeratosis
    Definition / general
    • Keratinization disorder which presents at birth with generalized erythema, widespread bullae and desquamation resulting in denuded skin
    • Form of ichthyosis, an abnormality of epidermal maturation resulting in skin fragility

    • "Epidermolytic hyperkeratosis" is a descriptive term which refers to specific microscopic features (i.e. vacuolar degeneration of the spinous layer, keratin filament clumping and hyperkeratosis), but these findings may be seen in various entities, not just this disease (Int J Dermatol 2005;44:1):
      • Generalized: bullous ichthyosis
      • Systematized or linear: epidermal nevus variant
      • Palmoplantar: palmoplantar keratoderma variant (Vorner type)
      • Solitary: epidermolytic acanthoma
      • Multiple discrete: disseminated epidermolytic acanthoma
      • Incidental: focal epidermolytic hyperkeratosis
      • Solar keratosis related: a rare variant of solar keratosis
      • Follicular: nevoid follicular epidermolytic hyperkeratosis
      • Mucosal: epidermolytic leukoplakia
    Terminology
    • Other names:
      • Epidermolytic ichthyosis
      • Bullous congenital ichthyosiform erythroderma
      • Bullous ichthysosis
      • Bullous erythroderma ichthyosiformis congenita of Brocq
      • Disorder of cornification type 3
      • Vorner's syndrome
    Epidemiology
    Sites
    Pathophysiology
    • Defects in keratin genes (KRT1 and KRT10) cause excessive and abnormal keratinization
    • Mutations occur in carboxy-terminal of rod domain of keratin 1 (K1) and in amino-terminal of rod domain of keratin 10 (K10)
    Clinical features
    • Presents at birth with generalized erythema, followed by development of thick verrucous scaling on flexor surfaces
    • Distinct foul odor due to secondary bacterial colonization of thickened skin (Dermatol Online J 2014;20:21248)
    • Complications include infection, sepsis, dehydration and electrolyte imbalances due to compromised skin barrier (Dermatol Online J 2014;20:21248)
    • Palmar and plantar involvement occurs in 60% of patients, most commonly in those with K1 mutations (Dermatol Online J 2014;20:21248)

    • Linear epidermolytic hyperkeratosis is associated with genetic mosaicism (Dermatol Online J 2014;20:21248)
      • Linear hyperkeratosis in streaks along Blaschko lines
      • Unilateral or bilateral, localized or generalized
      • Absence of blistering (unlike classical epidermolytic hyperkeratosis)
    Diagnosis
    • Characteristic histological and ultrastructural findings (see below)
    • Immunofluorescence to determine which keratin type is involved (Hum Mol Genet 2006;15:1133)
    Case reports
    Treatment
    • Symptom management only (Int J Dermatol 2005;44:1):
      • Daily bathing and moisturizing twice daily
      • Alpha hydroxy acids to reduce xerosis
      • Antibiotic ointment applied to fissures
      • Retinoids in severe cases, but use with caution as may exacerbate blistering (An Bras Dermatol 2011;86:S72)
    Clinical images

    Images hosted on other servers:

    Corrugated cardboard-like

    Palms

    Anterior ankles

    Close up of ankle

    Presenting as erythroderma and blistering


    Hyperkeratotic papules

    Papules in proximal thigh

    Left palmar hyperkeratosis

    Plantar hyperkeratosis

    Thickened red hyperkeratotic corrugated plaques

    Plaques distributed over neck, back and flexural surfaces

    Gross description
    • Varies based on age
      • Infants have generalized erythema accompanied by erosions and bullae
      • In a few years, lesions evolve into verrucous plaques and scales
    Microscopic (histologic) description
    • Epidermolytic hyperkeratosis (characteristic feature disease is named for), also called granular degeneration, is seen in upper epidermis
      • Granular and vacuolar degeneration of spinous and granular cell layers
      • Microscopically this appears as cleared spaces of variable size around nuclei in upper layers of nucleated epidermis

    • Granular layer contains an increased number of irregularly shaped keratohyaline granules
    • Overlying hyperkeratosis
    • Bullae form due to separation of edematous keratinocytes; keratinocytes fall apart (epidermolysis) due to lack of normal cytoplasmic keratin filament infrastructure that normally gives keratinocytes their strength
    • Mild lymphocytic inflammatory infiltrate in upper epidermis
    • Many more mitoses are present than in normal epidermis
    Microscopic (histologic) images

    Images hosted on other servers:

    Focal epidermolytic hyperkeratosis

    Continuous epidermolytic hyperkeratosis

    Mosaic

    Epidermolytic ichthyosis

    Parakeratosis, hypergranulosis
    and vacuolar degeneration of
    granular and spinous layer

    Positive stains
    • Suprabasal epithelium shows increased expression of keratin 14
    Electron microscopy description
    • Increase and clumping of tonofilaments
    • Increased and premature formation of keratohyaline granules of variable size
    • At periphery of cells are numerous keratohyaline granules embedded in thick shells of irregularly-clumped tonofilaments
    • Desmosome attachment to keratinocytes is abnormal due to disturbed desmosome-tonofilament associations, which leads to blister formation
    Differential diagnosis

    Epstein-Barr virus (EBV)
    Definition / general
    • Ubiquitous virus in humans; also known as human herpes virus type 4 (Wikipedia)
    Epidemiology
    • EBV infectious mononucleosis causes viral pharyngitis in all age groups, but particularly in young adults
    • Infection usually occurs by age 15 - 25
    • In US, nearly 50% seroconvert by age 5; 12% of college aged adults convert each year, and half develop acute infectious mononucleosis (US Centers for Disease Control)
    Etiology
    • EBV is transmitted via saliva, and initially infects B cells of oropharyngeal epithelium
    • The circulating infected B cells spread to entire reticuloendothelial system in liver, spleen and peripheral lymph nodes
    Clinical features
    • Incubation period ranges from 1 - 2 months
    • Primary EBV infections are either asymptomatic or cause infectious mononucleosis, also known as glandular fever
    • Infectious mononucleosis has symptoms of fever, lympahedenopathy and sore throat; also petechia of posterior pharynx (can also be seen in group A streptococcal pharyngitis), not present in other pharyngitis
    • May affect other organs
    • Skin involvement in 10%; described as maculopapular exanthema with faint, diffuse, non-itchy rash, initially of trunk and upper arms, lasting for one week; rash is less intense than the transient, morbilliform, hypersensitivity reaction type rash of extensor surfaces appearing 7-10 days after treatment with beta-lactam antibiotics (ampicillin, amoxicillin and cephalosporin)
    • Complications include spontaneous splenic rupture, which requires surgical intervention
    • Can cause oral hairy leukoplakia or non-Hodgkin lymphoma in patients with severe immunosuppression, including HIV
    • In areas within Africa, prediposes children to Burkitt lymphoma
    • Japanese or other Asians may develop chronic infections

    • Chronic active EBV infection: chronic recurrent infectious mononucleosis-like symptoms, high mortality due to virus-associated hemophagocytic syndrome, NK/T cell lymphoma, EBV related cardiovascular disease and large vessel arteritis; also hypersensitivity to mosquito bites in 1 / 3 (may be initial symptom), causing necrotic skin ulceration and scarring
    Laboratory
    • Nonspecific tests are CBC with differential, elevated ESR, liver function tests
    • Specific tests are heterophile antibody tests, monospot test (sensitivity of 85%, specificity of 100%); if original monospot test is negative, repeat in 6 weeks if high suspicion
    • Also antibodies to EBV capsid and nuclear antigen (for surgical specimens); PCR assay for EBV DNA
    Case reports
    • 11 year old Japanese-Filipino girl with chronic active EBV infection and mosquito bite hypersensitivity, leading to EBV+, NK/T cell lymphoma (Hum Pathol 2005;36:212)
    Treatment
    • Usually bed rest and analgesia during febrile stage
    • Acyclovir and ganciclovir sometimes prescribed to reduce EBV shedding, but effectiveness is questionable
    • Short course of corticosteroids are indicated if hemolytic anemia, thrombocytopenia, CNS involvement or severe tonsillar enlargement
    • May have prolonged malaise for several months
    Microscopic (histologic) description
    • Oropharyngeal epithelium has intense lymphoproliferative response
    Cytology description
    • Characteristic atypical lymphocytes
    Cytology images

    Images hosted on other servers:

    Atypical lymphocytes have indented and slightly larger cytoplasm with nucleoli

    Differential diagnosis

    Erysipelas
    Definition / general
    • Erysipelas is a superficial form of cellulitis and is almost always caused by Streptococcus pyogenes (Wikipedia)
    Epidemiology
    • Can affect any age group but commonly affect infants and elderly
    • Tracing back to the Middle Ages, it historically occurred on the face and was referred to as St. Anthonys Fire, after an Egyptian healer (eMedicine)
    • More recently, its distribution has shifted to the lower extremities due to an aging population with risk factors such as leg venous insufficiency; 75% of cases now occur in legs (Int J Dermatol 2010;49:1012)
    • Its incidence has declined throughout the 20th century, possibly due to antibiotics and improved sanitation
    Clinical features
    • Lesions predominantly appear on lower limbs, but when it involves the face, it gives rise to a characteristic butterfly distribution on the cheeks and bridge of the nose
    • Usually abrupt onset; can be distinguished from other forms of cellulitis by a sharply demarcated, raised border
    • Causes upper dermal edema with blockage of the superficial lymphatics, causing the clinical orange peel or peau dorange appearance of the affected skin and localized lymphadenopathy
    • May be accompanied by constitutional symptoms of fever, chills, malaise
    • Preexisting lymphedema is a known risk factor (Acta Dermatovenerol Alp Panonica Adriat 2009;18:63)

    • Rare complications include:
      • Infective endocarditis or septic arthritis with septicemia
      • Post-streptococcal glomerulonephritis in children
      • Facial erysipelas can cause cavernous sinus thrombosis
      • Streptococcal toxic shock syndrome
    Treatment
    • IV and PO penicillin
    • Erythromycin can be used in those with penicillin allergies
    • Treatment requires 10 - 14 days of antibiotics; no scarring occurs
    • Prognosis is excellent with appropriate antibiotic therapy; however, local recurrence has been reported in up to 20% of patients with predisposing conditions
    • Recurrent erysipelas is treated with local antiseptics, general wound care and long term management of lymphedema (Neth J Med 2007;65:89)
    • Currently, there are no guidelines for long term antibiotics; one regimen is 2.4 MU benzathin-penicillin G IM every 2 weeks, for up to 2 years (J Eur Acad Dermatol Venereol 2006;20:818)
    Clinical images

    Images hosted on other servers:

    Various images

    Microscopic (histologic) description
    • Marked dermal edema, vascular dilatation and streptococcal invasion of lymphatics and tissues
    • Dermal inflammatory infiltrate consist of neutrophils and mononuclear cells
    Additional references

    Erythema annulare centrifugum
    Definition / general
    • Annular, erythematous, well circumscribed, mildly pruritic eruption with classic trailing scale (scale is seen behind the advancing erythematous edge)
    • Histopathology shows tightly cuffed or coat sleeve perivascular lymphohistiocytic inflammation
    • Classified into superficial and deep type, based on depth of inflammation; superficial type is more likely to have epidermal changes, such as spongiosis and mounds of parakeratosis (Calonje: McKee’s Pathology of the Skin, 5th Edition, 2019)
    Essential features
    • Clinically, annular erythematous plaques with central clearing and trailing scale, with a superficial or deep coat sleeve perivascular lymphohistiocytic inflammation, with or without spongiosis and parakeratotic scale
    • Clinicopathologic correlation is necessary, since the superficial variant may be indistinguishable from pityriasis rosea (Patterson: Weedon’s Skin Pathology, 4th Edition, 2015)
    • Erythema annular centrifugum (EAC) is often recurrent (Ann Dermatol 2016;28:257)
    Terminology
    ICD coding
    • ICD 10: L53.1 - erythema annulare centrifugum
    Epidemiology
    Sites
    Pathophysiology
    • Regarded as a hypersensitivity type reaction in response to a variety of triggers but the true pathophysiology remains unknown (J Dermatol 2002;29:61)
    Etiology
    Clinical features
    • Lesions start as urticarial smooth erythematous papules that enlarge centrifugally in annular erythematous plaques; these clear centrally and have a characteristic trailing scale
    • Advancing erythematous edges are typically raised
    • Superficial type may have pruritus and scale
    • Deep type may have indurated borders and no scale
    • Often recurrent, with relapsing / remittent course, including some cases showing seasonal or annual recurrences (Calonje: McKee’s Pathology of the Skin, 5th Edition, 2019)
    Diagnosis
    • Easily made on examination of a punch biopsy specimen taken from the edge of the lesion with trailing scale
    Laboratory
    • Dependent on the underlying association, nonspecific
    Prognostic factors
    • One study showed that the superficial type may show shorter duration of skin lesion and better response to therapy but may have higher rate of recurrence (Ann Dermatol 2016;28:257)
    Case reports
    Treatment
    Clinical images

    Contributed by Silvija P. Gottesman, M.D.

    Annular eruption with scale

    Microscopic (histologic) description
    Microscopic (histologic) images

    Contributed by Silvija P. Gottesman, M.D.

    Perivascular lymphohistiocytic inflammation

    Subtle epidermal changes

    Virtual slides

    Images hosted on other servers:

    Perivascular coat sleeve inflammation

    Negative stains
    • PAS special stain in search of hyphae should be performed on all EAC biopsies to exclude dermatophytosis
    Sample pathology report
    • Skin, right upper leg, punch biopsy:
      • Superficial perivascular lymphohistiocytic inflammation with spongiosis and parakeratotic scale crust (see comment)
      • Comment: Multiple levels were examined showing a tightly cuffed perivascular lymphohistiocytic infiltrate with subtle basilar spongiosis with parakeratosis. The histopathologic differential includes erythema annulare centrifugum and pityriasis rosea. Submitted clinical image was reviewed; annular erythema with trailing scale is noted and a diagnosis of erythema annulare centrifugum is favored. PAS stain for fungal organisms is negative.
    Differential diagnosis
    Board review style question #1
    A 55 year old man presents with a 2 week history of circular rash on his bilateral thighs. The notes mention that although he had a positive fecal occult blood test (FOBT) last year, he has not been to the gastroenterologist for follow up, due to concerns regarding cost. On exam, the dermatologist notes the lesions are annular and raised, with trailing scale and central clearing. A punch biopsy is sent for histopathologic evaluation. Histology shows a well demarcated perivascular lymphohistiocytic infiltrate in the superficial dermis with subtle spongiosis and scale. What is the most likely diagnosis and appropriate next step?

    1. Erythema annulare centrifugum; include comment regarding association with malignancy among other triggers and need for further clinical evaluation
    2. Pityriasis rosea; inquire about recent upper respiratory symptoms and monitor for resolution in 2 - 6 weeks
    3. Secondary syphilis, the great mimicker, can have various histopathology outside the classic; perform an RPR
    4. Tinea corporis; perform a PAS stain to confirm
    Board review style answer #1
    A. Erythema annulare centrifugum; include comment regarding association with malignancy among other triggers and need for further clinical evaluation. The most likely diagnosis is erythema annulare centrifugum (EAC); a clinicopathologic diagnosis combining the findings of annular clinical eruption with scale and a perivascular lymphohistiocytic infiltrate on histopathologic sections. EAC will sometimes show an association with underlying malignancy (sometimes called paraneoplastic erythema annulare centrifugum or PEACE). Given the patient’s history of positive FOBT and this new onset of rash, a comment or phone call to the clinician may be warranted to ensure this patient receives appropriate follow up care (Calonje: McKee’s Pathology of the Skin, 5th Edition, 2019).

    Comment Here

    Reference: Erythema annulare centrifugum
    Board review style question #2


    A 35 year old woman, with a history of COVID-19 infection 2 months prior, presents to the dermatologist with 1 month history of a raised circular rash. Pathology is shown in the images above. What is the most likely diagnosis?

    1. Annular elastolytic giant cell granuloma
    2. Erythema annulare centrifugum
    3. Erythema elevatum diutinum
    4. Granuloma annulare
    Board review style answer #2
    B. Erythema annulare centrifugum. There are many similar sounding names in dermatopathology, which can be confusing. Of the available choices, erythema annulare centrifugum is the only one that fits the histologic and clinical picture. Erythema elevatum diutinum is a vasculitic process presenting with papules / nodules on extensor surfaces with histologic features similar to leukocytoclastic vasculitis and onion skin fibrosis. Granuloma annulare (GA) and annular elastolytic giant cell granuloma (AEGC) may present with annular lesions but histology will show granulomatous inflammation with degenerating elastic fibers in the case of AEGC and central necrobiotic collagen and mucin in the case of GA (Calonje: McKee’s Pathology of the Skin, 5th Edition, 2019).

    Comment Here

    Reference: Erythema annulare centrifugum

    Erythema dyschromicum perstans (pending)

    Erythema elevatum diutinum
    Definition / general
    • Rare, chronic fibrosing vasculitis associated with a variety of systemic diseases
    Essential features
    • Red-brown plaques and nodules on extensor and acral surfaces
    • Histopathology reflects spectrum of overlapping findings dependent on stage of disease
      • Early: neutrophilic infiltrate and leukocytoclastic vasculitis
      • Established: mixed infiltrate with granulomas, granulation and fibrosis
      • Late: prominent fibrosis obscures sparse neutrophils; xanthomatization
    ICD coding
    • ICD-10: L95.1 - erythema elevatum diutinum
    Epidemiology
    • Rare
    • Most commonly affects adults between 30 - 60 years of age but earlier onset observed in association with human immunodeficiency virus (HIV) infection
    • No sex or racial predilection (J Am Acad Dermatol 1992;26:38)
    Sites
    Pathophysiology
    Etiology
    • Unknown
    Clinical features
    • Erythematous or purpuric patches or thin, soft plaques in early disease
    • Red-brown, violaceous or yellow-brown indurated papules, plaques or nodules in established or late disease; lesions may be asymptomatic or associated with pruritus or dysesthesia (J Am Acad Dermatol 1992;26:38)
    • Bulky nodules may be associated with underlying HIV infection (J Cutan Pathol 2018;45:94)
    • Annular plaques simulating granuloma annulare are rare (J Cutan Pathol 2019;46:97)
    • Vesiculobullous presentations are also described (Clin Exp Dermatol 2017;42:777, Indian J Dermatol 2020;65:164)
    • Associated disorders and systemic manifestations:
      • Infections: HIV, hepatitis B, hepatitis C and Streptococcus
      • Gastrointestinal: inflammatory bowel disease and celiac disease
      • Hematologic: IgA monoclonal gammopathy, paraproteinemia, Waldenström macroglobulinemia, lymphoma
      • Autoimmune connective tissue: granulomatosis with polyangiitis, dermatomyositis, systemic lupus erythematosus, rheumatoid arthritis, relapsing polychondritis
      • Ocular: keratitis, scleritis, uveitis, blindness
      • Other: pregnancy, solid organ malignancy (J Eur Acad Dermatol Venereol 2014;28:1594)
    Diagnosis
    • Skin biopsy with histologic examination of tissue
    Laboratory
    • Given the associated disorders, the following laboratory evaluation is recommended:
      • Complete blood count
      • HIV and hepatitis B and C serologies
      • Serum protein electrophoresis or immunofixation
      • Antistreptolysin O
      • Antinuclear antibody
      • Antineutrophil cytoplasmic antibodies (ANCA) testing
    Prognostic factors
    Case reports
    Treatment
    • Oral dapsone monotherapy is effective in 80% of cases, particularly in early disease
    • Alternative medical treatments can be combined with dapsone for refractory cases
      • Topical, intralesional or oral corticosteroids
      • Sulfonamides: sulfasalazine, sulfapyridine
      • Antimicrobials: tetracyclines, macrolides
      • Colchicine
    • Medical therapy including dapsone ineffective for chronic lesions with prominent fibrosis
    Clinical images

    Contributed by Kiran Motaparthi, M.D.
    Plaque on the elbow

    Plaque on the elbow

    Nodules on the shin and foot

    Nodules on the shin and foot

    Nodule on the hand

    Nodule on the hand

    Microscopic (histologic) description
    • Histopathologic features vary based on the stage of disease
      • Early:
        • Leukocytoclastic vasculitis with fibrinoid necrosis of small vessel walls, along with predominantly neutrophilic dermal infiltrate
        • Prominent dermal edema may simulate Sweet syndrome or result in vesiculation (J Am Acad Dermatol 1992;26:38)
      • Established:
        • Polymorphous infiltrate includes neutrophils but also lymphocytes, histiocytes, plasma cells and eosinophils
        • Granulation tissue
        • Perivascular fibrosis in onion skinning pattern
        • Granuloma formation is variable (Clin Exp Dermatol 1992;17:87)
      • Late:
    Microscopic (histologic) images

    Contributed by Kiran Motaparthi, M.D.
    Neutrophilic infiltrate

    Neutrophilic infiltrate

    Nodular mixed infiltrate

    Nodular mixed infiltrate

    Dilated vessels and prominent fibrosis

    Dilated vessels and prominent fibrosis

    Small vessel vasculitis

    Small vessel vasculitis

    Storiform fibrosis

    Storiform fibrosis

    Virtual slides

    Images hosted on other servers:

    Erythema elevatum diutinum

    Immunofluorescence description
    Negative stains
    Electron microscopy description
    Sample pathology report
    • Skin, extensor forearm, punch biopsy:
      • Erythema elevatum diutinum
    Differential diagnosis
    Board review style question #1

      A 34 year old man presents with multiple firm red-brown nodules distributed over the acral and extensor surfaces. Which of the following features is observed in all stages of this disease?

    1. Cholesterol clefts
    2. Dermal edema
    3. Fibrosis
    4. Granulomas
    5. Neutrophils
    Board review style answer #1
    E. Neutrophils. This is erythema elevatum diutinum.

    Comment Here

    Reference: Erythema elevatum diutinum
    Board review style question #2

      Which of the following disorders demonstrates overlapping histopathologic features with early erythema elevatum diutinum?

    1. Palisaded neutrophilic granulomatous dermatitis
    2. Rheumatoid nodule
    3. Sclerotic fibroma
    4. Sweet syndrome
    5. Tuberous xanthoma
    Board review style answer #2
    D. Sweet syndrome

    Comment Here

    Reference: Erythema elevatum diutinum

    Erythema induratum (pending)

    Erythema migrans
    Definition / general
    • Erythema migrans is the most common manifestation of Lyme disease
    • Classically presents as erythematous, annular, homogeneous patches with central punctum
    • Characteristic bullseye appearance with central clearing may be present in only around 9% of cases (Clin Dermatol 2006;24:509)
    • Usually caused by spirochete bacteria Borrelia burgdoferi, which is most frequently transmitted by the tick Ixodes scapularis
    • Reference: Infect Dis Clin North Am 2015;29:211
    Essential features
    • Primary erythema migrans is the first stage of Lyme disease (early localized disease)
    • No serology is recommended if only primary erythema migrans is present given the low true seropositivity for early localized disease (< 40%)
    • Histology of erythema migrans commonly shows nonspecific superficial and deep perivascular infiltrates with or without periadnexal involvement
    • Prognosis is excellent with > 90% of patients returning to their previous health status when prompt antimicrobial treatment is initiated
    • Reference: Nat Rev Dis Primers 2016;2:16090
    Terminology
    • Erythema chronicum migrans
    ICD coding
    • ICD-10:
      • A69.2 - Lyme disease, erythema chronicum migrans due to Borrelia burgdorferi
      • A26.0 - cutaneous erysipeloid, erythema migrans
    • ICD-11:
      • 1C1G.0 - early cutaneous Lyme borreliosis
    Epidemiology
    • An estimated 300,000 cases of Lyme disease occur annually in the U.S. (Infect Dis Clin North Am 2015;29:211)
      • Up to 95% of cases occur in New England, the Middle Atlantic and North Central regions
      • Incidence of erythema migrans is likely underestimated due to unnoticed skin lesions, a lack of systemic or local symptoms and unreported cases due to false negative serologic testing
      • 2 peaks in age distribution
        • 5 - 14 years old
        • 45 - 54 years old
      • Majority of cases occur in late spring and summer due to an increase in outdoor activity
    Sites
    • Erythema migrans lesions can occur on almost every part of the body, including the trunk, all extremities, groin, axilla and buttocks (Infect Dis Clin North Am 2015;29:211)
      • Mucous membranes, palms and soles are exempt from developing erythema migrans lesions
      • Some patients may present with multiple annular secondary lesions
        • These contain viable spirochetes due to hematogenous or lymphatic spread from the original tick bite
    • In children, the most common regions affected are the head and neck; in adults, typically the extremities or pelvis area (J Autoimmun 2015;57:82)
    Pathophysiology
    • Borrelia burgdoferi (spirochetes) replicate in the midgut of Ixodes scapularis (tick) and subsequently migrate to the salivary glands (Biophys J 2014;106:763)
      • As the Ixodes tick feeds on human skin the B. burgdoferi spirochetes are transported into the dermis
        • This process of spirochete migration from the tick's midgut to the dermis takes at least 36 - 48 hours of direct feeding / tick attachment (Infect Dis Clin North Am 2015;29:211)
        • At the end of the blood meal, a small punctum on the skin at the site of spirochete inoculation becomes present
      • In the dermis, spirochetes replicate rapidly locally and may travel into the bloodstream where they can spread diffusely
      • Spirochetes remain bound to the extracellular matrix during replication and may adhere
      • Innate and adaptive immune responses are activated and release proinflammatory cytokines which result in localized erythema and swelling
      • Bullseye appearance is likely a result of spirochete replication / spreading, phagocytic cell activation / migration and deactivation of host phagocytes once the bacteria are cleared
      • Other Ixodes species may less commonly transmit Lyme disease, including I. pacifcus and I. ricinus
    Etiology
    • Usually transmitted to humans by a bite from Ixodes ticks infected with B. burgdorferi (Biophys J 2014;106:763)
    • Rarely may be due to other tick borne pathogens, such as Amblyomma americanum (Am J Med 2017;130:231)
    Diagrams / tables

    Images hosted on other servers:

    Lyme disease U.S. distribution

    Ixodes scapulais life cycle

    Clinical features
    • 3 stages of Lyme disease are: (1) early localized, (2) early disseminated and (3) late disseminated (J Autoimmun 2015;57:82)
    • Primary erythema migrans is the first stage (early localized disease) (J Cutan Pathol 2016;43:32)
      • Presents in ~70 - 95% of patients within the first 3 weeks of inoculation
      • Lesions have either a central clearance with ovoid expansion or a targetoid appearance
      • Size of lesions can be highly variable, although > 5 cm is required according to the CDC for a diagnosis of primary erythema migrans (Infect Dis Clin North Am 2015;29:211)
      • Local pruritus and burning are not uncommon
      • Represents local cutaneous spirochete infection in the area of tick bite
    • Secondary erythema migrans presents as multiple lesions with similar annular appearance / characteristics, although their morphology may vary (Indian J Dermatol 2013;58:167)
      • Frequently affected sites include the face and extremities
      • More likely to be accompanied by systemic symptoms, such as malaise, lymphadenopathy and fever
      • Suggests disseminated spirochetes in the blood or lymph
    Diagnosis
    • Erythema migrans is typically a clinical diagnosis (e.g., characteristic skin lesion appearance, epidemiologic / exposure history) (Infect Dis Clin North Am 2015;29:211)
      • As only ~25% of U.S. patients with erythema migrans recall a preceding tick bite, this history is not essential
      • Serology may be negative in early disease, and patients with the charactersitic erythema migrans rash and suggestive history should be treated clinically
      • In later stages of disease (e.g., cardiac manifestations), 2 tier confirmation with quantitative screening for serum antibodies to B. burgdorferi via enzyme of antibody immunoassays followed by Western blot in positive or equivocal specimens is recommended
      • PCR positivity for Borrelia in skin samples may be positive in approximately 77% of cases (PLoS One 2015;10:e0136600)
      • Culture positivity with isolation of B. burgdorferi is the gold standard for confirming the diagnosis via laboratory studies
        • However, this is the least practical for diagnosis as it requires special media and takes a long time for growth
        • Positive serology and quantitative polymerase chain reaction (PCR) analyses of blood and skin samples are also supportive
    Laboratory
    • IgM seropositivity for B. burgdorferi (or other infectious agents) (J Cutan Pathol 2016;43:32)
      • IgG seronegativity for the same etiologic agent of erythema migrans is more likely in acute infections
      • Seropositivity may be more likely in males than females and less likely in children / adolescents (PLoS One 2012;7:e41321)
    • B. burgdorferi can be isolated from leading margins of erythema migrans lesions and cultured (PLoS One 2015;10:e0136600)
      • Quantitative PCR analysis on tissue / culture is supportive
    • Routine laboratory tests (e.g., complete blood counts, liver enzyme assays, sedimentation rate) are often not helpful (except in cases of coinfection with other infectious agents) (Infect Dis Clin North Am 2015;29:211)
    Prognostic factors
    • Prognosis is excellent with > 90% of patients returning to their previous health status when prompt antimicrobial treatment is initiated (Eur J Clin Microbiol Infect Dis 2019;38:201)
    • Erythema migrans lesions often resolve spontaneously (usually within 4 weeks) even when no treatment is administered (Infect Dis Clin North Am 2015;29:211)
      • Untreated patients may experience recurrent primary erythema migrans, secondary lesions or both
      • Some patients may experience evanescent lesions
      • Manifestations of later stages of Lyme disease may develop in patients that do not receive treatment
        • These include joint symptoms, neurologic abnormalities and cardiac involvement
      • 20% of untreated patients have no subsequent manifestation of Lyme disease after the resolution of erythema migrans lesions
    • New lesions of erythema migrans occurring after successful antimicrobial treatment of a prior episode of Lyme disease are likely due to repeat tick bites (Infect Dis Clin North Am 2015;29:211)
      • Human immune systems are not fully protected against reinfection
      • Strain variability of B. burgdorferi is likely contributive
      • Patients with later stages of Lyme disease are unlikely to develop reinfection due to a greater developed immune response
    Case reports
    Treatment
    • Oral amoxicillin, doxycycline and cefuroxime axetil are recommended initial therapies to treat erythema migrans (Infect Dis Clin North Am 2015;29:211)
      • Amoxicillin and cefuroxime axetil are preferred for pediatric patients (< 8 years old) and pregnant patients
    • Alternative therapies include oral azithromycin, clarithromycin and erythromycin (Infect Dis Clin North Am 2015;29:211)
      • These drugs have lower efficacy and are reserved for patients who are intolerant of tetracyclines, penicillins and cephalosporins
    Clinical images

    Images hosted on other servers:

    Large erythema migrans on abdomen

    Erythema migrans on leg

    Multiple erythema migrans lesions

    Microscopic (histologic) description
    • Commonly nonspecific superficial and deep perivascular lymphohistiocytic infiltrate with or without periadnexal involvement (Am J Dermatopathol 2012;34:834)
      • Central eosinophils and peripheral plasma cells are typically present
      • Spongiosis, peripheral neutrophils, periadnexal without perivascular involvement and absence of plasma cells may all be seen in certain cases
      • Accompanied focal interface dermatitis may be suggestive (Am J Dermatopathol 2020;42:745)
    • Epidermal changes may occur centrally in conjunction with dermal changes typical of an arthropod bite reaction (lymphocytes, histiocytes, eosinophils, mast cells) while peripheral changes tend to be more lymphoplasmacytic without any epidermal changes
    • Increased vascularity and even epidermal necrosis may be seen
    • Warthin-Starry stain may identify organisms
    • IHC and tissue PCR on formalin fixed, paraffin embedded (FFPE) tissue also described
    Positive stains
    Molecular / cytogenetics description
    • No serology is recommended if only primary erythema migrans is present given the low true seropositivity for early localized disease (< 40%) (JAMA 2016;315:1767)
      • If other symptoms are present or suspected later stages of Lyme disease, the CDC recommends 2 step serologic testing consisting of (step 1) enzyme immunoassay followed by (step 2) western blot analysis if IgM / IgG results for B. burgdorferi are positive or equivocal for step 1 (JAMA 2016;315:1780)
    Sample pathology report
    • Left upper extremity skin, biopsy:
      • Gyrate erythema, most consistent with erythema migrans (see comment)
      • Comment: Specimen shows superficial and deep mixed perivascular inflammatory infiltrates, including eosinophils, lymphocytes, histiocytes and plasma cells. Periadnexal involvement is detected as well as focal interface dermatitis. While the changes are nonspecific, the associated clinical history of annular erythematous rash with central punctum with a recent history of outdoor travel are most suggestive of the diagnosis above.
    Differential diagnosis
    • If an erythematous lesion develops in the setting of a suspected bite (Infect Dis Clin North Am 2015;29:211):
      • Arthropod bite hypersensitivity reaction:
        • Largest diameter is < 5 cm
        • Can be associated with significant pruritus
        • Lesions may fade spontaneously within 24 - 48 hours
        • Lack of systemic symptoms
      • Brown recluse spider bite:
        • Focal necrosis and ulceration with or without the red, white and blue sign
        • Preferentially occurs on extremities and spreads centrifugally
        • Endemic to south central United States (such as southeastern Nebraska and southern Ohio)
    • Other causes of erythematous lesions (Infect Dis Clin North Am 2015;29:211):
      • Bacterial cellulitis:
        • Homogenous band-like erythema that develops suddenly and is usually painful
        • Patient often presents with leukocytosis, toxic appearing
        • Distal lower extremities are preferentially affected
        • History of recent trauma at the site of the lesion, underlying peripheral vascular disease or surgical history involving the venous or lymphatic systems
      • Herpes simplex / zoster:
        • Grouped vesicles on an erythematous base
        • Oral, hand / finger, genital locations (Herpes simplex)
        • Dermatomal distribution, prior history of chickenpox (Herpes zoster)
        • Extremely painful
      • Dermatophyte infection / tinea:
        • Scaling, thin and irregularly raised borders with central clearing
        • Resolves slowly (over a matter of weeks)
        • Pruritic
        • Typically not associated with systemic symptoms, targetoid appearance (Infect Dis Clin North Am 2015;29:211)
    • Figurate erythemas (J Dtsch Dermatol Ges 2021;19:963):
      • Erythema annulare centrifugum (EAC):
        • Classically, initial lesions present with urticaria-like papules that spread centrifugally with central clearing
        • Persistent annular erythema that may resolve spontaneously
        • Peak incidence mid adult life, M:F = 1:1
        • Most common locations include trunk and proximal extremities
        • 2 histologic variants: superficial and deep
          • Superficial erythema annulare centrifugum tends to show spongiosis, parakeratosis and superficial perivascular lymphocytes and histocytes
            • Lesions may demonstrate trailing scale and pruritus clinically
          • Deep erythema annulare centrifugum does not show epidermal changes and has mid to deep dermal mononuclear infiltrates
            • Lesions tend to be nonscaling and nonpruritic with cord-like borders
      • Erythema gyratum repens (EGR):
        • Mainly affects Caucasian males (M:F = 2:1) with average age of onset ~63 years old
        • Clinically characterized by rapidly migrating (e.g., 1 centimeter / day), concentric, annular, erythematous bands with peripheral scale and pruritus
        • May present with palmoplantar keratoderma, hypereosinophilia or ichthyosis
        • Common locations include large areas of body (e.g., trunk, extremities), with acral and facial sparing
        • Histology, although nonspecific, may show focal parakeratosis, spongiosis and superficial perivascular lymphocytes with eosinophils
        • Clinical diagnosis and is strongly associated with underlying malignancy, such as lymphoma, lung, esophageal, breast, stomach and genitourinary cancer
      • Erythema marginatum:
        • Also known as erythema marginatum rheumaticum (EMR)
        • Migratory annular and polycyclic erythematous lesions that exhibit fluctuance, fading and reappearing
        • Cutaneous presentation of acute rheumatic fever (ARF), which is autoimmune disease secondary to group A beta hemolytic streptococcal pharyngitis
        • Often has a 2 - 5 week latency period after initial pharyngitis infection
        • Lesions favor proximal extremities and trunk, while sparing the face and acral surfaces
        • Often copresents with other acute rheumatic fever signs, including carditis, polyarthritis, chorea and subcutaneous nodules
        • Patients may also have fever, elevated inflammatory markers (e.g., C reactive protein) or prolonged PR interval on electrocardiogram
        • Clinical diagnosis
        • Presents more frequently in children than adults
    Board review style question #1

    A 65 year old man presents with a rash. He states that he recently went on a weekend hiking trip and noticed a rash on his abdomen that has been slowly enlarging over the past 5 days. He endorses mild pruritus but denies any other associated symptoms. On physical exam, an erythematous, annular patch with central punctum and targetoid appearance is present on the patient’s abdomen. What is the most likely etiology of this patient’s condition?

    1. Amblyomma americanum
    2. Babesia canis
    3. Borrelia burgdorferi
    4. Ehrlichia chaffeesis
    5. Mycoplasma pneumoniae
    Board review style answer #1
    C. Borrelia burgdorferi is a spirochete bacteria and is the most common pathogen responsible for Lyme disease. Early localized Lyme disease presents as erythema migrans, which are enlarging, bullseye, annular, erythematous lesions in sites of Ixodes scapularis (tick) bites. Clinical history is essential and typically includes a prior history of outdoor activity, namely in endemic regions such as the Northeast United States. A central punctum may be seen in the center of the lesion and ~25% of patients are unable to specify prior history of a known tick bite / blood feed.

    Comment Here

    Reference: Erythema migrans
    Board review style question #2
    For how long must direct Ixodes scapularis blood feeding take place for spirochete inoculation in primary erythema migrans?

    1. 2 - 4 hours
    2. 6 - 12 hours
    3. 14 - 20 hours
    4. 24 - 32 hours
    5. 36 - 48 hours
    Board review style answer #2
    E. 36 - 48 hours. The process of spirochete migration from the tick’s midgut to the dermis in erythema migrans takes at least 36 - 48 hours of direct feeding and tick attachment.

    Comment Here

    Reference: Erythema migrans

    Erythema multiforme
    Definition / general
    • Acute, self limited, hypersensitivity reaction to infections (coccidioidomycosis, herpes simplex, histoplasmosis, leprosy, mycoplasma, typhoid), drugs (penicillin, phenylbutazone, phenytoin, salicylates, sulfa), carcinoma / lymphoma, or collagen vascular disorders
    • Affects skin (distal extremities, palms, soles) and mucous membranes with target lesions
    • Also sore throat and malaise
    • Any age
    • Commonly recurs but rarely persists
    Clinical features
    • Variable (multiform) lesions, including papules, macules, vesicles, bullae, target lesions
    • Commonly in mucous membranes; also elbows, knees, extensor surface of extremities
    Clinical images

    Contributed by Mark R. Wick, M.D.
    Microscopic (histologic) description
    • Subepidermal bullae with basement membrane in bullae roof due to dermal edema
    • Severe dermal inflammatory infiltrate (includes lymphocytes, histiocytes)
    • Eosinophils may be present, but neutrophils are sparse or absent
    • Overlying epidermis often demonstrates liquefactive necrosis and degeneration, dyskeratotic keratinocytes
    • May also have dermoepidermal bullae with basal lamina at floor of bullae
    • Variable epidermal spongiosis and eosinophils
    • No leukocytoclasis, no microabscesses, no festooning of dermal papillae

    • Note: erythema multiforme may have variable histologic changes from toxic epidermal necrolysis to dermal disturbances
    Microscopic (histologic) images

    Contributed by Mark R. Wick, M.D.



    Images hosted on other servers:

    Various images

    Positive stains
    • Granular C3 and IgM at basement membrane and in vessels
    Differential diagnosis

    Erythema nodosum
    Definition / general
    • Panniculitis with tender red nodules, usually on both shins
    Terminology
    • Erythema nodosum migrans (subacute nodular migratory panniculitis, migratory panniculitis): asymmetrical, unilateral and distributed solely on the legs; marked female predominance; older age group
    • Chronic erythema nodosum: nodules appear over months / years; otherwise indistinguishable from typical condition
    Clinical features
    • Red, painful, bilateral, symmetrical nodules, elevated above the skin surface
    • On anterior surface of legs, arms, face, calves and trunk
    • Usually involutes in days / weeks, leaving depressed, pigmented lesions
    • No ulceration
    • Immune mediated but precise mechanism is unknown
    • May be associated with streptococcus infection, tuberculosis, sarcoidosis, coccidioidomycosis, ulcerative colitis, Behçet disease, drug reactions or idiopathic; also Hodgkin lymphoma, renal cell carcinoma and carcinomas of colon, pancreas and uterine cervix
    Clinical images

    Images hosted on other servers:

    Various images

    Microscopic (histologic) description
    • A prototype of septal panniculitis, characterized by vascular changes, septal inflammation, hemorrhage and variable acute and chronic panniculitis
    • Both septal and lobular panniculitis
    • Marked septal fibrosis, infiltrated by lymphocytes, neutrophils, histiocytes and granulomas with giant cells
    • Septal infiltrate spills over to affect the fat lobules
    • Dermis shows perivascular and periadnexal chronic inflammatory cell infiltrate
    • Early, the septal inflammation is acute and characterized by neutrophils, soon replaced by lymphocytes and histiocytes
    • Variable eosinophils, variable vasculitis

    • Miescher radial granuloma: characteristic finding of erythema nodosum; septal collection of histiocytes surrounding a cleft (appear to look like spaces); reported in Sweet syndrome, nodular vasculitis and necrobiotic lipodica
    Differential diagnosis
    • Nodular vasculitis or subacute nodular migratory panniculitis: usually septal
    • Weber-Christian disease associated panniculitis: usually lobular inflammation

    Erythema toxicum neonatorum
    Definition / general
    • Skin condition in 50% of newborns, with red patches on forehead, face, chest, trunk and extremities that have raised clear bumps in the center; usually resolves within a few days without treatment
    Clinical features
    • Pathogenesis unknown; may be an acute graft versus host reaction, resulting from the transfer of maternal lymphocytes during delivery
    • Erythematous pustular eruption
    • Can be diagnosed clinically based on classic appearance (Am Fam Physician 2008;77:47)
    Clinical images

    Images hosted on other servers:

    Red patches with raised clear central bumps

    Microscopic (histologic) description
    • Early erythematous lesion: shows perivascular inflammatory cell infiltrate with conspicuous eosinophils
    • Intraepidermal pustule (subcorneal): filled with eosinophils and occasional neutrophils
    Differential diagnosis
    • Incontinentia pigment: prominent eosinophilic spongiosis, not seen in erythema toxicum neonatorum
    • Miliaria rubra (heat rash): vesicles are related to sweat ducts, not hair follicles, and typically contain mononuclear cells, not eosinophils

    Erythrasma (pending)

    Familial benign chronic pemphigus (Hailey-Hailey disease)
    Definition / general
    • Autosomal dominant genodermatosis characterized by intraepidermal vesicles located predominately in intertriginous area
    • Vesicles may progress to bullae, then rupture, forming a crust
    • Histology shows intraepidermal acantholysis due to nonfunctional desmosomal protein complexes
    Essential features
    • Genodermatosis with autosomal dominant inheritance pattern (mutation in ATPC1 gene), 33% of cases are sporadic
    • Presents in second to forth decades, improves with age
    • Vesicles / bullae in intertriginous areas (axillae, groin, perianal and inframammary areas). exacerbated by perspiration
    • Rarely involves other areas
    • Relapsing remitting clinical course
    • Secondary infection (bacterial, herpetic) can be problematic and serious
    • Main treatment is topical steroids and topical antimicrobials
    Terminology
    • Also called benign familial pemphigus
    • Papular acantholytic dermatosis in genitocrural region may be a localized form of H-H disease
    ICD coding
    • ICD-10: Q82.8 - Other specified congenital malformations of skin
    Epidemiology
    Sites
    • Axillae, groin, perianal and inframammary areas
    • Rarely involves other sites
    Pathophysiology
    • Mutation in ATP2C1, a calcium pump important in normal function of desmosomal protein complex
    • Dissociation of intracellular and extracellular domains of desmosomal cadherin and E-cadherin (adherens junction associated protein)
    • Faulty calcium pump action leads to disorganized function of desmogleins, which are calcium dependent adherence proteins (cadherins) (Indian Dermatol Online J 2016;7:147)
    • Hailey-Hailey and Darier disease share similar pathogenesis, as opposed to pemphigus vulgaris, in which autoantibodies develop against desmosomal proteins
    Etiology
    • Blistering lesions in affected individuals may be induced by trauma, heat, UV light, perspiration
    • May be complicated by secondary infection with scabies, bacteria, herpes or yeast
    Clinical features
    • Pruritic / burning, often malodorous lesions
    • Papular, verrucous, annular and vesiculopustular variants are rare
    • Nikolski sign may be positive
    • 33% of cases are sporadic
    • Healing accompanied by hyperpigmentation, but scarring is rare
    • Longitudinal leukonychia (asymptomatic white longitudinal bands on the fingernails in 70% of patients)
    • Superinfection by Candida albicans, herpes virus and Staphylococcus aureus are frequent complications
    • Cases of complication by squamous cell carcinoma have been reported
    • Symptoms worsen during summer, often disappear during winter
    Case reports
    Treatment
    • Topical steroids and topical antimicrobials are the first line treatments
    • Botulinum toxin A injections
    • Cryotherapy
    • Photodynamic therapy
    • Oral magnesium chloride
    • Laser ablation
    • Electron beam radiotherapy
    • Dermabrasion
    • Glycopyrrolate
    • Afamelanotide
    Clinical images

    Images hosted on other servers:
    Missing Image

    Hailey-Hailey in right axilla

    Missing Image

    Hyperpigmented plaques over groin

    Missing Image

    Hailey-Hailey lesions on left groin

    Missing Image

    Figure 2b: Showing
    longitudinal leukonychia
    and ridging
    Microscopic (histologic) description
    • Suprabasilar and intraepidermal clefting
    • Acantholysis of keratinocytes resembling dilapidated brick wall
    • Epidermal hyperplasia
    • No pronounced dyskeratosis but dyskeratotic keratinocytes have well defined nucleus and preserved cytoplasm (unlike pemphigus vulgaris)
    • Corps ronds and grains are rare
    • Adnexal structures are spared
    • Dermis shows variable chronic inflammatory infiltrate
    • Parakeratotic crust may contain neutrophils and bacteria
    • Immunofluorescence negative
    Microscopic (histologic) images

    Contributed by Savita Ries, M.D.
    Missing Image

    Broad intraepidermal vesicles with dilapidated brick appearance



    Images hosted on other servers:
    Missing Image

    Figure 1: HHD on
    right sole, groin
    and trunk

    Molecular / cytogenetics description
    • Autosomal dominant condition with incomplete penetrance
    • ATPase calcium transporting type 2C member 1 gene (ATP2C1) on chromosome 3q21-q24 encodes a CA2+/ Mn2+ ATPase channel pump SPCA1
    Differential diagnosis
    • Darier disease:
      • ATP2A2 mutation
      • Less acantholysis, more dyskeratosis
      • Involves "seborrheic" areas of the body, including scalp, face, upper chest, back
      • Acantholytic zones involve adnexal structures (particularly hair follicles)
      • Numerous corps ronds and grains
    • Grover disease:
      • Narrow vesicles involving only a few rete ridges but may be histologically identical to Hailey-Hailey disease
      • Different clinical distribution of lesions
      • Acquired rather than inherited (no family history)
    • Papular acantholytic dermatosis of the genitocrural region:
      • Localized variant of Hailey-Hailey or Darier disease involving only the perigenital areas
    • Pemphigus vulgaris:
      • No predilection for intertriginous areas
      • Intact epithelium in adjacent epidermis
      • Involvement of adnexal structures
      • Eosinophils on histology
      • Less acantholysis, more dyskeratosis
      • Autoimmune mediated direct immunofluorescence is positive
    • Relapsing linear acantholytic dermatosis:
      • Vesicles follow the lines of Blaschko
    Additional references
    Board review style question #1
      What is the underlying pathogenesis of Hailey-Hailey disease?

    1. ATP2A2 mutation
    2. ATP2C1 mutation
    3. Autoantibodies against desmosomal proteins
    4. Autoantibodies against non collagenous domain of type VII collagen
    5. Autoantibodies against the hemidesmosomal antigens bullous pemphigoid antigen 1 (BP1) and 2 (BP2)
    Board review style answer #1
    B. Hailey-Hailey is caused by an autosomal dominant mutation in ATP2C1, a calcium pump important in normal function of desmosomal protein complex. This mutation causes dissociation of intracellular and extracellular domains of desmosomal cadherin and E-cadherin (adherens junction associated protein), and a faulty calcium pump action leads to disorganized function of desmogleins, which are calcium dependent adherence proteins (cadherins).

    Comment Here

    Reference: Hailey-Hailey disease
    Board review style question #2
      What differential diagnosis of Hailey-Hailey is also inherited in an autosomal dominant manner?

    1. Darier disease
    2. Grover disease
    3. Papular acantholytic dermatosis of the genitocrural region
    4. Pemphigus vulgaris
    5. Relapsing linear acantholytic dermatosis
    Board review style answer #2
    A. Darier disease is the only other genodermatosis transmitted in an autosomal dominant manner. The disease is due to the mutation in ATP2A2 that encodes sarco / endoplasmic reticulum Ca(2+) ATPase isoform 2 (SERCA2).

    Comment Here

    Reference: Hailey-Hailey disease

    Fibroelastotic papulosis (pending)
    [Pending]

    Folliculitis
    Definition / general
    • Primary inflammation of a hair follicle, either infectious or noninfectious
    Terminology
    • Perifolliculitis: presence of inflammatory cells in the perifollicular tissues that may involve the adjacent reticular dermis; either primarily lymphocytic (lichen planopilaris, pityriasis rubra pilaris) or granulomatous (perioral dermatitis, rosacea)

    • Pseudolymphomatous folliculitis: facial lesion with dense, polymorphic, mixed lymphocytes around hair follicles and infiltrating follicular epithelium
    Sites
    • Common sites are the face, scalp, thighs, axilla and inguinal area
    Etiology
    • Caused by infection, friction and other causes of follicular trauma, excessive perspiration and occlusion
    • Infectious cases are either superficial (fungi, bacteria, syphilis, viral) or deep (usually granulomatous and due to either fungi or bacteria)
    • Fungal forms may be endothrix (spores are within hair shaft) or ectothrix (spores are on outer surface of hair shaft)
    • Noninfectious cases are either superficial / suppurative (acne vulgaris, rosacea, follicular mucinosis, steroid induced), deep / granulomatous (acne vulgaris-conglobate and keloidal forms or perforating) or spongiotic (Fox-Fordyce disease, atopic dermatitis, pruritic folliculitis of pregnancy)
    • Epidermal growth factor receptor (EGFR) inhibitors appear to be relevant in folliculitis by causing abnormal epidermal differentiation that leads to follicular obstruction and subsequent inflammation (Br J Dermatol 2001;144:1169)
    Clinical features
    • Superficial folliculitis is more common but is often self-limited
    • Patients with recurrent or persistent superficial folliculitis or with deep folliculitis are more likely to seek medical care
    • In recalcitrant folliculitis that failed standard therapy, consider cultures for sensitivity, Gram stain, potassium chloride (KOH) preparation to rule out fungal folliculitis
    • Clinical presentation of superficial folliculitis is acute onset of mildy tender papules and pustules
    • Deep folliculitis is usually associated with more pain and may have suppurative drainage; may cause scarring and permanent hair loss
    Treatment
    • For uncomplicated superficial folliculitis, use antibacterial soaps, good handwashing
    • For recurrent and deep lesions, treat empirically with topical antibiotics
    • Antiobiotics should kill Staphylococcus aureus, the most common pathogen; recommended to use dicloxacillin and cephalosporins (eMedicine)
    • For MRSA, use clindamycin, bactrim, minocycline or linezolid
    • For Staphylococcus aureus carriers, treat patient and family with mupirocin ointment BID x 5 days or rifampin 600 mg/day x 10 days
    Clinical images

    Images hosted on other servers:

    Folliculitis

    Microscopic (histologic) description
    • Superficial folliculitis has moderate inflammatory cells in the follicular ostium and upper regions of the follicle
    • Infiltrate initially consists of neutrophils, later becomes more mixed with lymphocytes and macrophages
    Microscopic (histologic) images

    Images hosted on other servers:

    Micro image of folliculitis

    Differential diagnosis

    Folliculitis decalvans
    Definition / general
    • Folliculitis decalvans is a neutrophilic scarring alopecia characterized by pustules, perifollicular erythema and follicular tufting
    Essential features
    • Neutrophilic scarring alopecia variant
    • More common in men
    • Presents with scarring alopecia, follicular pustules and resultant compound follicles that give the clinical appearance of tufting
    • Typical histologic findings include a dense neutrophilic and mixed inflammatory cell infiltrate involving the upper follicle with accompanying perifollicular and interfollicular fibrosis, loss of sebaceous glands and compound follicle formation
    • Staphylococcus aureus may be cultured from pustules but their pathogenic role is unclear
    • Untreated disease may result in extensive and irreversible hair loss
    • Reference: Dermatol Ther 2008;21:238
    ICD coding
    • ICD-10: L66.2 - folliculitis decalvans
    • ICD-11: ED70.50 - folliculitis decalvans
    Epidemiology
    Sites
    Pathophysiology
    Etiology
    • Possible role for S. aureus
    Clinical features
    • Patients frequently complain of pain, pruritus and a burning scalp sensation (Dermatol Ther 2008;21:238)
    • Presents with erythematous papules and pustules, which eventuate in patches of scarring hair loss
    • Pustules may be present around the periphery of scarred areas (Indian J Dermatol Venereol Leprol 2021;87:569)
    • Aberrant healing of destroyed infundibular epithelium results in numerous terminal hairs protruding from one follicular ostium (compound follicles or tufting)
    • Reported dermoscopic findings include a lack of follicular ostia, tufted hairs, perifollicular starburst pattern and yellow crusts (J Am Acad Dermatol 2021;85:1185)
    • Extensive scarring may be seen in longstanding disease
    Diagnosis
    • History (sex, presence of pustules, location of hair loss)
    • Examination (findings as discussed above)
    • Biopsy
    Prognostic factors
    • If untreated, may lead to extensive scarring (J Am Acad Dermatol 2021;85:1185)
    • Timely treatment in patients with visible remaining terminal or vellus hairs in involved areas may lead to retention of hair
    • Squamous cell carcinoma may occur at sites of longstanding, untreated disease (Dermatol Online J 2019;25:13030)
    Case reports
    Treatment
    Clinical images

    Contributed by Chico J. Collie, M.B.B.S. and Jonathan D. Ho, M.B.B.S., D.Sc.
    Folliculitis decalvans affecting the vertex

    Folliculitis decalvans affecting the vertex

    Microscopic (histologic) description
    Microscopic (histologic) images

    Contributed by Chico J. Collie, M.B.B.S. and Jonathan D. Ho, M.B.B.S., D.Sc.
    Decrease in follicular density

    Decrease in follicular density

    Superior follicle

    Superior follicle

    Deep follicle

    Deep follicle

    Perifollicular infiltrate and follicular destruction

    Perifollicular infiltrate and follicular destruction


    Mixed inflammatory cell infiltrate

    Mixed inflammatory cell infiltrate

    Intrafollicular neutrophils

    Intrafollicular neutrophils

    Compound follicle

    Compound follicle


    Positive stains
    • Gram stain may reveal gram positive cocci in clusters consistent with S. aureus
    • Not routinely performed
    Negative stains
    • PAS / GMS stains should be performed to exclude tinea capitis
    Sample pathology report
    • Skin, scalp, punch biopsy:
      • Neutrophilic scarring alopecia consistent with folliculitis decalvans (see comment)
      • Comment: Transverse sections reveal 10 hairs in anagen and terminal in size. Sebaceous glands are markedly reduced in number. There is a moderately dense perifollicular, interfollicular and perivascular inflammatory cell infiltrate composed of neutrophils, plasma cells, lymphocytes and scattered eosinophils. Intrafollicular neutrophils with superficial pustule formation are seen. The inflammation increases in density towards the infundibulum. There is both concentric perifollicular fibrosis and interfollicular fibrosis. Fibrous tracts and naked hair shafts with a focal foreign body granulomatous reaction are present at deeper levels. Compound follicles in groups of up to 8 hairs are identified. The overlying epidermis is acanthotic and scale crust containing neutrophils are present. PAS stain is negative for fungal elements.
    Differential diagnosis
    • Dissecting cellulitis / folliculitis of the scalp (perifolliculitis capitis abscedens et suffodiens) (Histopathology 2010;56:24):
      • Another form of neutrophilic scarring alopecia
      • Primarily in young black men with large pus filled nodules on the scalp and scarring alopecia
      • Bottom heavy mixed inflammatory cell infiltrate resembling granulation tissue in the deep dermis and subcutis
      • May have sinus tract formation
    • Tinea capitis:
      • PAS positive fungi within hair shafts
    • Central centrifugal cicatricial alopecia (Clin Cosmet Investig Dermatol 2016;9:175):
      • Black women
      • Progressive and symmetric peripheral expansion of the lesions
      • Lymphocyte predominant inflammatory cell infiltrate in the upper portion of the hair follicle
      • Perifollicular but no interfollicular fibrosis
      • With follicular rupture, occasional neutrophils may be seen but compound follicles of > 4 hairs are unusual in this entity
    • Lichen planopilaris / frontal fibrosing alopecia (Int J Dermatol 2006;45:375):
      • Patients with patches of scarring alopecia or loss of frontal hairline; more common in White women but may occur in persons of either sex and any race / ethnicity
      • Lymphocytic scarring alopecia
      • Perifollicular but no interfollicular fibrosis
      • Basal layer vacuolation of the follicular epithelium
    Board review style question #1

    A 25 year old man presents with hair loss involving the vertex of the scalp. Examination reveals numerous pustules, scarring and multiple hairs exiting through a single follicular ostium. A punch biopsy demonstrates the features shown in the associated photomicrographs. PAS stain is negative for fungal elements. What is the most likely diagnosis?

    1. Folliculitis decalvans
    2. Frontal fibrosing alopecia
    3. Lichen planopilaris
    4. Male pattern hair loss
    5. Tinea capitis
    Board review style answer #1
    A. Folliculitis decalvans. This is folliculitis decalvans, which is a neutrophilic scarring alopecia characterized by scarring alopecia, a neutrophilic / mixed inflammatory cell infiltrate and compound follicles (numerous hair shafts in one follicular ostium). Answers B and C are incorrect because frontal fibrosing alopecia and lichen planopilaris are lymphocytic scarring alopecias and do not typically present with pustules. Answer D is incorrect because pattern hair loss is a noncicatricial alopecia characterized by a decreased terminal to vellus ratio and retention of sebaceous glands. Answer E is incorrect because although tinea capitis may have a significant neutrophilic infiltrate and follicular destruction similar to folliculitis decalvans, the PAS stain should demonstrate fungi within hair shafts. Additionally, compound follicles are not a feature of tinea capitis.

    Comment Here

    Reference: Folliculitis decalvans
    Board review style question #2
    Which of the following features favors folliculitis decalvans over dissecting cellulitis of the scalp?

    1. Concentric perifollicular fibrosis
    2. Decreased follicular density
    3. Loss of sebaceous glands
    4. Neutrophilic infiltrate involving the isthmus and infundibulum
    5. Premature desquamation of the inner root sheath
    Board review style answer #2
    D. Neutrophilic infiltrate involving the isthmus and infundibulum. While both folliculitis decalvans and dissecting cellulitis of the scalp are neutrophilic scarring alopecias, folliculitis decalvans involves the superior follicle while the latter involves the inferior follicle. Answers A, B, C and E are incorrect because the features listed are nonspecific may be seen in a variety of scarring alopecias.

    Comment Here

    Reference: Folliculitis decalvans

    Foreign body reaction
    Definition / general
    • Inflammatory response, most often granulomatous, to an exogenous or endogenous substance that is regarded as foreign material by the cutaneous immune system
    Essential features
    • Foreign body reaction refers to the inflammatory response of the cutaneous immune system to either exogenous material or an endogenous substance that is altered in some way so that it is regarded as nonself / foreign
    • Foreign material may be introduced intentionally (such as tattoos of cosmetic fillers) or accidentally (such as metals or minerals at sites of trauma) (Dermatol Clin 2015;33:497)
    • Keratin is one of the most common substances to elicit a foreign body reaction, as it is typically confined to immune privileged sites
    • Though other reaction patterns to foreign material can be seen, granulomatous is the most common (Int J Dermatol 1984;23:531)
    Terminology
    • Foreign body granuloma refers to foreign body reaction with granulomatous inflammation, which is the most common pattern
    ICD coding
    • ICD-10: L92.3 - foreign body granuloma of the skin and subcutaneous tissue
    Epidemiology
    Sites
    • Any site can be affected but sites of cutaneous injury most commonly affected
    Pathophysiology
    • Exogenous material or endogenous material typically hidden from the immune system (e.g. follicular keratin) becomes exposed to innate immune system through trauma (e.g. accidental injury, injection, surgery, tattooing), topical application or systemic exposure
    • Initial response is a neutrophilic infiltrate, which typically fails to deal with the foreign material (Dermatol Clin 2015;33:497)
    • Local tissue reactions may be lichenoid, eczematous or pseudolymphomatous; however, granulomatous is the most common reaction pattern seen with foreign bodies (Br J Dermatol 1995;133:665, Biomed Res Int 2014;2014:354608, Int J Dermatol 1984;23:531)
    • Granuloma formation:
    Etiology
    • Exposure of immune system to exogenous material or an endogenous substance that is typically protected from the immune system
    Clinical features
    • Varies with the nature of the exposure to the foreign body and specifics of the host response
    • Typically, an acute inflammatory response with erythema and pain is the earliest finding
    • Resolves over weeks, months or even years
    • Subsequently, a chronic inflammatory response may develop if the acute response did not entirely clear the foreign material
    • Red to brown papules, plaques and nodules, along with variable ulceration, may be seen
    • Fibrosis may result in tissue induration
    • Configuration of lesion(s) may a be clue to causative agent
    • Pyogenic granuloma-like lesions (Cases J 2009;2:7354)
    • Pseudofolliculitis barbae (Dermatol Clin 1988;6:407)
    • Foreign material may contain microorganisms (Arch Dermatol 1984;120:1209, Cutis 1995;56:41)
    • Associated lymphadenopathy may secondary to granulomatous inflammation and foreign material in lymph node (Oral Surg Oral Med Oral Pathol 1985;59:449, J Clin Pathol 2000;53:549)
    Diagnosis
    • Careful history and examination correlated with tissue sampling and exclusion of other diagnoses (e.g. granulomatous disease, infection)
    • Ultrasound can quantify the severity of tattoo reactions (Skin Res Technol 2014;20:257)
    • Identification of foreign substance can be ascertained using ultrastructural analytic techniques such as energy dispersive Xray spectroscopy / analysis (Clin Dermatol 1991;9:157)
    Radiology description
    • Ultrasound
      • Prominent echolucent band, the thickness of which correlates with thickness of cellular infiltration on histology (Skin Res Technol 2014;20:257)
    Case reports
    Treatment
    Clinical images

    Images hosted on other servers:

    Tongue nodule

    Lower lip nodule

    Microscopic (histologic) description
    • Varies depending on the specific response but many materials (e.g. silica, tattoo pigments, zinc, keratin, zirconium, silicone etc.) produce sarcoidal granulomas (Int J Dermatol 1984;23:531):
      • Discrete aggregates of epithelioid histiocytes and multinucleate giant cells (Langhans or foreign body type)
      • Sparse rim of surrounding lymphocytes and plasma cells
    • Specific patterns (Clin Dermatol 1991;9:157, Int J Dermatol 1984;23:531):
      • Tattoo: may see pigment within macrophages and in interstitium
      • Paraffin: cystic spaces of varying size (swiss cheese), which stain with oil red O on fresh tissue
      • Silicone: cystic spaces of varying size (swiss cheese), which do not stain with oil red O on fresh tissue
      • Silica: crystalline particles that are birefringent on polarized light
      • Talc: birefringent particles with polarized light
      • Starch: birefringent particles with Maltese cross shape on polarized light; stains with PAS
      • Zinc: birefringent particles with polarized light
      • Bovine collagen: stains pale gray-violet (rather than blue-green) with Masson trichrome
      • Hyaluronic acid: stains blue with high pH Alcian blue
      • Wood splinter: regularly sized cells on cross section
      • Keratin: potato chip bodies that are variably birefringent
    Microscopic (histologic) images

    Contributed by Matthew Franklin, M.D.
    Granulomatous inflammation

    Granulomatous inflammation

    Foreign body giant cells

    Foreign body giant cells

    Splinter with inflammation Splinter with inflammation

    Splinter with inflammation

    Virtual slides

    Images hosted on other servers:

    Talc granuloma

    Foreign material in orbit

    Positive stains
    • CD68 stains multinucleate giant cells
    Videos

    Histopathology of breast foreign body reaction

    Sample pathology report
    • Skin, left dorsal forearm, punch biopsy:
      • Tattoo with foreign body reaction (see comment)
      • Comment: Sections show a bisected punch specimen with an unremarkable epidermis with subjacent dermal scar. Underlying the dermal scar, there is a broad, well demarcated zone of granulomatous inflammation with numerous multinucleate giant cells and scattered lymphocytes. Black pigment is identified within histiocytes and in the surrounding interstitium. The findings are those of a granulomatous foreign body reaction and could be consistent with a prior traumatic tattoo. Clinical correlation is recommended.
    Differential diagnosis
    • Sarcoidosis:
      • Exclude foreign material and infection (e.g. with microscopic examination with polarized light and stains for microorganisms)
    • Infection (e.g. abscess):
    • Pyogenic granuloma:
      • May arise in the setting of a foreign body reaction
      • Exclude an underlying foreign body reaction (e.g. with careful history and microscopic examination with polarized light)
    Board review style question #1

    The photomicrograph shown above depicts an excision specimen from a carpenter presenting with a painful nodule of the right palm. What is the diagnosis?

    1. Epidermoid cyst
    2. Foreign body granuloma
    3. Osteoma cutis
    4. Pilomatricoma
    Board review style answer #1
    B. Foreign body granuloma

    Comment Here

    Reference: Foreign body reaction
    Board review style question #2
    Which inflammatory pattern is most commonly seen in a foreign body reaction?

    1. Eczematous
    2. Granulomatous
    3. Lichenoid
    4. Psoriasiform
    Board review style answer #2
    B. Granulomatous

    Comment Here

    Reference: Foreign body reaction

    Fox-Fordyce disease (pending)
    [Pending]

    Gangrene
    Definition / general
    • Dry gangrene: coagulation necrosis of extremity due to slowly developing vascular occlusion
    • Wet gangrene: infection and liquefaction of “dry gangrenous” tissue by saprophytic bacteria
    • Associated with diabetes (Intern Med 2011;50:1303)
    • See also Fournier's gangrene of penis / scrotum
    Treatment
    • Debridement or possibly amputation; also antibiotics, hyperbaric oxygen, vascular surgery
    Clinical images

    Images hosted on other servers:

    Dry gangrene in patient with diabetes

    Dry gangrene due to cryoglobulinemic vasculitis

    Wet gangrene

    Videos

    Dry gangrene


    Graft versus host disease
    Definition / general
    • Graft versus host disease (GVHD) is a multisystem immunologic disorder following an allogenic hematopoietic cell transplant and rarely after solid organ transplant or transfusion
    • It can be acute or chronic and can affect many organ systems, including the skin, gastrointestinal tract, liver and lungs
    Essential features
    • GVHD refers to multiorgan syndromes of tissue inflammation or fibrosis that primarily affect skin
    • Donor (graft) T cells attack and destroy recipient (host) cells
    • 2 types: acute or chronic
    • Histologically, acute GVHD presents with vacuolar interface dermatitis with necrotic keratinocytes surrounded by lymphocytes and chronic GVHD shows either lichenoid or sclerodermoid changes
    Terminology
    • GVHD: graft versus host disease
    • aGVHD: acute graft versus host disease
    • cGVHD: chronic graft versus host disease
    ICD coding
    • ICD-10:
      • D89.81 - graft versus host disease
      • D89.810 - acute graft versus host disease
      • D89.811 - chronic graft versus host disease
      • D89.812 - acute on chronic graft versus host disease
      • D89.813 - graft versus host disease, unspecified
    • ICD-11: 4B24 - graft versus host disease
    Epidemiology
    Sites
    • Acute GVHD usually affects skin: ears, palms, soles, lateral neck, cheeks and upper back
    • Acute extracutaneous GVHD involves liver, gastrointestinal tract, lungs and lymphoid tissue (Australas J Dermatol 2010;51:1)
    • Chronic GVHD affects primarily the skin; however, nearly all organ systems can be involved (mucous membranes, liver, lung, eye, joints and fascia, gastrointestinal tract, genitalia) (Blood 2012;120:2545)
    Pathophysiology
    • GVHD can develop in different clinical scenarios when tissues containing T cells (blood products, bone marrow and solid organs) are transferred from one person to another and the T cells are not able to be removed (Lancet 2009;373:1550)
    • Immune cells transferred from a nonidentical donor (graft) into the recipient (host) recognize the host cells as foreign and initiate a graft versus host reaction (N Engl J Med 2017;377:2167)
    • There are 3 pathophysiologic phases of acute GVHD (Semin Hematol 2006;43:3, Dermatol Clin 2019;37:569)
      1. Host tissue damage from the radiation and chemotherapy that precede the transplant
      2. Activation and multiplication of donor T cells as a result of interaction with antigens from the host
      3. These activated T cells lead to tissue damage once target organs are reached
    • There are also 3 pathophysiologic phases of chronic GVHD (N Engl J Med 2017;377:2565)
      1. Activation of innate immune cells, endothelial cells and fibroblasts resulting from tissue cytotoxic injury, infections and acute GVHD
      2. Increased response of the adaptive immune system and decrease in immune cell regulators (upregulation of Th1, Th2 and Th17 cells and downregulation of T regulatory cells)
      3. Activation of macrophages resulting in production of transforming factor b and platelet derived growth factor, which activate fibroblasts leading to abnormal tissue repair
    Etiology
    • Allogeneic hematopoietic cell transplant, solid organ transplant or transfusion of blood products
    Clinical features
    • Previously GVHD was classified as acute and chronic based on a 100 day cutoff
    • Current NIH consensus criteria use clinical findings to differentiate between acute and chronic GVHD (Biol Blood Marrow Transplant 2015;21:389)
      • Acute GVHD (Australas J Dermatol 2010;51:1)
        • Affects skin, gastrointestinal tract, liver and hematopoietic system
        • Skin involvement is often the earliest and presents as macular and papular eruption, which may resemble a sunburn; may be pruritic or painful and can become confluent
        • Smooth or hyperkeratotic papules and postinflammatory hyperpigmentation may occur
        • In severe GVHD, rash can form bullous lesions with epidermal necrosis mimicking SJS / TEN
        • Staging of skin involvement is based on severity
          • Stage 1: maculopapular rash involving < 25% of body area
          • Stage 2: rash involving 25 - 50% of body area
          • Stage 3: generalized erythroderma
          • Stage 4: generalized erythroderma with bullous formation
      • Chronic GVHD (Biol Blood Marrow Transplant 2003;9:215, Australas J Dermatol 2010;51:1)
        • Skin is the most common site of involvement; all organs can be affected
        • Either isolated to one organ / site or widespread disease
        • Can occur with or without history of acute GVHD
        • Major cause of morbidity and mortality after transplant
        • Onset is usually > 3 months after transplantation
        • 3 diagnostic clinical features: lichen planus-like lesions, sclerotic skin lesions and poikilodermatous changes (Biol Blood Marrow Transplant 2005;11:945)
          • Lichen planus-like lesions: erythematous, violaceous papules or plaques, usually on dorsal hands and feet, forearms and trunk; can occur in a follicular distribution masquerading keratosis pilaris
          • Sclerosis: morphea-like GVHD (involves dermis, presents as hyperpigmented, hypopigmented or skin colored plaques), lichen sclerosus-like (typically plaques on the upper back with epidermal atrophy and superficial dermal fibrosis), deep sclerosis / eosinophilic fasciitis-like (usually medial arms and thighs, subcutaneous sclerosis) (J Am Acad Dermatol 2005;53:591)
          • Poikiloderma: patches with pigmentation and telangiectasia showing a mixture of atrophy, hypo and hyperpigmentation
        • Hair disorders following allogenic hematopoietic stem cell transplantation: scarring alopecia (LPP / FFA) and nonscarring alopecia (alopecia areata)
        • Nail disorders: dystrophy, anonychia, longitudinal ridging, splitting or brittle nails, onycholysis, pterygium
    Diagnosis
    • Combination of clinical and pathologic findings
    • Acute and chronic GVHD: skin punch biopsy
    • In chronic GVHD, deep incisional biopsy can be useful in evaluating fascial involvement
    • MRI can help to diagnose subcutaneous disease and fasciitis (Arch Dermatol 2009;145:918)
    Prognostic factors
    • Associated with lower survival for acute GVHD (Bone Marrow Transplant 2013;48:587)
      • Advanced disease status
      • Increasing age of patients
    • Associated with higher survival for acute GVHD (Bone Marrow Transplant 2013;48:587)
      • Late onset
      • Involving skin only
    • Associated with lower survival for chronic GVHD (Ann Hematol 2015;94:1727)
      • Low platelet counts < 100,000/μl at the time of diagnosis
      • Female donor
      • IgG levels above normal at the time of diagnosis
      • Hyperbilirubinemia and small intestinal / colonic involvement
    • No impact of prior acute GVHD on outcome of patients with chronic GVHD (Ann Hematol 2015;94:1727)
    Case reports
    • 54 year old man underwent liver transplantation for alcohol use disorder related cirrhosis and presented with dermatitis, bone marrow failure and enteritis (Sci Prog 2022;105:368504221117070)
    • 57 year old man status post-kidney transplant with sudden onset diarrhea and skin rash (Medicine (Baltimore) 2017;96:e7333)
    • 62 year old man underwent allogenic bone marrow transplantation for acute myeloid leukemia and few years later developed sclerodermatous GVHD involving the trunk and limbs (J Cutan Pathol 2022;49:373)
    • 72 year old man with a history of recent cadaveric liver transplantation presented with fever, generalized skin rash and hemorrhagic lesions in the oropharynx (Hepatol Forum 2022;3:30)
    Treatment
    Clinical images

    Contributed by Pooja Rambhia, M.D. and Adele Shenoy, M.D.
    skin with hyperpigmentation and denudement

    Skin with hyperpigmentation and denudement

    Skin with hyperpigmentation, denudement and hemorrhage

    Skin with hyperpigmentation, denudement and hemorrhage

    Palmar skin with erythema

    Palmar skin with erythema

    Hypo and hyperpigmented coalescing patches

    Hypo and hyperpigmented coalescing patches



    Images hosted on other servers:

    Maculopapular rash; erythroderma

    Rash affecting the palms

    Acute GVHD mimicking toxic epidermal necrolysis

    Microscopic (histologic) description
    • Acute GVHD (Patterson: Weedon's Skin Pathology, 4th Edition, 2015)
      • Mild to moderate superficial perivascular lymphocytic infiltrate with exocytosis of inflammatory cells into the epidermis and basal vacuolation, interface dermatitis
      • Scattered, shrunken, eosinophilic keratinocytes with pyknotic nuclei, at all levels of the epidermis; often accompanied by 2 or more lymphocytes, producing the satellite cell necrosis - lymphocyte associated apoptosis
      • Occasionally, rare eosinophils can be present; melanin incontinence is prominent in patients with darker skin types
      • If severe, subepidermal microvesicles, subepidermal blisters and epidermal necrosis
      • Lymphocytic infiltrate in GVHD after solid organ transplantation is usually brisk in comparison to the sparser inflammation following bone marrow transplantation
    • Chronic GVHD (Biol Blood Marrow Transplant 2015;21:589, Patterson: Weedon's Skin Pathology, 4th Edition, 2015)
      • Chronic lichenoid GVHD
        • Acanthosis, orthohyperkeratosis, parakeratosis with hypergranulosis
        • Underlying band-like lymphocytic infiltrate with basal layer vacuolization and apoptotic keratinocytes
        • Periadnexal inflammation can be present
        • May be difficult to distinguish from lichen planus without clinicopathologic correlation; satellite cell necrosis is the most helpful clue in GVHD biopsies to distinguish it from lichen planus
      • Chronic sclerodermoid GVHD
        • Dermal sclerosis (thickened compact collagen with loss of periadnexal fat)
        • Manifests with localized morphea-like features, diffuse sclerosis or lichen sclerosus-like features
        • Localized morphea-like features and diffuse sclerosis are largely confined to the reticular dermis or subcutaneous septa with little or no epidermal involvement
        • Lichen sclerosus-like GVHD: collagen alteration is confined to the papillary dermis (hyalinzation of the papillary dermis), often with residual interface changes characterized by the presence of mild vacuolar alteration, melanophages and sparse superficial perivascular lymphocytic infiltrate
        • In the fasciitis variant, biopsy specimens show only fibrous thickening of the fascia with adjacent inflammation without epidermal or dermal involvement
      • Rare variants: psoriasiform
    Microscopic (histologic) images

    Contributed by Silvija P. Gottesman, M.D. and Ohoud Aljarbou, M.D.
    Denuded skin

    Denuded skin

    Follicle with interface changes

    Follicle with interface changes

    Interface dermatitis with pigment incontinence

    Interface dermatitis with pigment incontinence

    Satellite cell necrosis

    Satellite cell necrosis


    Acute GVHD with sparse inflammation and abundant dyskeratotic keratinocytes

    Dense perivascular and interstitial eosinophilic infiltrate


    Acute GVHD with vacuolar interface reaction and abundant dyskeratotic keratinocytes

    Focal formation of subepidermal blister


    Psoriasiform GVHD with regular acanthosis and parakeratosis

    Virtual slides

    Images hosted on other servers:

    Satellite cell necrosis

    Immunofluorescence description
    • Acute GVHD: immunofluorescence testing is not indicated but can show nonspecific IgM and complement deposits at the basement membrane zone (J Immunol 1978;120:1485)
    Sample pathology report
    • Skin, forearm, punch biopsy:
      • Vacuolar interface dermatitis (see comment)
      • Comment: Subtle basilar vacuolar changes with scattered dyskeratotic keratinocytes are seen involving the epidermis and follicular epithelium. In foci, lymphocyte associated apoptosis of keratinocytes (satellite cell necrosis) is noted. There is sparse superficial perivascular lymphocytic infiltrate and absence of eosinophils. Melanin incontinence is noted in the superficial dermis. In the correct clinical setting, these findings are compatible with graft versus host disease. Clinical correlation is recommended.
    Differential diagnosis
    Board review style question #1
    A 40 year old woman with a history of AML and status post-bone marrow transplant 2 months ago presented with fever and pruritic maculopapular eruption on palms and soles. Physical exam showed smooth and hyperkeratotic papules. Patient shares that at times she is noncompliant with taking her prescribed medications. Lately she has started using herbal supplements recommended by a friend. Punch biopsy from the foot was taken and microscopic exam showed mild vacuolar interface dermatitis with superficial perivascular lymphocytic infiltrate in the dermis and no eosinophils. Focally in the epidermis, eosinophilic degenerated keratinocytes surrounded by lymphocytes were noted. What is the most likely diagnosis?

    1. Acute graft versus host disease
    2. Chronic graft versus host disease
    3. Drug reaction
    4. HHV6 infection
    Board review style answer #1
    A. Acute graft versus host disease. The patient has a history of bone marrow transplant, medication noncompliance and presents with a rash. Biopsy shows vacuolar interface dermatitis with superficial perivascular lymphocytic infiltrate in the dermis, no eosinophils and rare dyskeratotic keratinocytes. The combined clinical and morphologic features listed above are most consistent with acute graft versus host disease. Answer B is incorrect because chronic graft versus host disease would show lichenoid or sclerodermoid features. Answer C is incorrect because drug eruption would present with tissue eosinophilia. Answer D is incorrect because HHV6 infection shows nonspecific microscopic features.

    Comment Here

    Reference: Graft versus host disease
    Board review style question #2
    A 30 year old man with a history of hematologic malignancy and bone marrow transplant 1 year prior presents to the dermatology clinic with erythematous and violaceous papules and plaques on the trunk, dorsal hands and forearms. Punch biopsy from the forearm showed vacuolar interface dermatitis with associated lichenoid inflammation in the papillary dermis, pigment incontinence and absence of eosinophils. Interface changes extend down follicular epithelia. What is the most likely diagnosis?

    1. Chronic lichenoid graft versus host disease
    2. Chronic sclerodermoid graft versus host disease
    3. Lichenoid drug eruption
    4. Lichen planus
    Board review style answer #2
    A. Chronic lichenoid graft versus host disease. The patient presents with a rash and has a history of bone marrow transplant. Biopsy shows vacuolar interface dermatitis associated with lichenoid inflammation in the papillary dermis, pigment incontinence and absence of eosinophils. The combined clinical and morphologic features listed above are most consistent with chronic lichenoid graft versus host disease. Answer B is incorrect because chronic sclerodermoid graft versus host disease would show dermal sclerosis, which is absent in this case. Answer C is incorrect because lichenoid drug eruption would show prominent eosinophils, which are not appreciated here. Answer D is incorrect because lichen planus has characteristic clinical presentation of intensely pruritic papules and the inflammatory infiltrate is more prominent than in GVHD.

    Comment Here

    Reference: Graft versus host disease
    Board review style question #3

    A 15 year old boy, day 47 post-bone marrow transplantation, presented with abrupt onset of diffuse erythematous papular eruption. Which of the following is the most likely diagnosis?

    1. Discoid lupus erythematosus
    2. Graft versus host disease
    3. Pityriasis lichenoides chronica
    4. Viral exanthema
    Board review style answer #3
    B. Graft versus host disease. Biopsy shows epidermis with basal vacuolization and dyskeratotic keratinocytes. There is mild superficial dermal and perivascular lymphocytic infiltrate. The given clinical history along with the pathological findings support the diagnosis of graft versus host disease (GVHD). Answer A is incorrect because while discoid lupus erythematosus histologically can resemble GVHD (basal vacuolar interface change), it shows dermal mucin, which is usually absent in GVHD. Answer C is incorrect because while pityriasis lichenoides chronica and GVHD show similar histologic features, clinical correlation favors GVHD in this case. Answer D is incorrect because viral exanthem has nonspecific histologic findings, including spongiosis, perivascular lymphocytic infiltrate and sometimes lymphoexocytosis and necrotic keratinocytes in epidermis; eosinophils are usually absent. Clinically presents as a morbilliform rash. Clinical correlation and history of bone marrow transplant favors GVHD.

    Comment Here

    Reference: Graft versus host disease

    Granular parakeratosis
    Definition / general
    • Granular parakeratosis is an acquired, idiopathic, uncommon, benign, cutaneous condition characterized by scaly eruptions, most commonly in intertriginous areas, with characteristic histopathologic features
    Essential features
    Terminology
    • First described in 1991 as axillary granular parakeratosis (J Am Acad Dermatol 1991;24:541)
    • Has been seen in other intertriginous and nonintertriginous regions of the skin
    • Hyperkeratotic flexural erythema
    ICD coding
    • ICD-10:
      • L85.8 - other specified epidermal thickening
      • L85.9 - epidermal thickening, unspecified
    Epidemiology
    • More common in women
    Sites
    • Intertriginous regions, axilla in particular
    • Nonintertriginous
    Pathophysiology
    • Pathophysiology thought to involve a defective profilaggrin - filaggrin pathway
    Etiology
    Clinical features
    • Associations established with metabolic syndrome and obesity
    Diagnosis
    • Clinically, patients present with erythematous or hyperpigmented papules that may coalesce into plaques
    • Biopsy of the lesion to identify characteristic histopathologic features
    Prognostic factors
    • Favorable with self limiting course; resolves in a period of 1 month to 1 year
    • Cases of granular parakeratosis chronically recurring for up to 20 years have been reported
    • Reference: Int J Dermatol 2022;61:973
    Case reports
    • 29 year old man with scrotal and groin erythema for 1 month (Clin Cosmet Investig Dermatol 2022;15:1367)
    • 46 year old overweight woman presented with a 2 month history of axillary rash (Cutis 2020;105:E42)
    • 47 year old woman with a 1 week history of asymptomatic, multiple, linear, horizontal, brown, hyperpigmented scaly papules in the infra-abdominal fold (Cureus 2022;14:e24085)
    • 52 year old obese woman with dark brown, scaly, nontender and nonpruritic eruptions in both axillae that gradually had enlarged and thickened over 6 weeks (Cutis 2013;92:61)
    • 52 year old Thai man with multiple tiny filiform hyperkeratotic papules on his face 2 weeks after using antimelasma cream (Skin Appendage Disord 2021;7:144)
    • 85 year old woman with possible varicella zoster infection (S D Med 2021;74:480)
    Treatment
    Clinical images

    Images hosted on other servers:
    Red macules and patches in groin and scrotum

    Red macules and patches in groin and scrotum

    Hyperpigmented papules in the infra-abdominal fold

    Hyperpigmented papules in the infra-abdominal fold

    Discrete, tiny, filiform, hyperkeratotic papules on forehead

    Discrete, tiny,
    filiform, hyper-
    keratotic papules
    on forehead

    Microscopic (histologic) description
    Microscopic (histologic) description

    Contributed by Shabnam Momtahen, M.D.
    Hyperkeratosis, retention of keratohyalin granules, acanthosis Hyperkeratosis, retention of keratohyalin granules, acanthosis Hyperkeratosis, retention of keratohyalin granules, acanthosis

    Hyperkeratosis, retention of keratohyalin granules, acanthosis

    Sample pathology report
    • Skin, biopsy:
      • Granular parakeratosis (see comment)
      • Comment: Sections show thickened stratum corneum with abnormal retention of keratohyalin granules and parakeratosis. The subjacent epidermis shows acanthosis and mild spongiosis. The findings are compatible with granular parakeratosis.
    Differential diagnosis
    • Hailey-Hailey disease:
      • Presence of suprabasilar and intraepidermal cleft along with acantholysis of keratinocytes resembling dilapidated brick wall
      • Absence of retained keratohyalin granules
    • Verruca:
      • Presence of hyperkeratosis, papillomatosis, hypergranulosis
      • Columns of parakeratosis over projecting dermal papillae and intracorneal hemorrhage
      • Vacuolated superficial keratinocytes with pyknotic raisin-like nuclei (koilocytes)
      • Absence of retained keratohyalin granules
    • Contact dermatitis:
      • Presence of prominent spongiosis with parakeratosis, Langerhans cells and superficial perivascular inflammation with eosinophils
      • Absence of retained keratohyalin granules
    • Acanthosis nigricans:
      • Presence of orthokeratotic hyperkeratosis and papillomatosis of stratum spinosum
      • Hyperpigmentation of basal cell layer without melanocytic hyperplasia
      • Absence of retained keratohyalin granules
    • Confluent and reticulated papillomatosis:
      • Presence of hyperkeratosis, papillomatosis, focal acanthosis with increased melanin pigmentation and melanosomes
      • Absence of retained keratohyalin granules
    • Inverse psoriasis:
      • Presence of regular acanthosis, often with elongated rete (psoriasiform), alternating zones of hypogranulosis and hypergranulosis in the epidermis
      • Thinning of the suprapapillary plates
      • Areas of parakeratosis in the stratum corneum with mounds of neutrophils (Munro microabscesses)
      • Perivascular, predominantly lymphocytic infiltrate in the upper and middle portions of the dermis
      • Dilated and tortuous vessels in the dermal papillae
      • Absence of retained keratohyalin granules
    Board review style question #1


    A 36 year old woman presented to her primary care provider with a 1 month history of axillary erythema, without significant itching, that clinically resembled eczema. The erythema worsened and the skin lesions increased after topical application of steroid ointment. Dermatological examination revealed well demarcated erythema in the axillae with central hyperpigmentation and peripheral dry skin. What is the best diagnosis?

    1. Acanthosis nigricans
    2. Contact dermatitis
    3. Granular parakeratosis
    4. Hailey-Hailey disease
    Board review style answer #1
    C. Granular parakeratosis. Histological analysis reveals hyperkeratosis and parakeratosis with retention of numerous keratohyalin granules in the stratum corneum. Acanthosis nigricans (answer A) does not show retained keratohyalin granules. Contact dermatitis (answer B) shows prominent spongiosis without retained keratohyalin granules. Biopsy of Hailey-Hailey disease (answer D) shows acantholysis without retained keratohyalin granules.

    Comment Here

    Reference: Granular parakeratosis
    Board review style question #2
    A 57 year old woman presented with hyperpigmented, scaly and nonpruritic eruptions affecting the left axilla that have progressively enlarged over the past 4 weeks. There has been no other intertriginous involvement. Her medical history was notable for mild asthma, seasonal allergies and obesity. What is the best diagnosis?

    1. Darier disease
    2. Granular parakeratosis
    3. Seborrheic dermatitis
    4. Warts
    Board review style answer #2
    B. Granular parakeratosis. Darier disease (answer A) is defined by acantholysis with characteristic dyskeratosis forming corp ronds and grains. Seborrheic dermatitis (answer C) is characterized as acute, subacute or chronic spongiotic dermatitis. Acute form shows spongiosis with overlying scale crust containing a few neutrophils. In subacute lesions, there is psoriasiform hyperplasia with mild spongiosis. Chronic lesions show more pronounced psoriasiform hyperplasia and only minimal spongiosis. Warts (answer D) are defined by the presence of hyperkeratosis, papillomatosis, hypergranulosis; columns of parakeratosis, intracorneal hemorrhage; koilocytes.

    Comment Here

    Reference: Granular parakeratosis

    Granuloma annulare
    Definition / general
    • Idiopathic, self limited dermatosis typically characterized by erythematous papules and plaques and characterized histologically by palisaded or interstitial granulomatous inflammation
    Essential features
    • Benign, self limited dermatosis characterized by erythematous papules and plaques, classically in an arciform or annular configuration
    • 2 classic morphologic patterns
      • Palisaded granulomas with central necrobiosis (collagen degradation) with peripheral histiocytes and admixed lymphocytes
      • Histiocytes intercalating among collagen bundles with interstitial mucin
    • Associations with hyperlipidemia and diabetes mellitus
    • No proven treatment for this recurring, idiopathic disease
    ICD coding
    • ICD-10: L92.0 - granuloma annulare
    Epidemiology
    Sites
    Pathophysiology
    Etiology
    Clinical features
    Diagnosis
    • Histopathologic evaluation confirms diagnosis
    Laboratory
    Prognostic factors
    Case reports
    Treatment
    Clinical images

    Contributed by David Crowe, M.D.
    Annular patch Annular patch

    Annular patch

    Annular plaque

    Annular plaque

    Erythematous annular plaque

    Erythematous annular plaque

    Microscopic (histologic) description
    • Palisaded or interstitial granulomatous inflammation with mucin unites all forms (Arch Dermatol 1977;113:1681)
    • Interstitial may be most common pattern (Indian Dermatol Online J 2018;9:409, Arch Dermatol 1977;113:1681)
    • Variable histopathology
    • Necrobiotic (collagenolytic) granulomas
      • Granulomas characterized by central areas of degraded collagen surrounded by lymphocytes and histiocytes, occasionally palisaded, are present in dermis
      • Occasional multinucleate giant cells
      • Deep variants
        • Subcutaneous
        • Pseudorheumatoid nodule
          • Nodules composed of palisaded granulomas with central eosinophilic material (degraded collagen) are separated by thickened collagen bundles
          • Minimal or absent mucin
    • Interstitial (or incomplete)
      • Busy dermis (interstitial lymphohistiocytic inflammation)
      • Histiocytes and lymphocytes arranged around vessels and intercalating between collagen bundles with conspicuous interstitial mucin
      • Lacks necrobiosis
      • Patch granuloma annulare more likely to be only or predominantly interstitial pattern (without palisaded granulomas) (J Cutan Pathol 2020;47:785)
    • Nonnecrobiotic (sarcoidal or tuberculoid)
      • Mimics sarcoidosis or tuberculous granulomas (Am J Dermatopathol 2015;37:547)
      • Increased mucin or eosinophils may suggest granuloma annulare over mimics
    • Eosinophils in ~40% of cases (J Cutan Pathol 1985;12:13)
    • In generalized granuloma annulare, interstitial and palisading patterns roughly equal in frequency (Ann Dermatol 2009;21:113)
    • Transepithelial excretion of altered collagen in perforating granuloma annulare
    • Minimal epidermal changes
    • Potential overlap with other granulomatous disorders
    Microscopic (histologic) images

    Contributed by Stephen Somach, M.D.
    Deep dermal palisaded granuloma

    Deep dermal palisaded granuloma

    Reticular dermal palisaded granuloma

    Reticular dermal palisaded granuloma

    Increased dermal mucin in necrobiotic zone

    Increased dermal mucin in necrobiotic zone

    Predominantly sarcoidal granulomas Predominantly sarcoidal granulomas

    Predominantly sarcoidal granulomas


    Interstitial histiocytic infiltrate

    Interstitial histiocytic infiltrate

    Small interstitial histiocytes

    Small interstitial histiocytes

    Palisaded granuloma with fibrinoid collagen degeneration

    Palisaded
    granuloma with
    fibrinoid collagen
    degeneration

    Deep palisaded granuloma with fibrinoid degeneration

    Deep palisaded granuloma with fibrinoid degeneration

    Virtual slides

    Images hosted on other servers:

    Palisaded granuloma annulare

    Alcian blue, granuloma annulare

    Immunofluorescence description
    Positive stains
    Electron microscopy description
    Videos

    Granuloma Annulare
    Dr. James Carton

    Sample pathology report
    • Skin, left forearm, punch biopsy:
      • Consistent with granuloma annulare (see comment)
      • Comment: Sections show a punch biopsy specimen with a normal epidermis. Within the reticular dermis is a palisaded array of histiocytes associated with central collagen degeneration and mucin deposition.
    Differential diagnosis
    Additional references
    Board review style question #1

    A 44 year old woman presents with an eruption involving her proximal upper extremities characterized by slightly erythematous papules coalescing into annular and arciform plaques. Which of the following systemic abnormalities is most strongly associated with granuloma annulare?

    1. Obesity
    2. Hematologic malignancy
    3. HIV
    4. Hyperlipidemia
    5. Intravenous drug abuse
    Board review style answer #1
    D. Hyperlipidemia

    Comment Here

    Reference: Granuloma annulare
    Board review style question #2

    A 51 year old woman with diabetes presents with skin colored papules on her chest and abdomen that have a vague annular configuration. A 4 mm punch biopsy is taken from her abdomen. Sections demonstrate palisaded histiocytes forming granulomas with peripheral lymphocytes and central eosinophilic material. Which interstitial extracellular substance would you expect to be most prevalent in this granulomatous dermatosis?

    1. Calcium
    2. Mucin
    3. Urate crystals
    4. Lipid
    5. Amyloid
    Board review style answer #2
    B. Mucin

    Comment Here

    Reference: Granuloma annulare

    Granuloma faciale
    Definition / general
    Essential features
    • Solitary or multiple asymptomatic red-brown plaques on the face
    • Persistent and refractory to treatment
    • Mixed infiltrate with neutrophils, plasma cells and eosinophils
    • Fibrosing vasculitis: leukocytoclasia with perivascular and often storiform fibrosis
    ICD coding
    • ICD-10: L92.2 - Granuloma faciale (eosinophilic granuloma of skin)
    Epidemiology
    Sites
    • Face - forehead, cheeks and nose - most common
    • Extrafacial - trunk or extremities - not uncommon
    Pathophysiology
    Etiology
    • Unknown at this time
    Clinical features
    Diagnosis
    Prognostic factors
    • Persists indefinitely and frequently refractory to treatment
    • Spontaneous resolution is rare
    Case reports
    Treatment
    • First line
      • Topical tacrolimus
    • Second line
      • Intralesional corticosteroids
      • Cryotherapy
    • Refractory cases
    Clinical images

    Images hosted on other servers:
    Plaques on the forehead

    Plaques on the forehead

    Plaque on the nose

    Plaque on the nose

    Plaques on the forehead and cheek

    Plaques on the forehead and cheek

    Microscopic (histologic) description
    • Grenz zone
    • Diffuse, polymorphous inflammatory infiltrate involves the upper half of the dermis
      • Neutrophils, plasma cells, eosinophils, lymphocytes and histiocytes
    • Leukocytoclasia (karryorrhexis), extravasated red blood cells and hemosiderin
      • Fibrinoid necrosis of small vessels is variable
    • Perivascular fibrosis with clefting between collagen bundles results in storiform fibrosis
    Microscopic (histologic) images

    Contributed by Kiran Motaparthi, M.D.
    Polymorphous infiltrate with grenz zone

    Polymorphous infiltrate with grenz zone

    Plasma cells, neutrophils and fibrosis

    Plasma cells, neutrophils and fibrosis

    Fibrosing vasculitis

    Fibrosing vasculitis

    Caption

    Eosinophils

    Virtual slides

    Images hosted on other servers:
    Granuloma faciale Granuloma faciale

    Granuloma faciale

    Immunofluorescence description
    • Immunoglobulin and complement deposition within vessel walls and at the dermoepidermal junction are variable (J Cutan Pathol 2006;33:508)
    Sample pathology report
    • Skin, nasal dorsum, punch biopsy:
      • Granuloma faciale
    Differential diagnosis
    Board review style question #1

    A 43 year old man presents with an asymptomatic red-brown plaque on the cheek. Which of the following features is characteristic of this disorder?

    1. Cholesterol clefts
    2. Epithelioid endothelial cells
    3. Fibrosis
    4. Germinal centers
    5. Granulomatous inflammation
    Board review style answer #1
    C. Fibrosis. This is granuloma faciale.

    Comment Here

    Reference: Granuloma faciale
    Board review style question #2
    Compared with granuloma faciale, which of the following disorders demonstrates nearly identical histopathologic features?

    1. Angiolymphoid hyperplasia with eosinophilia
    2. Eosinophilic angiocentric fibrosis
    3. Eosinophilic cellulitis
    4. Eosinophilic folliculitis
    5. Eosinophilic fasciitis
    Board review style answer #2
    B. Eosinophilic angiocentric fibrosis

    Comment Here

    Reference: Granuloma faciale

    Grossing
    Table of Contents
    Biopsy | Grossing
    Biopsy
    • If possible, biopsy a primary lesion
    • Proper selection of anatomical site and best lesion to perform a biopsy are very important (An Bras Dermatol 2009;84:507)
    • Biopsy characteristic areas in their diagnostic stage, not ruptured bullae, secondarily infected / scratched areas or involuting lesions
    • Some lesions are best diagnosed early (blisters, vasculitis) or late (psoriasis, discoid lupus erythematosus, lichen planus)
    • Punch biopsy of nearby normal skin may be helpful, particularly if changes in abnormal areas are quantitative (hyperkeratosis, acanthosis, etc.)
    • Must recognize that biopsy captures histopathology only at one point in time in evolution of a lesion
    • For inflammatory lesions, biopsy should correlate with clinical differential diagnosis
    • For widespread dermatoses, avoid biopsying lesions on the legs; at this site but not elsewhere, the biopsy heals slowly and lesions are often hemorrhagic
    • For alopecias, biopsy the border of active lesions but if evaluation for hair regrowth is needed, biopsy the most advanced area
    • For metabolic abnormalities, electron microscopy of axillary skin biopsy may be helpful, although results may be nonspecific (Hum Pathol 2001;32:649)
    Grossing
    • Breadloafing (serial section) of ellipses is often best cut perpendicular to long axis at 3 mm intervals
    • Always ink margins for tumors, including deep margin
    • Bisect punch biopsies if 4 mm or larger

    • Biopsy for alopecia: can be cut perpendicular or horizontal depending of the pathology

    • Tzanck preparation (smear):
      • Used to identify Tzanck (multinucleated) cells of herpes simplex, Varicella and herpes zoster, pemphigus vulgaris or cytomegalovirus
      • Scrape the base of the blister / ulcer with the round base of the scalpel
      • Smear on a glass slide
      • Air-dry and stain with Wright's or Giemsa stains

    • Scabies (scrapping):
      • Place a drop of immersion oil on a microscope slide
      • Place a drop of immersion oil on the scalpel
      • Scrape the stratum corneum off the surface of the lesion
      • Numerous lesions should be scraped and placed on the same microscope slide

    • Submit fresh tissue for:
      • Immunofluorescence (can also use Michels medium)
      • Bacterial, fungal, and viral cultures
      • Chromosomal analysis
      • Antigen mapping of basement membrane zone / BMZ (saline splitting)

    Gyrate erythema
    Definition / general
    • Various erytematous eruptions with a curvilinear ("gyrate") appearance:
    • Erythema annulare centrifugum: most common gyrate erythema, but etiology unknown; may grow over weeks, resolves in 1 - 2 months
    • Erythema marginatum rheumaticum: due to rheumatic fever, now extremely rare
    • Erythema gyratum repens: uncommon, paraneoplastic process associated with internal malignancy (eMedicine)
    • Erythema chronicum migrans: occurs after a tick bite, may be associated with Lyme disease (Wikipedia)
    Terminology
    • Also called figurate erythema
    Case reports
    • 64 year old woman with pruritic, erythematous plaques on abdomen and back that appeared and disappeared (Case #71)
    Clinical features
    • Multiple waves of curvilinear erythema and scale
    • Rash may migrate and be pruritic
    Clinical images

    Images hosted on other servers:

    Erythema annulare centrifugum - pregnant woman

    Erythema gyratum repens

    Erythema chronicum migrans

    Microscopic (histologic) description
    • Nonspecific changes
    • Dense perivascular lymphocytic infiltrate of superficial dermis and possibly deep reticular dermis
    • Lymphocytes are well demarcated and adjacent to vessels ("coat sleeve" or "pipe stem" appearance in erythem annulare centrifugum)
    • Variable epidermal spongiosis, mild ancanthosis and parakeratosis; may have focal vacuolar changes
    Microscopic (histologic) images

    Case #71

    Erythema gyrated repent



    Images hosted on other servers:

    Erythema annulare centrifugum - pregnant woman

    Erythema chronicum migrans

    Positive stains
    • Erythema gyratum repens: occasional presence of C3, C4 and IgG at the basement membrane zone with direct immunofluorescence

    Herpes simplex / zoster
    Definition / general
    • Painful diseases caused by herpes simplex virus (HSV) or varicella zoster virus (VZV, also causes chickenpox)
    Clinical features
    • For both viruses, after primary infection, viral particles reside in sensory ganglia and are dormant until they erupt as recurrent herpes simplex virus or shingles (zoster)
    • The two viruses are differentiated by culture (difficult to culture zoster) or immunologic methods

    • Herpes simplex:
      • Historically, HSV1 was associated with herpes labialis (90%), and HSV2 was associated with herpes genitalis (90%), although in some recent studies, most genital lesions are caused by HSV1
      • Visceral involvement is most often seen in the lung, liver and brain
      • Today, diagnosis is often confirmed by PCR or immunohistochemistry
      • The multinucleated cells are the diagnostic features of the historic Tzank test, a Giemsa stained smear of vesicle contents
      • In the past, the laboratory diagnosis of herpes infection was confirmed by growth in tissue culture, electron microscopy, immunofluorescence of viral specific protein or viral DNA hybridization

    • Varicella zoster:
      • Varicella zoster is associated with leukemia and lymphoma, SLE and postradiation or postchemotherapy status
      • Occurs in 40 - 50% of patients in the first year following bone marrow transplantation
      • Shingles has dermatomal distribution or severe involvement of trigeminal nerve-first division (ophthalmic division) with corneal ulceration and herpetic keratitis

    • Description:
      • Grouped vesicles on an erythematous base, later become pustules, then crusts
    Case reports
    Treatment
    • Acyclovir; also valacyclovir, penciclovir and famciclovir (eMedicine)
    Clinical images

    Contributed by Mark R. Wick, M.D.

    Breast skin



    Images hosted on other servers:

    Dermatomal distribution of zoster

    Verrucous varicella zoster on leg

    Microscopic (histologic) description
    • Keratinocytes are multinucleated, acantholytic with distinct nuclear inclusions, found initially in follicular epithelium
    • Late epidermal necrosis or full-thickness acantholysis
    • Dermal nerve twigs may exhibit a perineural infiltrate of lymphocytes and neutrophils, sometimes associated with intraneural involvement
    • Schwann cell hypertrophy and frank neural necrosis are occasionally encountered
    Microscopic (histologic) images

    Contributed by Hillary Rose Elwood, M.D.

    Viral cytopathic effect of multinucleation, molding, nuclear margination



    Case #423

    Herpes virus vesicular dermatitis



    Images hosted on other servers:

    Multinucleated cells, some with intranuclear inclusions


    Hidradenitis suppurativa
    Definition / general
    • Chronic inflammatory disorder of pilosebaceous apocrine unit characterized by nodules, abscesses, fistulae and sinus tracts, with scarring
    • Most commonly affects the axillae and groin, among other areas of young adults
    Essential features
    • Inflammatory disorder of terminal hair bearing skin, most commonly in the axillae and groin, manifesting with nodules, open comedones and draining sinuses with prominent scarring
    • Risk factors include female sex, obesity, smoking and family history
    • Associated with metabolic syndrome, adverse cardiovascular events and other inflammatory disorders, e.g. psoriasis
    • Follicular occlusion is the first step in pathogenesis, followed by epithelial rupture with ensuing inflammation, foreign body reaction, occasionally apocrine involvement and bacterial colonization
    • Effective treatment (may involve antibiotics, biologic or surgical measures) is often challenging
    Terminology
    ICD coding
    • ICD-10: L73.2 - hidradenitis suppurativa
    • ICD-11: ED92.0 - hidradenitis suppurativa
    Epidemiology
    • Estimated prevalence between 0.00033% and 4.10% (Exp Dermatol 2019;28:769)
    • Prevalence in U.S. population recently estimated to be 0.10% (JAMA Dermatol 2017;153:760)
      • More than twice as common in women
      • Highest among patients aged 30 - 39 years
      • Prevalence is more than 3 times greater in African American patients than Caucasian patients
    • Onset before 11 years of age estimated to account for 2% of cases (Pediatr Dermatol 1999;16:292)
    Sites
    Pathophysiology
    Etiology
    Clinical features
    Diagnosis
    Laboratory
    Radiology description
    • Ultrasound (Skin Res Technol 2020;26:11)
      • Thickened dermis
      • Widened hair follicles
      • Tracts
      • Hypoechoic fluid pockets
      • Decreased surrounding tissue echogenicity
      • Abscess
    Radiology images

    Images hosted on other servers:
    Hidradenitis nodule (a) and abscess (b)

    Hidradenitis nodule (a) and abscess (b)

    Case reports
    Treatment
    Clinical images

    Images hosted on other servers:

    Axilla

    Microscopic (histologic) description
    Microscopic (histologic) images

    Contributed by Matthew Franklin, M.D. and @JMGardnerMD on Twitter
    Dermal sinus tract

    Dermal sinus tract

    Sinus tract with rupture

    Sinus tract with rupture

    Suppurative and granulomatous inflammation

    Suppurative and granulomatous inflammation

    Follicular occlusion and inflammation

    Follicular occlusion and inflammation

    Perifollicular abscess

    Perifollicular abscess

    Epithelial remnants with inflammation

    Epithelial remnants with inflammation


    Hidradenitis suppurativa Hidradenitis suppurativa

    Hidradenitis suppurativa

    Hidradenitis suppurativa Hidradenitis suppurativa

    Hidradenitis suppurativa

    Molecular / cytogenetics description
    Sample pathology report
    • Right proximal thigh, excision:
      • Dermal epithelial lined sinus tract with cyst rupture and surrounding acute and chronic inflammation, consistent with hidradenitis suppurativa
    Differential diagnosis
    • Other entities within the follicular occlusion tetrad (acne conglobata, pilonidal cyst and dissecting cellulitis) have similar histopathology
    • Clinicopathologic correlation, especially with anatomic site, is necessary
    • Ruptured follicular cyst / folliculitis:
      • Similar histopathology, except dissecting tracts would not be expected
    • Soft tissue infection:
      • Special stains for microorganisms, as well as tissue cultures, help to exclude this possibility
    Additional references
    Board review style question #1
    Dermal sinus tract Follicular occlusion and inflammation


    A 25 year old woman with diabetes presents for draining boils in her axillae and groin. Abnormalities in which of the following cutaneous structures is thought to initiate the pathogenic cascade leading to the inflammatory sequelae of this disease?

    1. Apocrine glands
    2. Arrector pili muscle
    3. Eccrine glands
    4. Infundibular follicular epithelium
    5. Sebaceous glands
    Board review style answer #1
    D. Infundibular follicular epithelium

    Comment Here

    Reference: Hidradenitis suppurativa
    Board review style question #2
    Which of these is a risk factor for the development of hidradenitis suppurativa?

    1. Alcohol consumption
    2. Low BMI
    3. Male sex
    4. Smoking
    Board review style answer #2

    Histology
    Definition / general
    • Membrane covering the exterior of the body composed of epidermis and dermis
    Essential features
    • Skin is the largest organ of the body with a weight of approximately 5 kg and an area close to 2 m²
    • General functions of the skin:
      • Protection (against UV light, dehydration, microbial invasion, mechanical injury, chemical and thermal stresses)
      • Retains water and excretes drugs and waste
      • Thermoregulation through blood vessels and eccrine glands
      • Metabolic: vitamin D production
      • Sensorial: contains peripheral endings of sensory nerves
    Terminology
    • Also called integument
    Physiology
    • Epidermis
      • Outer stratified squamous epithelium composed of epidermal and intraepidermal adnexal (acrotrichial and acrosyringeal) keratinocytes
      • Main purpose is cornification
      • Keratinocytes are connected by desmosomes, adherens junctions, tight junctions and gap junctions
      • Basal layer
        • 10% are stem cells
        • Produces keratin 5
      • Squamous layer
        • Desmosomes develop and composition of keratin intermediate filaments change (↑ intracellular calcium)
        • Produces keratins 1 and 10
        • Langerhans cells and Merkel cells present
      • Granular layer
        • Profilaggrin which forms filaggrin: facilitates filament aggregation
        • Loricrin and involucrin: contribute to the formation of the insoluble cell envelope
        • Cells begin to lose nuclei
        • Production of the cell envelope
          • Located beneath the cell membrane
          • Composed of crosslinked proteins dependent on transglutaminase
        • Odland bodies
          • Lipid rich lamellated granules (mainly ceramide) secreted into intercellular spaces
          • Contribute to permeability barrier
      • Cornified layer
        • Formed because of keratinocytic maturation (cells flatten as they ascend to the surface)
        • Synthesis of lamellar granules and proteins
        • Cells lose their nuclei, cytoplasmic organelles, metabolic activity and eventually desquamate
    • Dermal epidermal junction: components bind the epidermis to the dermis
      • 4 major structural regions proceeding from the epidermis to the dermis
        • Basal cell plasma membrane
        • Lamina lucida: low electron density region, contains laminins (major laminin 332)
        • Lamina densa: electron dense region; its major component is collagen IV
        • Sublamina densa: upper papillary dermis contains loops of type VII collagen (anchoring fibrils)
      • Attachment structures
        • Hemidesmosomes: plaque proteins extend from basal plasma membrane of the keratinocyte to the lamina lucida (associated to the anchoring filaments)
        • Anchoring filaments: thread-like structures that cross the lamina lucida
        • Anchoring fibrils: extend from the lower lamina densa to the upper reticular dermis
      • Total epidermal renewal time, approximately 2 months
        • Cells take 26 - 42 days to transit from the basal layer to the granular layer
        • 14 days for keratin layer to be shed
    • Melanocytes
      • Neural crest derived dendritic cells that synthesize melanin
      • Melanin absorbs ultraviolet (UV) and protects from damaging UV Induced mutations
      • Melanin (produced from tyrosine) is transferred in melanosomes (lysosome type organelle), through melanocytic dendritic processes into adjacent keratinocytes and hair shafts
      • 2 types of melanin are produced: pheomelanin (yellow-red) and eumelanin (black-brown)
      • Racial skin color is due to amount of melanin in keratinocytes, not number of melanocytes
    • Dermis
      • Mainly supports the epidermis and plays varied other roles due to the presence of vessels, nerves and adnexa
      • Blood vessels provide nutrients and help regulate temperature
      • Sucquet-Hoyer canals
        • Specialized acral arteriovenous anastomoses
        • Allow blood shunting from arterioles to venules, bypassing capillaries
        • Surrounded by modified smooth muscle glomus cells (latter function as sphincters)
        • Primary function is of thermoregulation
      • Lymphatics
        • Play a role in tumor spread and removal of debris (fluid, cells and macromolecules)
        • Represent the primary route for Langerhans cells to reach regional lymph nodes
      • Nerves
        • Somatic sensory nerves mediate pain, itch temperature and touch
        • Autonomic motor nerves mediate vascular tone, pilomotor response and sweating
        • Afferent nerves consist of myelinated and nonmyelinated free nerve endings
        • 2 types of sensory receptors are present
          • Specialized (encapsulated): Meissner corpuscles have tactile function and Pacinian corpuscles detect pressure
          • Unspecialized: sensory non-encapsulated nerves linked to Merkel cells
    • Subcutis
      • Plays a role in cushioning, insulation, endocrine function and energy stores
      • 2 types of fat:
        • White
        • Brown: mostly present in infants and children, rich in mitochondria and produces heat
    • Adnexa
      • Skin associated structures each with specific functions including hair follicles, sebaceous glands, eccrine sweat glands and apocrine glands
      • Folliculosebaceous apocrine units: functional complex of hair follicle, sebaceous gland, erector pili muscle and (depending on site) apocrine gland
    • Hair follicle
      • Functions include temperature regulation, protection of other structures and tactile sensory input
      • Types of hair
        • Lanugo: present during late gestation and first month of life
        • Vellus: fine hair (face, trunk and extremities)
        • Terminal: coarse (scalp, eyebrows, eyelashes)
      • Cycle of hair
        • Anagen: growing phase (lasts 2 - 7 years)
        • Catagen: involuting phase (lasts 2 - 3 weeks, 1 - 2% of total hair)
        • Telogen: resting phase (lasts 100 days, 10 - 20% of total hair)
    • Sebaceous glands
      • Main function is protection by production and release of sebum
      • Secretion (governed by androgens) plays a role in waterproofing, control of epidermal water loss and inhibition of fungal and bacterial growth
      • Glands are holocrine (secretion depends on degeneration of acini with release of cells and lipid)
      • Sebaceous secretions (sebum) carry to the surface a mixture of fat, corneocytes and normal flora (yeasts, bacteria and mites)
      • Flora includes Propionobacterium acnes, Staphylococcus epidermidis and Demodex brevis
    • Apocrine sweat glands
      • Concentrated in axilla, groin, perineum, face, periumbilical, external auditory meatus, eyelid and areola
      • Function in humans is unknown; in other mammals they produce scent and play a role in sexual attraction
      • Odorless secretion is initially released and then modified by superficial bacteria producing body odor
    • Eccrine sweat glands
      • Regulate body temperature
      • Present almost everywhere in the skin except oral lips, clitoris, labia minora and external auditory canal
      • Eccrine duct possesses a conduit and metabolic (modifies secretion and reabsorbs water) function
      • Possess merocrine secretion
    • Nail unit
      • Protects tissues of the distal fingertip from injuries and enhances delicate movements by counterpressure
    • Langerhans cells
      • Bone marrow derived dendritic cells, described in 1868 by Paul Langerhans, a German pathologist, physiologist and biologist (Am J Dermatopathol 1985;7:347)
      • Function as intraepidermal macrophages phagocytosing antigens and then migrate to regional lymph nodes where they present antigens to T cells
    • Merkel cells
      • Part of the affector limb mechanoreceptors related with touch sensation
      • Described by in 1875 by Friedrich Merkel a Germany anatomist and histopathologist (Am J Dermatopathol 1982;4:521)
      • Located in the basal epidermis and concentrated in tactile areas of hairy skin, glabrous skin, lips, eccrine sweat glands and anal canal
      • Their close relation with nerve fibers represents a Merkel cell neurite complex
      • Heavily granulated cells containing keratin filaments and neuropeptides (Anat Rec A Discov Mol Cell Evol Biol 2003;271:225)
    Diagrams / tables

    Contributed by Mariantonieta Tirado, M.D.
    Normal nail anatomy

    Normal nail anatomy

    Microscopic (histologic) description
    • Epidermis
      • Composed of 4 layers
        • Basal cell layer (stratum basale)
        • Prickle cell layer (stratum spinosum or Malpighian layer)
        • Granular cell layer (stratum granulosum)
        • Corneocyte layer (stratum corneum, horny layer)
      • Basal layer
        • Proliferating cell population with cuboidal shape, larger nuclei, conspicuous nucleoli and basophilic cytoplasm
        • Few mitotic figures may be present
        • Melanocytes surrounded by clear halo are present
        • Toker cells are clear cells present in the basal and suprabasal layers of the nipple epidermis of both males and females
      • Squamous layer
        • Several layers of larger eosinophilic polygonal cells with oval nuclei and conspicuous nucleoli
        • Cells attached to each other by spine-like processes (intercellular bridges)
      • Granular layer: 3 layers of flattened, diamond shaped cells with keratohyaline granules
      • Cornified layer: composed of flat, eosinophilic corneocytes without nuclei
      • Stratum lucidum
        • Homogenous eosinophilic zone
        • Present only in soles and palms, between granular and cornified layer
    • Melanocytes
      • Appear as clear cells (truly an artifact of fixation, secondary to shrinkage of the cytoplasm), with dendritic cytoplasm and a smaller and more basophilic nucleus than that of a basal keratinocyte
      • Ratio of melanocytes to basal cells ranges from approximately 1:4 on the cheek to 1:10 on the limbs
    • Dermis and subcutis
      • Divided into superficial papillary dermis and deeper reticular dermis
        • Papillary dermis: thin collagen fibers, located beneath the epidermis and around adnexa
        • Reticular dermis: thicker, extends from the base of the papillary dermis to the surface of the subcutis
      • Varies in thickness depending on anatomic location (eyelid: 0.5 mm; back: 5 mm)
      • Consists of connective tissue composed of collagen, elastic fibers and ground substance of mucopolysaccharides and mucoproteins
      • Harbors:
        • Scattered cells (fibrocytes, dendrocytes, histiocytes, mast cells, Langerhans cells and rare lymphocytes)
        • Adnexa
        • Smooth muscle
        • Nerves
        • Vessels: small arteries, arterioles and lymphatics
      • Small arteries, arterioles, venules, lymphatics and nerves conform a network of 2 connected plexuses parallel to the surface
        • Superficial plexus: located in the upper reticular dermis, supplies the papillary dermis with a capillary loop system
        • Deep plexus: located in the lower reticular dermis
        • Sensory receptors
          • Meissner corpuscles: ellipsoid lamellated structures, localized in the papillary dermis of lips, palms and soles
          • Pacinian corpuscles: ovoid structure with concentric lamellae, localized in the deep dermis and subcutis of genitalia, lips, palms and soles
      • Subcutis
        • Contains lobules of mature adipose tissue divided by thin connective tissue septa
        • Composed of adipocytes with a single globule of lipid that compresses the nucleus to the periphery
    • Hair follicles
      • Segments of the hair follicle in longitudinal sections
        • Upper segment: stationary
          • Infundibulum: from ostium of the follicle to the opening of the sebaceous duct; shape of a funnel with similar layers as the epidermis, with granular layer
          • Isthmus: from the opening of the sebaceous duct to the attachment of the arrector pili muscle at the hair bulge; contains a basal layer, spinous layer, absence of granular layer and an eosinophilic cornified layer
        • Lower segment: transient
          • Stem: from base of the isthmus to Adamson fringe (latter area between anucleated cells of the stem and nucleated cells of the bulb)
          • Bulb: contains matrix cells with large pale nucleus and prominent nucleoli and melanocytes that surround the dermal papillae
      • Layers of a terminal anagen hair follicle of the suprabulbar area in horizontal sections, from the center to the periphery
        • Hair shaft (medulla, cortex and cuticle)
        • Inner root sheath
          • Cuticular layer of the inner root sheath: 1 cell thick
          • Huxley layer: 2 cells thick with abundant eosinophilic trichohyalin granules
          • Henle layer: 1 cell thick, bright eosinophilic trichohyalin granules
        • Outer root sheath: composed of clear keratinocytes and keratohyaline granules
        • Vitreous and external fibrous layer (perifollicular connective tissue sheath)
      • Hair often contains Demodex folliculorum mites, clumps of Staphylococcus epidermidis or Pityrosporum yeasts
    • Sebaceous glands
      • Lobulated structures mostly connected to hair follicles
      • Distributed all over the skin with exception of palms, soles and dorsum of the feet
      • Have outer cuboidal or flattened, basophilic germinative cells that differentiate, move inward and accumulate intracytoplasmic lipid droplets, causing multivacuolation and indentations of nuclei
      • Excretory ducts are lined by keratinizing squamous epithelium
    • Apocrine sweat glands
      • Empty into the follicle above the sebaceous duct
      • Includes 2 components
        • Secretory
          • Located in the deep dermis or subcutis
          • Possesses an outer layer of myoepithelial cells and an inner layer of cuboidal to columnar eosinophilic cells
          • Shows luminal "decapitation" secretion
        • Ductal
          • Connects with the pilosebaceous follicle
          • Composed by a double layer of cuboidal cells
          • Histologically indistinguishable from eccrine ducts
    • Eccrine sweat glands
      • Includes 2 components
        • Secretory
          • Located deep in the dermis or subcutis
          • Has an outer layer of myoepithelial cells
          • Also possesses an inner layer of large clear pyramidal cells (secrete water) and smaller darker cells (secrete glycoproteins, mostly line the luminal surface)
        • Ductal
          • Opens directly into the epidermis
          • Composed of a double layer of basophilic cuboidal cells
          • Luminal surface is lined by an eosinophilic cuticle
          • Divided in 4 subunits: coiled secretory unit, coiled dermal duct, straight dermal duct, coiled intraepidermal duct (acrosyringium)
    • Nail unit
      • Comprised of the nail plate and surrounding tissues
      • Located in the dorsal aspect of the distal phalanx of fingers and toes
      • Anatomic structures include
        • Proximal nail fold: layer that extends superficially with the skin and deeply with the nail matrix
        • Eponychium (cuticle): cornified layer of the nail fold located between the nail plate and matrix
        • Nail matrix: produces the superficial and ventral portions of the nail plate
        • Lunula: white crescent shaped area representing the junction between the matrix and the bed
        • Nail plate: consists of corneocytes and is attached to the nail bed
        • Nail bed: epithelium lying over a vascularized dermis that provides support to the nail plate
        • Hyponychium: intermediate epithelium between the junction of the distal ventral edge of the free nail and the fingertip skin
        • Lateral nail folds: lateral overhanging skin folds that guide the growth of the nail plate
    • Langerhans cells
      • Dendritic cells with reniform nucleus scattered in the superficial epidermal spinous layer into the granular layer and in the dermis, difficult to see on H&E
    • Merkel cells
      • More common in outer root sheath of hair follicles and tactile hair discs
      • Not identified with H&E but with immunohistochemistry and electron microscopy
    Microscopic (histologic) images

    Contributed by Mariantonieta Tirado, M.D.
    Layers of the epidermis

    Layers of the epidermis

    Melanocytes

    Melanocytes

    MelanA+ melanocytes

    MelanA+ melanocytes

    HMB45+ melanocytes

    HMB45+ melanocytes

    Stratum lucidum

    Stratum lucidum

    Superficial and deep neurovascular plexuses

    Superficial and deep neurovascular plexuses


    Pacinian corpuscle

    Pacinian corpuscle

    Meissner corpuscles

    Meissner corpuscles

    Brown fat

    Brown fat

    Segments of hair follicle

    Segments of hair follicle

    Layers of terminal anagen hair follicle

    Layers of terminal anagen hair follicle

    Demodex folliculorum mites

    Demodex folliculorum mites


    Sebaceous gland

    Sebaceous gland

    Adipophilin+ sebaceous gland

    Adipophilin+ sebaceous gland

    Eccrine gland, myoepithelial cell

    Eccrine gland, myoepithelial cell

    Eccrine duct

    Eccrine duct

    Intraepidermal portion of eccrine duct (acrosyringium)

    Intraepidermal portion of eccrine duct (acrosyringium)

    Apocrine gland

    Apocrine gland


    Apocrine gland with decapitation secretion

    Apocrine gland with decapitation secretion

    CD1a+ Langerhans cells

    CD1a+ Langerhans cells

    CK20+ Merkel cell

    CK20+ Merkel cell

    Positive stains
    Negative stains
    Electron microscopy description
    • Melanosomes: spherical membrane bound particle with periodic longitudinal concentric lamellae
    • Birbeck granules (rod shaped structure with zipper-like striations, often with bulbous end)
    Electron microscopy images

    Images hosted on other servers:

    Birbeck granules

    Board review style question #1

    What is this structure and what does it make?

    1. Apocrine gland produces odorless secretion
    2. Eccrine gland produces merocrine secretion
    3. Hair follicle produces hair shafts
    4. Sebaceous gland produces sebum
    Board review style answer #1
    A. Apocrine gland produces odorless secretion

    Comment Here

    Reference: Basic skin histology
    Board review style question #2

    Which skin cell type is CK20+?

    1. Keratinocyte
    2. Langerhans cell
    3. Melanocyte
    4. Merkel cell
    Board review style answer #2
    D. Merkel cell

    Comment Here

    Reference: Basic skin histology
    Board review style question #3

    Which immunostain is expected to be expressed by this cell?

    1. CD1a
    2. CK5/6
    3. CK20
    4. MelanA
    Board review style answer #3
    D. MelanA

    Comment Here

    Reference: Basic skin histology

    Histoplasma capsulatum
    Definition / general
    • Infection is caused by inhalation of airborne spores of Histoplasma capsulatum, a dimorphic saprophytic fungus found in soil contaminated with bird or bat feces
    Epidemiology
    • Highest incidence is in North America, particularly in the central states along the Mississippi River basin
    • Histoplasma is endemic to Southeast U.S. (80% of this population may have positive intradermal histoplasmin skin test), Mexico, Africa, Asia
    • Transmission of disease is by inhalation of spores that can be found in feces of bats, starlings (type of bird) and chickens; causes a primary pulmonary pneumonia
    Clinical features
    • There are several clinical manifestations of histoplasmosis:
      • Acute pulmonary histoplasmosis: almost 90% patients have no symptoms
      • Chronic pulmonary histoplasmosis: occurs mostly in patients with underlying lung disease
      • Progressive disseminated histoplasmosis: occurs mostly in immunosuppressed patients
      • Ocular histoplasmosis syndrome: eye disease, occurs in 1 - 10% of the patients
      • Cutaneous histoplasmosis: erythema nodosum or erythema multiforme

    • Pneumonia is self-limited in immunocompetent patients, but disseminates in immunocompromised (very young, very old, HIV+) to liver, spleen, bone marrow, nodes, lung, rarely to skin
    • Disseminated disease: strongly associated with AIDS; patients have fever, weight loss, splenomegaly; variable cutaneous lesions
    • Cutaneous lesions are nodules, papules, ulcers; less commonly macules, pustules or vesicles
    Diagnosis
    • Culture shows tan-white-brown wooly mold at 25 - 30 °C on Sabouraud dextrose agar
    • Organisms have delicate, septate hyphae, 1 - 2 microns thick, with large rough walled macroconidia 5 - 15 microns
    • Reverts to yeast at 37 °C on sheep blood agar
    • Yeast is 2 - 4 microns, budding, single nuclei, round / oval with thin rigid walls
    Case reports
    Treatment
    • Most with minimal disease require no treatment
    • Moderate to severe cases require antifungal drugs, usually Itraconazole x 9 months in immunocompetent patients, Itraconzazole followed by IV amphotericin B in immunosuppressed patients
    Clinical images

    Contributed by Mark R. Wick, M.D.

    Breast skin, HIV+

    Microscopic (histologic) description
    • Isolated intracellular organisms and large aggregates surrounded by chronic inflammatory cells and fibroblasts (but no neutrophils or eosinophils); also epithelioid granulomas with variable caseation
    • May be narrow based budding of spores
    Microscopic (histologic) images

    Contributed by Mark R. Wick, M.D.

    Breast skin



    Images hosted on other servers:

    Within macrophages

    Positive stains
    Molecular / cytogenetics description
    • PCR probes are now available
    • Best diagnostic test is the urinary ELISA
    Differential diagnosis
    Additional references

    Hyperpigmentation
    Definition / general
    • Common, usually harmless condition, in which patches of skin become darker than normal surrounding skin (Wikipedia)
    • Due to melanocyte stimulation from drugs (Merck), heat, hormones, inflammation (eMedicine), malignancy, metabolic disease, scars, sunlight, various dermatoses or familial progressive hyperpigmentation (Eur J Dermatol 2006;16:246)
    • Patches near axilla may be postinflammatory due to hair plucking (Int J Cosmet Sci 2006;28:247)
    • Hyperpigmentation can be evaluated with Taylor hyperpigmentation scale (Cutis 2005;76:270)
    • See also solar lentigo: age / liver spots due to sun damage on hands or face
    Clinical features
    • Skin with darker pigmentation than surrounding healthy skin, due to increased melanin
    Case reports
    Treatment
    • None
    • Bleaching products with hydroquinone, retinol and antioxidants (Cutis 2008;81:365)
    • Laser therapy (Plast Reconstr Surg 2008;121:282)
    • Note: hydroquinone slows production of melanin, so darker areas gradually fade to match surrounding skin; tretinoin and cortisone take 3 - 6 months to produce improvement
    Clinical images

    Images hosted on other servers:

    Drug related:

    Amiodarone

    Bleomycin

    Diltiazem


    Imatinib (figs 1A - B)

    Imipramine
    (fig. 1)



    Other:

    Familial Progressive Hyperpigmentation

    Pre and post treatment for Vitamin B12 deficiency

       

    42 year old African American
    woman with lesions
    on left side of face
    as a result of acne excoriée

    Microscopic (histologic) description
    • Increased pigmentation of basal keratinocytes, increased transfer of melanin into adjacent keratinocytes
    • Variable melanophages, deposits in dermal cells and apoptotic cells
    • No atypia
    Microscopic (histologic) images

    Images hosted on other servers:

    Amiodarone

    Diltiazem


    Imatinib (figs 2A - B)

    Imipramine
    (fig. 2)

    Familial progressive hyperpigmentation

    Electron microscopy images

    Images hosted on other servers:

    Photoexposed pigmented skin


    Hypertrophic lupus erythematosus (pending)
    [Pending]

    Ichthyosis
    Definition / general
    • Disorder of epidermal maturation in which skin resembles fish scales
    • Associated with excessive keratin buildup due to a desquamation defect, leading to retention of abnormally formed scale
    • Usually apparent at birth
    • In adults, is associated with malignancies (lymphoma, carcinoma of bronchus, breast and cervix), sarcoidosis, lupus and drugs
    • Ichthyosis vulgaris: common type
    • Lamellar ichthyosis: rare inherited skin condition of newborn with shedding of plate-like layers of skin
    • X linked variant: deficiency in steroid sulfatase, which removes proadhesive cholesterol sulfate from intracellular spaces
    Clinical features
    • Skin resembles fish scales; rough scaly patches and plaques
    Clinical images

    Images hosted on other servers:

    Lamellar ichthyosis

    Microscopic (histologic) description
    • Increased stratum corneum with loss of normal basket weave pattern, minimal inflammation
    • Loss of granular layer in ichthyosis vulgaris

    Impetigo
    Definition / general
    Diagnosis
    • Skin examination
    • Culture of pus from skin lesions may help with pathogen identification and antimicrobial susceptibility testing
    Clinical images

    Images hosted on other servers:
    Missing Image

    Nonbullous impetigo

    Bullous impetigo in shoulder region

    Bullous impetigo in abdomen


    Crusted impetigo

    Case reports
    Treatment
    • Prevention through proper hygiene (washing affected areas and not sharing personal items with others)
    • Topical antibiotics such as mupirocin, retapamulin, fusidic acid
    • Oral antibiotics if topical therapy is impractical (amoxicillin / calvulanate, dicloaxacillin, clindamycin, doxycycline, macrolides)
    Microscopic (histologic) images

    Contributed by Jerad Gardner, M.D.
    Missing Image Missing Image

    Impetigo contagiosa



    Images hosted on other servers:

    Subcorneal blister with inflammatory cells


    Infestations-general
    Definition / general
    • Defined as parasites living in or on a host
    • Skin lesions due to direct irritant effects, immediate or delayed hypersensitivity reaction or specific effects of venom
    • Bites cause skin rash, inflamed papules or nodules, variable ulceration
    Epidemiology
    • Scabies: found worldwide and affects people of all races and social classes (CDC website)
      • Highest in endemic areas and in young, elderly, immunocompromised, nursing home residents, and resource poor, overcrowded populations
      • Prevalence ranges from 0.2% to 71.4%, highest in the Pacific and Latin American regions (Lancet Infect Dis 2015;15:960)
    Sites
    • Varies depending on infestation

    • Scabies:
      • Most commonly palms, palmar and lateral aspects of fingers, web spaces between fingers, flexor surfaces of wrists, female nipples, and male genitalia

    • Lice:
      • Scalp (Pediculus humanus capitis)
      • Body skin (Pediculus humanus corporis)
      • Hair in the pubic area (Pthirus pubis)

    • Helminths:
      • Subcutaneous areas
    Pathophysiology
    • Varies depending on the type of parasite and host response
    Etiology
    • Infestation and colonization of susceptible hosts following exposure
    • Arthropod assault or bite
    Clinical features
    • Varies depending on the type of parasite and host response
      • Scabies: burrows, marked pruritus, especially at night
      • Lice: intense itching and irritation, erosion, serous crusting and scaling of the scalp or other areas of the body
      • Helminths: subcutaneous nodules, rashes or ulcer
      • Tungiasis: subcutaneous nodules, rashes or ulcer
    • Extensive on the scalp
    • Can be associated with secondary infections
    • Erosions may progress to become open, non healing wounds and may be associated with secondary infections
    • Systemic symptoms may be present
    Diagnosis
    • Physical examination
    • Dermatoscopy is non invasive, important for diagnosis of skin infestation disease (Clin Dermatol 2014;32:315)
    • Histology is useful to find the diagnostic organism
    • PCR or other molecular tests may be performed on skin specimen for specific cases only and are useful in clinically atypical cases (Trends Parasitol 2013;29:35)
    Laboratory
    Case reports
    Treatment
    • Varies based on type of parasite and host response
    • Goals:
      • Eradicate the parasite: topical or rarely systemic medications or surgical excisions
      • Symptomatic relief: antihistamines and steroids
      • Prevent transmission to close contacts: isolation and treatment of contacts
    • Improving personal hygiene may be helpful
    Differential diagnosis
    • Dermal hypersensitivity reactions to internal antigens such as medications or external antigens including detergents, etc.
    • Lymphomatoid papulosis or other papular dermatoses

    Inflammatory linear verrucous epidermal nevus (pending)
    [Pending]

    Irritant contact dermatitis
    Definition / general
    • Due to contact with an irritant (soaps and detergents); unlike allergic contact dermatitis, prior sensitization is not required
    Clinical images

    Contributed by Mark R. Wick, M.D.
    Microscopic (histologic) description
    • Similar to allergic contact dermatitis, but no/rare eosinophils
    • May have superficial epidermal necrosis associated with intraepithelial neutrophils and scale crust
    Microscopic (histologic) images

    Contributed by Mark R. Wick, M.D.

    Jessner lymphocytic infiltration of skin
    Definition / general
    • May be a variant of lymphoid hyperplasia
    • Related to discoid lupus erythematosus
    • Violaceous nodules of head and neck or trunk
    Case reports
    Clinical images

    Contributed by Mark R. Wick, M.D.


    Images hosted on other servers:

    Bilateral lid ectropion

    Microscopic (histologic) description
    • Lymphocytes and plasmacytoid monocytes, no distinct epidermal changes
    Microscopic (histologic) images

    Images hosted on other servers:

    Chronic inflammatory cell infiltrate

    CD3


    Keratosis follicularis (Darier disease)
    Definition / general
    • Autosomal dominant genodermatosis
    • Clinical: greasy keratotic papules in a seborrheic distribution, nail changes and mucosal findings
    • Histologic: acantholytic dyskeratosis
    Essential features
    • Autosomal dominant genodermatosis due to mutations in ATP2A2 gene
    • Clinical features: family history, greasy hyperkeratotic papules in a seborrheic / flexural / acral distribution, mucosal lesions and nail changes
    • Histologic features: acantholytic dyskeratosis with corp rond and grain formation
    • Clinical correlation is crucial to distinguish from mimickers with identical histopathologic features
    Terminology
    • Darier-White disease, keratosis follicularis and dyskeratosis follicularis
    ICD coding
    • ICD-10: L11.8 - other specified acantholytic disorders
    • ICD-11: EC20.2 - hereditary acantholytic dermatoses
    Epidemiology
    Sites
    • Seborrheic distribution: central chest, back, marginal scalp and face
    • Dorsal hands
    • Palms and soles
    • Oral cavity
    • Nails
    • Intertriginous skin
    Pathophysiology
    Etiology
    Clinical features
    Diagnosis
    • Family history
    • Mucocutaneous findings
    • Skin biopsy
    • PCR DNA amplification to detect ATP2A2 mutations (Mol Med Rep 2015;12:1845)
    Prognostic factors
    Case reports
    Treatment
    Clinical images

    Contributed by Michael Fitz-Henley, M.B.B.S., D.M. and Viktoryia Kazlouskaya, M.D. (Case #517)
    Darier disease Darier disease

    Darier disease

    Flexural Darier disease

    Flexural Darier disease

    Subtle flat topped acral papules

    Subtle flat topped acral papules

    Nail changes in Darier disease

    Nail changes in Darier disease


    Leonine facies

    Leonine facies

    Keratotic papules

    Microscopic (histologic) description
    Microscopic (histologic) images

    Contributed by Jonathan D. Ho, M.B.B.S., D.Sc. and Viktoryia Kazlouskaya, M.D. (Case #517)
    Hyperkeratosis and<br>intraepidermal<br>clefting

    Hyperkeratosis and
    intraepidermal
    clefting

    Acantholytic dyskeratosis

    Acantholytic dyskeratosis

    Corp ronds and grains Corp ronds and grains

    Corp ronds and grains

    Grains forming tier

    Grains forming tier

    Villus formation

    Villus formation


    Acral papule Acral papule

    Acral papule

    Mild perivascular superficial lymphocytic inflammation, intraepidermal acantholysis and dyskeratosis

    Virtual slides

    Images hosted on other servers:

    Darier disease

    Positive stains
    • PAS stain - should be performed to rule out superimposed fungal infections but may not always be positive
    Molecular / cytogenetics description
    Sample pathology report
    • Skin of chest, punch biopsy:
      • Darier disease (see comment)
      • Comment: The specimen exhibits parakeratosis, epidermal hyperplasia, acantholytic dyskeratosis with prominent corp rond and grain formation, suprabasal clefting with formation of villi and a mild superficial perivascular lymphocytic infiltrate. Although similar acantholytic dyskeratosis may be seen in a number of entities, given the clinical history of greasy papules in a seborrheic distribution, a positive family history and persistence of lesions, the findings are most consistent with Darier disease.
    Differential diagnosis
    • Hailey-Hailey disease:
      • Intraspinous to full thickness acantholysis
      • Less prominent dyskeratosis
      • Dilapidated brick wall appearance
    • Acrokeratosis verruciformis of Hopf:
    • Pemphigus vulgaris:
      • Suprabasal and intraspinous acantholysis without corp rond and grain formation
      • Intraepidermal, intercellular deposition of IgG/C3
    • Transient acantholytic dermatosis (Grover disease):
      • May have identical histopathologic features
      • Distinction is easy based on clinical features (relapsing remitting pruritic papular eruption in middle aged to elderly males)
    • Warty dyskeratoma:
      • Identical Darier type acantholytic dyskeratosis but solitary lesion
      • Tends to have distinct cup shaped epidermal invagination and very prominent villus formation (J Dermatol 2017;44:232)
    • Acantholytic dyskeratotic acanthoma:
    • Acantholytic dermatosis of the genitocrural area:
      • Darier or Hailey-Hailey type histopathologic appearance
      • Limited to the genitocrural area
      • No other clinical features of heritable acantholytic disease
    • Focal acantholytic dyskeratosis:
    Additional references
    Board review style question #1

    A 35 year old man presents with a 10 year history of scaly papules on the face and chest. Biopsy reveals the findings shown in the image above. What is the most likely diagnosis?

    1. Darier disease
    2. Hailey-Hailey disease
    3. Herpes simplex virus infection
    4. Pemphigus vulgaris
    5. Seborrheic dermatitis
    Board review style answer #1
    A. Darier disease. The photomicrograph shows acantholysis with dyskeratosis (corp ronds and grains) as well as the formation of villi classically seen in Darier disease. While Hailey-Hailey disease may have acantholysis with dyskeratosis, prominent corp ronds and grains are lacking. Pemphigus has bland acantholysis and herpes simplex shows distinct viral cytopathic change. Seborrheic dermatitis is a spongiotic dermatitis.

    Comment Here

    Reference: Darier disease
    Board review style question #2
    Which of the following diseases may demonstrate histopathologic features identical to Darier disease?

    1. Grover disease
    2. Hailey-Hailey disease
    3. Inflammatory and linear verrucous epidermal nevus
    4. Pemphigus foliaceus
    5. Pemphigus vulgaris
    Board review style answer #2
    A. Grover disease. Grover disease has multiple histopathologic patterns including those with Darier type histology. All other options lack typical corp rond and grain formation.

    Comment Here

    Reference: Darier disease
    Board review style question #3
    Where is the abnormal protein located in Darier disease?

    1. Cytoplasm
    2. Endoplasmic reticulum
    3. Golgi apparatus
    4. Mitochondria
    5. Nucleus
    Board review style answer #3
    B. Endoplasmic reticulum. An abnormal SERCA2 protein is located in the endoplasmic reticulum and plays a role in the Ca2+ signaling pathway regulating cell to cell adhesion and differentiation of the epidermis (Nat Genet 1999;21:271).

    Comment Here

    Reference: Darier disease

    Leishmaniasis
    Definition / general
    • Protozoal disease caused by over more than 20 Leishmania species, in both tropical and subtropical regions
    • 3 main forms of the disease: visceral leishmaniasis (VL), cutaneous leishmaniasis (CL) and mucocutaneous leishmaniasis (MCL)
    Essential features
    • In H&E, presence of small hematoxylinophilic formations that are round, uniform in appearance, intracytoplasmic and sometimes distributed around the outer rim of the vacuoles (the marquee sign)
    • Weigert iron hematoxylin may stain parasites better than H&E or Giemsa
    • Immunohistostain using CD1a antibody is more sensitive than H&E
    Terminology
    • Oriental button
    ICD coding
    • ICD-10:
      • B55.0 - visceral leishmaniasis
      • B55.9 - leishmaniasis, unspecified
    Epidemiology
    • Cutaneous form is predominantly in the Middle East, whereas the mucocutaneous form occurs in South America (Brazil, Bolivia, Peru)
    Sites
    • Visceral (spleen, liver, bone marrow, lymph nodes and intestinal mucosa), cutaneous (skin) and mucocutaneous (skin and mucous membrane)
    Pathophysiology
    • Transmitted by the bite of infected female phlebotomine sandflies (infective stage: promastigotes)
    • Promastigotes phagocytized by macrophages and other types of mononuclear phagocytic cells
    • Promastigotes transform in these cells into the tissue stage of the parasite (i.e., amastigotes)
    • Parasite, host and other factors affect whether the infection becomes symptomatic and whether it results in cutaneous or visceral leishmaniasis
    • Sandflies become infected by ingesting infected cells during blood meals
    Diagrams / tables

    Images hosted on other servers:

    Life cycle of leishmaniasis

    Clinical features
    • Protozoal disease caused by more than 20 Leishmania species in both tropical and subtropical regions, such as Asia, Africa, America and Europe; not present in Australia and Pacific islands (StatPearls: Leishmaniasis [Accessed 15 March 2022])
    • 3 main forms of the disease: visceral leishmaniasis, cutaneous leishmaniasis and mucocutaneous leishmaniasis
    • Worldwide incidence of visceral leishmaniasis is 50,000 - 90,000 new cases per year (Front Cell Infect Microbiol 2022;11:839851)
    • Worldwide incidence of cutaneous leishmaniasis is 600,000 - 1 million new cases per year
    • Over 90% of mucocutaneous leishmaniasis occurs in Bolivia, Brazil, Ethiopia and Peru (Acta Trop 2022;228:106327)
    • Produces crusted, indurated papule that slowly enlarges; the disease could evolve into a self limited clinical form or into a fatal systemic illness
    • Possible coinfection Leishmania HIV
    • Visceral (also called kala azar); parasites disseminate to the spleen, liver, bone marrow, lymph nodes and intestinal mucosa causing fever, weight loss, hepatosplenomegaly, anorexia, diarrhea, anemia, thrombocytopenia, hypergammaglobulinemia
      • Rarely, it can also affect the skin: post kala azar dermal leishmaniasis (PKDL)
      • Can resolve spontaneously or after therapy (early response mediated by Th2 lymphocytes, elevated levels of IL10, IL4 versus interferon gamma and activation of B lymphocytes); if left untreated, and in some predisposed individuals, it can be fatal in over 95% of cases
      • Broadly divided into old world (the Eastern Hemisphere) and new world (the Western Hemisphere) (Parasitol Res 2021;120:1541)
      • Old world: Leishmania infantum and L. donovani
      • New world: L. braziliensis complex and L. mexicana complex
      • L. braziliensis may also produce espundia, a destructive mucocutaneous form
      • Human infection through bite of infected female phlebotomine sand flies; the sand flies inject the infective stage (i.e., promastigotes) from their proboscis during blood meals
      • Clinical presentation and prognosis vary based on species, duration of infection and immune status of patient
    • Cutaneous: usually located in exposed areas such as the face, scalp, arms or others
      • Both in the old world and the new world
      • Old world (also called button of the East world): L. major, L. tropica and skin strains of L. infantum
      • L. major (wet lesions): exudative ulcers (3 - 6 cm), rapid evolution, evolution into a disfigured scar
      • L. tropica (dry lesions): crusted plaques (1 - 2 cm), slow healing
      • New world: L. peruviana, L. panamenisis and L. amazonensis
        • Crusted papule (variable dimensions) with lymphatic dissemination around the surrounding areas which causes the formation of secondary ulcers (Microb Pathog 2021;152:104721)
        • Disseminated (if immune system doesn't respond to invading parasites), recurrent (recidivans cutaneous) or post kala azar (rare, years after visceral disease) (Microb Pathog 2021;152:104721)
    • Mucocutaneous: usually new world disease of rural and jungle regions
      • L. braziliensis and L. panamensis produce granulomatous inflammatory lesions, initially located on the nasal cartilaginous septum (tapir nose); possible involvement of oral, pharyngeal or laryngeal mucosa with ulceration and mutilation (Biol Trace Elem Res 2021;199:3918)
    Diagnosis
    • Visceral leishmaniasis: serodiagnosis (immunocompetent individuals) using common tests, such as the immunofluorescence antibody test (IFAT), enzyme linked immunoassay (ELISA) or direct agglutination test (DAT); the rapid immunochromatographic test is also available and in some cases, bone marrow aspirate is required
    • Cutaneous and mucocutaneous leishmaniasis: identification of parasites on H&E or in smears, by culture on specialized media, by Leishmania intradermal skin test (Montenegro test), by fluorescent antibody tests using the patient's serum or by PCR using species specific primers (DNA probes) (Appl Microbiol Biotechnol 2020;104:8105)
    Case reports
    • 9.5 month old girl with visceral leishmaniasis related hemophagocytic lymphohistiocytosis diagnosed by metagenomic next generation sequencing (mNGS) (Int J Infect Dis 2020;97:27)
    • 50 year old man presented with a left eye caruncular mass of 5 year duration (Orbit 2015;34:232)
    • 56 year old man with visceral leishmaniasis diagnosed by biopsy of an axillary lymph node (Genet Mol Res 2014;13:9960)
    • 67 year old man who presented with inflammatory skin lesions on the scalp and face for the past 7 years (Infection 2021;49:177)
    Treatment
    Clinical images

    Contributed by Gerardo Cazzato, M.D. and Domenico Bonamonte, M.D.

    Erythematous nodule



    Images hosted on other servers:

    Cutaneous and mucosal forms

    Microscopic (histologic) description
    • In the skin, usually a granulomatous inflammation with the presence of histiocytes / macrophages that can (not always, so called uninhabited lesions) contain tiny, uniform, round, hematoxylinophils, intracytoplasmic organisms (amastigotes)
    • In acute infections, there is a mixed inflammatory infiltrate with numerous amastigotes
    • Reference: J Am Acad Dermatol 2015;73:911
    Microscopic (histologic) images

    Contributed by Gerardo Cazzato, M.D. and Giuseppe Ingravallo, M.D., Ph.D.

    Skin lesion

    Leishmania amastigotes

    Marquee sign

    Positive stains
    Videos

    Leishmania infection

    Sample pathology report
    • Skin, biopsy (usually punch / shave):
      • Cutaneous leishmaniasis (see comment)
      • Comment: Sections of skin and subcutis, including mainly lymphomonocyte and histiocytic inflammatory infiltration, sometimes with necrosis phenomena. Within the histiocyto-macrophage component, the presence of small hematoxylinophilic formations, round, uniform in appearance, intracytoplasmic morphologically suggestive of Leishmania amastigotes is described. It is difficult to appreciate the distribution of these corpuscles around the outer rim of the vacuoles (the marquee sign). Immunostaining with CD1a was focal positive.
    Differential diagnosis
    • Other infection (fungal, bacterial, insect bite, parasitic, viral):
      • Exclude with tissue cultures and stains for microorganisms (e.g., Fite, GMS, PAS, Gram stain)
      • Histoplasmosis (similar size and appearance) but without kinetoplasts present in leishmaniasis (tissue and blood microbial cultures, urine antigen test, travel history)
    • Toxoplasma gondii
    • Penicillium marneffei
    Board review style question #1
    What types of vectors are the various species of leishmaniasis transmitted by?

    1. Bees
    2. Sandflies
    3. Ticks
    4. Wasps
    Board review style answer #1
    B. Sandflies

    Comment Here

    Reference: Leishmaniasis
    Board review style question #2
    What kind of microorganism is Leishmania?

    1. Ectoparasite
    2. Helminth
    3. Microsporidia
    4. Protozoan
    Board review style answer #2
    D. Protozoan

    Comment Here

    Reference: Leishmaniasis

    Leprosy
    Definition / general
    Terminology
    • Also called Hansen disease
    Epidemiology
    • Most U.S. cases occur in immigrants
    • Worldwide distribution due to travel and migration but endemic in tropics
    Clinical features
    • Mycobacterium leprae is an obligate intracellular gram positive and weakly acid fast organism
    • The complexity of presentation is related to the varied immunologic responses
    • The incubation period is usually 3 - 5 years
    • Tuberculoid leprosy occurs in individuals with good cell mediated immunity; patients develop granulomatous response
    • Lepromatous leprosy occurs in individuals with poor cell mediated immunity; do not develop a granulomatous response
    • Borderline leprosy is an intermediate form between tuberculoid and lepromatous leprosy
    • Transmitted by nasal discharge and digital impregnation of skin, as bacilli can be carried under nails and are inoculated under the skin by scratching
    • Lucio phenomenon is seen in Mexican and Central American patients who present with untreated, diffuse, non nodular lepromatous leprosy with hemorrhagic infarct
    • Planter lesions are at increased risk to develop squamous cell carcinoma (Indian J Lepr 1998;70:179)
    • Diagnosis is by PCR; most skin lesions have no identifiable bacteria (J Lab Physicians 2011;3:21)
    • Mitsuda reaction: intradermal injection of an armadilloderived lepra bacilli, is useful for classification

    • Description:
      • Tuberculoid leprosy has hypopigmented center and raised erythematous border
      • Lepromatous leprosy has macules, papules and plaques, but firm nodules may also be seen in the face
      • Borderline leprosy has hypopigmented macules
    Diagnosis
    • 16S ribosomal RNA gene PCR assay (can use paraffin block)
    Case reports
    Treatment
    • Clarithromycin, rifampin and dapsone
    Microscopic (histologic) description
    • Tuberculoid leprosy: epithelioid histiocytes surround small cutaneous nerves; Langhans giant cells may be seen but without necrosis; the infiltrate may involve the papillary dermis up to the epidermis; may destroy arrectores pilorum muscle; bacilli are usually scarce
    • Lepromatous leprosy: macrophages (Virchow cells, lepra) are found in poorly circumscribed masses in the dermis, with few / no lymphocytes; macrophages may be distended with large groups of leprosy bacilli (globi); bacteria are present in large numbers in cutaneous nerves and in endothelium and media of small and large vessels; may invade arrectores pilorum muscle; may have subcutaneous nodules (erythema nodosum leprorum)
    • Borderline leprosy: perineural fibrosis with lamellar or onion skin pattern; more circumscription of the granulomatous response, more lymphocytes and closer relationship to nerves
    • Indeterminate leprosy: scanty superficial and deep lymphohistiocytic infiltrate in the dermis with some tendency to localize around appendages; increased mast cells
    • Histiocytoid leprosy: spindle cell proliferation with storiform pattern suggestive of fibrous histiocytoma
    • Lucio phenomenon: leukocytoclastic vasculitis and epidermal infarction
    Microscopic (histologic) images

    Contributed by Mowafak Hamodat, M.Sc.

    Lepromatous leprosy



    Images hosted on other servers:

    Lepromatous leprosy, low magnification

    Mycobacterium lepromatosis infection

    Positive stains
    Differential diagnosis

    Lichen nitidus
    Definition / general
    • Uncommon, idiopathic, lichenoid disorder of the skin with characteristic clinical and histopathologic features
    Essential features
    • Papular eruption most commonly seen in the young
    • Discrete lymphohistiocytic infiltrate expanding 1 - 4 dermal papillae
    • Epidermal collarette gives ball and claw appearance
    • Vacuolar interface change may be prominent
    • Multinucleated giant cells often present
    • Atypical clinical variants have classical histopathologic findings
    ICD coding
    • ICD-10: L44.1 - Lichen nitidus
    • ICD-11: EA92 - Lichenoid dermatoses
    Epidemiology
    Sites
    Pathophysiology
    Etiology
    Clinical features
    Diagnosis
    • Classical clinical presentation
    • Histopathologic findings are diagnostic even in atypical variants
    Prognostic factors
    • Majority resolve spontaneously
    Case reports
    Treatment
    Clinical images

    Contributed by Jonathan D. Ho, M.B.B.S., D.Sc.

    Pinpoint koebnerized papules

    Subtle papules

    Microscopic (histologic) description
    • Discrete inflammatory cell infiltrate obscuring the dermoepidermal junction
    • Lichenoid infiltrate expands 1 - 4 dermal papillae
    • Epidermal collarette gives a ball and claw appearance
    • Variable admixture of lymphocytes and histiocytes with occasional giant cells
    • Basal layer vacuolation and colloid bodies may be prominent
    • Overlying hyperkeratosis with or without parakeratosis frequent
    • Epidermal atrophy may be noted
    • Melanophages prominent in richly pigmented individuals
    • Extravasated erythrocytes in purpuric variant (Dermatol Online J 2007;13:5)
    • Rarely, may have perifollicular distribution (Pediatr Dermatol 2013;30:e20)
    Microscopic (histologic) images

    Contributed by Jonathan D. Ho, M.B.B.S., D.Sc.

    Expanded dermal papillae

    Ball and claw appearance

    Lymphohistiocytic infiltrate

    Epidermal changes

    PAS stain

    Virtual slides

    Images hosted on other servers:

    Classical lichen nitidus

    Immunofluorescence description
    • Immunofluorescence not used for diagnosis
    • Immunoglobulins typically absent
    • May have fibrinogen in a linear pattern at the dermoepidermal junction (Arch Dermatol 1973;107:200)
    Positive stains
    • PAS highlights papillary dermal colloid bodies
    Sample pathology report
    • Left knee, punch biopsy:
      • Lichen nitidus (see comment)
      • Comment: The specimen exhibits focal parakeratosis, focal epidermal atrophy, basal layer vacuolation with associated apoptotic keratinocytes and a subjacent lichenoid lymphohistiocytic infiltrate expanding 1 - 2 dermal papillae. There are scattered multinucleated giant cells, colloid bodies and occasional melanophages. These features are diagnostic of lichen nitidus.
    Differential diagnosis
    Board review style question #1



    A 9 year old boy presents with an asymptomatic eruption of hypopigmented pinpoint papules involving the skin of his elbows, knees and penile shaft. The child is otherwise well. A punch biopsy reveals focal hyperkeratosis, epidermal atrophy, basal layer vacuolation and a discrete lymphohistiocytic infiltrate expanding a single dermal papilla and obscuring the dermoepidermal junction. What is the most likely diagnosis?

    1. Lichen nitidus
    2. Lichen planus
    3. Lichen scrofulosorum
    4. Lichen simplex chronicus
    5. Lichen striatus
    Board review style answer #1
    A. Lichen nitidus

    Comment Here

    Reference: Lichen nitidus
    Board review style question #2
    Which of the following histopathologic features favors a diagnosis of lichen nitidus over lichen planus?

    1. Basal layer vacuolar change
    2. Broad band-like lymphocytic infiltrate with scattered colloid bodies
    3. Multinucleated giant cells within a focal lichenoid infiltrate
    4. Scattered melanophages
    5. Wedge shaped hypergranulosis
    Board review style answer #2
    C. Multinucleated giant cells within a focal lichenoid infiltrate

    Comment Here

    Reference: Lichen nitidus

    Lichen planopilaris
    Definition / general
    • Lichenoid inflammation around the hair follicle
    Essential features
    • Most common scarring alopecia in White populations; discoid lupus erythematosus (DLE) and central centrifugal cicatricial alopecia (CCCA) predominate in richly pigmented populations (Am J Dermatopathol 2023;45:532)
    • Involvement of the infundibuloisthmic area of the hair structure with lymphocytic infiltration
    • Perifollicular mucinous fibroplasia
    • 3 variants
      • Classic lichen planopilaris (LPP)
      • Frontal fibrosing alopecia (FFA)
      • Graham-Little-Piccardi-Lassueur syndrome (GLPLS)
    Terminology
    • Follicular lichen planus
    • Frontal fibrosing alopecia (FFA)
    • Graham-Little-Piccardi-Lassueur syndrome (GLPLS)
    ICD coding
    • ICD-10: L66.1 - lichen planopilaris
    Epidemiology
    • F > M
    • Middle age > others
    • Most commonly occurs in White patients, followed by Black patients and then patients of other races (Int J Womens Dermatol 2018;4:180)
    Sites
    • LPP: scalp (more common in the vertex and parietal scalp)
    • FFA: frontotemporal with or without eyebrows, sideburns in males, limbs in both
    • GLPLS: scalp (scarring alopecia), axilla and groin (nonscarring alopecia), trunk and extremities (hyperkeratotic follicular papules)
    • References: An Bras Dermatol 2022;97:348, Front Med (Lausanne) 2021;8:737813
    Pathophysiology
    • Not completely understood
    • Loss of immune privilege in hair follicle stem cells
    • Autoimmune follicular damage by CD8+ T cell dominant lymphocytic infiltration
    • Disruption of peroxisome proliferator activated receptor gamma (PPARγ) and its lipid mediator stimulants
    • Epidermal - mesenchymal transition
    • FOXP3 overexpression
    • Mitochondrial malfunction
    • Increase in M2 macrophages: LPP > FFA
    • References: J Clin Med 2023;12:3259, JID Innov 2022;2:100113
    Etiology
    Clinical features
    • LPP (JID Innov 2022;2:100113, J Clin Med 2023;12:3259, Int J Trichology 2013;5:204)
      • Asymmetric, single or multifocal patches of scarring alopecia
      • Perifollicular erythema
      • Follicular scar
      • Follicular spine (plugging)
      • Symptoms: pruritus, burning or pain
      • Trichoscopy: diminished follicular ostia, perifollicular white scales and erythema, white dots, blue-gray dots with target pattern around follicles
      • Lichen planopilaris activity index (LPPAI) is used for quantification of symptoms and signs (J Am Acad Dermatol 2010;62:387)
    • FFA (JID Innov 2022;2:100113, J Clin Med 2023;12:3259)
      • Facial and extrafacial keratotic papules
      • Involvement of androgenetic dependent areas
      • Postmenopausal hair loss along the frontotemporal hairline and the eyebrows
      • Symmetric
      • Symptoms: pruritus, burning or pain
      • Trichoscopy: diminished follicular ostia, perifollicular white scales and erythema
      • Frontal fibrosing alopecia severity index (FFASI) is validated for assessing FFA (Br J Dermatol 2016;175:203)
      • Frontal fibrosing alopecia severity score (FFASS) is also validated for assessing FFA (J Am Acad Dermatol 2018;78:522)
    • GLPLS (An Bras Dermatol 2022;97:348)
      • Triad
        • Scarring alopecia of the scalp
        • Nonscarring alopecia of axilla and groin
        • Follicular papules on trunk and extremities
    Diagnosis
    Prognostic factors
    • Generally can be resistant to treatment
    • Some cases improve spontaneously or with treatment
    • Others last for years
    • Reference: Am Fam Physician 2011;84:53
    Case reports
    Treatment
    • Goal: to improve symptoms and stop progression of the alopecia
    • No written guidelines
    • Recommended therapies
      • LPP (Clin Cosmet Investig Dermatol 2018;11:91, An Bras Dermatol 2022;97:348)
        • Limited: topical / intralesional steroids
        • Extensive
          • Systemic treatment: hydroxychloroquine, methotrexate, cyclosporine, mycophenolate mofetil, pioglitazone and tofacitinib
          • Topical / intralesional steroids: if limited active lesions are present
          • Systemic steroids: only in severe cases to improve symptoms and stop rapid progression
      • FFA (An Bras Dermatol 2022;97:348)
        • Topical / intralesional steroids
        • Systemic treatment: 5 alpha reductase inhibitors, hydroxychloroquine, retinoids, etc.
    Clinical images

    Images hosted on other servers:
    Frontotemporal and eyebrow alopecia (FFA) and Multifocal scalp alopecia (LPP)

    Frontotemporal and
    eyebrow alopecia
    / multifocal scalp
    alopecia

    Trichoscopic view of LPP

    Trichoscopic view

    Microscopic (histologic) description
    Microscopic (histologic) images

    Contributed by Pooria Zare, M.D., M.P.H. and Mazaher Ramezani, M.D.
    Inflammation

    Inflammation

    Tufting Tufting

    Tufting

    Infundibuloisthmic involvement Infundibuloisthmic involvement

    Infundibuloisthmic involvement


    Upper follicular involvement Upper follicular involvement Upper follicular involvement

    Upper follicular involvement

    Scar Scar

    Scar


    Follicular plugging

    Follicular plugging

    Follicular lichenoid inflammation

    Follicular lichenoid inflammation

    Lymphohistiocytic infiltrate  and mucin

    Lymphohistiocytic infiltrate and mucin

    Immunofluorescence description
    • Could be positive in
      • Colloid bodies containing IgM or IgG in the dermis around follicular epithelium, with globular or clustered pattern (not linear)
      • Papillary dermis with irregular depositions of C3 or fibrinogen
    • Reference: An Bras Dermatol 2022;97:348
    Positive stains
    Negative stains
    Sample pathology report
    • Skin, scalp, punch biopsy:
      • Lymphocytic scarring alopecia compatible with lichen planopilaris (see comment)
      • Comment: Infundibuloisthmic involvement and perifollicular mucinous fibroplasia are features in favor of diagnosis. Clinicopathologic correlation is recommended.
    Differential diagnosis
    • Discoid lupus erythematosus (DLE):
      • Interfollicular epidermal involvement with lichenoid inflammation or vacuolization
      • Thick and tortuous epidermal basement membrane
      • Patchy lymphoplasmacytic infiltration of the superficial and deep (periadnexal and perivascular) dermis
      • Interstitial mucin deposition
      • Perieccrine inflammation
      • Dermal clusters of CD123 positive plasmacytoid dendritic cells
    • Folliculitis decalvans:
      • Mixed neutrophilic and lymphoplasmacytic inflammation
      • Absence of interface change in dermoepidermal junction
    • Central centrifugal cicatricial alopecia:
      • Mixed neutrophilic and lymphocytic inflammation
      • Syringoma-like dilated eccrine ducts (nonspecific, may be seen in other scarring alopecias; clinicopathologic correlation is necessary) (Am J Dermatopathol 2017;39:668)
    • Pseudopelade of Brocq:
      • End stage scarring alopecia
      • Most common cause of this disease is LPP
    Board review style question #1

    Which of the following features is most typical in the above case of lichen planopilaris (LPP) / frontal fibrosing alopecia (FFA)?

    1. Extension of inflammation to the subcutis
    2. Follicular tufting
    3. Involvement of the upper portion of the follicle
    4. Presence of miniaturization of hairs
    Board review style answer #1
    C. Involvement of the upper portion of the follicle. The picture shows a case of lichen planopilaris with involvement of the upper portion of the follicle, which is a hallmark of the disease. Answer D is incorrect because miniaturization of hairs is mostly seen in androgenetic alopecia and is not a feature of classic LPP. Answer B is incorrect because follicular tufting may be seen in LPP but it is also observed in other cases with follicular invasion by inflammatory cells, including folliculitis decalvans. Answer A is incorrect because the extension of inflammation to the subcutis is not a prominent feature in LPP; it is more commonly seen in discoid lupus erythematosus (DLE).

    Comment Here

    Reference: Lichen planopilaris
    Board review style question #2
    What is the most common cause of lymphocytic scarring alopecia in White predominant populations?

    1. Discoid lupus erythematosus
    2. Folliculitis decalvans
    3. Lichen planopilaris
    4. Pseudopelade of Brocq
    Board review style answer #2
    C. Lichen planopilaris. The most common cause of lymphocytic scarring alopecia is classic lichen planopilaris (LPP). Answer A is incorrect because discoid lupus erythematosus (DLE) is one of the examples of lymphocytic scarring alopecia but it is not the most common. It is associated with patchy lymphoplasmacytic infiltration of the superficial and deep dermis, including periadnexal and perivascular areas. Answer D is incorrect because pseudopelade of Brocq represents end stage scarring alopecia, mainly characterized by the absence of inflammation. Answer B is incorrect because folliculitis decalvans involves follicles with inflammatory cells, including lymphocytes, neutrophils and plasma cells.

    Comment Here

    Reference: Lichen planopilaris

    Lichen planus
    Definition / general
    • Clinical: the "6 Ps": purple, pruritic, planar (flat), polygonal papules and plaques
    • Histologic: prototypic lichenoid infiltrate dermatitis
    Essential features
    • The "6 Ps": purple, pruritic, planar (flat), polygonal papules and plaques
    • Sawtoothed rete ridges, band-like lymphohistiocytic infiltrate at the dermoepidermal junction
    • Wedge shaped hypergranulosis
    • Civatte bodies (dyskeratotic keratinocytes)
    ICD coding
    • ICD-10: L43.8 - other lichen planus
    • ICD-11: EA91 - lichen planus
    Epidemiology
    Sites
    • Skin (flexor, wrists and forearms, dorsal hands, shins, presacral area), mucous membranes (oral mucosa, genitalia), hair, nails, esophagus (rare)
    • May be confined to oral mucosa (J Dent Res Dent Clin Dent Prospects 2010;4:3)
    Pathophysiology
    • Possible delayed hypersensitivity reaction (Int J Dermatol 2009;48:682)
      • Cytotoxic CD8+ lymphocytes lead to apoptosis of basal keratinocytes
    Etiology
    • Currently unknown
    Clinical features
    • The "6 Ps": purple, pruritic, planar (flat), polygonal papules and plaques
    • Features may overlap with drug induced lichenoid eruptions; clinical history is key
    • Koebner phenomenon: new lesions at trauma sites (e.g. scratching)
    • Wickham striae: fine white lines on surface of lesions corresponding to areas of hypergranulosis
    • Clinical variants:
      • Atrophic (large lichen palnus plaques with central atrophy)
      • Hypertrophic (thick, hyperkeratotic plaques)
      • Actinic (young adults or children with lichen planus on sun exposed areas occurring during the spring and summer seasons)
      • Pigmentosus (lesions are brown, on the face and neck, more common in South Asia, frequent intertriginous involvement)
      • Ulcerative (palmoplantar ulcerations)
      • Bullous (rare, bullae within preexisting lichen planus lesions)
      • Pemphigoides (rare, circulating IgG autoantibodies against BP antigen like bullous pemphigoides)
      • References: N Engl J Med 2012;366:723, J Dtsch Dermatol Ges 2013;11:309, Int J Dermatol 2009;48:682
    Diagnosis
    • Made on H&E stain with light microscopy
    Prognostic factors
    • Disease course depends on variant:
      • Conventional (exanthematous) lichen planus: self limiting, resolves 1 - 2 years
      • Hypertrophic, oral, hair and nail lichen planus: longer, chronic course
    • Hyperpigmentation after resolution is common (postinflammatory hyperpigmentation) (Acta Derm Venereol 1991;71:242)
    • Oral lichen planus: associated with squamous cell carcinoma (oral lichen planus patients 4.8 times more likely to develop oral squamous cell carcinoma than patients without oral lichen planus) (Int J Dermatol 2019;58:296)
    Case reports
    Treatment
    • Topical corticosteroids
    • Intralesional corticosteroids
    • Oral glucocorticoids
    • Phototherapy
    • Oral retinoids
    • Oral antihistamines (for symptoms) (J Am Acad Dermatol 2019;81:1397)
    Clinical images

    Contributed by Jennifer M. McNiff, M.D.

    Lichen planus pigmentosus

    Classic lichen planus



    Images hosted on other servers:
    Lichen planus on the arm

    Lichen planus on the arm

    Wickham striae of lichen planus

    Wickham striae of lichen planus

    Lichen planus on the wrist

    Annular atrophic lichen planus

    Gross description
    Microscopic (histologic) description
    • Classic lichenoid dermatitis
    • Hyperkeratosis, acanthosis, wedge shaped hypergranulosis, sawtoothing of rete ridges, band-like lymphohistiocytic infiltrate obscuring the dermoepidermal junction (Int J Dermatol 1982;21:203)
    • Civatte / cytoid bodies (apoptotic basal keratinocytes, PAS+)
    • Basal cell squamatization (flattening of cells)
    • Parakeratosis: generally absent, think twice before diagnosing lichen planus
    • Artifactual cleft formation between epidermis and papillary dermis (Max Joseph space) (ScienceDirect: A clinicopathological study of cutaneous lichen planus [Accessed 16 October 2020])
    • Melanin pigment within macrophages of the dermis (pigment incontinence)
    • Occasional subepidermal bullae
    • Presence of eosinophils favors lichenoid drug reaction in conventional lichen planus (eosinophils can be seen in hypertrophic lichen planus) (J Cutan Pathol 2014;41:347)
    • Atrophic lichen planus: no acanthosis, fewer Civatte bodies, more pigment incontinence and minimal inflammation (Int J Dermatol 2007;46:1237)
    • Hypertrophic lichen planus: striking hyperkeratosis and acanthosis (pseudoepitheliomatous hyperplasia) (J Cutan Pathol 2014;41:347)
    • Ulcerative (erosive) lichen planus: epidermal ulceration with lichen planus changes at the ulcer margins; frequent plasma cells (Clin Exp Dermatol 1993;18:169)
    • Lichen planus pigmentosus: lymphocytic infiltrate tends to be deeper, with dermal melanin incontinence (Int J Dermatol 1992;31:90)
    • Actinic lichen planus: dermal melanin incontinence and focal parakeratosis; milder lymphocytic infiltrate (Clin Exp Dermatol 1989;14:65)
    • Bullous lichen planus: vesicle or bullae in conventional lichen planus (Clin Exp Dermatol 1989;14:150)
    • Lichen planus pemphigoides: bulla with mild perivascular lymphocytic infiltrate, may show neutrophils (J Am Acad Dermatol 1983;8:331)
    • Oral lichen planus: occasional parakeratosis, less acanthosis than in the skin, plasma cells in infiltrate
    • Lichen planopilaris: scarring alopecia with lichenoid interface dermatitis of the hair infundibulum, with fibrous tracts and Civatte bodies
    Microscopic (histologic) images

    Contributed by Simon F. Roy, M.D., Jennifer M. McNiff, M.D. and PathPresenter
    Lichenoid interface dermatitis

    Lichenoid interface dermatitis

    Civatte bodies in epidermis

    Civatte bodies in epidermis

    Hyperplastic epidermis with hyperkeratosis

    Hyperplastic epidermis with hyperkeratosis

    Lichenoid interface dermatitis

    Hypertrophic lichen planus

    Virtual slides

    Images hosted on other servers:
    Lichen planus

    Lichen planus

    Mucosal lichen planus

    Mucosal lichen planus

    Immunofluorescence description
    • Direct immunofluorescence:
    Immunofluorescence images

    Contributed by Simon F. Roy, M.D. and Jennifer M. McNiff, M.D.
    Lichen planus immunofluorescence

    Lichen planus immunofluorescence

    Positive stains
    • PAS: Civatte bodies
    Videos

    Hypertrophic lichen planus

    Sample pathology report
    • Skin, left forearm, punch biopsy:
      • Lichen planus (see comment)
      • Comment: Sections show acanthosis, hypergranulosis and a lichenoid infiltrate with Civatte bodies. The histologic features are consistent with lichen planus. If this is a solitary lesion, a benign lichenoid keratosis is favored. Clinicopathologic correlation is recommended.
    Differential diagnosis
    • Lichenoid drug reaction:
      • Lichenoid interface infiltrate with basal cell squamatization and Civatte bodies
      • Distinguishing features from lichen planus: clinical history of potential culprit medication present and eosinophils generally, sometimes plasma cells and parakeratosis
    • Benign lichenoid keratosis:
      • Lichenoid interface infiltrate with basal cell squamatization and Civatte bodies
      • Distinguishing features from lichen planus: diagnose only if solitary lesion rather than several papules and plaques
    • Chronic discoid lupus:
      • Vacuolar interface dermatitis with adnexal involvement, follicular plugging, hyperkeratosis, hypergranulosis and dermal mucin deposition
      • Distinguishing features from lichen planus: can resemble hypertrophic variant of lichen planus, although presence of epidermal atrophy, follicular plugging and adnexal involvement are in favor of lupus
    • Secondary syphilis:
      • Several histopathological patterns possible but classically a psoriasiform dermatitis, demonstrating slender acanthosis of the rete pegs and a prominent plasma cell infiltrate in the dermis
      • Distinguishing features from lichen planus: plasma cells and psoriasiform acanthosis
        • In rare occurences, syphilis may show a lichenoid interface dermatitis, although spirochetes would be absent by Warthin-Starry stain or Treponema pallidum immunohistochemistry
    • Squamous cell carcinoma (keratoacanthoma type):
      • Crateriform well differentiated epithelial proliferation with central keratinization, enlarged keratinocyte cells with ample eosinophilic cytoplasm, dyskeratosis and mitoses
      • Distinguishing features from lichen planus: absent wedge shaped hypergranulosis, present cytologic atypia, marked solar elastosis, lymphovascular or perineural invasion, perforating elastic fibers upon elastic staining and absence of eosinophils
    Board review style question #1
      Which of the following is true about cutaneous lichen planus?

    1. Civatte bodies are PAS negative
    2. Eosinophils are common
    3. Dyskeratotic keratinocytes are frequently seen at the basal layer
    4. Mucosal involvement is seldom seen
    Board review style answer #1
    C. Dyskeratotic keratinocytes are frequently seen at the basal layer. Dyskeratotic keratinocytes (so called Civatte bodies) are frequently seen at the basal layer. Civatte bodies are PAS positive. Eosinophils are uncommon in conventional cutaneous lichen planus. Lichen planus may occur on mucosal surfaces such as the oral mucosa or genitalia.

    Comment Here

    Reference: Lichen planus
    Board review style question #2

      Which histologic feature of lichenoid drug eruption is absent in classic lichen planus?

    1. Civatte bodies
    2. Eosinophils
    3. Hypergranulosis
    4. Hyperkeratosis
    Board review style answer #2
    B. Eosinophils. Classic lichen planus and lichenoid drug eruption both share similar histopathological features (civatte bodies, wedge shaped hypergranulosis and hyperkeratosis). However, eosinophils favor a lichenoid drug eruption.

    Comment Here

    Reference: Lichen planus

    Lichen sclerosus
    Definition / general
    • Immune mediated chronic mucocutaneous disorder involving both genital and extragenital skin
    Essential features
    • Sclerotic shiny, ivory colored papules and plaques
    • Most commonly female or male genitalia, less often extragenital skin
    • Lichenoid interface dermatitis with epidermal atrophy and progressive sclerosis / homogenization of papillary dermal collagen fibers
    Terminology
    • Current preferred nomenclature of lichen sclerosus (LS) initially proposed in 1975 (J Cutan Pathol 1976;3:159)
    • Other names
      • Lichen sclerosus et atrophicus
      • White spot disease
      • Csillag disease
      • Lichen albus
      • Leukoplakia
      • Kraurosis vulvae
      • Balanitis xerotic obliterans
    ICD coding
    • ICD-10
      • L90.0 - lichen sclerosus et atrophicus
      • N90.4 - leukoplakia of vulva
      • N48.0 - leukoplakia of penis
    • ICD-11
      • EB60.Y - lichen sclerosus of other specified sites
      • EB60.0 - lichen sclerosus of vulva
      • EB60.1 - lichen sclerosus of penis
    Epidemiology
    • Female predominance with bimodal distribution, prepubertal children and postmenopausal females
    • Frequency in the U.S. (Int J Womens Dermatol 2020;6:260)
      • 0.05% is likely an underestimation
      • 0.01% in the pediatric population
    Sites
    Pathophysiology / etiology
    Clinical features
    • White, flat topped papules, plaques or atrophic patches
    • May be surrounded by an erythematous to violaceous halo
    • Atrophic lesions: skin is smooth, wrinkled and soft
    • Itching is frequently severe, especially in anogenital area
    • Dysuria, urethral and vaginal discharge, dyspareunia, burning pain, fissuring and erosion may occur
    • Obliteration of anatomic structures with loss of labia minora, clitoral hood and urethral meatus
    • Perianal and vulvar involvement results in a figure 8 appearance
    • Phimosis, paraphimosis and recurrent balanitis are common in men
    • Associated with increased risk for genital squamous cell carcinoma (SCC) (Int J Cancer 2017;140:1998)
    • Rare reports of pediatric and adult vulvar melanoma arising in association with LS (Pediatr Dermatol 2016;33:e190)
    • Can be seen in association with morphea / scleroderma as overlapping lesions
    Diagnosis
    • Clinical findings are helpful; however, biopsy provides a definitive diagnosis, especially in the following settings
      • Clinically equivocal lesions
      • Treatment resistant lesions
      • Suspicion for malignancy
    Prognostic factors
    • Early diagnosis, treatment and response to treatment may prevent progression of disease
    • Late diagnosis / longstanding lesions, incomplete treatment compliance / response and coexisting neoplasia (VIN) may contribute to an increased risk of SCC, particularly in females (Gynecol Oncol 2020;157:671)
    Case reports
    Treatment
    • High potency topical corticosteroids (first line)
    • Topical calcineurin inhibitors
    • Intralesional corticosteroids, topical calcipotriol, hydroxychloroquine, cyclosporine, penicillamine, methotrexate and phototherapy (reserved for refractory cases)
    • Reference: Front Med (Lausanne) 2023;10:1106318
    Clinical images

    Images hosted on other servers
    Genital and extragenital LS

    Genital and extragenital LS

    Gross description
    • White, flat topped papules, plaques or atrophic patches
    • Erythematous to violaceous halo may be present
    • Fissuring and erosion may occur
    Microscopic (histologic) description
    • Low power: striped, trilayered or red, white and blue appearance
      • Red: compact orthokeratosis
      • White: papillary dermal edema / homogenized papillary dermal collagen bundles
      • Blue: band-like lymphocytic infiltrate
    • Early lesions
      • Often subtle with changes commonly seen in a periadnexal distribution including follicular hyperkeratosis and hypergranulosis (Histopathology 2005;47:340)
      • Variably dense lichenoid or interstitial inflammatory infiltrate
      • Homogenization of papillary dermal collagen fibers may be focal
    • Established lesions: hyperkeratosis, follicular plugging, epidermal atrophy with loss of rete ridges, basal layer vacuolization, homogenized / sclerotic papillary dermal collagen bundles, a band-like lymphocytic infiltrate and dilated blood vessels
    • Acanthosis, dilated blood vessels and eosinophils noted in genital LS (Zhongguo Yi Xue Ke Xue Yuan Xue Bao 2021;43:849)
    • Genital LS cases (93.4%) with dense inflammatory infiltrates can demonstrate a mycosis fungoides-like, monoclonal (50%), intraepidermal lymphocytic infiltrate (Am J Surg Pathol 2022;46:83)
    Microscopic (histologic) images

    Contributed by Candice E. Brem, M.D.
    Tiered appearance

    Tiered appearance

    Follicular plugging

    Follicular plugging

    Basal layer vacuolization

    Basal layer vacuolization

    Homogenized dermal collagen

    Homogenized dermal collagen

    Subepidermal split

    Subepidermal split

    Hemorrhagic and bullous LS

    Hemorrhagic and bullous LS

    Virtual slides

    Images hosted on other servers
    Genital / vulvar LS

    Genital / vulvar LS

    Genital / penile LS (balanitis xerotica obliterans)

    Genital / penile LS (balanitis xerotica obliterans)

    Positive stains
    • Immunohistochemical studies generally not needed for diagnosis
    Negative stains
    • Immunohistochemical studies generally not needed for diagnosis
    Videos

    Lichen sclerosus in 5 minutes

    Lichen sclerosus mimicking mycosis fungoides

    Basics of lichen sclerosus

    Sample pathology report
    • Skin, lower abdomen, shave biopsy:
      • Lichen sclerosus (et atrophicus) (see comment)
      • Comment: The specimen exhibits compact orthokeratosis, epidermal atrophy, focal basal layer vacuolization, homogenization of papillary dermal collagen fibers and a moderate to dense underlying band-like lymphocytic infiltrate. These findings support the histologic diagnosis.
    Differential diagnosis
    • Lichen planus:
      • Subepidermal band-like lymphocytic infiltrate
      • Absence of superficial sclerosis
    • Mycosis fungoides, patch / plaque stage:
      • Absence of typical histopathologic features of lichen sclerosus
      • Monoclonal rearrangement of the T cell receptor gene rearrangement
      • Diagnosis of mycosis fungoides in genital skin should be made only upon compelling clinicopathologic criteria
    • Vulvar intraepithelial neoplasia (VIN) / penile intraepithelial neoplasia (PeIN):
      • Presence of significant keratinocytic atypia
      • Abnormal / non-wild type p53 immunostaining
    • Lichen simplex chronicus:
      • Hypergranulosis
      • Epidermal hyperplasia rather than atrophic changes
      • Absence of basement membrane degeneration / vacuolization
      • Absence of superficial subepidermal sclerosis
    • Localized scleroderma / morphea:
      • Sparing of the overlying epidermis and superficial dermis
      • Involvement of the reticular dermis with extension into the subcutaneous tissue
      • Sparse deep periadnexal and perivascular inflammatory infiltrate
      • Overlap syndromes can occur
    • Chronic radiation dermatitis:
      • Radiation induced changes: elastosis, vascular ectasia, enlarged and atypical fibroblasts (radiation fibroblasts)
      • Loss of adnexal structures
    Board review style question #1

    A 67 year old woman presents with a longstanding history of pruritic, atrophic, ivory colored vulvar papules and plaques. A subsequent shave biopsy is performed (see above image). What is the best histologic diagnosis of this lesion?

    1. Lichen amyloidosis
    2. Lichen planus
    3. Lichen sclerosus
    4. Lichen simplex chronicus
    5. Vulvar intraepithelial neoplasia
    Board review style answer #1
    C. Lichen sclerosus. The clinical presentation taken in conjunction with the histologic findings of epidermal atrophy and homogenized papillary dermal collagen fibers with an adjacent focally band-like lymphocytic infiltrate are consistent with lichen sclerosus. Answer B is incorrect because lichen planus, while it generally presents as a band-like inflammatory infiltrate, should not have these changes in the papillary dermal collagen. Answer D is incorrect because lichen simplex chronicus should demonstrate epidermal hyperplasia and lacks the papillary dermal collagen changes and band-like inflammatory infiltrate. Answer A is incorrect because although there is homogenization of the papillary dermal collagen fibers, no amorphous eosinophilic material in the papillary dermis consistent with amyloid and low papillomatous epidermal hyperplasia are seen. Answer E is incorrect because no significant epidermal cytologic atypia is appreciated.

    Comment Here

    Reference: Lichen sclerosus

    Lichen simplex chronicus
    Definition / general
    • Chronic dermatitis characterized by localized areas of thickened, hyperkeratotic skin resulting from chronic rubbing or scratching
    • Absence of underlying dermatologic condition
    • These changes, known in conjunction as lichenification, are occasionally superimposed on other dermatoses, inflammatory and neoplastic
    Essential features
    Terminology
    • Neurodermatitis circumscripta
    • Neurodermatitis
    ICD coding
    • ICD-10: L28.0 - lichen simplex chronicus
    • ICD-11: EA83 - lichen simplex or lichenification
    Epidemiology
    Sites
    Pathophysiology
    Etiology
    • Pruritus
      • Underlying stimulus and most important symptom (Dermatol Ther 2008;21:42)
      • Hypothesized relation to an increased sensitivity to local neurotransmitters and their neuromodulation, in response to primary stimulus (i.e. temperature, pressure, etc.) (Semin Diagn Pathol 2017;34:220)
      • May be secondary to other dermatosis, medical conditions or psychological disorders (Dermatol Ther 2008;21:42)
      • Associated with psychiatric disorders, including depression, anxiety, dissociation and obsessive compulsive disorder (Dermatol Ther 2020;33:e14389)
    Clinical features
    • Circumscribed
    • Early lesions may appear erythematous
    • Chronicity results in hyperkeratosis with thickened scaly plaques, excoriated papules, leukodermic (hypopigmented) center and increased skin markings (Dermatol Ther 2008;21:42)
    • Longstanding lesions may also have depigmentation or hypopigmentation
    Diagnosis
    Laboratory
    • As pertinent, if an underlying medical condition is suspected as the etiology of pruritus
    Prognostic factors
    Case reports
    Treatment
    Clinical images

    Contributed by Cristian Rocha, M.D. and Jonathan D. Ho, M.B.B.S., D.Sc.
    Left lower back lesion

    Left lower back lesion

    Dorsal foot plaques

    Dorsal foot plaques

    Microscopic (histologic) description
    • Early lesions may demonstrate spongiosis (Semin Diagn Pathol 2017;34:220)
    • Evolves to demonstrate (Semin Diagn Pathol 2017;34:220)
      • Compact orthokeratosis with focal parakeratosis
      • Hypergranulosis
      • Occasional pseudoepitheliomatous hyperplasia
      • Prominent irregular acanthosis with curvilinear, blunt rete ridges
      • Papillary dermal fibrosis, with vertically oriented collagen bundles
      • Superficial chronic perivascular inflammatory infiltrate
      • Rare mast cells and eosinophils
      • As lichenification may be superimposed on other dermatoses, careful search for other disorders is advised
    Microscopic (histologic) images

    Contributed by Jose A. Plaza, M.D.
    Psoriasiform pattern

    Psoriasiform pattern

    Vertically oriented dermal fibrosis

    Vertically oriented dermal fibrosis

    Sample pathology report
    • Skin, left lower leg, shave biopsy:
      • Lichen simplex chronicus (see comment)
      • Comment: Sections demonstrate hyperkeratosis, focal parakeratosis, a thickened granular layer and marked acanthosis. Characteristic vertically oriented, thickened collagen bundles are seen in the superficial dermis. The underlying dermis shows mild to moderate chronic inflammation. These findings, in the right clinical setting, are consistent with the above diagnosis.
    • Note: Avoid descriptive diagnosis that will not help the clinician to properly treat their patients. Breaking the itch / scratch cycle is essential to treat this entity (Mod Pathol 2020;33:107).
    Differential diagnosis
    • Psoriasis (Semin Diagn Pathol 2017;34:220, Dermatol Ther 2004;17:8):
      • Regular acanthosis with club shaped rete ridges
      • Confluent parakeratosis
      • Hypogranulosis
      • Intracorneal neutrophils
      • Acanthosis
      • Vascular congestion of papillary dermis
    • Other chronic spongiotic dermatitides:
      • Spongiosis
      • Acanthosis
      • Exocytosis
      • Perivascular inflammatory infiltrate, predominantly lymphohistiocytic and occasional neutrophils and eosinophils (the latter may be numerous in allergic contact dermatitis)
      • Their differentiation may not be possible if superimposed lichenification is present
    • Prurigo nodularis
      • Same findings as in lichen simplex chronicus but in nodular configuration rather than plaque-like
    Board review style question #1

    A pathologist is reviewing a skin case. The clinical history field on the requisition indicates chronic pruritus. Hematoxylin and eosin stained sections of a punch skin biopsy submitted as lower leg lesion demonstrate the features shown in the photomicrograph. This entity most likely represents:

    1. Dermatophyte infection
    2. Lichen simplex chronicus
    3. Mycosis fungoides
    4. Psoriasis
    Board review style answer #1
    B. Lichen simplex chronicus

    Comment Here

    Reference: Lichen simplex chronicus
    Board review style question #2
    Which clinical scenario best fits a diagnosis of lichen simplex chronicus?

    1. 30 year old man with buttocks and scalp lesions (white / silver scaly, well demarcated plaques with underlying erythema)
    2. 45 year old woman with dorsal hand lesion (circumscribed, erythematous pruritic plaque with leukodermic center and increased skin markings)
    3. 50 year old woman with neck lesion (annular plaque of erythema, scale at advancing border)
    4. 60 year old man with numerous trunk lesions (initially pruritic patches with minimal scaling with evolution to erythematous plaques)
    Board review style answer #2
    B. 45 year old woman with dorsal hand lesion (circumscribed, erythematous pruritic plaque with leukodermic center and increased skin markings)

    Comment Here

    Reference: Lichen simplex chronicus

    Lichen striatus
    Definition / general
    • Small papules that expand over time to form bands or streaks, then disappear
    • Usually affects extremities of children in spring and summer
    • Case clustering has been documented
    Clinical features
    • Solitary unilateral, erythematous flesh colored lichenoid or sometimes psoriasiform scaly papule, which coalesce into a continuous or interrupted or curved bands
    Treatment
    • Self limiting, disappears within months or a year of onset
    Clinical images

    Images hosted on other servers:

    Lichen striatus

    Microscopic (histologic) description
    • Spongiosis and vacuolar interface change; also acanthosis and parakeratosis
    • Dermal lymphohistiocytic infiltrate around superficial and deep vessels and around adnexa
    Microscopic (histologic) images

    Images hosted on other servers:

    Various images


    Lichenoid drug reaction
    Fixed drug reaction
    Definition / general
    • May cause urticaria, erythema multiforme, erythema nodosum, folliculitis, pustules, purpura, hyperpigmentation or vasculitis
    • Procainamide may induce SLE-like disease
    • Anti-cancer drugs may cause acute necrotizing changes in sweat glands (neutrophilic eccrine hidradenitis or syringosquamous metaplasia)
    • Drug induced coma may cause necrosis of eccrine sweat coils and other adnexae, also epidermis
    • Other drugs causing cutaneous reactions include gold salts, thiazides, antimalarial drugs, color film developers, tetracycline, barbiturates, phenolphthalein
    • Fixed drug reaction: repeated administration causes recurrence of red-brown patch in same location, variable bullae

    Sites
    • Genitalia and face

    Treatment
    • Discontinue drug

    Clinical images

    Contributed by Mark R. Wick, M.D.

    Breast skin



    Images hosted on other servers:

    14 year old boy with reaction to sulfasalazine



    Microscopic (histologic) description
    • Eosinophils and marked vascular wall thickening
    • May cause granulomatous infiltrate
    • Also vacuolar interface changes and often lymphocytic exocytosis, dyskeratotic keratinocytes, parakeratosis with eosinophils and plasma cells in dermis

    Microscopic (histologic) images

    Contributed by Mark R. Wick, M.D.

    Breast skin



    Differential diagnosis

    Additional references
    Interstitial granulomatous drug reaction
    Definition / general
    • Reaction due to beta blockers, calcium channel blockers, lipid lowering agents, ACE inhibitors, diuretics, NSAIDs, antihistamines, antidepressants and oral hypoglycemic agents; often more than one of the above
    • Mean duration of 5 years before onset of eruptions
    • Resolves a mean 8 weeks (range 1 - 40) after drug is stopped
    • May be due to exaggerated immune response to drugs

    Clinical features
    • Violaceous to erythematous, nonpruritic plaques with bizarre shapes or annular morphology on skin folds (axilla, inner arms, groin, medial thighs and popliteal fossa)

    Case reports

    Microscopic (histologic) description
    • Diffuse granulomatous dermatitis with interstitial histiocytes and giant cells near collagen and elastic fibers, often with piecemeal fragmentation of collagen, giant cell engulfment of elastic fibers and variable interstitial mucin (J Cutan Pathol 1998;25:72)
    • Usually interface dermatitis with basilar vacuolopathy and focal dyskeratosis with lymphocytes at dermoepidermal junction
    • Usually tissue eosinophilia, lymphoid atypia with hyperchromatic nuclei, convoluted nuclear contours (including a few cerebriform nuclei in dermis), variable involvement of hair follicles and acrosyringia
    • Rare collagen necrobiosis
    • Usually no fibrin deposition within vessels or vasculitis

    Differential diagnosis
    Morbiliform drug reaction (pending)
    [Pending]

    Linear and whorled nevoid hypermelanosis
    Definition / general
    • Rare skin disorder, often in first weeks of life, with swirls and streaks of macular hyperpigmentation along lines of Blaschko (Wikipedia)
    • Usually not associated with other abnormalities (Pediatr Dermatol 2007;24:205)
    Diagrams / tables

    Images hosted on other servers:

    Lines of Blaschko

    Clinical features
    • Swirls and whorls of hyperpigmented macules in reticulate pattern along Blaschko lines
    Case reports
    Treatment
    • No satisfactory treatment
    Clinical images

    Images hosted on other servers:

    Whorled pattern over trunk (fig. 2)

    Macular hyperpigmentation

    Microscopic (histologic) description
    • Epidermal melanosis
    • No pigment incontinence or melanophages
    Microscopic (histologic) images

    Images hosted on other servers:

    Epidermal hypermelanosis

    Diffuse melanosis (fig. 1)


    Linear IgA bullous dermatosis
    Definition / general
    • Children or adults with acquired (drug induced or idiopathic) autoimmune disorder of pruritic subepidermal bullae or vesicles with linear deposits of IgA along the basement membrane
    Sites
    • Thighs, buttocks, lower trunk, genital area, scalp, perioral face
    • Mucous membrane involvement is importance because of associated scarring
    Etiology
    • Unclear, appears to be immune mediated
    • Target antigen of IgA autoantibodies is 120 kd secretory portion of BP180 antigen, but other antigens have also been reported
    • Often occurs after administration of antibiotics, predominantly vancomycin or penicillin, rarely after amoxicillin-clavulanic acid (Pediatr Dermatol 2007;24:E40)
    • Also after administration of nonsteroidal anti-inflammatory agents, diuretics, rarely interferon alpha-2A
    • Not associated with HLA DR3, B8 (Clin Exp Dermatol 2009;34:e123)
    Clinical features
    • Small tense blisters or annular bullous plaques ("cluster of jewels" or "string of beads") with erosions and crusting
    • Vancomycin associated cases may present with morbilliform (resembling measles) eruption without blistering (Arch Dermatol 2008;144:774)
    • In Tunisia, is most frequent childhood bullous dermatosis; usually spares mucosa; responds rapidly to dapsone with long stabilization; erythromycin and oxacillin may be alternative therapy (Pediatr Dermatol 2009;26:28)
    • Healing may be associated with hyper or hypopigmentation
    • Childhood cases are usually self healing; symptoms may last over 5 years, and occasionally extend into adult life
    • Adult cases follow a more prolonged course and rarely are refractory
    • Associated with internal malignancy, including lymphoma and bladder carcinoma
    Case reports
    Treatment
    Clinical images

    Images hosted on other servers:

    Annular bullous plaques ("cluster of jewels") with erosions and crusting, pre and posttherapy

    Microscopic (histologic) description
    • Subepidermal blisters with neutrophilic infiltration
    • Identical to dermatitis herpetiformis
    Microscopic (histologic) images

    Images hosted on other servers:

    Subepidermal blisters with neutrophils

    Neutrophilic microabscess

    (B) and (C), respectively: vancomycin associated cases

    Linear IgA basement membrane staining

    Positive stains
    • Linear deposition of IgA along the basement membrane zone of epidermis
    • IgA band is linear, at basement membrane, compared to dermatitis herpetiformis, which has granular band at dermal papillae
    • IgG in 25% of cases (Br J Dermatol 1997;116:293)
    • IgM and C3 are occasionally present (Int J Dermatol 1985;24:569)
    Differential diagnosis
    • Dermatitis herpetiformis: similar morphology but IgA band is usually granular; band is at dermal papillae; serum has anti-endomysial or anti-tissue transglutaminase IgA antibodies, symptoms improve on gluten-free diet (Dermatol Online J 2008;14:21)
    • Bullous lupus erythematosus: systemic symptoms; papillary dermal microabscesses, mucin in reticular dermis

    Lipodermatosclerosis
    Definition / general
    • Lipodermatosclerosis (hypodermitis sclerodermiformis and sclerosing panniculitis) presents as a chronic fibrosing disease commonly associated with venous insufficiency, arterial ischemia or thrombophlebitis
      • It is known to affect the subcutis of the lower extremities, commonly in obese, white women, older than 60
    • Hypodermitis sclerodermiformis was described in 1955 by Huriez et al. as a scleroderma-like hardening of the legs thought to result from infection of venous ulcers
      • In 1980, Burnand et al. used the term venous lipodermatosclerosis, which was characterized by changes in the skin and subcutaneous changes commonly known as postphlebitic leg (Arch Dermatol 1982;118:312)
    Essential features
    • Lipodermatosclerosis is a chronic inflammatory condition characterized by subcutaneous fibrosis and hardening of the skin on the lower extremities due to venous insufficiency
    • Lipodermatosclerosis is also known as venous stasis panniculitis or hypodermitis sclerodermaformis
    Terminology
    • Venous stasis panniculitis
    • Hypodermitis sclerodermiformis and sclerosing panniculitis
    • Chronic panniculitis with lipomembranous changes
    ICD coding
    • ICD-10: I83.10 - varicose veins
    Epidemiology
    Sites
    • Lower extremities
    • Pretibial or medial aspect of the leg
    Pathophysiology
    • Result of venous insufficiency, triggered by:
    • Resulting venous hypertension leads to an increase of leukocytes in the veins, which migrate to surrounding tissue
    • Leukocytes are activated, releasing proinflammatory cells and cytokines, inducing a chronic inflammatory state
    • Increased collagen production leads to the fibrosis of subcutaneous fat
    Etiology
    • Vascular damage, venous incompetence / hypertension and to a lesser extent, arterial ischemia
    • Abnormal fibrinolytic activity
    • Histologically shows an increase in dermal fibroblast proliferation and procollagen gene expression, suggesting involvement of these factors
    • Reference: Dermatol Ther 2010;23:375
    Clinical features
    • Acute phase
      • Similar to cellulitis, with induration, erythema, pain, itching and swelling
    • Chronic phase
      • Subcutaneous fibrosis, resulting in narrowing of the distal lower limb, causing the leg to have the classical upside down champagne bottle appearance; hard, wood-like appearance, which becomes circumferential in well developed lesions (Semin Cutan Med Surg 2013;32:169)
      • Overlying ulceration or crusting
      • Hyperpigmentation of the skin from hemosiderin deposition, varicose veins, venous eczema
    Diagnosis
    • Biopsy
    Case reports
    Treatment
    • Lifestyle: weight reduction, physical activity
    • Preventive measures: compression therapy and elevation
    • Medical treatment: stanozolol, pentoxifylline, hydroxychloroquine (Am Fam Physician 2019;100:298)
    • Ultrasound therapy
    • Ultraviolet radiation (UVA1)
    • Surgical treatment
    Clinical images

    Contributed by Merrick Kozak, M.D.
    Bilateral lower extremities

    Bilateral lower extremities

    Microscopic (histologic) description
    • Lobular and septal involvement of the subcutaneous tissue, fat necrosis, macropseudocyst and micropseudocyst formation and calcification of adipocytes
    • Lipomembranes (crenated appearance, eosinophilic feathery anuclear lining of cystic adipocytes) classic but not definitive for the diagnosis
    • Pseudoxanthoma elasticum-like septal elastosis with calcification is a consistent finding
    • Reference: Int J Dermatol 2016;55:303
    Microscopic (histologic) images

    Contributed by Shyam Raghavan, M.D.
    Pseudocyst formation Pseudocyst formation

    Pseudocyst formation

    Dermal sclerosis

    Dermal sclerosis

    Sample pathology report
    • Skin, left leg, biopsy:
      • Lipodermatosclerosis
    Differential diagnosis
    • Erythema nodosum:
      • Septal panniculitis with a mixed cellular infiltrate of lymphocytes, histiocytes, giant cells and occasional eosinophils, and a characteristic absence of vasculitis
    • Morphea (localized scleroderma):
      • Atrophic epidermis
      • Thickened, hyalinized collagen, with possible loss of appendageal structures
      • Inflammatory cell infiltrate (lymphocytes with or without plasma cells)
    • Necrobiosis lipoidica:
      • Layered inflammatory process and alternating zones of necrobiosis involving the full thickness of the dermis (stacked lasagne type appearance)
      • Histiocytic infiltrate with multinucleated giant cells surrounds these areas and accompanying inflammatory infiltrate is predominantly lymphocytic with plasma cells and occasional eosinophils
    • Cellulitis:
      • Neutrophil predominant perivascular inflammation
    Board review style question #1

    Which of the following is a microscopic feature of lipodermatosclerosis?

    1. Granuloma formation
    2. Neutrophil infiltration
    3. Predominant histiocytic infiltrate
    4. Pseudocyst formation
    Board review style answer #1
    D. Pseudocyst formation. The biopsy shows classical findings of lipodermatosclerosis, including fat necrosis, pseudocyst formation, calcification of adipocytes and the presence of lipomembranes (eosinophilic feathery anuclear lining of cystic adipocytes) are diagnostic; however, neutrophil infiltration, predominant histiocytic infiltrate and granuloma formation are not historically associated with lipodermatosclerosis.

    Comment Here

    Reference: Lipodermatosclerosis
    Board review style question #2
    Which of the following is the underlying major pathophysiology for lipodermatosclerosis?

    1. Allergic reaction
    2. Bacterial infection
    3. Chronic venous stasis
    4. Repeated microtrauma
    Board review style answer #2
    C. Chronic venous stasis. Lipodermatosclerosis is an inflammatory skin condition resulting from underlying venous insufficiency, which is caused by incompetent venous valves, venous outflow obstruction or dysfunction of the calf muscle pump.

    Comment Here

    Reference: Lipodermatosclerosis

    Lipodystrophy (pending)

    Lipoid proteinosis
    Definition / general
    • Autosomal recessive condition caused by mutations in the ECM1 gene, which leads to deposits of hyaline material in the skin and mucosal surfaces
    Essential features
    • Deposits of PAS and PASD positive homogenous, eosinophilic material in the dermis and lamina propria corresponding clinically to waxy yellowish white lesions
    • Variable clinical presentation (see clinical features below)
    • Autosomal recessive inheritance
    Terminology
    • Also known as hyalinosis cutis et mucosae or Urbach-Wiethe disease
    Epidemiology
    • Men and women are equally affected
    • Typically occurs in consanguineous families
    • Rare, only a few hundred known patients
    • Increased incidence in South Africa (likely due to founder mutation, Mol Syndromol 2016;7:26)
    Pathophysiology
    • Uncertain
    • ECM1 (extracellular matrix protein 1) is a component of the extracellular matrix and patients with LP lack functional ECM1 - this leads to hyaline deposits through an unknown mechanism
    Etiology
    • Autosomal recessive disease
    Clinical features
    • Clinical features are variable but include (Vahidnezhad: GeneReviews - Lipoid Proteinosis, 2016):
      • Hoarse cry as an infant, hoarse voice
      • Moniliform blepharosis (papules on the eyelid margins)
      • Vesicles with hemorrhagic crust on skin and oral mucosa in early lesions
      • Lesions evolve into waxy yellowish white papules, nodules or plaques
      • Poor wound healing
      • Calcifications in the brain, particularly a "bean shaped" calcification above the pituitary fossa
      • Seizures
      • Behavioral issues such as memory impairment, paranoia, aggressive behavior, hallucinations and lack of fear (involvement of the amygdala)
      • Parotitis and poor dental hygiene
      • Upper respiratory infections
      • Shortened tongue, difficulty in protruding tongue
      • Patchy alopecia
    Diagnosis
    • Clinical suspicion followed by sequencing of the ECM1 gene
    • Biopsy may be helpful but is not required for the diagnosis
    Laboratory
    • There are no specific laboratory findings
    Radiology description
    • Brain CT can identify calcifications but is not needed for diagnosis
    Prognostic factors
    • Patients with lipoid proteinosis have a normal life expectancy
    • However, deposits in the larynx can lead to airway obstruction and may need to be ablated
    Case reports
    Treatment
    • Lesions can be excised or ablated but there is no curative treatment
    Clinical images

    Case #433

    Beaded papules over bilateral eyelids

    Waxy papules over back of forearm

    Gross description
    • Skin or mucosa with waxy yellowish white nodules
    Microscopic (histologic) description
    • Amorphous, eosinophilic mostly acellular hyaline material in the papillary dermis and extending around sweat glands and blood vessels
    • In mucosal biopsies can be seen surrounding salivary gland acini
    Microscopic (histologic) images

    Case #433

    Eosinophilic homogenous material in the papillary dermis

    PAS positive

    Positive stains
    Negative stains
    Differential diagnosis
    • Amyloidosis
      • Deposits are similar on H&E but special stains for amyloid will be positive
    • Porphyria
      • Histologic appearance of early lesions in lipoid proteinosis can be similar to those of porphyria
      • Both show hyaline deposits involving blood vessels
      • Lesions in porphyria are limited to sun exposed areas, which is not true of lipoid proteinosis
    • Smokeless tobacco keratosis (Arch Pathol Lab Med 2004;128:e17)
      • Similar deposits of amorphous, eosinophilic PAS and PASD+ material in the lamina propria around vessels and salivary glands
      • More focal than lipoid proteinosis and limited to oral cavity
      • Lesions of smokeless tobacco keratosis are reversible when use of smokeless tobacco products is discontinued
    Additional references
    Board review style question #1
    The deposits seen in lipoid proteinosis are:

    1. Known to be made of amyloid AA
    2. Negative for PASD
    3. Positive for Congo red
    4. Positive for PAS
    Board review style answer #1
    D. The deposits in lipoid proteinosis are positive for PAS, positive for PASD, negative for Congo red and are of unknown composition (but are not amyloid)

    Comment Here

    Reference: Lipoid proteinosis
    Board review style question #2
    The histologic appearance of lipoid proteinosis is best described as:

    1. An infiltrate of histiocytes and foreign body giant cells engulfing elastotic fibers
    2. Dense, band-like lymphocytic infiltrate in dermis that obscures the dermoepidermal junction
    3. Hyaline deposits in the dermis, particularly around blood vessels and sweat glands
    4. Marked fatty infiltration of the dermis
    5. Orthokeratotic hyperkeratosis and papillomatosis of stratum spinosum
    Board review style answer #2
    C. Hyaline deposits in the dermis, particularly around blood vessels and sweat glands

    Comment Here

    Reference: Lipoid proteinosis

    Livedoid vasculopathy
    Definition / general
    • Manifestation of a thrombogenic vasculopathy in which occlusion of small dermal vessels by fibrin thrombi is the primary event
    Terminology
    • Also called atrophie blanche of Milian
    Clinical features
    • Affects lower legs, particularly ankles and dorsum of feet, rarely the extensor arm surfaces below the elbows
    • Elevated levels of fibrinopeptide A are suggestive of a thrombogenic state
    • Several cases with lupus-type anticoagulant and increased level of anticardiolipin antibodies have been reported
    • Other associations include protein C deficiency, factor V Leiden mutation, prothrombin mutation, plasminogen activator inhibitor-1 promoter mutation (OMIM 173360), antithrombin III deficiency, methylenetetrahydrofolate reductase mutation (OMIM 607093), homocysteinemia and essential cryoglobulinemia, all of which contribute to a hypercoagulable condition

    • Description: painful purpuric ulcers with reticular patterning on the lower extremities
    Treatment
    • Antiplatelet drugs and antithrombotic drugs may be useful
    • Also dapsone, nicotinic acid, intravenous immunoglobulin, PUVA therapy, danazol and hyperbaric oxygen therapy
    • Warfarin therapy is sometimes used to treat the associated / concurrent abnormalities in one or more clotting factors
    • Also cessation of smoking, low dose aspirin, oral pentoxifylline and oral dipyrimadole
    • Folic acid is recommended for those with an MTHFR mutation, hydrochloroquine for patients with antiphospholipid antibodies and anabolic steroids (stanozolol or danazol) for patients with cryofibrinogenemia
    Microscopic (histologic) description
    • Dermal capillaries have focal endothelial proliferation and wall thickening by PAS+ material, with fibrin thrombi causing occlusion
    • Specific changes vary by age of lesion
    • Primary event is formation of hyaline thrombi in lumen of small vessels in upper and mid dermis; rarely deeper vessels are involved; PAS+ diastase resistant fibrinoid material also present in walls of blood vessels and in perivascular stroma; also infarction of superficial dermis, often with a small area of ulceration
    • Sometimes a thin parakeratotic layer is present overlying infarcted or atrophic epidermis
    • Epidermis adjacent to ulceration may be spongiotic; a sparse perivascular lymphocytic infiltrate may be present, but there is no vasculitis
    • Neutrophils, if present, are usually sparse and confined to infarcted upper dermis and ulcer base
    • Often extravasated red cells in upper dermis
    • Small blood vessels often increased in adjacent papillary dermis, but this is a common feature in biopsies from lower legs
    • In older lesions, there is thickening and hyalinization of dermal vessels with some endothelial edema and proliferation
    • Fibrinoid material may also be present in vessel walls
    • In even older lesions, there is dermal sclerosis and scarring with some dilated lymphatics and epidermal atrophy; there may be a small amount of hemosiderin in upper dermis
    • As old lesions may become involved again, it is possible to find dermal sclerosis in some early lesions
    Positive stains
    • Immunofluorescence will demonstrate fibrin in vessel walls in early lesions; in later stages there are also immunoglobulins and complement components in broad bands about vessel walls

    Lobomycosis (pending)

    Lupus miliaris disseminatus faciei (acne agminata) (pending)
    [Pending]

    Lupus panniculitis
    Definition / general
    Essential features
    • Tender subcutaneous plaques and nodules
    • Autoimmune lobular lymphocytic panniculitis
    • Patients often do not have or develop systemic lupus
    • Must be distinguished from subcutaneous panniculitis-like T cell lymphoma
    • Presence of high Ki67 or hotspots favors a diagnosis of subcutaneous panniculitis-like T cell lymphoma (SPTCL)
    • Patients with overlapping lupus erythematosus panniculitis and SPTCL features may develop hemophagocytic lymphohistiocytosis (HLH)
    Terminology
    • Lupus panniculitis
    • Lupus erythematosus profundus (when there is concomitant discoid lupus erythematosus)
    ICD coding
    • L93.2
    Epidemiology
    • Women more often than men
    • Usually young to middle aged adults
    • Fewer reported cases in childhood and in patients of advanced age
    • Rare manifestation of autoimmune connective tissue disease, representing less than 3% of patients with cutaneous or systemic lupus erythematosus
    Sites
    • Trunk, breast, proximal extremities (upper more often than lower), face and scalp
    • Distal extremities are less commonly involved
    Clinical features
    • Tender, indurated and erythematous plaques and nodules
    • Overlying skin occasionally shows features of discoid lupus (scaling, plugged follicles, scarring and atrophy)
    • Lesions may spontaneously regress and recur
    • Occasional severe lipoatrophy with resolution of lesions
    • Patients often do not have a history of cutaneous or systemic lupus
    • Some patients develop mild systemic symptoms but do not meet full criteria for a diagnosis of systemic lupus
    Laboratory
    • Antinuclear antibodies are identified in a subset of patients with LEP
    • The absence of antinuclear antibodies does not exclude LEP
    Case reports
    Treatment
    • Antimalarial agents
    • Topical corticosteroids for concomitant discoid lupus
    • Systemic corticosteroids and other immunosuppressants are sometimes used temporarily at disease onset
    Microscopic (histologic) description
    • Lobular or mixed lobular and septal panniculitis
    • Hyaline lipomembranous changes punctuated by foci of active lobular panniculitis
    • Infiltrate consists of lymphocytes, histiocytes and plasma cells
    • Germinal center lymphoid follicles present throughout the lesion
    • Plasma cells may be prominent and form large clusters
    • Lymphocytes are small, bland and splay apart adipocyte lobules
    • Lymphocytic vasculitis, granulomas and dermal mucin are variable
    • Features of discoid lupus are present in a subset of patients (this is termed lupus erythematosus profundus)
    Microscopic (histologic) images

    Contributed by Robert E. LeBlanc M.D.

    H&E (10x, 20x, 40x)

    CD4, CD8, Ki67

    Positive stains
    • CD123 highlights clusters of plasmacytoid dendritic cells
    • CD4 highlights a majority of the lymphocytes in the fat
    Negative stains
    • Ki67 is generally very low with no foci of increased staining
    • CD8 should not highlight a preponderance of the lymphocytes
    Differential diagnosis
    • Injection site reaction or drug reaction: this distinction relies heavily on clinical correlation
    • Other connective tissue disease associated panniculitidies: this distinction relies heavily on clinical correlation
    • Subcutaneous panniculitis-like T cell lymphoma:
      • Ki67 is either expressed diffusely throughout the lesion or in hotspots enriched with adipotropic cytotoxic CD8+ T cells
      • The lymphoma cells are slightly enlarged and hyperchromatic with irregular nuclear contours
      • They surround individual fat lobules but also appear within the disrupted cell membranes of involved adipocytes
      • Paired T cell gene rearrangement studies may be helpful to assess clonality
    Board review style question #1
    Subcutaneous panniculitis-like T cell lymphoma is characterized by which of the following immunophenotypes?

    1. CD4+ T cells, high Ki67 expression
    2. CD4+ T cells, low Ki67 expression
    3. CD8+ T cells, high Ki67 expression
    4. CD8+ T cells, low Ki67 expression
    Board review style answer #1
    C. SPTCL has a cytotoxic CD8 possitive immunophenotype and high expression of Ki67 that can often permit its distinction from lupus erythematosus panniculitis.

    Comment Here

    Reference: Lupus panniculitis
    Board review style question #2
    Which of the following microscopic findings would favor the diagnosis of lupus erythematosus profundus over lupus erythematosus panniculitis?

    1. Epidermal vacuolar interface changes
    2. Hyaline lipomembranous changes
    3. Lobular lymphohistiocytic panniculitis
    4. Lymphocytic vasculitis
    Board review style answer #2
    A. Lupus erythematosus profundus is the preferred terminology for lesions of lupus erythematosus panniculitis that have overlying changes of discoid lupus, namely vacuolar interface dermatitis, basement membrane thickening, scale, follicular plugging and both perifollicular and periadnexal inflammation. The other answer options can be present in both lupus erythematosus profundus and lupus erythematosus panniculitis.

    Comment Here

    Reference: Lupus panniculitis

    Maduramycosis
    Definition / general
    • Maduromycosis usually involves feet, and appears as a nodule or abscess, which progresses over months to years to a chronic infection with granulomatous nodules drained by skin sinuses, leading to deformities involving the bones
    • May be due to bacteria (70%) or fungi (30%)
    • Fungi include Madurella mycetomi; bacteria include Nocardia brasiliensis
    • First described in Madurai, India
    • Mycetoma caused by microaerophilic actinomycetes is termed actinomycetoma; mycetoma caused by true fungi is called eumycetoma (eMedicine)

    • Note: actinomycosis affects the cervical-facial, thoracic and pelvic areas; these bacteria are smaller than in maduromycosis
    Case reports
    Clinical images

    Images hosted on other servers:

    Madura foot

    Microscopic (histologic) images

    Contributed by Venna Maheshwar, M.D., Kiran Alam, M.D. and Anshu Jain, M.D.



    Images hosted on other servers:

    Various images


    Malakoplakia
    Definition / general
    • Greek derivation: Malako-soft and Plako-plaque
    • Chronic granulomatous inflammation
    • Most commonly occurs in the genitourinary tract
    • Rarely occurs in skin (< 100 cases reported, Arch Pathol Lab Med 2008;132:113)
    • Co-involvement of the gastrointestinal tract and other visceral organs also rarely occurs
    Terminology
    Etiology
    • Likely due to inadequate killing of bacteria by macrophages or monocytes that exhibit defective phagolysosomal activity (eMedicine - Malakoplakia)
    • Partially digested bacteria accumulate in monocytes or macrophages, leading to deposition of calcium and iron on residual bacterial glycolipid
    • Basophilic inclusion structure that contains calcium is called Michaelis-Gutmann body, and is considered pathognomonic
    • Patients typically suffer from immunosuppression (HIV, cancer, lymphoma, post-transplant) or autoimmune diseases (SLE, rheumatoid arthritis)
    • Most cases are associated with E.coli
    Clinical features
    • Most often affects the urinary tract, but may affect GI tract, lymph nodes, brain, bone, adrenals and skin
    • May present with papules, plaques, polyps, ulcers and sinuses
    • Skin lesions are non progressive but persistent firm nodules up to 2 cm in diameter
    • Skin colored, yellow or pink
    • May contain a central dimple or draining sinus
    Diagnosis
    • Histologic diagnosis required
    Case reports
    Treatment
    • Antibiotics that concentrate in macrophages (quinolones, trimethoprim-sulfamethoxazole) are associated with a high cure rate
    • Antibiotic therapy against E. coli with surgery is most effective
    Clinical images

    Images hosted on other servers:
    Missing Image

    Purple plaque

    Missing Image

    After treatment

    Microscopic (histologic) description
    • Confluent sheets of histiocytes (von Hanseman cells) with eosinophilic granular cytoplasm and eccentric nuclei
    • Cells contain characteristic basophilic bodies containing calcium (von Kossa+)
    • Round, sometimes laminated structures are known as Michaelis - Gutmann bodies
    • The targetoid pattern is accentuated by staining with PAS
    • Also histiocytes, neutrophils, plasma cells, lymphocytes and granulation tissue
    Microscopic (histologic) images

    Images hosted on other servers:
    Missing Image Missing Image

    Michaelis-Gutmann body

    Missing Image

    Calcified structures

    Missing Image

    Periodic acid Schiff stain

    Missing Image

    PAS, Von Kossa, Prussian blue

    Positive stains
    Electron microscopy description
    • Histiocytes containing phagolysosomes with intact or partially digested bacteria
    • The granules within the macrophages, which stain with PAS, contain engorged lysosomes that contain bacteria debris

    Massive localized lymphedema
    Definition / general
    • Huge masses in thigh, popliteal fossa, scrotum, suprapubic region, inguinal region and abdomen of morbidly obese patients, lasting 1 - 8 years
    • Clinically resembles lipoma or recurrent cellulitis, but also suspicious for malignancy
    • May be due to obstruction of efferent lymphatics by prior surgery, massive abdominal pannus or hypothyroidism
    Clinical features
    • Nonencapsulated, poorly defined, mean 51 cm (range 38 - 75 cm), mean weight 6.8 kg (range 2 - 12 kg)
    • Overlying skin has marbled appearance resembling chronic lymphedema with induration and peau díorange
    Microscopic (histologic) description
    • Fibrous bands intersect lobules of adipose tissue
    • Lymphatic vascular ectasia, chronic inflammatory infiltrate, edema between collagen fibers, infarction and fat necrosis
    • No atypia
    Microscopic (histologic) images

    Contributed by Mark R. Wick, M.D.

    Differential diagnosis

    Mechanical blister
    Definition / general
    • Essentially identical to friction blister with the extension of cytolysis through the basal layer resulting in a hemorrhagic blister
    • Intraepidermal cleavage due to cytolysis and necrosis of keratinocytes in the upper stratum malpighii
    Essential features
    • Mechanical blister associated with manual labor
    • Cytolysis extends through the basal layer causing hemorrhage
    • Mostly clinical diagnosis
    • Symptomatic treatment
    Terminology
    • Friction blister: blister caused by shearing force, which subsequently fills with clear fluid
    • Blood blister: blister caused by excessive force, which fills with blood
    • Hemorrhagic bullae: larger blisters (> 0.5 cm), often multiple and associated with underlying disease process and more often deeper (suprabasilar or subepidermal)
    ICD coding
    • S90.82 - blister (nonthermal) of foot
    Epidemiology
    • Common in athletes, dancers or those wearing poorly fitting shoes
    • Associated with manual labor
    Sites
    • Hands and feet; sites subjected to repetitive friction
    Pathophysiology
    • Shearing forces within the epidermis cause friction blisters in the areas where epidermis is thick and firmly attached to the underlying tissue
    Etiology
    • Prolonged walking or repetitive actions
    Diagrams / tables

    Images hosted on other servers:

    Friction blister

    Clinical features
    • Hemorrhagic blister usually occurring on exposed skin
    • Most commonly occurring on hands and feet
    Diagnosis
    • Typically, a clinical diagnosis
    • If solitary, diagnosis is usually a simple blood blister, rarely biopsied
    • Multiple blood blisters of unknown origin may indicate an underlying process (see "differential diagnosis" below)
    Treatment
    • Symptomatic treatment
    Clinical images

    Images hosted on other servers:

    Blood blisters

    Gross description
    • Cutaneous vesicle or bulla filled with sanguinous fluid
    Microscopic (histologic) description
    • Intraepidermal split
    • The roof of the blister is composed of the stratum corneum, variable stratum granulosum and amorphous cellular debris
    • Most of the degenerated keratinocytes are pale and are located at the floor of the cleft
    • The deeper part of the epidermis consists of undamaged cells
    • In severe cases, the cytolysis may extend through the basal layer leading to hemorrhagic blisters
    Microscopic (histologic) images

    Contributed by Sara C. Shalin, M.D., Ph.D.


    Images hosted on other servers:

    Friction blister

    Electron microscopy description
    • Clumped tonofilaments, intracellular edema, small vacuoles at the cell periphery, areas devoid of organelles
    Differential diagnosis


    Melasma
    Definition / general
    • Acquired large areas of darkened skin, usually due to hormonal changes (birth control pills, pregnancy) and usually on both sides of face (eMedicine)
    • Clinical patterns are centrofacial, malar and mandibular
    • Classification (based on level of increased melanin in skin determined by Wood's light examination, J Am Acad Dermatol 1981;4:698):
      • Epidermal: 70% of cases, increased melanin in basal and suprabasal epidermis; skin pigmentation enhanced under Wood's light; responds best to bleaching agents
      • Dermal: 10% of cases, increase in melanophages in upper dermis; no enhancement of skin pigmentation under Wood's light, responds poorly to bleaching agents
      • Mixed: 20% of cases, mixture of epidermal and dermal features; patchy enhancement of skin pigmentation under Wood's light
      • Indeterminate: 2% of cases; not possible to characterize pigmentation pattern
    Terminology
    • Also called chloasma
    Clinical features
    • 90% women; more common in dark skinned individuals
    • In pregnant women, hormones cause "mask of pregnancy" on face and darkened skin on abdomen and elsewhere
    • Associated with freckles, lentigines and 4+ nevi (BMC Dermatol 2008;8:3)
    • Some cases associated with phenytoin, autoimmune thyroid disease, UV light in men (J Am Acad Dermatol 2012;66:642)
    • May be due to increase in stem cell factor and c-kit expression (Br J Dermatol 2006;154:1094)
    Treatment
    Clinical images

    Images hosted on other servers:

    Confluent hyperpigmented macules in a malar distribution

    Microscopic (histologic) description
    • Mild lymphohistiocytic infiltrate in 75%
    • Increase in epidermal melanin but no increase in number of melanocytes
    • Melanocytes are larger with prominent dendrites (Am J Dermatopathol 2005;27:96)
    Electron microscopy description
    • More melanosomes in keratinocytes, melanocytes and dendrites
    Differential diagnosis

    Miliaria
    Definition / general
    • Obstruction of sweat gland ducts during high heat and humidity
    Epidemiology
    • Most often seen in children but may affect any age
    Clinical features
    • Miliaria crystallina: obstruction is within stratum corneum, resulting in small clear vesicles located on trunk with high fever and pronounced sweating
    • Miliaria profunda: seen in tropical climates, obstruction is at sweat duct, resulting in small papules on trunk and occasionally on extremities
    • Miliaria pustulosa: miliaria rubra with pustules
    • Miliaria rubra (prickly heat): obstruction within prickle cell layer, resulting in extremely pruritic erythematosus papules and vesicles usually located about the trunk and intertrigenous regions; common in infants
    Clinical images

    Images hosted on other servers:

    Miliaria profunda

    Microscopic (histologic) description
    • Miliaria crystallina: subcorneal vesicle with few neutrophils; obstruction is within stratum corneu
    • Miliaria rubra: intraepidermal spongiotic vesicle
    • In both variants, lesion is centered around an intraepidermal eccrine sweat duct
    • Miliaria profunda: spongiosis of dermal portion of eccrine sweat duct; often with adjacent chronic inflammation
    • Miliaria pustulosa: features of miliaria plus intraepidermal or subcorneal pustules
    Additional references

    Molluscum contagiosum
    Definition / general
    Essential features
    • Infectious dermatosis most commonly affecting children and immunocompromised patients
    • Majority of infections are caused by MCV1 (98% of cases)
    • In contrast to most double stranded DNA viruses, poxvirus replicates in the cytoplasm
    • Henderson-Paterson bodies, also known as molluscum bodies, are large (up to 35 microns) intracytoplasmic eosinophilic inclusion on keratinocytes
    ICD coding
    • ICD-10: B08.1 - Molluscum contagiosum
    Epidemiology
    Sites
    Pathophysiology
    Etiology
    Clinical features
    Diagnosis
    Prognostic factors
    Case reports
    Treatment
    • Self limited
    • Mechanical methods: cryotherapy, curettage, pulse dye laser
    • Chemical methods: potassium hydroxide, podophyllotoxin, trichloroacetic acid, salicylic acid, lactic acid, glycolic acid, benzoyl peroxide and tretinoin
    • Immunomodulatory methods: imiquimod, oral cimetidine, interferon alfa, candidin and diphencyprone
    • Antivirals: cidofovir (Clin Cosmet Investig Dermatol 2019;12:373)
    Clinical images

    Contributed by Marely Santiago-Vazquez, M.D.

    Dermoscopy features

    Molluscum contagiosum morphology

    Clustered papules

    Microscopic (histologic) description
    Microscopic (histologic) images

    Contributed by Jorge L. Sánchez, M.D. and @zaalruwai83 on Twitter

    Epidermal hyperplasia

    Henderson-Patterson bodies

    Eosinophilic inclusion bodies

    Inflammatory infiltrate


    Large intracytoplasmic inclusions

    Molluscum contagiosum Molluscum contagiosum Molluscum contagiosum

    Molluscum contagiosum

    Virtual slides

    Images hosted on other servers:

    Molluscum contagiosum

    Videos

    Molluscum versus myrmecia wart - dermpath lookalikes

    Sample pathology report
    • Left superior eyelid skin, biopsy:
      • Molluscum contagiosum (see comment)
      • Comment: There is epidermal hyperplasia with large eosinophilic cytoplasmic bodies within keratinocytes.
    Differential diagnosis
    • Common wart:
      • Verrucous or filliform flesh colored papules
    • Herpes simplex:
      • Grouped vesicles that rupture quickly, leading to painful ulcerations with scalloped borders
      • Nuclear viral inclusions (multinucleation, nuclear molding, margination of chromatin)
    • Juvenile xanthogranuloma:
      • Dome shaped, yellowish orange papules or nodules
      • Triple association between juvenile xanthogranuloma, neurofibromatosis type 1 and juvenile promyelomonocytic leukemia has been reported
      • Touton giant cells, histiocytes, lymphocytes and eosinophils in reticular and papillary dermis
    • Spitz nevi:
      • Lesions typically located in face and extremities
      • Melanocytic nests with epithelioid cells at the dermal-epidermal junction
      • Kamino bodies within the epidermis
    • Basal cell carcinoma:
      • Most common in chronically sun exposed areas
      • Nest of basaloid cells with peripheral palisading and retraction artifact
    Board review style question #1

    A 6 year old boy presents with multiple scattered, flesh colored, small skin papules on the trunk and upper extremities. His medical history was unremarkable and his older sister had similar lesions in the past. A skin biopsy of one of the lesions is shown. Which of the following is the most likely cause of his condition?

    1. Human papillomavirus type 1 (HPV1)
    2. Human papillomavirus type 2 (HPV2)
    3. Human papillomavirus type 3 (HPV3)
    4. Molluscum contagiosum virus type 1 (MCV1)
    5. Molluscum contagiosum virus type 2 (MCV2)
    Board review style answer #1
    D. Molluscum contagiosum virus type 1 (MCV1). The histology shows molluscum contagiosum, which is caused by the molluscum contagiosum virus (MCV) of the Poxviridae family (double stranded DNA). There are several types of MCV but the most common by far is MCV1 (98% of cases). MCV1 most commonly affects children, whereas MCV2 is usually seen in patients with HIV. Although verruca vulgaris from HPV (especially type 1) is common in children, the specimen is characteristic for molluscum contagiosum as it shows epidermal hyperplasia with large eosinophilic inclusion bodies on the keratinocytes (Henderson-Patterson bodies).

    Comment Here

    Reference: Molluscum contagiosum
    Board review style question #2
    A patient is diagnosed with molluscum contagiosum after a skin biopsy was performed by his dermatologist. Which of the following type of viruses is responsible for causing his lesions?

    1. Double stranded DNA virus replicating in the cytoplasm
    2. Double stranded RNA virus replicating in the nucleus
    3. Single stranded DNA virus replicating in the cytoplasm
    4. Single stranded DNA virus replicating in the nucleus
    5. Single stranded RNA virus replicating in the cytoplasm
    Board review style answer #2
    A. Double stranded DNA virus replicating in the cytoplasm. Molluscum contagiosum is part of the Poxviridae family. The poxvirus replication is unusual as it is a double stranded DNA virus replicating in the cytoplasm instead of the nucleus.

    Comment Here

    Reference: Molluscum contagiosum

    Morphea and systemic sclerosis
    Definition / general
    Essential features
    Terminology
    • Systemic sclerosis
    • Localized scleroderma / morphea
    ICD coding
    • ICD-10:
      • L94.0 - localized scleroderma (morphea)
      • M34 - systemic sclerosis (scleroderma)
      • M34.0 - progressive systemic sclerosis
      • M34.1 - CREST syndrome
      • M34.2 - systemic sclerosis induced by drug and chemical
      • M34.8 - other forms of systemic sclerosis
      • M34.81 - systemic sclerosis with lung involvement
      • M34.82 - systemic sclerosis with myopathy
      • M34.83 - systemic sclerosis with polyneuropathy
      • M34.89 - other systemic sclerosis
      • M34.9 - systemic sclerosis, unspecified
    • ICD-11:
      • 4A42.Z - systemic sclerosis, unspecified
      • EB61 - morphoea
    Epidemiology
    Sites
    • Localized scleroderma
      • Skin, subcutaneous tissue, underlying muscle and bone
      • When present on the face, head and underlying central nervous system may rarely be involved (J Am Acad Dermatol 2011;64:217)
    • Systemic sclerosis
      • Skin and internal organs (heterogeneous presentation) (J Eur Acad Dermatol Venereol 2017;31:1401)
        • Limited cutaneous systemic sclerosis
          • Face and distal to elbows and knees
          • Internal organ involvement in later stages
        • Diffuse cutaneous systemic sclerosis
          • Face, trunk, proximal and distal extremities
    Pathophysiology
    Etiology
    Diagrams / tables

    Images hosted on other servers:
    Localized scleroderma

    Localized scleroderma

    Clinical features
    • Localized scleroderma: see Diagrams / tables (J Eur Acad Dermatol Venereol 2017;31:1401)
    • Systemic sclerosis
      • Skin (J Eur Acad Dermatol Venereol 2017;31:1401)
        • Raynaud phenomenon: > 90% of patients
          • Typically affects hands, less commonly feet; it may also involve ears, nose and tongue
        • Onset: edema of fingers and hands
        • Sclerotic changes cause dermatogenic contractures, sclerodactyly, perioral plications, microstomia and mask-like facial stiffness
        • With or without hair loss, decreased sweating, hyperpigmentation / hypopigmentation / depigmentation and pruritus
        • Digital ulcers
        • Calcinosis cutis
        • Late stage: diminished skin fibrosis of trunk and proximal extremities
      • Musculoskeletal (J Eur Acad Dermatol Venereol 2017;31:1401)
        • Arthralgia
        • Musculoskeletal pain
        • Muscle weakness
        • Overlap syndromes (myositis and rheumatoid arthritis)
      • Pulmonary (J Eur Acad Dermatol Venereol 2017;31:1401)
        • Interstitial lung disease (65%)
        • Pulmonary arterial hypertension (15%)
      • Gastrointestinal manifestations
        • Involvement of esophagus (80%), stomach (40 - 70%) and small and large intestine
        • Upper gastrointestinal involvement in nearly all patients in longstanding disease
      • Cardiovascular (J Eur Acad Dermatol Venereol 2017;31:1401)
        • Myocardial fibrosis, manifestations depend on severity and extension
      • Renal (J Eur Acad Dermatol Venereol 2017;31:1401)
        • Slowly progressive obliterative vasculopathy
        • Scleroderma renal crisis (malignant hypertension and oligo / anuric acute renal failure) - rare but severe
    Diagnosis
    • Localized scleroderma
      • Routine systemic workup is not recommended
      • Clinical presentation
      • Histopathology - not possible to differentiate between localized scleroderma and systemic sclerosis nor to differentiate among different localized scleroderma subtypes
        • Biopsy should be sufficiently deep
        • Only if the clinical presentation is inconclusive
      • Laboratory
        • Blood differential and chemistry
        • Creatine kinase (suspected myositis)
        • Autoantibodies may be present but their routine screening is not recommended
      • Imaging
        • Brain MRI and head CT (i.e. en coup de sabre or progressive facial hemiatrophy):
          • Detect muscle, joint or bone involvement
          • Detect potential subcortical calcifications or brain atrophy
          • Surgical planning
    • Systemic sclerosis
      • Clinical presentation
        • Physical examination
        • Capillaroscopy
        • Modified Rodnan skin score (mRSS)
      • Histopathology - not possible to differentiate between localized scleroderma and systemic sclerosis nor to differentiate among different localized scleroderma subtypes
        • Biopsy should be sufficiently deep
      • Laboratory
        • Blood differential and chemistry
          • Creatine kinase (suspected myositis)
      • Autoantibodies
        • Antinuclear antibody (ANA) (95%)
        • Systemic sclerosis associated antibodies (Lancet 2017;390:1685, J Eur Acad Dermatol Venereol 2017;31:1401)
          • Centromere
          • Topoisomerase 1
          • RNA polymerase III
          • Exosomal ribonucleoprotein (PM / Scl)
          • Uridine rich small nuclear ribonucleoprotein (U1-snRNP)
        • Systemic sclerosis overlap syndromes (features of other connective tissue disorders combined with systemic sclerosis)
          • Anti Ro / La, anti centromere - Sjögren overlap syndrome
          • Cyclic citrullinated peptide (CCP) - rheumatoid arthritis overlap syndrome
          • Rheumatoid factor (Fc portion of IgG) - rheumatoid arthritis overlap syndrome
          • Mitochondrial Ag M2 - primary biliary cirrhosis
        • Organ oriented diagnostic workup
          • Laboratory
          • Imaging
    Radiology images

    Images hosted on other servers:
    13 year old boy with juvenile systemic sclerosis

    13 year old boy with juvenile systemic sclerosis

    Calcifications

    Calcifications

    Calcinosis cutis in a 15 year old boy

    Calcinosis cutis in a 15 year old boy

    Whole body MRI in a juvenile with scleromyositis

    Whole body MRI in a juvenile with scleromyositis

    Prognostic factors
    • Localized scleroderma
      • 25% of patients experience reactivation of disease after resolution (Br J Dermatol 2015;172:722)
        • Factors associated with reactivation:
        • Skin and soft tissue atrophy, contractures (may affect functionality)
    • Systemic sclerosis
      • Chronic and unpredictable course
      • Factors related to poor outcome (Semin Arthritis Rheum 2014;44:208):
        • Male gender
        • Older age
        • Lung, kidney and heart involvement
        • Skin extension
        • High erythrocyte sedimentation rate
      • Quality of life and psychosocial well being are often affected
      • Multidisciplinary care is required due to involvement of several organs
      • Leading cause of mortality in last 2 decades is cardiopulmonary involvement (73% of all systemic sclerosis related deaths) (Semin Arthritis Rheum 2014;44:208)
    Case reports
    • 22 year old woman, gravida 1 para 0, at 7 months gestation complaining of skin thickening on the breasts (Dermatol Online J 2017;23:13030)
    • 28 year old woman with myasthenia gravis who developed linear scleroderma 7 years after diagnosis (J Dermatol 2000;27:31)
    • 44 year old woman status post lumpectomy and chemoradiation with skin erythema, sclerosis and contracture of her right breast 15 months after completion of treatment (Histopathology 2018;72:342)
    Treatment
    • Localized scleroderma
      • Topical therapy
        • Corticosteroids
        • Calcipotriol
        • Tacrolimus
      • Systemic therapy
        • Systemic corticosteroids
        • Methotrexate
        • Mycophenolate mofetil
        • Others systemic medications
      • Phototherapy
      • Physical therapy
      • Surgical interventions
    • Systemic sclerosis
      • Organ based approach (determine organ involvement and disease activity)
    • Reference: J Eur Acad Dermatol Venereol 2017;31:1401
    Clinical images

    Contributed by Cristian Rocha, M.D.
    En coup de sabre

    En coup de sabre

    Generalized localized scleroderma / morphea

    Generalized localized scleroderma / morphea

    Microscopic (histologic) description
    • Localized scleroderma and systemic sclerosis have mostly indistinct histologic features (J Eur Acad Dermatol Venereol 2017;31:1401)
      • Some authors have suggested histopathological differences (Am J Dermatopathol 1998;20:242):
        • Density and distribution of inflammatory infiltrate (systemic sclerosis may be less inflammatory on biopsy)
        • Thickness of collagenized tissue, found to be mostly lower reticular dermis and subcutis in systemic sclerosis compared to involvement of all levels of dermis in localized scleroderma
    • Biopsy is performed when clinical features are inconclusive
    • Spectrum of changes over time leading to thickened, collagenized dermis
      • Preservation of elastic fibers by elastic van Gieson stain with variable changes:
        • Areas with thickened collagen show wider, compressed and parallel straightened fibers (rather than the wavy, branching distribution of normal skin) (J Cutan Pathol 2009;36:952)
    • Punch biopsy shows rigid, squared silhouette at low power
    • Early disease (J Eur Acad Dermatol Venereol 2017;31:1401)
      • Thickened collagen bundles within papillary and reticular dermis
      • Dense perivascular and periadnexal inflammatory infiltrate, as well as between collagen bundles and at the limit of the dermis and subcutis
      • Inflammatory infiltrate: predominantly lymphocytes; plasma cells, histiocytes and eosinophils are also seen
      • Overlying epidermis may be unaffected or atrophic
      • Blood vessel wall edema may be seen
    • Late / fibrotic stage (J Eur Acad Dermatol Venereol 2017;31:1401)
      • Relatively avascular skin
      • Adnexal structures are atrophic or absent
        • Entrapped atrophic adnexal structures may show loss of surrounding adipose tissue
      • Minimal inflammation
      • Highly packed, eosinophilic collagen bundles
      • Blood vessel wall sclerosis
    • Can affect subcutaneous fat
      • Morphea profunda, morphea of childhood, generalized morphea, linear morphea
      • Panniculitis is predominantly septal, resulting in widening and thickening
    • Some subtypes are restricted to the superficial dermis
      • Superficial morphea
    • Rare variants
      • Bullous morphea:
        • Bullae arise on indurated plaques (subepidermal)
        • Marked papillary dermal edema
        • Ectatic lymphatic vessels
      • Keloidal / nodular morphea:
        • Keloidal lesions in association with morphea plaques
      • Guttate morphea:
        • Localized and superficial
        • Usually associated with larger plaques
    Microscopic (histologic) images

    Contributed by Jose A. Plaza, M.D., Mark R. Wick, M.D. and Joel Pinczewski, M.D., Ph.D.
    Early scleroderma lesion Early scleroderma lesion Early scleroderma lesion

    Early scleroderma lesion

    Breast skin, morphea

    Late stage morphea lesion

    Sample pathology report
    • Skin, trunk, biopsy:
      • Scleroderma (see comment)
      • Comment: Sections show a punch biopsy with squared silhouette and thickened dermal collagen. Adnexal structures are atrophic and entrapped within the collagenous stroma. There is dense perivascular and periadnexal lymphoplasmacytic infiltrate, as well as between collagen bundles and at the dermal subcutis junction. Altogether, these findings are consistent with scleroderma. Clinical correlation is advised for subclassification.
    Differential diagnosis
    • Scleredema:
      • Hypertrophied collagen bundles with intervening mucin
      • No inflammatory infiltrate
      • Edematous lesions may be difficult to distinguish
    • Chronic radiation dermatitis:
      • Variable epidermal atrophy, acanthosis, hyperkeratosis
      • Radiation induced changes (atypia, elastosis and superficial capillary ectasia)
      • Loss of adnexal structures
    • Lichen sclerosus et atrophicus:
      • Sclerosis limited to papillary dermis
      • Vacuolar change in basal cells
      • Edema of superficial dermis
      • Band-like or lichenoid inflammatory cell infiltrate in active lesions
    • Keloid:
      • Raised lesion
      • Dense and thickened dermal collagen bundles
      • Markedly eosinophilic, hyalinized (bubble gum) collagen bundles
      • Decreased dermal vascularity, with occasional ectatic vessels
      • Keloidal / nodular morphea shows associated morphea plaques
    • Hypertrophic scar:
      • Raised lesion
      • Dense and thickened dermal collagen bundles
      • Lacks hyalinized collagen bundles
      • Perpendicularly oriented vessels
      • Active lesions show a cellular appearing dermis with numerous myofibroblasts arranged in fascicles and nodules
    • Scleromyxedema:
      • Mucin deposition in upper and mid dermis
      • Increased disorganized fibroblastic proliferation and collagen deposition
    • Nephrogenic systemic fibrosis:
      • Clinical history remarkable for renal insufficiency and gadolinium exposure
      • Disorganized collagen bundles in deep dermis and adipose septae
      • Spindle cells in between collagen bundles
      • Increased mucin deposition
      • Osseous sclerotic bodies and multinucleated giant cells may be seen
    Board review style question #1

    What histopathologic characteristic helps to differentiate between localized and systemic sclerosis?

    1. Edema of superficial dermis
    2. Fibrosis limited to papillary dermis
    3. Histological features are indistinct
    4. Perivascular inflammation
    Board review style answer #1
    C. Histological features are mostly indistinct. Localized scleroderma and systemic sclerosis have indistinct histologic findings and require clinical correlation for their classification and subclassification.

    Comment Here

    Reference: Scleroderma
    Board review style question #2
    Which of the following histopathological findings are most consistent with a diagnosis of morphea?

    1. Edema and hypertrophied collagen bundles with intervening mucin
    2. Sclerosis limited to papillary dermis, vacuolar change in basal cells and edema of superficial dermis
    3. Thick and dense collagenized dermis, entrapped atrophic adnexal structures with loss of surrounding adipose tissue
    4. Variable epidermal atrophy, acanthosis, atypia and loss of adnexal structures
    Board review style answer #2
    C. Thick and dense collagenized dermis, entrapped atrophic adnexal structures with loss of surrounding adipose tissue. Some of the most characteristic findings in localized scleroderma and systemic sclerosis include a spectrum of changes over time leading to a thickened, collagenized dermis. The early phases of disease may present with perivascular and periadnexal inflammatory infiltrate as well as between collagen bundles and at the limit of the dermis and subcutis. The late / fibrotic stages are usually less inflammatory. The adnexal structures are atrophic or absent and may show loss of surrounding adipose tissue.

    Comment Here

    Reference: Scleroderma

    Mycetoma (pending)

    Myiasis
    Definition / general
    • Infestation by larvae of the human botfly (Dermatobia hominis) and tumbu fly (Cordylobia anthropophaga) are common causes of myiasis
    • Skin lesions are due to growth and burrowing of the larva, which feed on the host tissues, liquid body substance or ingested food
    • Other cutaneous locations such as eye, mucosa, breast and gastrointestinal tract can be affected (West Afr J Med 2013;32:149)
    Terminology
    • Human botfly, Dermatobia hominis, myiasis
    Epidemiology
    • Tropical and subtropical areas with warm and humid climate, such as the Americas, Central and South Africa are most common (CDC: Parasites - Myiasis [Accessed 10 October 2018])
    • Rarely seen in the continental United States; most commonly seen in travelers
    Sites
    Etiology
    Life cycle:
    • In endemic areas, D. hominis flies typically lay their eggs on mosquitoes, which in turn deposit them on warmblooded mammals such as humans
    • In wound myiasis, an open wound or orifice attracts flies to deposit their eggs
    • The eggs hatch in the skin, stimulated by the heat from the host
    • The larvae mature through 3 stages during which they develop posterior spines, which makes them difficult to dislodge
    • 5 - 10 weeks after deposition, the mature third stage larvae drop from the host and complete the pupal stage in soil
    • Adult botfly emerges from soil after 4 - 11 weeks
    Diagrams / tables

    Images hosted on other servers:

    Life cycle of D. hominis

    Clinical features
    • May clinically resemble insect bites, allergic reactions, herpes virus or molluscum contagiosum
    • Furuncular myiasis: a pruritic papule that develops within 24 hours of penetration, enlarging to 1 - 3 cm in diameter
    • These lesions can be painful or tender and may become crusted and purulent
    • In wound myiasis, the larvae are deposited in a suppurating wound or on decomposing flesh
    • Creeping (or migratory) cutaneous myiasis resembles cutaneous larva migrans, but the larvae migrate more slowly, persist for longer (often months) and are larger than helminth larvae
    • In most cases, myiasis is self limited with minimal morbidity
    • Secondary infection by bacteria may be a complication
    Diagnosis
    • Thorough physical examination and documentation of relevant exposure history, including travel to warm humid climates
    • Surgical extraction and histologic examination can confirm the presence of larvae
    • Supportive travel history
    Case reports
    Treatment
    • The major reasons for treatment are reduction of pain, cosmesis, psychological relief and to prevent secondary infections
    • Surgical extraction of larvae is the key treatment
    • Systemic medication: broad spectrum antibiotics, oral steroid and ivermectin may also be used (Ocul Immunol Inflamm 2011;19:444)
    Clinical images

    Images hosted on other servers:

    Close up, adult and larvae of D. hominis

    Gross description
    Gross images

    Contributed by Bobbi Pritt, M.D.

    Dermatobia hominis, the human botfly

    Microscopic (histologic) description
    • Histologically, a mixed acute and chronic inflammatory infiltrate composed of varying proportions of neutrophils (early), lymphocytes, plasma cells and histiocytes (late), admixed with eosinophils (J Am Acad Dermatol 2004;50:S26)
    • Histologically, larvae are characterized by an undulating chitinous exoskeleton and pigmented spines (setae) protruding from exoskeleton
    Microscopic (histologic) images

    Contributed by Priya Nagarajan, M.D., Ph.D.

    2x magnification

    10x magnification

    Videos

    Differential diagnosis

    Necrobiosis lipoidica
    Definition / general
    • Atrophic, yellow depressed plaques, usually on legs of diabetic patients
    Clinical features
    • Also associated with hypo and hyperthyroidism, inflammatory bowel disease and vasculitis
    • Atrophic, yellow depressed plaques, telangiectasia and active inflammatory edge
    • Chronic lesions may show ulceration and crusting
    • Solitary or multiple, often symmetrical in lower extremities in pretibial area
    • Involvement of penis with a lesion resembling chronic balanitis has been described
    • Rarely, squamous cell carcinoma may arise in chronic lesions
    Clinical images

    Images hosted on other servers:

    Necrobiosis lipoidica diabeticorum

    Microscopic (histologic) description
    • Epidermal changes may be inconspicuous or absent; variable acanthosis, atrophy or hyperkeratosis
    • Palisading, necrobiotic granuloma consist of a large confluent area of necrobiosis centered in the superficial dermis and subcutaneous fat
    • Usually epithelioid histiocytes, giant cells and sometimes a well formed granuloma
    • Eosinophilic, swollen or degenerate collagen appears hyalinised with surrounding infiltrate of lymphocytes and histiocytes
    • Linear infiltrate of histiocytes between collagen bundles; occasionally lipomemebranous fat necrosis
    • Blood vessel wall thickening with intimal proliferation and narrowing of the lumen; also mild to moderate perivascular lymphocytic infiltrate
    • Plasma cells are almost always present
    • Cholesterol clefts are rare
    • Loss of elastic tissue
    Microscopic (histologic) images

    Contributed by Gaurav Jain, M.B.B.S., M.D.
    Degenerated collagen alternating with inflammation

    Degenerated collagen alternating with inflammation

    Layers of degenerated collagen

    Layers of degenerated collagen

    Degenerated collagen

    Degenerated collagen

    Negative stains
    Differential diagnosis
    • Granuloma annulare: mucin+, lysozyme+
    • Necrobiotic xanthogranuloma: head and neck of patients with paraproteinemia, not associated with diabetes

    Necrobiotic xanthogranuloma
    Definition / general
    • Head, neck and trunk lesions similar to necrobiosis lipoidica but associated with IgG paraproteinemia and not diabetes
    Case reports
    Gross description
    • Yellowish papules / nodules
    Microscopic (histologic) description
    • Marked necrobiosis alternating with foci of xanthogranulomatous infiltration through the reticular dermis with extension into subcutaneous fat
    • Septal distribution may mimic panniculitis
    • Granulomatous infiltrate is associated with epithelioid and foamy histiocytes in addition to conspicuous giant cells, many of Touton type; also foreign body giant cells, lymphocytes and plasma cells, cholesterol clefts
    • Asteroid bodies often found in cytoplasm of giant cells
    • Granulomatous and necrobiotic process may also affect muscular arteries
    Microscopic (histologic) images

    Images hosted on other servers:

    Necrobiotic xanthogranuloma

    Various images

    Positive stains
    • Oil Red O may reveal focal lipid droplets
    • Direct immunofluorescence shows IgM, C3 and fibrinogen deposits in blood vessel wall
    • Alcian blue staining may reveal small amount of interstitial mucin
    Negative stains
    • No elastic fibers in necrobiotic areas

    Nephrogenic systemic fibrosis
    Clinical images

    Contributed by Joel Pinczewski, M.D., Ph.D.

    Indurated and hyperpigmented skin lesions

    Microscopic (histologic) description
    • Elastic fibers
      • Known as sclerotic bodies or elastocollagenous balls
      • Originally thought to be pathognomonic of nephrogenic systemic fibrosis, but subsequently described in other lesions in patients exposed to gadolinium
    Microscopic (histologic) images

    Contributed by Joel Pinczewski, M.D., Ph.D.

    Increased dermal mucin and spindled fibroblasts


    Neutrophilic dermatosis associated with gastrointestinal and hepatobiliary disease
    Definition / general
    • Occurs in 14 - 44% of patients with Crohn's disease
    • Includes pyoderma gangrenosum, erythema nodosum and so called metastatic Crohn's disease / nonnecrotizing sarcoid-like granulomatous dermatitis (Hum Pathol 2003;34:1185)
    • Above are all associated with vascular injury, typically with prominent extravascular neutrophilic or histiocytic dermal infiltrate
    • No microbes found in skin lesions
    Treatment
    • Immunosuppressive therapy
    Microscopic (histologic) description
    • Variable dermal edema and neutrophilic dermatosis
    • Perivascular and interstitial neutrophilic infiltrate
    • Variable numbers of lymphocytes and histiocytes
    • Abundant karyorrhexis, resembling Sweet disease
    • Also leukocytoclastic vasculitis and pustular vasculitis in some cases
    • Inflammation often limited to dermis but may involve subcutaneous fat, resulting in erythema nodosum or erythema nodosum like panniculitis
    Differential diagnosis

    Neutrophilic eccrine hidradenitis
    Definition / general
    • A benign, self limiting chemotherapy associated neutrophilic dermatosis characterized by neutrophilic infiltration of eccrine sweat glands (Case Rep Dermatol 2013;5:340)
    Essential features
    Sites
    Pathophysiology
    Etiology
    Clinical features
    Diagnosis
    • Skin biopsy
    Prognostic factors
    Case reports
    Treatment
    Clinical images

    Images hosted on other servers:

    Violaceous plaques on the upper back

    Erythematous plaques on the chest and abdomen

    Solitary papules on the forearm

    Erythematous, papular lesions on bilateral palms

    Microscopic (histologic) description
    Microscopic (histologic) images

    Contributed by Jasmine Saleh, M.D., M.P.H. and Jodi Speiser, M.D.

    Interface dermatitis

    Neutrophils and squamous syringometaplasia

    Sample pathology report
    • Skin, upper back, biopsy:
      • Neutrophilic eccrine hidradenitis
    Differential diagnosis
    • Idiopathic plantar hidradenitis (neutrophilic eccrine hidradenitis in children):
      • Neutrophilic inflammation is centered on the coiled duct and proximal straight duct, with the secretory apparatus usually being spared or only minimally affected
      • Eccrine syringometaplasia is not seen
    • Sweet syndrome:
      • Dense diffuse infiltration of neutrophils in the dermis
      • No eccrine involvement
    • Cellulitis:
      • Diffuse infiltration of neutrophils
      • Positive culture or special stains
      • No eccrine involvement
    • Pyoderma gangrenosum:
      • Nonspecific ulceration with dermal acute and chronic inflammation
      • No eccrine involvement
    Board review style question #1

    Which of the following is true about neutrophilic eccrine hidradenitis (shown in the image)?

    1. Dermal edema, squamous syringometaplasia and necrosis of the eccrine coils and glands may be present
    2. It is highly malignant with poor prognosis
    3. It most frequently occurs in infants and children after viral infection
    4. It presents as a tan-brown pustule
    Board review style answer #1
    A. Dermal edema, squamous syringometaplasia and necrosis of the eccrine coils and glands may be present

    Comment Here

    Reference: Neutrophilic eccrine hidradenitis
    Board review style question #2
    Which of the following can be seen in neutrophilic eccrine hidradenitis?

    1. Dense diffuse infiltration of neutrophils in the dermis
    2. Mucinous degeneration in periadnexal fibroadipose tissue
    3. Ulceration
    4. Vasculitis
    Board review style answer #2
    B. Mucinous degeneration in periadnexal fibroadipose tissue

    Comment Here

    Reference: Neutrophilic eccrine hidradenitis

    Nonscarring alopecia in systemic lupus erythematosus (pending)
    [Pending]

    North American blastomycosis
    Definition / general
    • Skin lesions usually secondary to pulmonary blastomycosis when soil containing mycelia is disturbed and airborne conidia are inhaled (Clin Microbiol Rev 2010;23:367); pulmonary disease may be subclinical
    • Rarely occurs at site of penetrating injuries
    • Due to Blastomyces dermatitidis, a spherical, double-contoured, 12 micron yeast that reproduces by budding
    • Endemic areas are Mississippi, Missouri and Ohio river valleys (USA) and southern Canada
    • Risk factors are immunosuppression, collagen vascular disease, being an outdoor worker, having a coworker with blastomycosis (Can J Infect Dis Med Microbiol 2009;20:117)
    • May affect healthy patients via spore inhalation; usually men > women because they participate more in activities that disturb infected soil
    Clinical features
    • Slowly enlarging verrucous plaques containing multiple small abscesses
    Diagnosis
    Case reports
    Treatment
    • Itraconazole or long term amphotericin
    Clinical images

    Images hosted on other servers:

    Various images

    Microscopic (histologic) description
    • Marked pseudoepitheliomatous hyperplasia of epidermis
    • Granulomatous and neutrophilic infiltrate
    • Fungi are within giant cells
    • Blastomyces dermatitidis is a 12 micron, spherical, double-contoured yeast with broad based buds
    Microscopic (histologic) images

    Images hosted on other servers:

    Pseudoepitheliomatous hyperplasia, epidermal abscesses and prominent dermal infiltrate
    High power shows fungal organisms


    Budding yeast

    Differential diagnosis
    • Alternaria species: may have similar histologic features, but differentiate based on culture (J Cutan Pathol 2011;38:923)
    • Squamous cell carcinoma: may resemble pseudoepitheliomatous hyperplasia of fungal infection, but has marked atypia and invasion
    Additional references

    Nutritional deficiency (pending)
    [Pending]

    Ochronosis
    Definition / general
    • Disorders characterized by blue-gray to black pigmentation due to deposition of ochre colored pigment in connective tissues
    • 2 main forms: alkaptonuria / endogenous ochronosis (homogentisate 1,2 dioxygenase deficiency) and exogenous ochronosis (overuse of topical hydroquinone and other medicaments)
    Essential features
    • Deposition disorder characterized by deposition of homogentisic acid in connective tissue
    • May result from an inborn error of metabolism (alkaptonuria) or from overuse of topical hydroquinone or other phenol containing medications (exogenous ochronosis)
    • Alkaptonuria presents with hyperpigmentation, arthropathies, kidney disease and heart disease
    • Exogenous ochronosis patients primarily present with hyperpigmentation involving the face, neck or upper limbs
    • Classical histologic findings in the skin include formation of banana shaped, yellow-brown deposits and heavy elastosis in the superficial dermis
    ICD coding
    • Alkaptonuria
      • ICD-10: E70.29 - Other disorders of tyrosine metabolism, ochronosis
      • ICD-11: 5C50.10 - Alkaptonuria, ochronosis
    • Exogenous ochronosis
      • ICD-10: L81.9 - Disorder of pigmentation, unspecified
      • ICD-11: ED6Y - Other specified disorders of skin pigmentation, exogenous ochronosis
    Epidemiology
    Sites
    • Alkaptonuria: skin (ear, face, acral surfaces and axillae), sclera, bone cartilage, kidney and cardiac valves
    • Exogenous ochronosis: skin of the malar areas, temples, lower cheeks, neck, forearms and hands
    Pathophysiology
    • Alkaptonuria
      • Inborn error of tyrosine metabolism secondary to autosomal recessive loss of function mutations in HGD / HGO (Nat Genet 1996;14:19)
      • Resultant deficiency of homogentisate 1,2 dioxygenase leads to accumulation of homogentisic acid and eventual deposition in collagen rich tissue (Calcif Tissue Int 2017;101:50)
      • Long term deposition leads to degenerative arthritis, cardiac valvulopathy, renal disease and oculocutaneous pigmentation (N Engl J Med 2002;347:2111)
    • Exogenous ochronosis
    Etiology
    Clinical features
    Diagnosis
    Laboratory
    Radiology description
    • Xray findings in alkaptonuric arthropathy include joint space narrowing, osteophyte formation, joint space calcification and reactive sclerosis of articular surfaces (Radiol Med 2005;110:349)
    Radiology images

    Images hosted on other servers:

    Preoperative Xrays of right knee

    MRI of right knee

    Prognostic factors
    • Alkaptonuria
    • Exogenous ochronosis
      • Late diagnosis and extensive hyperpigmentation at time of diagnosis connote poor prognosis
    Case reports
    Treatment
    Clinical images

    Contributed by Jonathan D. Ho, M.B.B.S., D.Sc.

    Forearm macules



    Images hosted on other servers:

    Pigmented knee cartilage

    Pigmented papules

    Facial pigmentation

    Microscopic (histologic) description
    • Features of ochronosis are similar in both alkaptonuria and exogenous ochronosis
    • Early changes include homogenization of collagen and degenerating elastotic fibers (Am J Dermatopathol 2014;36:989, Am J Dermatopathol 2019 Nov 12 [Epub ahead of print])
    • Yellow-brown, banana shaped, sickled or round ochronotic bodies deposit in the connective tissue
    • Occasional bodies may be seen in direct continuity with collagen or elastic fibers (Am J Dermatopathol 2019 Nov 12 [Epub ahead of print])
    • Exogenous ochronosis
      • Extensive solar elastosis-like change is noted in an interfollicular distribution
      • Decreased basal keratinocyte pigmentation
      • Occasionally, ochronotic bodies are seen in association with colloid milium and are designated pigmented colloid milium (J Dermatol Case Rep 2015;9:42)
      • Granulomatous reaction has also been reported (J Cutan Pathol 1990;17:236)
    • Similar ochronotic bodies are seen in the connective tissues of cardiac, scleral and bony tissue when involved
    Microscopic (histologic) images

    Contributed by Jonathan D. Ho, M.B.B.S., D.Sc.

    Ochronotic bodies

    Ochronotic bodies embedded in elastosis

    Ochronotic bodies staining with Verhoeff van Gieson

    Pigmented, ochronosis associated colloid milium

    Swollen collagen bundles

    Electron microscopy description
    • Electron dense material within elastic fibers
    • Degeneration of elastic fibers with fragmentation, clearing and a moth eaten appearance
    • Reference: Arch Dermatol 2010;146:1021
    Electron microscopy images

    Images hosted on other servers:

    Exogenous ochronosis

    Molecular / cytogenetics description
    Sample pathology report
    • Right cheek, punch biopsy:
      • Ochronosis (see comment)
      • Comment: The specimen exhibits extensive elastosis in an interfollicular distribution and scattered yellow-brown, banana shaped deposits in the superficial dermis in keeping with exogenous ochronosis.
    Differential diagnosis
    • Melasma:
      • Increased keratinocyte melanization, which may involve all layers of the epidermis
      • Variable papillary dermal melanophages
      • Solar elastosis
      • Telangiectasia
      • Increased mast cells
    • Acquired bilateral nevus of Ota-like macules (Hori nevus):
      • Scattered dendritic melanocytes in the superficial dermis
      • Cells typically heavily pigmented and express melanocyte markers
    • Argyria:
      • Fine, black granules deposited on the basement membrane of blood vessels and skin adnexa
    • Lichen planus pigmentosus:
      • Subtle interface dermatitis (active lesion)
      • Focal band-like lymphocytic infiltrate (active lesion)
      • Band-like or perivascular heavily pigmented melanophages (active and burnt out lesions)
    • Facial acanthosis nigricans:
      • Hyperkeratosis
      • Papillomatosis
      • Only subtle increase in basal keratinocyte pigmentation
    Board review style question #1

    A 40 year old Afro-Caribbean woman with a history of long term use of skin lightening products presents with an asymptomatic hyperpigmented eruption involving the face. A punch biopsy reveals yellow-brown, banana shaped deposits in the superficial dermis. What is the most likely diagnosis?

    1. Acanthosis nigricans
    2. Argyria
    3. Exogenous ochronosis
    4. Lichen planus pigmentosus
    5. Melasma
    Board review style answer #1
    C. Exogenous ochronosis. This image shows extensive elastosis with the formation of banana bodies / ochronotic bodies. These findings are diagnostic of ochronosis. The association with skin lightening agents favor exogenous ochronosis rather than heritable ochronosis.

    Comment Here

    Reference: Ochronosis
    Board review style question #2
    A 60 year old man presents for hip replacement. On preoperative evaluation, black discoloration of the sclera and ear cartilage are noted and a diagnosis of alkaptonuria is entertained. What enzyme is deficient in patients with alkaptonuria?

    1. Fumarylacetoacetate hydrolase
    2. Homogentisate 1,2 dioxygenase
    3. Tyrosinase
    4. Tyrosine aminotransferase
    5. 4-hydroxyphenylpyruvate dioxygenase
    Board review style answer #2
    B. Homogentisate 1,2 dioxygenase. Alkaptonuria is an inborn error of tyrosine metabolism secondary to autosomal recessive loss of function mutations in HGD / HGO, resulting in a deficiency of homogentisate 1,2 dioxygenase. This leads to the accumulation of homogentisic acid with deposition in collagen rich tissue. Longterm tissue deposition leads to degenerative arthritis, cardiac valvulopathy, renal disease and oculocutaneous pigmentation

    Comment Here

    Reference: Ochronosis

    Onchocerciasis
    Definition / general
    • Chronic dermatitis accompanied by progressive keratitis, uveitis and loss of sight caused by the filarial nematode, Onchocerca volvulus which is transmitted to humans through the bite of a blackfly (simulium species) (WHO: Onchocerciasis (river blindness) - disease information [Accessed 9 April 2018])
    • Larval worms (microfilariae) migrate in the skin and the eye and lead to irreversible blindness and skin diseases
    Essential features
    • Caused by the filarial nematode Onchocerca volvulus, through the bite of a blackfly
    • Dying larvae evoke focal inflammation resulting initially in dermal microabscesses followed by granuloma formation
    • Pigmentation and atrophy of skin at advanced stage
    • Treatment is a single dose of ivermectin (150 mcg / kg) every month for 3 - 6 months until patient becomes asymptomatic
    Terminology
    • Also called river blindness
    Epidemiology
    • 90% in sub-Saharan Africa
    • Also found in Yemen as well as Central and Southern America
    • 37 million people infected worldwide in 2006 (PLoS Med 2006;3:e260)
    Sites
    • Eye and skin
    Etiology
    Clinical features
    • Onchocercal nodules are located close to bony prominences outside the inguinal and cervical regions (Postgrad Med J 2010;86:578 )
    • Acute and chronic onchodermatitis: scattered pruritic papules, vesicles or pustules distributed over the shoulders, waist or buttocks (Int J Dermatol 2004;43:170)
    • In advanced disease, the lesions can be spottily depigmented "leopard skin", scaly and atrophic "lizard skin" or thickened and hyperkeratotic "elephant skin"
    • May also have lymphedema of the groin or "hanging groin" and skin atrophy (Br J Dermatol 1993;129:260)
    Diagnosis
    Case reports
    • 5 year old girl with a subcutaneous nodule on forehead and histopathologic analysis of the nodule revealed the presence of Onchocerca volvulus worm (Pediatr Dev Pathol 2015;18:164)
    • 7 year old girl with a conjunctival nodule (Neth J Med 2015;73:437)
    • 9 year old girl with papular, indurated and itching skin lesions located on the limbs and positive anti-filarial antibodies in serum (Klin Padiatr 2006;218:41)
    • 53 year old man presented with a 4 month history of intense, migratory urticaria (Int J Dermatol 2005;44:125)
    • 60 year old woman developed leopard skin-like changes, rashes and pruritus on the left leg and onchocercal microfilariae were identified by a skin snip (Pan Afr Med J 2015;22:298)
    Treatment
    • Ivermectin (150 mcg / kg) administered orally as a single dose and repeated every 3 to 6 months until the patient is asymptomatic (Lancet 2002;360:203)
    Clinical images

    Images hosted on other servers:

    Blackfly (Simulium damnosum)

    Microscopic (histologic) description
    • At early stage, microfilariae concentrate in the papillary dermis with clusters of inflammatory cells surrounding vessels and adnexa
    • Focal microabscesses and granuloma is evoked by dying larvae
    • At advanced stages, secondary acanthosis, parakeratosis, pigment incontinence and melanophagocytosis appear
    • Worms may be calcified or degenerated with dermal scaring and atrophy
    • Onchocercoma (onchocercal nodule) is a subcutaneous ball of worms embedded in inflammatory granulation tissue
    • In cross - section, onchocerca typically has a cuticle, subjacent thin layer of muscle
    • Within the lumen are paired uteri containing microfilaria (J Am Acad Dermatol 2015;73:947)
    Microscopic (histologic) images

    Contributed by Eddie Fridman, M.D.
    Missing Image Missing Image Missing Image Missing Image

    36 year old Ethiopian man with no previous history and a clinical diagnosis of inclusion cyst / pilomatrixoma of left thigh



    Images hosted on other servers:
    Missing Image Missing Image

    Figs C and D: Microfilariae of Onchocerca volvulus in tissue

    Missing Image Missing Image

    Figs B and D: Adults of Onchocerca volvulus

    Differential diagnosis
    • Other helminthic diseases such as:
    Board review style question #1
      Which description of Onchocerciasis is not correct?

    1. A chronic dermatitis caused by the filarial nematode, Onchocerca volvulus which is transmitted to humans through the bite of a blackfly (simulium species).
    2. Dying larvae can evoke focal microabscesses followed by granuloma formation.
    3. Skin snips are the gold standard to diagnose Onchocerciasis.
    4. The disease occurs most commonly in South America.
    5. Treatment is a single dose of Ivermectin (150 mcg/kg) every 3 months.
    Board review style answer #1
    D. 90% of the disease occurs in sub-Saharan Africa.

    Comment Here

    Reference: Onchocerciasis

    Onychomycosis
    Definition / general
    • Spread of fungi to nail apparatus, caused by Trichophyton rubrum and Trichophyton mentagrophytes; also Candida albicans and Scopulariopis brevicaulis
    • Often multiple fungal species
    • Common in patients with diabetes; associated with NOT washing feet daily (Int J Nurs Stud 2011;48:1101)
    • Proposed clinical classification: distal and lateral subungual (beneath the nail), superficial, endonyx (diffuse milky-white discolouration of affected nail but no nail bed hyperkeratosis and no onycholysis), proximal subungual, mixed, totally dystrophic and secondary onychomycosis (J Am Acad Dermatol 2011;65:1219)
    Treatment
    Clinical images

    Images hosted on other servers:

    Toenail infections


    Other porphyrias
    Definition / general
    • Noninflammatory blistering disorder due to disturbance of porphyrin metabolism (porphyrins are present in hemoglobin, myoglobin and cytochromes)
    Clinical features
    • Urticaria and vesicles that heal with scarring, and are exacerbated by sunlight
    • Types: acute intermittent, congenital erythropoietic, hepatoerythropoietic porphyria, porphyria cutanea tarda and mixed
    • Acute intermittent porphyria: second most common type of porphyria but still rare; autosomal dominant metabolic disorder with porphobilinogen deaminase deficiency; typically asymptomatic, but triggered by hormones, drugs and dietary changes; presents with abdominal pain or psychiatric symptoms; skin is typically not involved
    • Congenital erythropoietic porphyria: rare autosomal recessive disease with mutation in uroporphyrinogen-III synthase gene, leading to porphyrin accumulation in many tissues with marked skin photosensitivity, hemolytic anemia with splenomegaly and decreased life expectancy
    • Hepatoerythropoietic porphyria: rare (< 50 cases reported) autosomal recessive form of porphyria cutanea tarda, with markedly deficient activity of uroporphyrinogen decarboxylase (UROD); manifests during infancy or early childhood with extreme photosensitivity, skin fragility in sun-exposed areas, hypertrichosis, erythrodontia and pink urine; may resemble child abuse (Arch Dermatol 2010;146:529)
    • Porphyria cutanea tarda: Most frequent type of porphyria; autosomal dominant disorder due to catalytic deficiency of uroporphyrinogen decarboxylase (UROD), the fifth enzyme in heme biosynthesis (Best Pract Res Clin Gastroenterol 2010;24:735); reduced complement deposition during clinical remission (An Bras Dermatol 2010;85:827)
    Clinical images

    Images hosted on other servers:

    Porphyria cutanea tarda

    Hepatoerythropoietic porphyria

    Microscopic (histologic) description
    • Subepidermal vesicle, marked thickening of superficial dermal vessels (may appear rigid)
    • No / minimal inflammation
    • Porphyria cutanea tarda: dermal papillae protrudes into bulla with festooned pattern; roof of blister has eosinophilic, PAS+, and diastase resistant linear globules
    Microscopic (histologic) images

    Images hosted on other servers:

    Porphyria cutanea tarda associated with HIV;
    Subepidermal bulla formation has resulted
    in loss of the epidermis (note the
    rigid papillary dermal capillary walls)

    Positive stains
    • IgG; to lesser extent IgM, fibrinogen and C3, outline donut shaped blood vessels in the papillary dermis
    • Immunoreactivity also at the dermo-epidermal junction and within basement membrane region of eccrine sweat glands and ducts
    Additional references

    Oxalosis
    Table of Contents
    Definition / general
    Definition / general
    • Excess oxalate accumulation in the kidney resulting in acute and chronic tubulointerstitial injury
    • May be primary genetic (primary hyperoxaluria) resulting in oxalate overproduction or secondary due to excess intake or decreased clearance of oxalate
    • May cause skin manifestations, such as livedo reticularis, acrocyanosis, peripheral gangrene and ulcerations, due to vascular deposition of oxalate (Arch Dermatol 2011;147:1302)

    Pancreatic panniculitis
    Definition / general
    • Due to acute pancreatitis or pancreatic carcinoma
    • Lesions may be widespread, may drain chalky material
    • Associated with elevated serum amylase and lipase
    Epidemiology
    • Males are affected more than females
    Clinical features
    • Tender violaceous and erythematosus nodules, usually in trunk, buttocks and lower extremities
    • Also joint involvement, pleural effusion, ascitis and pericardial effusion
    • Peripheral blood eosinophilia is quite common
    Microscopic (histologic) description
    • Changes are lobular in distribution and characterized by ghost cells, which are anucleate cells composed of amorphous granular debris and a rim of eosinophilia; also stippled basophilia due to calcification
    • Usually neutrophils around foci of fat necrosis and hemorrhage
    • Uninvolved surrounding fat is heavily infiltrated by acute and chronic inflammatory cells including large numbers of macrophages, many with foamy cytoplasm due to ingested lipid, and occasional multinucleated giant cells
    • No evidence of vasculitis
    • Birefringent crystals have been described in the mesenteric fat and within affected joints, but not in subcutaneous fat
    Microscopic (histologic) images

    Contributed by Angel Fernandez-Flores, M.D., Ph.D.


    Panniculitis
    Definition / general
    • Inflammation of subcutaneous fat affecting connective tissue septa separating lobules, usually in lower legs
    • Poorly defined, tender, erythematous nodules
    • Variable fever, malaise
    • Lesions are transient
    • Either septal (involving fibrous septa) or lobular (involving lipocyte lobules)
    • Inadequate biopsy specimens, particularly punch biopsies, may include no subcutaneous fat, and make diagnosis difficult if not impossible

    • Lobular panniculitis: due to nodular vasculitis (erythema induratum), subcutaneous fat necrosis of newborn, pancreatic fat necrosis, lupus panniculitis, physical and factitious panniculitis, other
    • Lupus panniculitis: also called lupus profundus; may develop in normal skin or preexisting skin lesion of patient with systemic or discoid lupus erythematosus
    • Physical and factitious panniculitis: due to blunt force, pinching, cold or injection of foreign substances for secondary gain; a diagnosis of exclusion
    • Septal panniculitis: due to erythema nodosum or alpha-1-antitrypsin deficiency
    Case reports
    Microscopic (histologic) description
    • Lupus panniculitis: vacuolar interface change and lymphoid infiltrates in dermis in lupus-related lesions; in non lupus-related skin lesions, see ghost cells due to loss of nuclei and basophilic material replacing cell membranes, causing basophilic sclerosis of lobules; dense lymphoplasmacytic infiltrates in lobules and septa; variable dermal and septal mucin; fibrosis over time
    • Septal panniculitis: lobular neutrophils, variable focal fat necrosis; refractile material may be present if substances were injected

    Paracoccidiodes (pending)

    Parvovirus B19
    Definition / general
    • Viral infection that typically causes mild rash in children
    Terminology
    • Clinical syndrome is called "Fifth disease" or erythema infectiosum
    Etiology
    • Parvovirus B19 is spread by respiratory droplets, and has an incubation period of 5 - 14 days
    • Viral receptor is the erythrocyte P antigen, which is expressed on endothelial cells
    • Parvovirus attacks erythroblasts; may cause acute anemic crisis in patients with accelerated erythropoiesis, immunocompromised or minimal reserve (chronic anemia, hemolytic anemia, sickle cell, fetuses), although usually recipients have no symptoms (Transfusion 2005;45:1811)
    • Pregnant women may pass virus to fetus, where it may cause marked fetal anemia and hydrops
    • Rash may be due to delayed type hypersensitivity, antibody dependent cellular immunity against microbial antigens in epidermis or endothelium, or circulating immune complexes
    Clinical features
    • Causes "Fifth disease" in children, a mild illness with a "slapped cheek" facial rash
    • Characteristic skin manifestations are petechial eruption in a glove and stocking distribution, reticular truncal erythema and "slapped cheek" sign (Hum Pathol 2000;31:488)
    • May produce influenza-like illness, miscarriage, fetal hydrops and neonatal angioedema
    • Atypical presentations include generalized petechial rash (Pediatrics 2010;125:e787), asymptomatic papular eruption or lower extremity palpable purpura, red baby syndrome (Indian J Pediatr 2009;76:309), papular purpuric gloves and socks syndrome (Med Oral Patol Oral Cir Bucal 2007;12:E4); may resemble Sweet syndrome, myopathic dermatomyosis or lupus erythematosus (Intern Med 1998;37:708)
    • Immune responses usually clear infection and provide lifelong protection but virus may persist in blood or tissue (Transfus Med 2007;17:263)
    • Parvovirus B19 has been associated with some cases of Rosai-Dorfman disease (J Clin Pathol 2006;59:1320)
    Case reports
    Treatment
    • Usually self resolving
    • Can treat symptoms of fever, joint pain, swelling and itching
    • Children and healthy adults have complete recovery
    Clinical images

    Images hosted on other servers:

    Lace-like rash on extremities

    Microscopic (histologic) description
    • Interstitial histiocytic infiltrate with piecemeal fragmentation of collagen and mononuclear cell-predominant vascular injury pattern
    • Extravasated red blood cells
    • Erythrocyte precursors may have intranuclear inclusions
    • Eosinophils and occasional neutrophils may be present
    • Perineuritis may also be present
    • Also interface dermatitis, eczematous alterations, papillary dermal edema; occasionally mesenchymal mucinosis, leukocytoclastic vasculitis
    Microscopic (histologic) images

    Images hosted on other servers:

    Bone marrow

    Fetal spleen


    Parvovirus B19 infection in formalin - fixed and paraffin - embedded placenta and fetal tissues

    B19 positive immunostain

    Positive stains
    • B19 immunostain
    Molecular / cytogenetics description
    • Virus identified in endothelial cells of lesional skin by several methods, including PCR
    • Also identified in keratinocytes

    Pemphigoid gestationis
    Definition / general
    • Rare, self limiting, autoimmune, subepidermal bullous disease, occurring during or soon after pregnancy or in women taking oral contraceptives
    Terminology
    • Formerly called herpes gestationis due to herpetiform nature of blisters but disease is NOT related to herpes infection
    • Called pruritus gravidarum when occurs without significant cutaneous stigmata
    Epidemiology
    • Occurs in 1 per 50,000 pregnancies
    • Rarely complicates hydatidiform mole and gestational choriocarcinoma
    • Rarely present in postpartum period
    • May follow a change in sexual partner
    Sites
    • Pruritic lesions of abdomen, chest, back and extremities
    Etiology
    • Due to circulating autoantibodies against placental collagen XVII (BP180, BPAG2) a hemidesmosomal transmembrane protein and less frequently BP230 (J Cell Biochem 1999;72:356)
    Clinical features
    • Usually urticarial papules, also blisters and rash
    • Usually resolves within weeks to months after delivery
    • Tends to recur with subsequent pregnancy
    • Associated with premature delivery, small for gestational age infants
    Prognostic factors
    Case reports
    Treatment
    Clinical images

    Contributed by Mark R. Wick, M.D.

    Breast skin



    Images hosted on other servers:

    36 year old woman at 24 weeks gestation

    Annular erythematous-edematous lesions

    Exudative and erythematous lesions

    Various images

    Microscopic (histologic) description
    • Similar to bullous pemphigoid - subepidermal blister, with eosinophils in lumen
    • Marked edema in papillary dermis
    • Perivascular infiltrate consists of lymphocytes, histiocytes and large numbers of eosinophils
    • Eosinophilic spongiosis may be seen
    Microscopic (histologic) images

    Contributed by Mark R. Wick, M.D.

    Breast skin

    Collagen IV stain

    IgG stain



    Images hosted on other servers:

    H&E and C3

    H&E, subepidermal bulla with eosinophils

    Perivascular infiltrate of lymphocytes and eosinophils

    Linear C3 staining

    Positive stains
    Differential diagnosis
    • Pruritic urticarial papules of pregnancy: typically begins in stretch mark areas of abdomen and usually ends within 2 weeks after delivery; no antibody deposition
    • Pregnancy prurigo: usually develops in the third trimester of pregnancy, presents with pruritic papules and nodules; histologic changes are those of low grade nonspecific spongiotic dermatitis

    Pemphigus
    Definition / general
    • Pemphigus is a group of acantholytic conditions caused by autoantibodies against various epidermal cell junction proteins, commonly presenting with flaccid blisters, erosions or scaling
    Essential features
    • Group of acantholytic conditions including 4 main forms: pemphigus vulgaris, pemphigus foliaceus, paraneoplastic pemphigus and a newly described IgA pemphigus
    • Caused by IgG or IgA autoantibody against epidermal antigens (commonly desmoglein 1 and desmoglein 3)
    • Most frequent clinical presentation: flaccid bullae or crusted erosions involving skin or mucosa
    • Prototypic histopathological feature: intraepidemal acantholysis with intact basal layer (tombstone sign) and intraepidermal eosinophils
    • Direct immunofluorescence shows intercellular IgG / IgA and C3
    ICD coding
    Epidemiology
    Sites
    • Skin, mucosae, nails (paronychia)
    Pathophysiology
    • IgG or IgA autoantibody against epidermal antigens (Exp Dermatol 2005;14:861, Clin Rev Allergy Immunol 2018;54:1):
      • Pemphigus vulgaris: IgG to desmoglein 1 (skin) or desmoglein 3 (mucosa)
      • Pemphigus foliaceus: IgG to desmoglein 1
      • Paraneoplastic pemphigus: IgG to desmoplakin I, desmoplakin II, plectin, periplakin, envoplakin, BP230 or A2ML1
      • IgA pemphigus: IgA to desmocollin 1 (subcorneal pustular dermatosis [SPD] variant); IgA to desmoglein 1, desmoglein 3 (intraepidermal neutrophilic [IEN] variant)
    • Compensation theory: desmoglein 1 compensates for desmoglein 3 in the skin of patients with mucosal lesions and vice versa (J Clin Invest 1999;103:461)
    • Cell mediated toxicity plays a role in paraneoplastic pemphigus with T cells attacking epidermis resulting in lichenoid inflammation
    Etiology
    • Association with human leukocyte antigen class II: HLA-DR4 and HLA-DR14, HLA-DRB1 0402, 1401, 0503, 0302 (Front Med (Lausanne) 2018;5:226)
    • Endemic pemphigus foliaceus: Simulium nigrimanum (black fly) is a possible vector
    • Drug induced pemphigus:
      • Thiol drugs (e.g. penicillamine, captopril)
        • Penicillamine → pemphigus foliaceus
      • Nonthiol drugs (e.g. penicillin, cephalosporin, enalapril, rifampin, NSAID) (Br Med J 1976;2:1423)
    • Paraneoplastic pemphigus:
      • B cell lymphoproliferative disorders, Castleman disease, Waldenström macroglobulinemia, carcinomas (bronchus, pancreas, liver, uterus, breast, thyroid, liver), thymoma, sarcoma (N Engl J Med 1990;323:1729)
    Clinical features
    • Pemphigus vulgaris:
      • Mucosal erosions (oral or genital), painful flaccid blisters
        • Hoarseness if larynx or pharynx are involved
      • Rarely isolated crusted lesions on the scalp, paronychia, foot ulcers and macroglossia
    • Pemphigus vulgaris variants:
      • Pemphigus vegetans:
        • Neumann type (severe) and Hallopeau type (mild)
        • Affects intertriginous areas, blisters evolving into hypertrophic vegetations and pustules, cerebriform tongue (Ankara Univ Hekim Fak Derg 1988;15:137)
    • Pemphigus foliaceus:
      • Affects seborrheic areas (face, scalp, upper trunk)
      • Crusted superficial erosions
    • Pemphigus foliaceus variants:
      • Pemphigus erythematosus (Senear-Usher syndrome):
        • Plaques with crusted erosions on the face (malar areas)
        • Patients may have underlying lupus erythematosus
      • Endemic pemphigus foliaceus: small blisters, erosions or papules (J Dermatol 2015;42:18)
      • Pemphigus herpetiformis:
        • Usually a variant of pemphigus foliaceus, rarely of pemphigus vulgaris
        • Urticarial plaques and vesicles in herpetiform pattern
    • Paraneoplastic pemphigus:
      • Mucosal involvement (severe stomatitis and conjunctivitis)
      • Erythema multiforme-like (targetoid) lesions on the palms and soles
      • Lichenoid lesions
      • Bronchiolitis obliterans
    • IgA pemphigus:
      • Small superficial blisters / pustules arranged in annular / arcuate plaques with crusting in the center; mucosae are usually not involved (Oncol Lett 2014;8:62)
      • Superficial flaccid pustular blister (SPD variant)
      • Generalized crusted macules with peripheral pustules (IEN variant)
    • Signs:
      • Nikolsky sign: induction of blister / erosion by slight friction
      • Asboe-Hansen sign: lateral extension of intact blister with gentle pressure
    Diagnosis
    • Made on clinical, histopathologic, immunopathologic and serologic findings (Turk Patoloji Derg 2016;32:91)
    • Histopathology
      • Biopsy for H&E taken from the edge of the blister
    • Immunopathology
      • Direct immunofluorescence (most sensitive)
      • Biopsy taken from the perilesional skin
    • Serology
    • Endoscopy may help identify extraoral mucosal lesions (Clin Endosc 2014;47:452)
    • Paraneoplastic pemphigus may utilize chest Xray, CT scans, pulmonary function tests and cancer screening
    Laboratory
    • Indirect immunofluorescence: intercellular deposition of IgG or IgA
      • Pemphigus vulgaris and variants:
        • IgG against desmoglein 3 or desmoglein 1 and 3
        • Monkey esophagus used as a substrate
      • Pemphigus foliaceus and variants:
      • Paraneoplastic pemphigus:
        • IgG against envoplakin, periplakin, desmoglein 1 and desmoglein 3
        • Rat bladder used as a substrate for plakin
      • IgA pemphigus:
        • IgA against desmocollin 1 (SPD variant)
        • IgA against desmoglein 1, desmoglein 3 (IEN variant)
    • Enzyme linked immunosorbent assay (ELISA):
      • Pemphigus vulgaris (IgG against desmoglein 3 or 1)
      • Pemphigus foliaceus (IgG against desmoglein 1)
      • Paraneoplastic pemphigus (antibodies against envoplakin and periplakin)
      • IgA pemphigus (IgA and sometimes IgG to desmocollin 1, desmoglein 1 and desmoglein 3)
    • Immunoblotting and immunoprecipitation (rarely used today)
    • Other findings:
      • Possible autoimmune related thrombocytopenia, neutropenia and anemia
    Radiology description
    • Bronchiolitis obliterans in pemphigus paraneoplastica presents with reduction of vessels and areas of bronchiectasis
    Radiology images

    Images hosted on other servers:

    Bronchiolitis obliterans, paraneoplastic pemphigus

    Castleman disease, paraneoplastic pemphigus

    Prognostic factors
    • Historically, 5 year mortality > 90%
    • Pemphigus foliaceus has better prognosis than pemphigus vulgaris
    • Poor prognosis factors (J Eur Acad Dermatol Venereol 2010;24:947):
      • Younger age at presentation
      • Mucosal involvement at presentation
      • Relapses
    • Paraneoplastic pemphigus (Arch Dermatol 2012;148:1165):
      • 5 year survival is about 40%
      • Poor prognosis factors: erythema multiforme-like lesions, bronchiolitis obliterans, extensive involvement
    Case reports
    Treatment
    • Rituximab is a first line in moderate and severe cases (J Eur Acad Dermatol Venereol 2020;34:1900)
    • Systemic steroids are treatment of choice if rituximab is not available
    • Systemic immunosuppressive medications (azathioprine, mycophenolate mofetil, methotrexate, cyclosporine and dapsone) are used as steroid sparing agents
    • Local skin care
    • Prevention of secondary infection (especially herpes simplex virus)
    • Intravenous immune globulin, immunoadsorption, plasmapheresis, extracorporeal photopheresis or cyclophosphamide for refractory cases
    • Dapsone may be used as a first line agent for IgA pemphigus (Indian J Dermatol 2012;57:495)
    Clinical images

    Contributed by Natalia Zhovta, M.D., Yi C. Lai, M.D., Maria Hannaway, M.D., Jamerson Loyal, M.D., Iryna Tsikhanouskaya, M.D., Ph.D., Timothy Patton, D.O. and Juliana Jung, M.D.

    Pemphigus vulgaris:
    Erosions on oral mucosa

    Erosions on oral mucosa

    Erosions on the hands

    Erosions on the hands

    Erosions on the knees

    Erosions on the knees

    Erosions on the back

    Erosions on the back


    Superficial erosions

    Superficial erosions

    Erosions on the scalp

    Erosions on the scalp

    Verrucous plaques

    Verrucous plaques



    Pemphigus foliaceus:
    Superficial erosions

    Superficial erosions

    Crusted plaques

    Crusted plaques

    Superficial crusts and erosions

    Superficial crusts and erosions



    Paraneoplastic pemphigus:
    Severe stomatitis Severe stomatitis

    Severe stomatitis

    Tense bullas

    Tense bullas



    IgA pemphigus:
    Superficial flaccid bullas

    Superficial flaccid bullas

    Annular lesions, flaccid bullas

    Annular lesions, flaccid bullas

    Superficial pustules

    Superficial pustules



    Images hosted on other servers:

    Paraneoplastic pemphigus

    Paraneoplastic pemphigus associated with non-Hodgkin lymphoma

    Pemphigus vulgaris

    Microscopic (histologic) description
    • Pemphigus vulgaris:
      • Suprabasal acantholysis
      • Basal layer remains attached (tombstone sign)
      • Intraepidermal vesicles with round acantholytic keratinocytes
      • Intraepidermal eosinophils and sometimes neutrophils
      • Follicular involvement
      • No significant dyskeratosis
      • Miniaturization of the sebaceous glands in the scalp lesions (also seen in pemphigus foliaceus) (J Cutan Pathol 2017;44:835)
    • Pemphigus vegetans:
      • Subtle suprabasal acantholysis
      • Hyperplastic epithelium, pseudoepitheliomatous hyperplasia
      • Intraepidermal eosinophils, microabscesses
    • Pemphigus foliaceus, endemic pemphigus foliaceus, pemphigus erythematosus:
      • Subcorneal acantholysis; minority of cases may have acantholysis in the lower layers of the epidermis (J Cutan Pathol 2014;41:880)
      • Rarely intact blisters
      • Detached stratum corneum may show granular keratinocytes as cling ons
      • Neutrophilic exocytosis
      • Intraepidermal eosinophils
    • Paraneoplastic pemphigus:
      • Variable
      • Features of pemphigus vulgaris
      • Lichenoid and vacuolar interface changes
      • Dyskeratosis and spongiosis
      • Rare eosinophils
    • IgA pemphigus:
      • Subcorneal pustules (subcorneal pustular dermatosis variant)
      • All layer epidermal pustules (intraepidermal neutrophilic variant)
    Microscopic (histologic) images

    Contributed by Viktoryia Kazlouskaya, M.D., Ph.D., Juliana Jung, M.D. and Case #56

    Pemphigus vulgaris:
    Suprabasilar acantholysis

    Suprabasilar acantholysis

    Suprabasilar split

    Suprabasilar split

    Intact basal layer

    Intact basal layer

    Oral ulcer

    Oral ulcer

    Intraepidermal spilt on musoca

    Intraepidermal spilt on musoca



    Pemphigus foliaceus:
    Acanthosis and subcorneal split

    Acanthosis and subcorneal split

    Subcorneal split

    Subcorneal split

    Minimal subcorneal split

    Minimal subcorneal split

    Focal subcorneal split

    Focal subcorneal split

    Subcorneal split

    Subcorneal split

    Subcorneal acantholysis

    Subcorneal acantholysis


    Pemphigus foliaceus

    Pemphigus foliaceus



    Paraneoplastic pemphigus:
    Blister with lichenoid inflammation Blister and lichenoid inflammation

    Blister with lichenoid inflammation

    Virtual slides

    Images hosted on other servers:

    Pemphigus vegetans: eosinophilic collections

    Pemphigus vulgaris: acantholysis and eosinophils

    Pemphigus vulgaris with acantholysis

    Cytology description
    • Rarely used but may be helpful as a quick tool or if other diagnostic resources are limited
    • Tzanck smears from the erosions or blister bottom (Sci Rep 2020;10:18314)
    • Acantholytic cells are larger round cells with large nucleus surrounded by clear halo and inconspicuous nucleoli
    Cytology images

    Images hosted on other servers:
    Large round acantholytic cells

    Large round acantholytic cells

    Immunofluorescence description
    • Pemphigus vulgaris: intercellular IgG and C3 in the epidermis (chicken wire pattern) and adnexal epithelium (J Int Oral Health 2015;7:59)
    • Pemphigus foliaceus: intercellular pattern, stronger fluorescence in the upper epidermis
    • Paraneoplastic pemphigus: linear or granular IgG at dermal epidermal junction; in 50% of cases, intercellular deposition is also seen
    • Pemphigus erythematosus: granular IgG at dermal epidermal junction
    • IgA pemphigus: intercellular IgA in the upper epidermis (SPD variant) or lower epidermis (IEN variant)
    Immunofluorescence images

    Contributed by Viktoryia Kazlouskaya, M.D., Ph.D. and Case #56
    Intercellular IgG deposits

    Intercellular IgG deposits

    Intercellular IgG deposits

    Intercellular C3 deposits

    Pemphigus foliaceus: C3 (left) and IgG (right)



    Images hosted on other servers:
    Intercellular deposits of IgA

    Intercellular deposits of IgA

    Negative stains
    • Fungal stains (PAS, GMS, etc.)
    Electron microscopy description
    • Dilatation of intercellular space, stretched desmosomes, acantholytic cells
    Electron microscopy images

    Images hosted on other servers:
    Loose keratinocytes, pemphigus

    Loose keratinocytes, pemphigus

    Sample pathology report
    • Skin, punch biopsy:
      • Intraepidermal acantholysis, suggestive of (or consistent with) pemphigus vulagaris (see comment)
      • Comment: Examination of the specimen shows intraepidermal acanthosis in the lower portions of the epidermis and spongiosis with eosinophil and neutrophil exocytosis. There is a perivascular infiltrate with eosinophils in the dermis. Neither fungal microorganisms nor basement membrane changes are seen with interpretation of PAS histochemical stain.
    • Skin, punch for immunofluorescence:
      • Intercellular epidermal IgG and C3 deposition (see microscopic and comment)
      • Comment: IgG and C3 immunofluorescence is seen throughout the epidermis with the intercellular pattern suggestive of pemphigus vulgaris but correlation with clinical findings is necessary.
      • 5 immunofluorescence stains are scored subjectively on a scale from 0 - 3+
      • Staining results:
        • IgG: 2+, intercellular epidermal
        • IgM: 0, negative
        • IgA: 0, negative
        • C3: 2+, intercellular epidermal
        • Fibrin: 0, negative
    Differential diagnosis
    • Pemphigus vulgaris:
      • Grover disease:
        • Apparent dyskeratosis, focal acantholysis, follicular acantholysis is rare, usually a small lesion, negative immunofluorescence (J Cutan Pathol 2019;46:6)
        • Clinically, itchy nonfollicular papules on the trunk of middle age and elderly patients
      • Darier disease:
        • Apparent dyskeratosis, epidermal hyperplasia, follicular acantholysis is rare, negative immunofluorescence
        • Clinically, congenital, autosomal dominant, starts in adolescence, presents with crusted papules on the intertriginous areas and scalp
      • Warty dyskeratoma:
        • Verrucous epidermal hyperplasia, exoendophytic silhouette, apparent dyskeratosis, negative immunofluorescence
        • Clinically, solitary nodule
      • Hailey-Hailey disease:
        • Psoriasiform hyperplasia, exuberant crust may be present, apparent dyskeratosis, follicular involvement is rare, acantholytic cells throughout the full thickness of the epidermis (brick wall look), negative immunofluorescence
    • Pemphigus foliaceus:
      • Bullous impetigo:
        • Evidence of bacteria, negative immunofluorescence, more common in children, clinical presentation and history
      • Staphylococcal scalded skin syndrome:
        • May be indistinguishable with pemphigus foliaceus, clinical correlation is required
      • Fungal infections:
        • Positive fungal stain, negative immunofluorescence
    • Pemphigus vegetans:
      • Pyostomatitis vegetans:
        • Neutrophilic rather than eosinophilic abscesses
      • Infections:
        • Positive fungal, bacterial stains, negative immunofluorescence
    • Paraneoplastic pemphigus:
    • IgA pemphigus:
    Board review style question #1

    What are the expected histopathological features of the condition in the above image?

    1. Acantholytic dyskeratotic cells and parakeratotic columns with negative direct immunofluorescence
    2. Dilapidated brick wall appearance with negative direct immunofluorescence
    3. Focal acantholysis with various pattern and negative direct immunofluorescence
    4. Intraepidermal acantholysis with chicken wire pattern IgG on direct immunofluorescence
    Board review style answer #1
    D. Intraepidermal acantholysis with chicken wire pattern IgG on direct immunofluorescence

    The image demonstrates crusted erosions and bullae on the scalp and chest and is consistent with pemphigus vulgaris. Histopathological features of pemphigus vulgaris include intraepidermal acantholysis and chicken wire pattern IgG on direct immunofluorescence. Answer A is incorrect because acantholytic, dyskeratotic cells and parakeratotic columns are seen in Darier disease, which typically presents as keratotic plaques and papules. Answer B is incorrect because dilapidated brick wall appearance is seen in Hailey-Hailey disease, which typically presents in intertriginous areas. Answer C is incorrect because focal acantholysis with various patterns is seen in Grover disease, which typically presents as small papules on the chest.

    Comment Here

    Reference: Pemphigus
    Board review style question #2
    A patient presents with oral, ocular and polymorphous skin lesions. Biopsy shows suprabasal acantholysis and lichenoid changes. What is the most likely underlying etiology?

    1. B cell lymphoproliferative disorders
    2. Contact allergen
    3. Idiopathic autoantibody
    4. Thiol drug
    5. Viral infection
    Board review style answer #2
    A. B cell lymphoproliferative disorders

    Suprabasal acantholysis with lichenoid changes and oral and conjunctival involvement is characteristic of paraneoplastic pemphigus, with B cell lymphoproliferative disorders as the most common underlying cause. Answer B is incorrect because contact allergic pemphigus has been reported with histopathology analogous to pemphigus vulgaris but not paraneoplastic pemphigus. Answer C is incorrect because idiopathic autoantibody is consistent with primary pemphigus variants. Answer D is incorrect because thiol drug is reported to cause drug induced pemphigus. Answer E is incorrect because viral infection has not been linked with paraneoplastic pemphigus-like lesions.

    Comment Here

    Reference: Pemphigus
    Board review style question #3
    Which of the following is true regarding pemphigus foliaceus?

    1. Caused by desmoglein 3 autoantibody
    2. Has worse prognosis than pemphigus vulgaris in general
    3. Involves the lower half of the epidermis
    4. More prevalent than pemphigus vulgaris in general
    5. Rarely affects mucosa
    Board review style answer #3
    E. Rarely affects mucosa

    The mucosa expresses less desmoglein 1 compared with desmoglein 3 and is typically spared in pemphigus foliaceus. Answers A and C are incorrect because pemphigus foliaceus characteristically demonstrates desmoglein 1 autoantibodies and affects the superficial epidermis of the skin. Answers B and D are incorrect because pemphigus foliaceus is generally less common than pemphigus vulgaris and has a better prognosis.

    Comment Here

    Reference: Pemphigus

    Perforating dermatoses
    Definition / general
    • Elastosis perforans serpiginosum: rare skin disease in which abnormal elastic tissue fibers, other connective tissue elements and cellular debris are expelled from papillary dermis through epidermis (eMedicine: Elastosis Perforans Serpiginosum [Accessed 27 August 2018]); often back of neck in teenage boys
    • Reactive perforating collagenosis: rare skin disease of transepidermal elimination of altered collagen via the epidermis, either inherited or acquired and associated with diabetes melliltus or end stage renal disease
    Epidemiology
    • Either idiopathic, drug induced (D-penicillamine), or associated with other inherited disorders (Down syndrome, Ehlers-Danlos syndrome, osteogenesis imperfecta, Marfan syndrome, Rothmund-Thomson syndrome)
    Microscopic (histologic) description
    • Clumps of coarse elastic fibers penetrate epidermis and cause focal epidermal hyperplasia
    • Marked increase in elastic tissue in both reticular and papillary dermis
    • The vertically oriented fibers of the papillary dermis are thicker than normal
    • The center of the lesion shows characteristic transepithelial, parafollicular or transfollicular fibers that are straight, wavy or screw-like in configuration
    • The perforating canal contains degenerated epithelial cells, inflammatory debris and numerous elastic fibers
    • The epithelium on either side of the perforating canal is acanthotic, often with pseudoepitheliomatous hyperplasia
    • Foreign body giant cell reaction is common in superficial dermis; occasionally elastophagocytosis is present
    Microscopic (histologic) images

    Images hosted on other servers:

    Various images

    Positive stains
    • Elastic stains (highlights elastic fibers)
    Differential diagnosis
    • Kyrle disease: transepidermal degeneration of parakeratin and inflammatory debris
    • Perforating folliculitis: transepidermal elimination of degenerative connective tissue
    • Reactive perforating collagenosis: transepidermal elimination of collagen

    Periodic fever syndromes (pending)
    [Pending]

    Pigmented purpuric dermatosis
    Definition / general
    • Chronic purpuric skin disorders include several skin disorders characterized by macules, patches and petechiae on a brown, red or yellow base resulting from erythrocyte extravasation and hemosiderin deposition
    Essential features
    Terminology
    ICD coding
    • ICD-10: L81.7 - pigmented purpuric dermatosis
    • ICD-11: EF40.0 - capillaritis
    Epidemiology
    Sites
    Pathophysiology
    Etiology
    Clinical features
    • Different subtypes are mainly distinguished based on the clinical presentations (StatPearls: Pigmented Purpuric Dermatitis [Accessed 18 April 2023])
      • Schamberg disease (progressive pigmentary purpura)
        • Most common type; pinpoint to pinhead reddish puncta known as grains of cayenne pepper, purpuric macules forming patches, bilateral with a chronic course and numerous exacerbations and remissions
      • Purpura annularis telangiectodes of Majocchi disease
        • Annular patches with telangiectasias and central atrophy
        • Linear or arciform patches may be present, bilateral with a chronic course, relapses and remissions
      • Pigmented purpuric lichenoid dermatitis of Gougerot and Blum
        • Lichenoid and violaceous plaques with superimposed purpura, 1 to several centimeters in diameter, pruritic, may resemble Kaposi sarcoma, bilateral, chronic
      • Lichen aureus
        • Rusty yellow or gold lichenoid papules, patches and plaques, unilateral, acute onset, persistent course
      • Eczematid-like purpura of Doucas and Kapetanakis
        • Eczematous features, with scale overlying purpuric and petechial macules, pruritic with mild lichenification, acute onset and fluctuating course
      • Itching purpura or disseminated pruriginous angiodermatitis
        • Orange to brown purpuric macules with acute onset and chronic course
      • Unilateral linear capillaritis or segmental pigmented purpura and quadrantic capillaropathy
        • Linear, dermatomal or segmental distribution with self resolving course
      • Granulomatous pigmented purpura
        • Red to brown purpuric macules or papules, mostly involving knee and ankles, with a persistent course, more often in Asians
    • Pigmented purpuric dermatosis-like variant of mycosis fungoides can overlap with mycosis fungoides or mycosis fungoides can evolve from pigmented purpuric dermatosis (Am J Dermatopathol 1997;19:108, Cutis 2014;94:297)
      • This form shows large areas of confluence, violaceous hue and pruritus usually lasting > 1 year
    • Dermoscopy: red or brown dots and globules, copper colored pigmentation in background, linear vasculatures, linear and reticular brown lines, rosette structure, red patches, follicular opening (An Bras Dermatol 2016;91:584, Postepy Dermatol Alergol 2019;36:687)
    Diagnosis
    • Diagnosis is often made by clinical presentation
    • Biopsy is used for confirmation
    • Blood work up and coagulation screening to exclude other conditions
    • Reference: J Clin Med 2021;10:2283
    Laboratory
    • No associated laboratory abnormalities
    Prognostic factors
    • Variable progression and resolution course
    • Often chronic with multiple relapses
    • Pigmented purpuric dermatosis with monoclonal lymphocytic infiltrate: extensive disease (Am J Clin Pathol 2007;128:218)
    • Progression to mycosis fungoides rarely reported (J Cutan Pathol 2021;48:611)
    Case reports
    Treatment
    Clinical images

    Contributed by Michael Franzetti, M.D.
    Schamberg disease

    Schamberg disease

    Annular patch

    Annular patch



    Images hosted on other servers:

    Schamberg disease

    Purpura annularis telangiectodes

    Diet induced pigmented purpuric dermatosis

    Pigmented purpuric dermatosis of Gougerot-Blum

    Microscopic (histologic) description
    • Common histologic features in all subtypes (StatPearls: Pigmented Purpuric Dermatitis [Accessed 18 April 2023])
      • CD4+ perivascular lymphocytic infiltrate
      • Occasional CD1a dendritic cells and macrophages around superficial small blood vessels
      • Erythrocyte extravasation
      • Dilated blood vessels and endothelial cell swelling
      • Hemosiderin deposition and variable pigmentation
      • Mild epidermal spongiosis and lymphocyte exocytosis
        • Present in all subtypes except lichen aureus
    • Possible associated features
      • Epidermis: focal parakeratosis, basal layer vacuolar degeneration
      • Dermal - epidermal junction: interface changes (Ann Dermatol 2015;27:404)
      • Vascular: endothelial edema, proliferation with narrowing of vessel lumen
    • Progressive pigmentary dermatosis (Schamberg disease) (Dermatol Clin 1985;3:165)
      • Variable degree of epidermal spongiosis
    • Pigmented purpuric lichenoid dermatitis of Gougerot and Blum
      • Band-like lymphocytic infiltrate in the upper dermis with or without epidermal spongiosis
      • Patchy parakeratosis (J Clin Med 2021;10:2283)
    • Itching purpura
      • Marked spongiosis and neutrophils, hemosiderin can be minimal in early lesions
    • Eczematoid purpura of Doucas and Kapetanakis (Ann Dermatol 2015;27:404)
      • Epidermal spongiosis and lymphocytic exocytosis are more prominent in this subtype
    • Lichen aureus (Br J Dermatol 1985;112:307)
      • Band-like lichenoid lymphocytic inflammation at the dermal - epidermal junction with Grenz zone
    • Granulomatous pigmented purpura (Am J Dermatopathol 2012;34:746, J Dermatol 1996;23:551)
      • Dense granulomatous infiltrate in the papillary dermis, monocyte infiltrate, thickened capillaries and hemosiderin deposition
    Microscopic (histologic) images

    Contributed by Sepideh N. Asadbeigi, M.D., Shabnam Momtahen, M.D. and Jeffrey McBride, M.D.
    Perivascular lymphocytes Perivascular lymphocytes Perivascular lymphocytes

    Perivascular lymphocytes

    Schamberg disease

    Schamberg disease


    Vacuolization Vacuolization

    Vacuolization

    Erythrocyte extravasation

    Erythrocyte extravasation

    Hemosiderin deposition

    Hemosiderin deposition

    Immunofluorescence description
    Molecular / cytogenetics description
    • 2 principal categories: polyclonal and monoclonal; ~40% of monoclonal cases have clinical and pathologic features of mycosis fungoides (Am J Clin Pathol 2007;128:218)
    Sample pathology report
    • Skin, left lower leg, punch biopsy:
      • Pigmented purpuric dermatosis (see comment)
      • Comment: The histopathologic examination shows mild spongiosis with vacuolization of the basal layer of the epidermis. There is cuffing of the small vessels in the papillary dermis by lymphocytes. Erythrocyte extravasation and superficial hemosiderin deposition in the dermis is present. The overall histologic features are consistent with pigmented purpuric dermatosis.
    Differential diagnosis
    Additional references
    Board review style question #1

      A 38 year old woman with an unremarkable medical history presents with a 2 week history of bilateral red macules on the shins. Physical examination reveals pinpoint petechiae resembling grains of cayenne pepper on her legs. CBC and coagulation studies are within normal limits. Biopsy shows a superficial perivascular lymphocytic infiltrate without luminal wall fibrin deposition. Mild epidermal spongiosis, erythrocyte extravasation and siderophages are present. What is the most compatible diagnosis in this patient?

    1. Cutaneous vasculitis
    2. Drug eruption
    3. Mycosis fungoides
    4. Schamberg disease
    Board review style answer #1
    D. Schamberg disease. This patient has a progressive pigmented dermatosis, consistent with Schamberg disease. Answer A is incorrect because there is no evidence of cutaneous vasculitis, which shows erythrocyte extravasation with vascular destruction, leukocytoclasia, fibrinoid necrosis and vascular luminal wall fibrin deposition. Answer B is incorrect because drug reactions would have a relevant medical history and often show eosinophils. Answer C is incorrect because although some forms of Schamberg disease may overlap with mycosis fungoides, these biopsies show cytologic atypia of epidermotropic lymphocytes and Pautrier microabscesses, which were not present here.

    Comment Here

    Reference: Pigmented purpuric dermatosis
    Board review style question #2
      Which of the following is true about pigmented purpuric dermatoses?

    1. Direct immunofluorescence studies show IgA depositions in dermal vessels
    2. Early stages of mycosis fungoides may have histologic overlap
    3. Fibrinoid necrosis of the vessel wall is a characteristic feature
    4. Hyperplasia of endothelial cells and an increase in superficial vessels are frequently observed
    Board review style answer #2
    B. Early stages of mycosis fungoides may have histologic overlap with pigmented purpuric dermatoses. Answer C is incorrect because fibrinoid necrosis of the vessel wall is a feature of cutaneous vasculitis and should not be present in pigmented purpuric dermatoses. Answer D is incorrect because hyperplasia of endothelial cells and an increase in superficial vessels are seen in stasis dermatitis, which should be considered in the differential diagnosis. Answer A is incorrect because IgA depositions are seen in the dermal vessels of IgA and lupus vasculitis. PPD is not an immune complex mediated disease.

    Comment Here

    Reference: Pigmented purpuric dermatosis

    Pityriasis lichenoides
    Definition / general
    Essential features
    • Asymptomatic red-brown and small sized papulosquamous lesions arise in crops and usually involve trunk, upper extremities and buttocks
    • Parakeratosis, spongiosis, intraepidermal lymphocytes and erythrocytes, superficial perivascular or lichenoid lymphocytic infiltrate, focal interface change, melanophages and extravasated erythrocytes
    Terminology
    • Guttate parapsoriasis, chronic lichenoid pityriasis, parapsoriasis lichenoides chronica
    ICD coding
    • ICD-10: L41.1 - pityriasis lichenoides chronica
    Epidemiology
    Sites
    • Trunk, proximal extremities and buttocks
    Pathophysiology
    • Not exactly understood; infectious and drug related hypersensitivity reactions and premycotic lymphoproliferative disorder are the main hypotheses
    Etiology
    • Associated with genetic and environmental factors (viral infections, autoimmune diseases, drug and vaccine induced, etc.); there is some evidence of association with lymphoproliferative disorder
    Clinical features
    • Acute form: erythematous, vesiculopustular and necrotic lesions with the symptoms of itching and burning
    • Chronic form: reddish brown papules, postinflammatory hypo and hyperpigmentation can be seen
    • Upper limbs and flexor surfaces are more frequently involved than lower limbs and extensor surfaces; central, diffuse and peripheral forms can occur and atypical presentation may appear (Dermatol Pract Concept 2013;3:7, J Drugs Dermatol 2019;18:690)
    Diagnosis
    Laboratory
    • Some patients have elevated serum immune complexes
    Prognostic factors
    • Relapsing and remitting course persists for months or a few years
    • Treatment may or may not relieve symptoms
    Case reports
    Treatment
    • Topical steroids
    • Topical immunomodulators (tacrolimus, pimecrolimus)
    • Oral antibiotics (erythromycin, tetracycline such as doxycycline)
    • Phototherapy
    • Systemic steroids
    • Reference: Am J Clin Dermatol 2007;8:29
    Clinical images

    Contributed by Mark R. Wick, M.D.

    Arms

    Breast skin



    Images hosted on other servers:

    Oval red-brown papulosquamous lesion

    Flattened scaly brown papules

    Microscopic (histologic) description
    • Parakeratosis, mild to moderate acanthosis, mild spongiosis, sparse necrotic keratinocytes, minimal vacuolar degeneration of the basal layer, focal exocytosis of small numbers of lymphocytes and intraepidermal erythrocytes
    • Mild superficial perivascular and lichenoid lymphocytic infiltrate, focal interface change, melanophages and extravasated erythrocytes without (occasionally focal) fibrinoid necrosis of vessels; neither eosinophils nor plasma cells present
    • Dilatation and congestion of superficial vessels, papillary dermal edema and pigment incontinence
    • Pityriasis lichenoides et varioliformis acuta (PLEVA, acute form of the disease) is similar to PLC but with more pronounced histologic features: more interface damage, more inflammation, more erythrocytes in dermis and epidermis
      • Acute and chronic refer to the characteristics of the lesions
    • Also shares clinical and some immune histologic features with lymphomatoid papulosis (Am J Surg Pathol 2012;36:1021)
    • CD4:CD8 ratio > 1; monoclonal T cell population may mimic MF (Indian J Dermatol 2012;57:424)
    • Some cases reported as a form of MF and T cell dyscrasias arising in association with pitryiasis lichenoides in children and adults (J Turk Acad Dermatol 2014;8:1483c3, Australas J Dermatol 2021 Nov 9 [Epub ahead of print])
    • Adnexotropism was described (Am J Dermatopathol 2020;42:1)
    Microscopic (histologic) images

    Contributed by Gülçin (Güler) Şimşek, M.D. and Mark R. Wick, M.D.

    Parakeratosis and exocytosis of lymphocytes

    Parakeratosis, spongiosis and exocytosis of lymphocytes

    Perivascular and lichenoid lymphocytic infiltrate

    Lichenoid interface dermatitis with melanophages

    Extravasated erythrocytes, fibrinoid necrosis


    Intraepidermal erythrocytes and lymphocytes

    Exocytosis of lymphocytes / extravasated erythrocytes

    Breast skin

    Immunofluorescence description
    • IgM and C3 in the walls of the superficial dermal blood vessels and along the dermoepidermal junction in some patients
    Sample pathology report
    • Trunk, papulosquamous lesion, punch biopsy:
      • Lichenoid / perivascular interface dermatitis (given the history, compatible with pitryiasis lichenoides chronica) (see comment)
      • Comment: Parakeratosis, mild superficial perivascular and lichenoid lymphocytic infiltrate, focal interface change, exocytosis of small numbers of lymphocytes and extravasated dermal and intraepidermal erythrocytes. The histologic features are consistent with pitryiasis lichenoides chronica. Clinicopathologic correlation is recommended.
    Differential diagnosis
    Board review style question #1


    An 11 year old boy presented with depigmented papulosquamous lesions on the anterior part of the trunk and upper extremities. Clinical picture and biopsy reveal the findings shown in the images above. What is the most likely diagnosis?

    1. Lichen planus
    2. Lichenoid drug eruption
    3. Mycosis fungoides
    4. Pityriasis lichenoides chronica
    5. Pityriasis lichenoides et varioliformis acuta
    Board review style answer #1
    D. Pityriasis lichenoides chronica

    Comment Here

    Reference: Pityriasis lichenoides chronica

    Pityriasis rosea
    Definition / general
    • Pityriasis rosea is a self limited viral exanthem usually seen in young patients that starts with a herald patch, followed by similar lesions on the trunk
    Essential features
    • Pityriasis rosea is a viral exanthem, commonly caused by reactivation of human herpesvirus 6 and human herpesvirus 7
    • Clinically presents with the herald patch manifesting first, followed by disseminated similar patches on the upper trunk
    • Histopathology of pityriasis rosea demonstrates spongiotic pattern
    ICD coding
    • ICD-10: L42 - pityriasis rosea
    Epidemiology
    Sites
    • Skin (usually trunk and extremities), lymphadenopathy
    Pathophysiology
    Etiology
    Clinical features
    Diagnosis
    • Clinical examination
    • Skin biopsy
    Laboratory
    • Usually not needed to establish the diagnosis
    Prognostic factors
    Case reports
    Treatment
    Clinical images

    Contributed by Natalia Zhovta, M.D. and Yi Lai, M.D.
    Elongated scaly patches

    Elongated scaly patches

    Inverse patch

    Inverse patch

    Scaly patches

    Scaly patches

    Disseminated scaly patches

    Disseminated scaly patches

    Herald patch

    Herald patch

    Disseminated patches

    Disseminated patches


    Herald patch

    Herald patch

    Disseminated elongated patches

    Disseminated elongated patches

    Microscopic (histologic) description
    • Subacute spongiotic dermatitis with perivascular lymphocytic infiltrate (Dermatologica 1982;165:551)
    • Mild acanthosis of the epidermis may be present, especially in herald patch
    • Foci of parakeratosis overlying spongiosis
    • Granular layer may be diminished or absent under area of parakeratosis (Indian J Dermatol Venereol Leprol 2000;66:244)
    • Erythrocyte extravasates may be focally present in the superficial dermis
    • Rare presence of dyskeratotic cells and eosinophils may be observed
    Microscopic (histologic) images

    Contributed by Viktoryia Kazlouskaya, M.D., Ph.D.
    Spongiosis with mounds of parakeratosis

    Spongiosis with mounds of parakeratosis

    Minimal spongiosis with mounds of parakeratosis Minimal spongiosis with mounds of parakeratosis

    Minimal spongiosis with mounds of parakeratosis

    Spongiosis, parakeratosis, erythrocyte extravasates Spongiosis, parakeratosis, erythrocyte extravasates Spongiosis, parakeratosis, erythrocyte extravasates

    Spongiosis, parakeratosis, erythrocyte extravasates


    Spongiosis, parakeratosis, superficial lymphocytic infiltrate Spongiosis, parakeratosis, superficial lymphocytic infiltrate

    Spongiosis, parakeratosis, superficial lymphocytic infiltrate

    Negative stains
    • Fungal stains (PAS, GMS, etc.)
    Sample pathology report
    • Skin, biopsy:
      • Subacute spongiotic dermatitis, suggestive for / consistent with pityriasis rosea (see comment)
      • Comment: Biopsy examination demonstrates subacute spongiosis with overlying mounds of parakeratosis. There is a mild inflammatory perivascular infiltrate composed predominantly of lymphocytes with erythrocyte extravasates. No fungal elements were identified with PAS stain. Clinical image was reviewed. Overall, the histopathological features and clinical presentation are consistent with pityriasis rosea or a pityriasis rosea-like drug reaction but only in an appropriate clinical setting. (If clinical images and history are not present, the report may be more descriptive and include differential diagnosis of other spongiotic conditions.)
    Differential diagnosis
    • Pityriasis rosea-like eruptions:
      • Triggered by medications or vaccinations
      • Clinically, may be itchier, often display peripheral eosinophilia and tend to persist unless the drug is stopped
      • Histopathologically: often indistinguishable; may have interface and eosinophilia (JAAD Case Rep 2018;4:800)
    • Guttate / eruptive psoriasis:
      • May be very similar clinically and histopathologically
      • Herald patch is usually not present in psoriasis
      • Histopathologically: neutrophils present in the parakeratosis; dilated vessels in the papillary plates are more common (Australas J Dermatol 2020;61:e481)
    • Secondary syphilis:
      • Clinically, may be similar but no herald patch seen
      • Histopathologically: psoriasiform and lichenoid dermatitis with perivascular and interstitial infiltrate with lymphocytes, histocytes and plasma cells
      • Sometimes, the infiltrate may be granulomatous
      • Swollen, prominent endothelial cells are common (J Am Acad Dermatol 2020;82:156)
    • Other eczematous eruptions (contact dermatitis, nummular dermatitis, seborrheic dermatitis):
      • May be indistinguishable histopathologically and clinical correlation is required
      • Although a few eosinophils may be present in pityriasis rosea, large numbers are uncommon
    • Drug eruptions:
      • Variable clinical presentation
      • Histopathology is variable; combination of interface pattern with spongiosis and eosinophil rich infiltrates are common
    • Pityriasis lichenoides:
      • Clinically, may have similar features; however, tends to have a chronic course, no herald patch present
      • Histopathologically: an interface dermatitis and perivascular lymphocytic infiltrate is present with erythrocyte extravasates
      • Presence of neutrophils in the parakeratosis
    Board review style question #1

    What are the expected histopathological features of this condition?

    1. Spongiosis, mixed perivascular infiltrate with lymphocytes and neutrophils, neutrophils in the stratum corneum and positive PAS stain
    2. Regular psoriasiform acanthosis, perivascular lymphocytic infiltrate, parakeratosis with neutrophils and negative PAS stain
    3. Acute spongiosis, wedge shaped perivascular and interstitial infiltrate with numerous eosinophils
    4. Subacute spongiosis, perivascular superficial lymphocytic infiltrate with erythrocyte exocytosis and mounds of parakeratosis overlying spongiosis
    5. Interface dermatitis, perivascular superficial lymphocytic infiltrate with erythrocyte exocytosis and parakeratosis with neutrophils
    Board review style answer #1
    D. The condition demonstrated in the picture is pityriasis rosea. The patient has scaly, slightly oval patches localized along Langer lines. Classical histopathological features of this condition are subacute spongiosis, superficial perivascular lymphocytic infiltrate (often with erythrocyte exocytosis) and small mounds of parakeratosis overlying spongiotic areas. The description from choice A corresponds to tinea infection; B) psoriasis; C) arthropod reaction; E) pityriasis lichenoides.

    Comment Here

    Reference: Pityriasis rosea
    Board review style question #2
    What virus plays the most significant role in the pathogenesis of pityriasis rosea?

    1. EBV
    2. HHV6
    3. HIV
    4. HSV1
    5. HSV2
    Board review style answer #2
    B. HHV6. Although the exact etiology of pityriasis rosea is unknown, the reactivation of HHV6 and HHV7 was reported to play a role.

    Comment Here

    Reference: Pityriasis rosea

    Pityriasis rubra pilaris
    Definition / general
    • Pityriasis rubra pilaris (PRP) is a rare, chronic, idiopathic disorder characterized by an acute cutaneous eruption with potential to quickly progress to erythroderma
    Essential features
    • Includes acquired and hereditable forms
    • Clinical presentation is characterized by papulosquamous / erythematosquamous dermatosis
    • Affects all ages with variable clinical courses
    • Can be associated with HIV
    • CARD14 mutation has been detected in patients with familial PRP
    • Histopathologic findings are characteristic
    ICD coding
    • ICD-10: L44.0 - pityriasis rubra pilaris
    Epidemiology
    Pathophysiology
    Etiology
    Clinical features
    • Can be localized or diffuse / erythroderma
    • Localized lesions are often found in elbows, knees and thighs
    • Characteristics are diffuse and confluent, orange-red, well defined plaques with normal appearing skin islands (sparing), scaling follicular papules and scaling on palms and soles
    • Variable degree of pruritus is common
    • Nail changes are common
    • Scalp alopecia or transient eruptive seborrheic keratosis with no evidence of an internal malignancy may occur (J Cutan Pathol 2021;48:133, Actas Dermosifiliogr 2014;105:955, Clin Exp Dermatol 2004;29:554, Acta Derm Venereol 1988;68:443, J Eur Acad Dermatol Venereol 2009;23:217, J Am Acad Dermatol 1989;20:801)
    • Cutaneous malignancies (e.g., basal cell carcinoma, Merkel cell carcinoma, squamous cell carcinoma and cutaneous Kaposi sarcoma) may also be present at the time of PRP diagnosis or ensuing months (Australas J Dermatol 2002;43:48, J Am Acad Dermatol 1996;35:781)
    • 6 types of PRP (Br J Dermatol 1996;135:1008)
      • Type I (classic adult type)
        • 55% of PRP patients
        • Presents with classic findings of PRP and spreads caudally
        • Majority clear within 3 years
      • Type II (atypical adult type)
        • 5% of PRP patients
        • Eczematous dermatitis with ichthyosiform scale on legs and palmoplantar keratoderma, occasional alopecia
        • Chronic clinical course
      • Type III (classic juvenile type)
        • 10% of PRP patients
        • Resembles type I
        • Erythroderma is less common
        • Majority clear within 3 years
      • Type IV (circumscribed juvenile type)
        • 25% of PRP patients
        • Circumscribed, erythematous follicular papules and scaly plaques on elbows and knees
        • Variable clinical course
      • Type V (atypical juvenile type)
        • 5% of PRP patients
        • Eczematous dermatitis with ichthyosiform scaly plaques, palmoplantar keratoderma, follicular hyperkeratosis and scleroderma-like changes of hands and feet
        • Onset in first few years of life
        • Chronic clinical course
      • Type VI
        • < 1% of PRP patients
        • Associated with HIV infection
        • Resembles type I
        • Follicular occlusion tetrad of dissecting cellulitis of the scalp, pilonidal sinus / cyst, hidradenitis suppurativa and acne conglobate may be accompanied
    Diagnosis
    • Definitive diagnosis is based on clinicopathologic findings
      • Lesions most often appear on the face, chest and scalp with caudal migration interspersed with areas of unaffected skin
      • Small, scaly papules surround hair follicles
      • Palms or soles may develop an orange-red waxy keratoderma (Am J Clin Dermatol 2010;11:157)
    • Diagnosis is confirmed via biopsy in conjunction with clinical characteristics
      • Checkerboard pattern of parakeratosis and orthokeratosis is characteristic for PRP
      • Follicular plugging is common
      • Acantholysis and increased granular cell layer
      • No blood test is available; however, diagnosis of familial forms of PRP can be supported by Sanger sequencing for mutations of the CARD14 gene (Am J Clin Dermatol 2018;19:377)
    Case reports
    Treatment
    Clinical images

    Contributed by VisualDx
    Erythroderma

    Erythroderma

    Perifollicular erythema and scale

    Perifollicular erythema and scale

    Orange palmoplantar keratoderma

    Orange palmoplantar keratoderma

    Ectropion and erythema

    Ectropion and erythema

    Microscopic (histologic) description
    • Orthohyperkeratosis alternating with parakeratosis in both horizontal and vertical directions (checkerboard pattern)
    • Follicular plugging
    • Mild, irregular, vaguely psoriasiform epidermal acanthosis with broad rete ridges, thick suprapapillary plates and preserved granular layer
    • Focal acantholysis, dyskeratosis may be seen (Am J Dermatopathol 1999;21:491)
    • Mild superficial perivascular and perifollicular lymphocytic infiltrate; rare eosinophils may be intermixed
    • Dilated vessels within papillary dermis
    Microscopic (histologic) images

    Contributed by Chau M. Bui, M.D.
    Hyperkeratosis Hyperkeratosis

    Hyperkeratosis and acanthosis

    Checkerboard pattern Checkerboard pattern

    Checkerboard pattern

    Follicular plugging Follicular plugging

    Follicular plugging

    Sample pathology report
    • Skin biopsy, face:
      • Pityriasis rubra pilaris, classical adult type I (see comment)
      • Comment: The clinical history and images showing widespread erythema with areas of noninvolved skin present on the face, neck and trunk were reviewed. Histopathologic examination reveals hyperparakeratosis and orthokeratosis in a checkerboard pattern with hyperkeratotic plugging of a hair follicle. The clinical - pathologic findings are consistent with pityriasis rubra pilaris.
    Differential diagnosis
    • Psoriasis (especially guttate):
      • Increased papillary dermal vascular ectasia
      • More parakeratosis
      • More likely to have hypogranulosis and intracorneal neutrophils
    • Digitate dermatosis (small plaque parapsoriasis):
      • More lymphocyte exocytosis
      • Less prominent orthoparakeratotic hyperkeratosis
    • Sézary syndrome:
    • Phrynoderma (vitamin A deficiency):
      • Less psoriasiform epidermal hyperplasia
    • CARD14 associated papulosquamous eruption:
    Board review style question #1

    What is the most specific histologic finding of pityriasis rubra pilaris (PRP)?

    1. Checkerboard hyperkeratosis pattern
    2. Hyperkeratosis and perivascular lymphocytic infiltrate
    3. Psoriasiform epidermal hyperplasia
    4. Sparse superficial inflammation
    Board review style answer #1
    A. Checkerboard hyperkeratosis pattern. Alternating vertical and horizontal orthoparakeratosis and parakeratosis (checkerboard pattern) is characteristic for PRP. Answer B is incorrect because perivascular lymphocytic infiltrate with hyperkeratosis is more commonly seen in cases of psoriasis. Answer D is incorrect because sparse superficial inflammation is more characteristic of eczema. Answer C is incorrect because psoriasiform patterns can be found in a wide variety of skin disorders and is not specific for PRP.

    Comment Here

    Reference: Pityriasis rubra pilaris
    Board review style question #2
    What is the mutation associated with pityriasis rubra pilaris (PRP)?

    1. BRAF
    2. CARD14
    3. KRAS
    4. TERT
    Board review style answer #2
    B. CARD14. Mutations in CARD14 have been identified in the familial form of PRP. Answers A and D are incorrect because these mutations are found more commonly in melanomas but not identified in PRP. Answer C is incorrect because KRAS mutations are found in a wide variety of cancers but have not been identified in PRP to date.

    Comment Here

    Reference: Pityriasis rubra pilaris

    Polymorphous light eruption
    Definition / general
    • The most common photodermatosis
    • Presents in young adults as recurrent erythematosus papules, vesicles or plaques on face, chest, upper back and extremities
    • Onset of lesions 18 - 24 hours after light exposure, usually in spring / summer
    • Associated with thyroid disease or lupus in some patients
    • Juvenile spring eruption appears to be either a form of polymorphous light eruption or a closely related disorder
    • Use photo testing to establish diagnosis
    Clinical images

    Images hosted on other servers:

    Polymorphic light eruption on arm

    Microscopic (histologic) description
    • Perivascular lymphohistiocytic infiltrate in superficial and sometimes deep dermis
    • Papillary dermal edema; epidermal acanthosis, spongiosis, occasional dyskeratotic cells and lymphocytic exocytosis
    • Basal cell vacuolation is mild
    • Periadnexal involvement may be seen; variable increase in eosinophils and neutrophils
    Positive stains
    • Direct immunofluorescence shows C3, IgM and IgG at basal layer of dermoepidermal junction
    Differential diagnosis
    • Actinic reticuloid: dense cellular and interstitial infiltrate of papillary and reticular dermis that may extend to subcutaneous fat; composed of variable lymphocytes, histiocytes, eosinophils and plasma cells; multinucleated giant cells are conspicuous; also large atypical hyperchromatic cerebriform lymphoid cells and blast forms
    • Gyrate erythema such as lymphocytic infiltration of Jessner
    • Reticulae erythematous mucinosis: has dermal mucin, not present in polymorphic light eruption
    • SLE
    Additional references

    Porphyria cutanea tarda
    Definition / general
    Clinical images

    Images hosted on other servers:

    Porphyria cutanea tarda

    Microscopic (histologic) description
    • Dermal papillae protrudes into bulla with festooned pattern; roof of blister has eosinophilic, PAS+ and diastase resistant linear globules
    Microscopic (histologic) images

    Images hosted on other servers:

    Porphyria cutanea tarda associated with HIV;
    Subepidermal bulla formation has resulted
    in loss of the epidermis (note the
    rigid papillary dermal capillary walls)


    Postinflammatory pigment alterations (pending)

    Pretibial myxedema
    Definition / general
    • Thick, dry, and waxy skin due to accumulation of mucopolysaccharides in dermis; similar to that in orbital tissues, caused by excess TSH secretion or hypothyroidism
    • Often in pretibial skin
    • Localized or generalized
    • Elephantiasis: extreme disease
    Clinical features
    • Pink, yellow and waxy plaques and nodules on the anterolateral aspect of leg
    • Indurations with prominence of follicles giving rise to peau d’orange appearance and secondary hypertrichosis is occasionally marked
    • Asymptomatic or mildly pruritic
    Case reports
    Treatment
    • Self limiting
    Microscopic (histologic) description
    • Epidermis is hyperkeratotic with follicular plugging and may be papillomatous and acanthotic
    • Dermis shows separation of collagen bundles by large quantities of mucin
    • Satellite fibroblasts are evident, but no increase in numbers except in elephantiasis form
    Positive stains
    Negative stains
    • Direct immunofluorescence is usually negative, although granular deposits of IgM have been identified
    Additional references

    Pseudoxanthoma elasticum
    Definition / general
    • Genetic disorder with ocular, vascular and skin lesions due to mutations of ABCC / MRP6 gene at 16p13.1, coding for ATP binding protein, and causing degeneration of elastic fibers
    • Variable penetrance, even within the same family
    • Autosomal dominant or recessive, usually women
    • Skin: yellow streaks and plaques, particularly in creases of axillae, groin and neck; appear before puberty
    • Eyes: angioid streaks in retina, causing hemorrhage and visual loss
    • Vessels: degenerative changes in arteries causing mineralization, occlusion or rupture
    Case reports
    Microscopic (histologic) description
    • Affected elastic fibers are basophilic and irregular, appearing as widely dispersed granular material amidst normal collagen fibers; may have a bizarre appearance reminiscent of a bishop's crook
    • Abnormal fibers are bright pink with an altered architecture, as they have lost their normal interlacing pattern but may be faintly basophilic due to calcium
    • Increased dermal mucin may be evident
    • Vascular involvement consists of fragmentation of the internal and external elastic laminae, accompanied by intimal thickening, resulting in weakness of the vessel wall and a tendency towards rupture and aneurysm formation
    • Eye: calcifications of Bruch’s membrane, which separates the choroid from the pigment epithelium of the retina
    Microscopic (histologic) images

    Case #332



    Verhoeff Van Gieson stain for elastic fibers

    Positive stains
    • Verhoeff Van Gieson stain for elastic fibers shows marked degeneration of fibers with a short and curled appearance
    • Von Kossa stain reveals calcium deposition
    • Fibers also stain positive with Alizarin red calcium stain
    Electron microscopy description
    • Fragmented and polymorphic elastic fibers in reticular dermis
    • Fibers may contain finely disbursed, mineral crystals
    • Similar changes in carriers
    Differential diagnosis
    • Cutis laxa: loss of elastic tissue in the papillary and reticular dermis; fibers are shortened, tapered and degenerate (Arch Dermatol 1965;92:373)
    • Late onset focal dermal elastosis: normal elastic fibers in the mid and deep dermis, with no evidence of calcification (Am J Dermatopathol 1999;21:381)
    • Manifestations of pseudoxanthoma elasticum, including cutaneous lesions, angioid streaks and vascular calcification, have been identified in many hemolytic conditions including beta thalassemia, sickle cell disease and hereditary spherocytosis (Br J Haematol 2003;122:852)
    • Penicillamine therapy: similar skin lesions but also produces elastosis perforans, serpingiosa and collagen defects
    • Pseudoxanthoma elasticum-like papillary dermal elastolysis occurs in elderly females, systemic lesions are absent; partial or complete elastolysis in the papillary dermis but calcification is typically absent with normal collagen fibers (Br J Dermatol 1998;139:141)
    • Saltpeter: similar dermal changes histologically and ultrastructurally
    Additional references

    Psoriasis
    Definition / general
    • Psoriasis is a chronic relapsing condition affecting skin, nails and joints that most commonly presents with well demarcated erythematous papules and plaques with silvery white scales
    Essential features
    • Most common clinical presentation: well demarcated erythematous plaques with silvery white scales
    • Complex pathogenesis including genetic, autoimmune and environmental factors
    • Common histopathological features: regular acanthosis, lymphocytic perivascular inflammation, parakeratosis with neutrophilic collections, spongiosiform pustules with neutrophils, dilated vessels in the papillary dermis, alternating areas of hyper and hypogranulosis and thinning of the suprapapillary plates
    • Several different clinical and histopathologic variants
    ICD coding
    • ICD-10:
      • L40 - Psoriasis
      • L40.0 - Psoriasis vulgaris
      • L40.1 - Generalized pustular psoriasis
      • L40.2 - Acrodermatitis continua
      • L40.3 - Pustulosis palmaris et plantaris
      • L40.4 - Guttate psoriasis
      • L40.5 - Arthropathic psoriasis
      • L40.8 - Other psoriasis
      • L40.9 - Psoriasis, unspecified
    Epidemiology
    • Adults:
      • Prevalence of 0.14 - 1.99%, higher in Western Europe, central Europe, North America and high income areas of South America (BMJ 2020;369:m1590)
      • Incidence ranges from 30.3 to 321.0 per 100,000 person years (BMJ 2020;369:m1590)
    • Children:
    • Peak incidence is at 20 - 30 years and at 50 - 60 years
    • Psoriatic arthritis occurs in 5 - 30% of patients with psoriasis
    • Associated with cardiovascular complications (increased rate of myocardial infarction, hypertension, hyperlipidemia, obesity, hyperlipidemia), depression, inflammatory bowel disease, diabetes, fatty liver disease and chronic kidney disease (J Am Heart Assoc 2020;9:e016956, J Am Acad Dermatol 2017;77:287, J Am Acad Dermatol 2017;76:393)
    Sites
    • Skin, nails, musculoskeletal system and rarely oral mucosa
    Pathophysiology
    • Complex pathogenesis, including genetic, autoimmune and environmental factors (J Allergy Clin Immunol 2017;140:645, Int J Mol Sci 2019;20:1475, J Dermatol 2018;45:264, J Immunol 2018;201:1605)
    • T cell (Th1) activation and release of cytokines (TNFα, IFγ, IL12, etc.)
    • Dendritic cells produce IL23, which activates Th17 cells, which then play a key role in the development of psoriasis; IL23 also induces macrophages to secrete TNFα
    • Th17 cells produce IL17, which promotes the production of other proinflammatory molecules (Int J Mol Sci 2017;18:2684)
    • IL22, produced by Th17 and Th22 cells, plays a role in stimulating the proliferation of keratinocytes and causing the development of psoriatic plaques
    • Overexpression of antimicrobial peptides (β-defensins, cathelicidin and S100) stimulate inflammation
    • NFκB induces expression of keratins 6 and 16, leading to acanthosis and increased turnover of keratinocytes
    Etiology
    • Genetic:
      • PSOR1 locus map on chromosome 6 is the most important genetic region responsible for the disease
      • HLA-Cw6 is associated with early onset of psoriasis
      • Additional associations include HLA-B27 (pustular psoriasis, psoriatic arthritis), HLA-B13, HLA-B17 and HLA-DR7
    • Triggering factors: numerous, most common are infections, HIV, hypocalcemia, stress, alcohol consumption, obesity and smoking (Front Immunol 2019;10:1807, Psoriasis (Auckl) 2016;6:65)
    • Drugs: antimalarials, lithium, beta blockers, terbinafine and imiquimod (Psoriasis (Auckl) 2017;7:87)
    • Paradoxically, TNF inhibitors, which are used to treat psoriasis, may also trigger psoriasis when used for other conditions
    • Streptococcus infections may trigger acute guttate psoriasis
    • Systemic steroid withdrawal may trigger pustular psoriasis, von Zumbusch type
    Clinical features
    • Variants:
      • Plaque psoriasis (most common): sharply demarcated erythematous plaques with silvery scales, commonly present on extensor surfaces (elbows, knees, etc.)
      • Guttate psoriasis: small scattered erythematous papules with silvery scales; often triggered by infections
      • Erythrodermic psoriasis: confluent plaques affecting nearly all of the skin but often sparing the central face; lymphadenopathy, electrolyte and thermoregulation abnormalities may also be present
      • Pustular psoriasis: presence of superficial pustules; generalized (von Zumbusch type), annular, palmar-plantar, localized and exanthemic variants are described
      • Acrodermatitis continua of Hallopeau: pustules on the distal parts of the fingers and toes, as well as on the nail bed, which may cause shedding of the nail plate
      • Scalp psoriasis: well demarcated plaques on the retroauricular areas and posterior upper neck
      • Sebopsoriasis lesions on the scalp, face, upper back and chest mimicking seborrheic dermatitis
      • Linear psoriasis: development of psoriasis lesions along Blaschko lines
      • Annular / figurate psoriasis: annular lesions of psoriasis with central clearing
      • Flexural (inverse) psoriasis: plaques affecting intertriginous folds, scaling may be less prominent
      • Napkin psoriasis: may be seen in napkin (diaper) area in newborns
      • Nail psoriasis: pits, splinter hemorrhages, oil drop sign (translucent yellow discoloration of the nail caused by parakeratosis of the nail bed), salmon patches and leukonychia are common on the nail plate
      • Follicular psoriasis: small keratotic follicular papules with scaling (Dermatol Online J 2020;26:3)
      • Psoriatic arthritis: inflammation of distal interphalangeal joints and proximal interphalangeal joints of the feet and hands
      • Oral psoriasis: red patches, geographic tongue, ulcerations, gingivitis or pustules
    • Signs:
      • Koebner phenomenon: new psoriatic papules at sites of skin trauma
      • Auspitz sign: pinpoint bleeding when scales are removed
      • Woronoff ring: an area of blanching surrounding resolving psoriatic plaques
    Diagnosis
    • Clinical examination
    • Skin biopsy
    • Xray if joints are affected
    • Complete blood count, comprehensive metabolic panel (kidney and hepatic function), pregnancy, tuberculosis, HIV and hepatitis tests may be considered if systemic treatment is planned
    Laboratory
    • Usually not needed to establish the diagnosis; should be performed if systemic treatment is desired
    • Rheumatoid factor is negative
    Radiology description
    • 5 main types of psoriatic arthritis:
      • Symmetrical peripheral polyarthritis resembling RA
      • Asymmetrical mono or oligoarthritis, usually involving the knee and small peripheral joints
      • Axial spondyloarthropathy
      • Predominant DIP joint involvement
      • Arthritis mutilans
    • Dactylitis, inflammatory changes in the distal interphalangeal joints and proximal interphalangeal joints, finger deformities (pencil in cup)
    • Increase of radiographic density of the distal phalanx due to extra bone formation (ivory phalanx)
    • Enthesitis
    • Abnormalities of the sacroiliac joints (J Ultrason 2016;16:65, J Ultrason 2016;16:163)
    Radiology images

    Images hosted on other servers:

    Polyarticular psoriatic arthritis

    Sacroiliac involvement in psoriatic arthritis

    Onychopathy and periostitis at base of phalanx

    Prognostic factors
    • Chronic disease with frequent relapses; early systemic treatment to achieve control and prevent systemic comorbidities
    Case reports
    Treatment
    • Topical treatment: corticosteroids (most common); vitamin D analogs (calcipotriene and calcitriol); topical retinoids; immunomodulators (pimecrolimus and tacrolimus); coal tar; keratolytics (salicylic acid and urea)
    • Intralesional steroids to resistant plaques
    • Light therapy (UVB, PUVA)
    • Systemic therapies (methotrexate, cyclosporine and mycophenolate mofetil, JAK inhibitors)
    • Biological therapies:
      • TNF inhibitors: infliximab, etanercept, adalimumab
      • IL12 / 23 inhibitor: ustekinumab
      • IL17 inhibitors: ixekinumab, brodalumab, secukinumab
      • IL23 inhibitors: guselkimab, tildrakizumb, risankizumab
      • Phosphodiesterase-4 inhibitors: apremilast
      • Reference: Int J Mol Sci 2019;20:1475
    Clinical images

    Contributed by Viktoryia Kazlouskaya, M.D., Ph.D., Mary-Katherine Collins, M.D. and Natalia Zhovta, M.D.

    Verrucous psoriasis on the knee

    Palmar psoriasis with well demarcated plaque

    Localized pustular psoriasis

    Plaque psoriasis, wrists

    Plaque psoriasis, elbows


    Intertriginous psoriasis

    Pustular psoriasis

    Erythrodermic psoriasis

    Guttate psoriasis



    Contributed by Mark R. Wick, M.D.

    Breast skin

    Gross description
    Gross images
    Microscopic (histologic) description
    • Regular acanthosis, often with elongated rete (psoriasiform)
    • Alternating zones of hypo and hypergranulosis in the epidermis
    • Thinning of the suprapapillary plates
    • Areas of parakeratosis in the stratum corneum with mounds of neutrophils (Munro microabscesses)
    • Perivascular, predominantly lymphocytic infiltrate in the upper and middle portions of the dermis; few neutrophils or eosinophils may be seen
    • Collections of neutrophils in the spinosum (spongiosiform pustules of Kogoj)
    • Dilated and tortuous vessels in the dermal papillae
    • Focal spongiosis may be present in the evolving lesions, intertriginous, acral and scalp variants
    • Subcorneal pustules in the pustular forms of psoriasis
    Microscopic (histologic) images

    Contributed by Viktoryia Kazlouskaya, M.D., Ph.D.

    Acanthosis with subcorneal pustule

    Subcorneal pustule

    Parakeratosis with neutrophilic collections

    Acanthosis and verrucous changes

    Dilated dermal capillaries


    Subcorneal pustules

    Regular acanthosis

    Acanthosis, parakeratosis with neutrophils

    Acral subcorneal pustule



    Contributed by Hillary Rose Elwood, M.D.

    Classic features of psoriasiform hyperplasia, confluent parakeratosis, loss of granular layer and dilated capillaries with thinned suprapapillary plate



    Contributed by Angel Fernandez-Flores, M.D., Ph.D.

    Regular psoriasiform acanthosis

    Dilated capillaries in the dermal papillae

    Neutrophils in stratum corneum

    Hypogranulosis and neutrophils in stratum corneum

    Immunofluorescence description
    • Direct and indirect immunofluorescence are negative
    Negative stains
    • Fungal stains (PAS, GMS, etc.)
    Videos

    Psoriasis: clinical presentation, histopathology and differential diagnosis

    Sample pathology report
    • Skin, biopsy:
      • Psoriasiform dermatosis, suggestive for / consistent with psoriasis (see comment)
      • Comment: Biopsy examination demonstrates a regular psoriasiform acanthosis of the epidermis. There is a mild inflammatory perivascular and interstitial infiltrate composed predominantly of lymphocytes. Thinning of suprapapillary plates, widened vessels in the superficial dermis and collections of neutrophils in the stratum corneum were noted. No fungal elements were identified with PAS stain.
      • Overall, the histopathological features are consistent with psoriasis but only in appropriate clinical setting; psoriasiform drug eruptions may be indistinguishable.
    Differential diagnosis
    • Subacute spongiotic processes (atopic dermatitis, nummular dermatitis, contact dermatitis, dyshidrotic eczema):
      • More prominent spongiosis
      • Sometimes abundant eosinophils in the infiltrate
    • Seborrheic dermatitis:
      • Spongiosis, mounds of parakeratosis at the follicular ostia
      • In some instances, differentiation is not possible
    • Pityriasis rosea:
      • Foci of spongiosis with small mounds of parakeratosis in the overlying stratum corneum
      • Herald patch may have acanthosis
    • Psoriasiform drug eruption:
      • May be indistinguishable, greater number of eosinophils may be present
    • Pityriasis rubra pilaris (Am J Dermatopathol 2019;41:37):
      • Alternating areas of ortho and parakeratosis vertically and horizontally in the stratum corneum (checkerboard pattern)
      • Acantholysis may be present
    • Lichen simplex chronicus:
      • Presence of vertical collagen bundles in the papillary dermis
      • No significant parakeratosis and collections of neutrophils in the stratun corneum
    • Inflammatory linear verrucous epidermal nevus (ILVEN):
      • Vertically alternating areas of ortho and parakeratosis in the stratum corneum with orthokeratotic areas overlying epidermal hypergranulosis and parakeratosis overlying hypogranulosis
      • No neutrophilic collections in the stratum corneum nor the epidermis
    • Porokeratosis ptychotropica:
      • Presence of cornoid lamellae and dyskeratotic cells
    • Fungal infections:
      • May have similar findings, stains (PAS or GMS should be performed when in question)
    • Nutritional dermatosis:
      • May have variable findings
      • In classical cases, a pallor of the superficial epidermis is seen
    • Clear cell acanthoma:
      • Usually a solitary lesion
      • Sharp demarcation between normal epidermis and clear cell areas
    • Pustular psoriasis:
    • Mycosis fungoides:
      • Epidermotropism, folliculotropism or syringotropism (tropism to eccrine glands)
      • Lymphocytes display atypical features (irregular contours, hyperchromasia, sometimes enlarged, etc.)
      • Presence of neutrophils in the stratum corneum is rare
      • Presence of thickened collagen bundles in the superficial dermis
    • Sezary syndrome:
      • May be indistinguishable unless clear history and histopathological features of psoriasis are present
    Board review style question #1

    What are the expected histopathological features of this condition?

    1. Regular psoriasiform acanthosis with subcorneal neutrophilic collections, PAS stain negative
    2. Regular psoriasiform acanthosis with subcorneal neutrophilic collections, PAS stain positive
    3. Irregular psoriasiform acanthosis with spongiosis
    4. Perifollicular pustules
    5. Intraepidermal bullae filled with neutrophils
    Board review style answer #1
    A. The image demonstrates pustular psoriasis. Histopathological features of pustular psoriasis include regular psoriasiform acanthosis with subcorneal neutrophilic collections. PAS stain is negative. Positive PAS staining is seen in fungal infections (answer B). Spongiosis, although may be focally seen in psoriasis, is more typical for eczematous conditions (answer C). Pustules in pustular psoriasis are not perifollicular (answer D). Intraepidermal bullae are not seen in psoriasis (answer E).

    Comment Here

    Reference: Psoriasis
    Board review style question #2
    A patient was treated for onychomycosis with a systemic medication and developed a rash. Biopsy of the rash demonstrated regular psoriasiform acanthosis, parakeratosis with neutrophils in the stratum corneum, thinning of parapapillary plated with dilated vessels in the superficial dermis. What was the most likely medication causing this condition?

    1. Griseofulvin
    2. Voriconazole
    3. Terbinafine
    4. Itraconazole
    5. Fluconazole
    Board review style answer #2
    C. The vignette describes psoriasis that started after administration of an oral antifungal agent. Terbinafine is known to exacerbate psoriasis. This side effect is not typical for griseofulvin (answer A) and azoles (voriconazole, itraconazole or fluconazole - answers B, D and E).

    Comment Here

    Reference: Psoriasis
    Board review style question #3
    What histopathological feature is helpful in differentiating psoriasis versus inflammatory linear verrucous epidermal nevus?

    1. Presence of spongiosis
    2. Presence of lichenoid infiltrate
    3. Presence of eosinophils in the inflammatory infiltrate
    4. Erythrocyte extravasates
    5. Alternating orthokeratosis with parakeratosis
    Board review style answer #3
    E. Alternating orthokeratosis with parakeratosis is the most specific sign that helps to differentiate ILVEN with psoriasis. Focal areas of spongiosis and erythrocyte extravasates may be seen in both conditions but these features are not specific. Eosinophils and lichenoid infiltrate are not typical for ILVEN and psoriasis.

    Comment Here

    Reference: Psoriasis

    Pyoderma gangrenosum
    Definition / general
    • Noninfectious neutrophilic dermatosis with painful nodules / pustules that progress to necrotic ulcers with a characteristic violaceous edge
    • Pyoderma gangrenosum is a diagnosis of exclusion based on clinical and histological findings
    Essential features
    • Painful ulcers
    • Up to 50% associated with systemic disease, particularly inflammatory bowel disease
    • Diagnosis of exclusion
    • Early lesions show neutrophilic folliculitis / perfolliculitis with dermal abscess
    • Later lesions are ulcerated with mixed dermal inflammation and neutrophilic abscess that undermines the ulcer edge
    ICD coding
    • ICD-10: L88 - pyoderma gangrenosum
    Epidemiology
    Sites
    • Any site; most common lower legs, peristomal
    Pathophysiology
    Etiology
    • Unknown etiology; 25 - 50% idiopathic
    • Up to 50% associated with systemic disease:
      • Inflammatory bowel disease (e.g. ulcerative colitis)
      • Arthritis
      • Hematological malignancy or monoclonal gammopathies
    • Other associated conditions: rheumatoid arthritis, seronegative arthritis, pregnancy, viral hepatitis, HIV, granulomatosis with polyangiitis
    • Reference: Clin Med (Lond) 2019;19:224
    Clinical features
    • 4 major subtypes:
      • Ulcerative / classic (most common): erythematous lesion that rapidly progresses to ulcer with undermined edge, violaceous border and purulent base
        • Painful, with or without fever
        • Possible pathergy (precipitated by minor trauma)
      • Bullous: often associated with a hematological malignancy (Arch Dermatol 1972;106:901)
      • Pustular: often associated with inflammatory bowel disease (Clin Exp Dermatol 2003;28:600)
      • Vegetative / superficial granulomatous: mildly painful, verrucous quality, often without systemic condition (J Am Acad Dermatol 1988;18:511)
    Diagnosis
    • Nonspecific clinical and histologic findings; a diagnosis of exclusion
    • Proposed diagnostic criteria by Delphi Consensus of International Experts (JAMA Dermatol 2018;154:461):
      • Need major criteria + ≥ 4 minor criteria
      • Major criteria: neutrophilic infiltrate on biopsy of ulcer edge
      • Minor criteria:
        • Exclude infection
        • Pathergy
        • History of inflammatory bowel disease or inflammatory arthritis
        • History of papule, pustule, vesicle within 4 days of ulceration
        • Peripheral erythema, undermining border and tenderness
        • Multiple ulceration, at least 1 on anterior lower leg
        • Healed ulcer site with wrinkled paper scar
        • Decreased ulcer size upon initiating immunosuppressant
    Prognostic factors
    • > 50% achieve complete wound healing in 1 year (Australas J Dermatol 2011;52:218)
    • Unfavorable prognostic factors: male, old age onset, bullous pyoderma gangrenosum associated with hematologic malignancy
    • Favorable prognostic factors: no underlying disease
    Case reports
    Treatment
    • General measures: control underlying disease, manage pain, wound care, avoid trauma
    • For limited disease: topical therapy with corticosteroid and calcineurin inhibitors
    • For extensive or advanced disease: first line therapy with systemic glucocorticoids or cyclosporine; second line / adjunctive therapy with tumor necrosis factor alpha (TNF alpha) inhibitors and anti-interleukin therapy
    • Reference: Am J Clin Dermatol 2017;18:355
    Clinical images

    Contributed by Michi Shinohara, M.D.
    Leg ulcer

    Leg ulcer

    Microscopic (histologic) description
    • Variable, dependent on the lesion age and site
    • Early lesion:
      • Neutrophilic folliculitis and perifolliculitis
      • Intradermal neutrophilic abscess formation
    • Later lesions:
      • Epidermal ulceration
      • Superficial dermal necrosis
      • Mixed inflammation with undermining at ulcer edge (characteristic) and abscess
      • Occasional giant cells (associated with inflammatory bowel disease)
      • Advancing edge often has subepidermal edema
      • Leukocytoclastic vasculitis and lymphocytic vasculitis may be present (J Cutan Pathol 2003;30:97)
      • Acanthosis in perilesional zone
    • Variants:
      • Ulcerative / classic variant: central neutrophilic abscess and peripheral angiocentric lymphocytic infiltrates
      • Bullous variant: subepidermal bullae and dermal neutrophilic infiltrates
      • Pustular variant: subcorneal pustules, subepidermal edema, dense dermal neutrophilic infiltrates
      • Vegetative / superficial granulomatous variant: reactive epidermal proliferation, dermal abscess, sinus tracts and palisading granulomas
    Microscopic (histologic) images

    Contributed by Michi Shinohara, M.D.
    Early lesion

    Early lesion

    Late lesion Late lesion

    Late lesion

    Videos

    Pyoderma gangrenosum versus nonspecific chronic ulcer changes

    Sample pathology report
    • Skin, leg, punch biopsy:
      • Ulcer with mixed dermal inflammation (see comment)
      • Comment: The findings are nonspecific but could be compatible with pyoderma gangrenosum. Pyoderma gangrenosum is a diagnosis of exclusion and other causes of ulceration, including infection, must be ruled out.
    Differential diagnosis
    • Sweet syndrome:
      • Not typically associated with ulcer, prominent karyorrhexis and deeper / more extensive inflammation
    • Infection or necrotizing fasciitis:
      • Involves deeper subcutaneous tissue; presence of microorganisms
    • Arthropod bite reaction:
      • Numerous eosinophils
    • Venous stasis ulceration:
      • Can show identical findings, usually without prominent neutrophils
      Other causes of ulcer, including factitial:
      • Can show identical findings
    Board review style question #1

    Which of the following systemic diseases is most often associated with pyoderma gangrenosum?

    1. Diabetes mellitus
    2. Peripheral vascular disease
    3. Systemic lupus erythematosus
    4. Ulcerative colitis
    Board review style answer #1
    D. Ulcerative colitis

    Comment Here

    Reference: Pyoderma gangrenosum
    Board review style question #2
    Which of the following is a feature of early pyoderma gangrenosum biopsies?

    1. Eosinophilic pustulosis
    2. Epidermal ulceration
    3. Neutrophilic folliculitis
    4. Subepidermal bullae
    Board review style answer #2
    C. Neutrophilic folliculitis

    Comment Here

    Reference: Pyoderma gangrenosum

    Reactive granulomatous dermatitis
    Definition / general
    • Dermal reaction pattern associated with autoimmune processes more often than infection or malignancy
    • Spectrum of clinicopathologic features that vary with lesion age
    • Comprises lesions consistent with palisaded neutrophilic granulomatous dermatitis and interstitial granulomatous dermatitis
    Essential features
    • Dermal reaction pattern associated with a systemic process, most commonly an autoimmune disease
    • Variable clinical presentations including crusted pink-brown papules and plaques on extensor extremities, umbilicated papules on acral site, annular plaques on trunk and indurated linear cords on flanks
    • Variable pathologic features including degenerated collagen surrounded by palisades of histiocytes, neutrophils and eosinophils, necrobiotic rosettes and leukocytoclastic vasculitis
    • Commonly resolves with management of associated systemic disease
    Terminology
    • Interstitial granulomatous dermatitis, palisaded neutrophilic granulomatous dermatitis, Winkelmann granuloma, superficial ulcerating rheumatoid necrobiosis
    ICD coding
    • ICD-10: L92.9 - granulomatous disorder of the skin and subcutaneous tissue, unspecified
    Epidemiology
    • F > M; adults > children
    • Associated with autoimmune disorders > malignancies or infections (Hum Pathol 2014;45:598, JAAD Case Rep 2017;3:425)
    • Palisaded neutrophilic granulomatous dermatitis pattern is most commonly associated with rheumatoid arthritis, systemic lupus erythematosus, granulomatosis with polyangiitis and eosinophilic granulomatosis with polyangiitis
    • Interstitial granulomatous dermatitis pattern is most commonly associated with rheumatoid arthritis (RA), seronegative inflammatory arthritis and autoimmune thyroiditis (Arch Dermatol 1994;130:1278)
    • Less common associations include malignant processes (e.g., myelodysplastic syndrome and myeloid leukemia), infections (e.g., human immunodeficiency virus [HIV], Epstein-Barr virus [EBV], parvovirus B19 and Streptococcus), autoimmune diseases (including systemic sclerosis and inflammatory bowel disease) and metabolic diseases (including diabetes) (Dermatol Clin 2015;33:373, Clin Case Rep 2019;7:695)
    Sites
    • Interstitial granulomatous dermatitis: lateral upper trunk, flanks, buttocks and proximal limbs
    • Palisaded neutrophilic granulomatous dermatitis: extensor surfaces of extremities
    Pathophysiology
    • Poorly understood but thought to be caused by immune complex deposition in small blood vessels → vasculitis, local ischemia and collagen degeneration → lymphohistiocytic response (Am J Clin Dermatol 2010;11:171)
    Clinical features
    • Palisaded neutrophilic granulomatous dermatitis: red crusted or umbilicated papules, plaques and nodules symmetrically distributed and typically on extensor surfaces
    • Interstitial granulomatous dermatitis: firm linear cords or annular erythematous plaques on lateral upper trunk, buttocks and proximal limbs
    • Usually asymptomatic but may be pruritic or tender
    • Significant overlap between palisaded neutrophilic granulomatous dermatitis and interstitial granulomatous dermatitis; lesions consistent with both entities may present in the same patient
    • Reference: Dermatol Clin 2015;33:373
    Diagnosis
    Prognostic factors
    Case reports
    Treatment
    • Management is focused on treatment of the underlying, triggering disorder
    • No standardized lesion directed therapy
    • Reports of improvement with topical corticosteroids, topical calcineurin inhibitors, systemic corticosteroids, tetracyclines, hydroxychloroquine, cyclosporin, etanercept and phototherapy (J Eur Acad Dermatol Venereol 2021;35:988)
    Clinical images

    Contributed by Christine J. Ko, M.D.
    Upper extremity reactive granulomatous dermatitis

    Upper extremity reactive granulomatous dermatitis

    Symmetric upper extremity reactive granulomatous dermatitis

    Symmetric upper
    extremity reactive
    granulomatous
    dermatitis



    Images hosted on other servers:
    Plaque on thigh

    Plaque on thigh

    Microscopic (histologic) description
    • Inflammatory infiltrate is variable among lesions and as the lesions age (J Eur Acad Dermatol Venereol 2021;35:988, JAAD Case Rep 2017;3:425, Am J Dermatopathol 2015;37:614, Lupus 2016;25:209)
      • May be consistent with classical palisaded neutrophilic granulomatous dermatitis or interstitial granulomatous dermatitis (see below) or may be neutrophil predominant, eosinophil predominant or histiocyte predominant
    • Interstitial granulomatous dermatitis: small rosettes of necrobiotic collagen surrounded by histiocytes and lymphocytes (floating sign)
      • Absent leukocytoclastic vasculitis and minimal or absent mucin deposition
      • Scarce neutrophils and eosinophils
    • Palisaded neutrophilic granulomatous dermatitis: large foci of necrobiotic collagen surrounded by palisades of histiocytes, lymphocytes and prominent neutrophils
      • Leukocytoclastic vasculitis may be present in early lesions
      • Mucin deposition is minimal or absent
    Microscopic (histologic) images

    Contributed by Christine J. Ko, M.D. and Kiran Motaparthi, M.D.

    Interstitial histiocytes and neutrophils

    Palisading granulomas with neutrophils

    Virtual slides

    Images hosted on other servers:

    Necrobiotic rosettes with histiocytes and neutrophils

    Sample pathology report
    • Upper trunk, punch biopsy:
      • Reactive granulomatous dermatitis, eosinophil rich (see comment)
      • Comment: There are palisades of histiocytes and eosinophils surrounding degenerated collagen in the superficial to mid-dermis. Special stains for bacteria, fungi and mycobacteria are negative. No vacuolar interface dermatitis is present to suggest interstitial granulomatous drug reaction and no mucin deposition is seen to suggest granuloma annulare.
    Differential diagnosis
    • Interstitial granulomatous drug reaction:
      • Firm papules, plaques and nodules that arise in response to exposure to medications, including antihypertensives, statins and tumor necrosis factor alpha inhibitors
      • Presence of vacuolar interface dermatitis helps to differentiate from reactive granulomatous dermatitis
    • Granuloma annulare:
      • Firm, red-brown smooth or umbilicated papules, patches and subcutaneous nodules that arise most commonly on the extremities but may also be generalized
      • Perforating variant may resemble umbilicated lesions of palisaded neutrophilic granulomatous dermatitis
      • No clear association with autoimmune disease
      • Abundant mucin and scarce neutrophils help to differentiate from reactive granulomatous dermatitis
    • Infectious granuloma:
      • Suppurative, sarcoidal or diffuse (interstitial) granulomas with positive infectious stains
    • Rheumatoid nodules:
      • Large nodules overlying extensor surfaces of joints in patients with rheumatoid arthritis
      • Palisading granulomas with fibrinoid necrosis, prominent neutrophilic infiltrate and absent mucin deposition in deep dermis or subcutaneous fat
    • Leukemia cutis:
      • Reactive granulomatous dermatitis has been described in the setting of myelodysplastic syndrome and myeloid leukemia
      • Distinguishing myeloid precursors from histiocytes enables differentiation
    • Leukocytoclastic vasculitis:
      • Purpuric macules and papules, petechiae, urticarial wheals and targetoid papules that favor dependent skin, are seen in small vessel vasculitis
      • Small or medium sized vessels with fibrinoid necrosis, erythrocyte extravasation and neutrophilic perivascular infiltrate with karyorrhexis
      • Lesions of palisaded neutrophilic granulomatous dermatitis may demonstrate leukocytoclastic vasculitis
      • Reactive granulomatous dermatitis may occur in the setting of systemic vasculitides including granulomatosis with polyangiitis and eosinophilic granulomatosis with polyangiitis
    • Acute febrile neutrophilic dermatosis (Sweet syndrome):
      • Pink, tender edematous papules and plaques that favor the upper trunk; often with concomitant fever and variable systemic involvement
      • Papillary dermal edema, dense neutrophilic infiltrate and karyorrhexis without vasculitis
    Board review style question #1

    A 55 year old woman with systemic lupus erythematosus presents with a 2 month history of pink-brown, crusted papules on the extensor surfaces of her bilateral arms. Punch biopsy findings are presented in the image above. Grocott methenamine silver and Ziehl-Neelsen stains are negative. Which of the following is the best diagnosis?

    1. Acute febrile neutrophilic dermatosis (Sweet syndrome)
    2. Erythema elevatum diutinum
    3. Granuloma annulare
    4. Reactive granulomatous dermatitis
    5. Rheumatoid nodule
    Board review style answer #1
    D. Reactive granulomatous dermatitis. The image shows necrobiosis surrounded by palisades of histiocytes and neutrophils, consistent with a diagnosis of reactive granulomatous dermatitis (answer D). Biopsy of acute febrile neutrophilic dermatosis (answer A) would be expected to show superficial dermal edema and exuberant neutrophilic infiltrate with karyorrhexis but without true vasculitis; additionally, Sweet syndrome classically presents with an acute eruption of edematous papules and plaques that may ulcerate rather than a subacute to chronic eruption of crusted papules, as is described in this case. Biopsy of erythema elevatum diutinum (answer B) would be expected to show leukocytoclastic vasculitis with onion skin fibrosis. Histologic variants of granuloma annulare (answer C) include palisading granuloma annulare, in which histiocytes surround areas of necrobiosis and mucin deposition, as well as interstitial granuloma annulare, in which histiocytes and mucin intercalate between dermal collagen bundles; significant neutrophilic infiltrate is not characteristic. Rheumatoid nodules (choice E) are characterized by palisading granulomas with fibrinoid necrosis in the deep dermis and subcutis.

    Comment Here

    Reference: Reactive granulomatous dermatitis
    Board review style question #2

    A 42 year old woman with poorly controlled rheumatoid arthritis presents with pink-brown, indurated, cord-like plaques on her trunk and flanks. A punch biopsy is performed and is shown above. Which of the following histopathologic features is characteristic of this entity?

    1. Degenerated collagen and fibrin surrounded by palisading histiocytes in the subcutaneous fat
    2. Degenerated collagen surrounded by palisades of histiocytes and neutrophils in the superficial to mid-dermis
    3. Foci of degenerated collagen with dense mucin deposition surrounded by palisades of histiocytes
    4. Naked granulomas with Langhans type giant cells and asteroid bodies
    5. Papillary dermal edema with dense neutrophilic infiltrate and karyorrhexis
    Board review style answer #2
    B. Degenerated collagen surrounded by palisades of histiocytes and neutrophils in the superficial to mid-dermis. The vignette describes the rope sign of interstitial granulomatous dermatitis (a subtype of reactive granulomatous dermatitis), in which indurated cord-like plaques develop on the trunk and flanks. The histologic image demonstrates degenerated collagen surrounded by palisades of histiocytes and neutrophils (answer B), further supporting the diagnosis of reactive granulomatous dermatitis. Answer A describes the histologic features of rheumatoid nodules. Answer C describes the histologic features of palisading granuloma annulare. Answer D describes the histologic features of sarcoidosis. Answer E describes the histology features of acute febrile neutrophils dermatosis (Sweet syndrome).

    Comment Here

    Reference: Reactive granulomatous dermatitis

    Reticulohistiocytic granuloma (solitary reticulohistiocytoma) (pending)
    [Pending]

    Rheumatoid nodule
    Definition / general
    • Rheumatoid nodules are the most common extra-articular manifestation of rheumatoid arthritis (RA) (Semin Cutan Med Surg 2007;26:100)
    • Subcutaneous rheumatoid nodules are firm, nontender and movable within the subcutaneous tissue; however, they could also be attached to underlying structures, such as the periosteum, fascia and tendons (Am J Med 1984;76:279)
    Essential features
    • Rheumatoid nodules are the most common extra-articular manifestation and characteristic lesion in rheumatoid arthritis patients (Clin Rheumatol 2019;38:3041)
    • Most common on pressure points (such as the olecranon process); however, they may occur at other sites, including within the lung and other internal organs (Clin Rev Allergy Immunol 2017;53:371)
    • Subcutaneous rheumatoid nodules are firm, nontender and movable within the subcutaneous tissue; however, they could also be attached to underlying structures, such as the periosteum, fascia and tendons (Am J Med 1984;76:279)
    • Rheumatoid nodules are clinical predictors of more severe arthritis, seropositivity, joint erosions and rheumatoid vasculitis (J Am Acad Dermatol 1983;8:439)
    • Presence of rheumatoid nodules often suggests a need for more aggressive treatment of the underlying rheumatoid arthritis to prevent sequelae (Can Fam Physician 2005;51:35)
    • Necrobiosis, with fibrin deposition and palisading epithelioid histiocytes, is a typical histologic feature of rheumatoid nodules (Clin Rheumatol 2019;38:3041)
    ICD coding
    • ICD-10: M06.30 - rheumatoid nodule, unspecified site
    Epidemiology
    • Rheumatoid arthritis is the most common inflammatory arthritis; affects 0.5 - 1.0% of the population (Rheum Dis Clin North Am 2001;27:269)
    • 30 - 40% of all patients with rheumatoid arthritis develop nodules (Ann Rheum Dis 2006;65:601)
    • Low concordance rate for rheumatoid arthritis in monozygotic twins highlights the importance of environmental factors (Br J Rheumatol 1993;32:903)
    • Smoking is an independent risk factor for rheumatoid arthritis, particularly rheumatoid factor (RF) positive disease (Arthritis Rheum 1997;40:1955)
    • 2 - 4 fold increase in mortality for patients with extra-articular manifestations, including rheumatoid nodules (J Rheumatol 2002;29:62)
    • Early presence of rheumatoid nodules is a predictor of severe extra-articular manifestations (Scand J Rheumatol 2000;29:358)
    • Nodules are found predominantly in seropositive patients; however, current or previous smoking is associated with the presence of nodules even after controlling for rheumatoid factor positivity (J Rheumatol 2000;27:630)
    • Unlike the production of rheumatoid factor, the development of subcutaneous nodules does not occur in healthy individuals who smoke (Ann Rheum Dis 1990;49:753)
    • Patients with rheumatoid arthritis and rheumatoid nodules are also more likely to develop vasculitis (J Rheumatol 2008;35:179)
    Sites
    Pathophysiology
    • Predominant infiltrating inflammatory cell in the rheumatoid nodule is the macrophage (Arthritis Rheum 1987;30:728)
    • Macrophages in the rheumatoid nodules express TNF alpha, IL1 beta and IL1Ra mRNA, particularly in perivascular cells of the stroma and in the palisading layer (Arthritis Rheum 1998;41:1783)
    • Cytokines, proteinases and other immune factors:
      • A variety of products, particularly cytokines, are produced within rheumatoid nodules; the fibroblasts in the nodules produce large quantities of metalloproteases (J Clin Invest 1972;51:2973)
      • Most of the cytokine profile, together with the ability of the tissue to produce metalloproteinases, establishes the rheumatoid nodule as a T helper 1 (Th1) granuloma (Arthritis Rheum 2003;48:334)
      • Lymphocytes generate IgG and IgM rheumatoid factor (Arthritis Rheum 2003;48:334)
    • There is evidence for (Arthritis Rheum 2008;58:1601):
      • Tumor necrosis factor (TNF) alpha
      • Interferon gamma
      • Interleukin (IL) 1 beta
      • IL1 receptor antagonist
      • IL10, IL15, IL18 and IL12
      • Adhesion molecules, including E-selectin
      • Intracellular adhesion molecule (ICAM) 1
      • Platelet endothelial cell adhesion molecule (PECAM)
      • Vascular cell adhesion molecule (VCAM)
    • Deposits of rheumatoid factor and the terminal components of complement are also found on the endothelium of small vessels within nodules (J Rheumatol 2000;27:1839)
    • Peptidyl arginine deiminases 2, 3 and 4 and myeloperoxidase have been shown to generate citrullinated and homocitrullinated antigens in necrotic tissue of rheumatoid nodules (Arthritis Res Ther 2016;18:239)
    • Neutrophils express these enzymes; through the generation of neutrophil extracellular traps (NETs), they may contribute to autoantigen externalization and may be targeted by anticitrullinated peptide antibodies (ACPA), thus contributing to the inflammatory process within rheumatoid nodules (Sci Transl Med 2013;5:178ra40)
    • Similar proinflammatory cytokines and cell adhesion molecules, in the rheumatoid nodule and synovial membrane, suggests that similar pathogenic processes result in the chronic inflammation and tissue destruction within these lesions (Arthritis Rheum 1998;41:1783)
    Etiology
    Clinical features
    • Subcutaneous rheumatoid nodules:
      • Firm, nontender and movable within the subcutaneous tissue; however, they can also be attached to underlying structures, such as the periosteum, fascia and tendons (Am J Med 1984;76:279)
      • Nodules vary in consistency, from firm immovable masses when they are small to soft mobile masses when they are large (Ann Rheum Dis 1993;52:625)
      • Nodules may ulcerate or may become gangrenous (Ann Rheum Dis 1993;52:625)
    • Pulmonary nodules (Med J Aust 1986;144:648):
      • Generally asymptomatic but can lead to complications, including:
        • Pleural effusion
        • Pneumothorax
        • Pyopneumothorax
        • Bronchopleural fistula
        • Hemoptysis
    • Cardiac nodules:
      • Presence of subcutaneous nodules is associated with valvular thickening, nodules and insufficiency (mainly of the mitral and aortic valves on echocardiography) (Rheumatol Int 1999;18:163)
      • Symptoms related to the presence of nodules are rare but syncope or death (due to heart block from a lesion situated in the conduction system) can occur (Ann Rheum Dis 1983;42:389)
      • Stroke or other manifestations of arterial embolization may result from nodules on a heart valve (J Heart Valve Dis 1997;6:77)
    Diagnosis
    • Subcutaneous rheumatoid nodules:
      • Multiple subcutaneous nodules, in the presence of rheumatoid factors and in the absence of any joint complaints, firmly suggest a diagnosis of rheumatoid nodulosis (Ann Rheum Dis 1949;8:1)
      • Biopsy of mature lesions, though often not necessary, shows necrobiosis, with fibrin deposition and palisading epithelioid histiocytes (Clin Rheumatol 2019;38:3041)
    Laboratory
    • Rheumatoid nodules are found predominantly in rheumatoid factor (RF) positive patients (Am J Med 1984;76:279)
    Case reports
    • 33 year old woman with a 3 year history of rheumatoid arthritis treated with methotrexate, after total thyroidectomy for hypothyroidism (J Med Case Rep 2019;13:159)
    • 56 year old man diagnosed 2 years earlier with seronegative rheumatoid arthritis (RA) presents with acute right sided hemiplegia and dysarthria (J Community Hosp Intern Med Perspect 2017;7:175)
    • 57 year old woman, with an overlap syndrome of both rheumatoid arthritis (RA) and autoimmune hepatitis, developed multiple skin nodules (Clin Rheumatol 2011;30:719)
    • 57 year old man with rheumatoid arthritis and myasthenia gravis developed an asymptomatic erythematous plaque on his nose and skin colored nodules on the elbows (Int J Dermatol 2007;46:33)
    Treatment
    • Subcutaneous nodules (G Ital Dermatol Venereol 2018;153:243):
      • Treatment of rheumatoid nodules is unnecessary because they are often asymptomatic
      • Nodules may ulcerate, become infected, or compress nerves or articular structures; surgical excision may be necessary and may include skin grafting for extremely large nodules
      • Intralesional injection of steroids may also be considered
      • Nodules managed with disease modifying antirheumatic drugs (DMARDs) may shrink, persist or worsen
    Clinical images

    Images hosted on other servers:

    Classic rheumatoid nodules on the elbow

    Faintly pink subcutaneous nodules

    Microscopic (histologic) description
    • Central area of fibrinoid necrosis surrounded by palisading macrophages and lymphocytes (Arthritis Rheum 1987;30:728)
    • Necrobiosis, with fibrin deposition and palisading epithelioid histiocytes (Clin Rheumatol 2019;38:3041)
    • Leukocytoclastic vasculitis (with associated immunoglobulin [Ig], fibrin deposition and complement activation) can be found in 33% of all rheumatoid nodules (Semin Cutan Med Surg 2007;26:100)
    • Granulomas are typically identified in the subcutis but occasionally may involve the dermis
    • Mucin deposition is characteristically absent
    Microscopic (histologic) images

    Contributed by Kiran Motaparthi, M.D.

    Palisading granulomas

    Palisading mononuclear histiocytes

    Subcutaneous palisading granulomas

    Granulomas surrounding fibrinoid necrosis

    Palisading epithelioid histiocytes

    Virtual slides

    Images hosted on other servers:

    Soft tissue, rheumatoid nodules

    Sample pathology report
    • Right index finger, excision:
      • Palisading granulomas in the subcutis surrounding areas of fibrinoid necrosis, consistent with rheumatoid nodule
    Differential diagnosis
    • Rheumatic fever nodules (Ann Rheum Dis 1993;52:625):
      • Localization: extensor sites, also over the spinous process of vertebrae
      • Depth: subcutaneous
      • Microscopy (Cureus 2021;13:e12577):
        • Histologic appearance varies depending on whether the biopsy is of an early or an established nodule
        • Features in early descriptions include increased numbers of dilated blood vessels with peripherally thickened walls, fibrinoid necrosis and fibrosis with attachment to tendons
        • Histologically similar to nodules seen in rheumatoid arthritis with central fibrinoid necrosis, surrounded by histiocytes and perivascular lymphocytes and neutrophils
      • Age of onset: children
      • Persistence: limited
      • Rheumatoid factor: absent
      • Concomitant joint disease: yes
      • Bone erosions: absent
    • Granuloma annulare:
      • Localization: extensor site or any other site
      • Depth: intradermal (subcutaneous)
      • Microscopy (Arch Dermatol 1977;113:1681):
        • Palisaded or interstitial granulomatous inflammation with mucin unites all forms
        • Minimal epidermal changes
      • Age of onset: children, young adults
      • Persistence: unpredictable
      • Rheumatoid factor: absent
      • Concomitant joint disease: none
      • Bone erosions: absent
    • Gouty tophi (Ann Rheum Dis 2013;72:1545):
      • Patient with a history of acute mono or oligoarticular arthritis
      • Most commonly involving the great toe metatarsophalangeal (MTP) joints
      • Hyperuricemia
      • White to yellow monosodium urate deposits
      • Diagnosis can be confirmed, when necessary, by aspiration and polarized microscopy
      • Microscopy (J Int Adv Otol 2016;12:216):
        • Tophi are composed of needle shaped aggregates of urate crystals with surrounding foreign body giant cell reaction
        • Urate crystals dissolve with routine processing, so fix a smear of crystals in absolute alcohol or nonaqueous fixation
        • Under polarized light microscopy, they have a needle-like morphology and strong negative birefringence
    • Calcinosis cutis (Semin Arthritis Rheum 2005;34:805):
      • Dystrophic calcification at sites of microtrauma or inflammation
      • Typically occurs in patients with systemic sclerosis, dermatomyositis or overlap connective tissue diseases
      • Lesions are composed of calcium hydroxyapatite
      • Multiple subcutaneous papules or nodules that are yellow-white in the hands, elbows or feet
      • May also occur in soft tissues of the trunk or legs
      • Calcinosis lesions may ulcerate and exude chalky white material
      • These nodules may be evident on plain radiography
      • Microscopy (Actas Dermosifiliogr 2015;106:785):
        • Irregular deposits of intensely basophilic acellular material in the dermis and subcutaneous tissue
        • Basophilia is so strong that the appearance is of a deep purple
        • Deposits are typically well circumscribed with a thin rim of eosinophilic hyalinization and frequently with a host giant cell reaction
    • Palisaded neutrophilic granulomatous dermatitis (JAAD Case Rep 2017;3:425):
      • Typically associated with underlying disease states, including autoimmune connective tissue disease, lymphoproliferative disorders and infections
      • Most common in patients with rheumatoid arthritis
      • Can be seen in patients with systemic lupus erythematosus and systemic vasculitis
      • Microscopy:
        • Early lesions present with neutrophilic infiltrates and leukocytoclastic vasculitis
        • Fully developed lesions feature palisaded granulomas with collagen trapping and neutrophil remnants
    • Epithelioid sarcoma (Adv Anat Pathol 2006;13:114):
      • Malignant mesenchymal neoplasm that exhibits epithelioid cytomorphology and a predominantly epithelial phenotype
      • Microscopy:
        • Classic or conventional, distal type:
          • Dermal / subcutaneous nodule(s)
          • Can simulate granulomatous process, with or without necrosis (pseudogranulomatous morphology)
          • Fairly uniform plump small to medium sized cells with eosinophilic cytoplasm (Adv Anat Pathol 2016;23:41)
          • Spindle cells can also be identified and often appear more conspicuous at the periphery of a nodule
          • Mildly atypical nuclei with vesicular chromatin and small nucleoli
          • Variable mitotic activity
          • Mixed chronic inflammatory infiltrate common
          • Dystrophic calcification and metaplastic bone formation
        • Proximal, large cell type (Am J Surg Pathol 1997;21:130):
          • Infiltrative growth pattern with less defined peripheral borders
          • Multiple large nodules
          • Large polygonal cells with abundant eosinophilic cytoplasm
          • Rhabdoid morphology common
          • Vesicular nuclei with prominent macronucleoli
          • Mitoses, necrosis and hemorrhage common
    • Necrobiosis lipoidica:
      • Atrophic, yellow depressed plaques, usually on legs of diabetic patients
      • Microscopy (Indian J Endocrinol Metab 2012;16:614):
        • Layered inflammatory process and alternating zones of necrobiosis involving the full thickness of the dermis
        • Changes tend to become more pronounced deeper in the dermis and may extend into the septal panniculus
        • Areas of necrobiosis are poorly defined and run into each other with broad foci of inflammatory infiltrate intervening
        • Variable histiocytic infiltrate with multinucleated giant cells surrounds these areas
        • Accompanying inflammatory infiltrate is predominantly lymphocytic with plasma cells and occasional eosinophils
    Board review style question #1

    A 61 year old woman presents with firm, nontender and movable olecranon nodules for the past 8 years. Representative histopathology is shown in the above image. Which factor is associated directly with the nodules?

    1. HLA-B27
    2. HLA-DQ2
    3. IL4 receptor (IL4R)
    4. IL13 receptor (IL13R)
    Board review style answer #1
    C. IL4 receptor (IL4R) single nucleotide polymorphisms are associated with rheumatoid nodules (Arthritis Res Ther 2010;12:R75)

    Comment Here

    Reference: Rheumatoid / rheumatic nodules
    Board review style question #2
    A 57 year old man with rheumatoid arthritis and myasthenia gravis developed an asymptomatic erythematous plaque on his nose and skin colored nodules on his elbows. If the elbow nodules get biopsied, what is the most probable microscopic pattern?

    1. Central area of fibrinoid necrosis surrounded by palisading macrophages and lymphocytes
    2. Layered inflammatory process and alternating zones of necrobiosis involving the full thickness of the dermis
    3. Mildly atypical nuclei with vesicular chromatin and small nucleoli
    4. Neutrophilic infiltrates and leukocytoclastic vasculitis
    Board review style answer #2
    A. Central area of fibrinoid necrosis surrounded by palisading macrophages and lymphocytes

    Comment Here

    Reference: Rheumatoid / rheumatic nodules

    Rhinosporidiosis (pending)

    Rosacea
    Definition / general
    • Common chronic dermatosis with erythema of central face, acneiform pustules and papules, telangiectasia and blepharitis (Am Fam Physician 2009;80:461)
    Terminology
    • Also called rhinophyma
    Clinical features
    • Exists in 5 clinical forms:
      • Erythematous, telangiectatic type (70% of cases)
      • Papulopustular type
      • Granulomatous type
      • Hyperplastic glandular type (phymatous rosacea, which results in irregular, bulbous enlargement of the nose; the condition known as rhinophyma)
      • Ocular disease
    • May be associated with abnormal TLR2 (toll-like receptor 2) expression, which leads to a calcium dependent release of kallikrein 5 from keratinocytes, which may cause rosacea (J Invest Dermatol 2011;131:688)
    Treatment
    • Topical metronidazole is well tolerated and efficacious for moderate to severe papulopustular rosacea
    • Also topical azelaic acid, azithromycin (once daily), minocycline (40 mg dose)
    • Pimecrolimus 1% cream is effective and well tolerated treatment for steroid induced rosacea
    Clinical images

    Images hosted on other servers:

    Various images

    Microscopic (histologic) description
    • Perinfundibular, lymphocytic or granulomatous inflammation
    • Occasional plasma cells, an important clue for the diagnosis
    • Variable features: mild dermal edema, solar elastosis, mild perifolliculitis
    • Sebaceous gland hypertrophy and scattered follicular plugging are present in most cases of rhinophyma
    • Papulopustular lesions have a more pronounced inflammatory infiltrate which is both perivascular and peripilar, involving the superficial and mid dermis; infiltrate may include a few neutrophils, as well as lymphocytes and plasma cells; Demodex mites are present in 20 - 50% of cases
    • Granulomatous form is usually characterized by a tuberculoid reaction, often in the vicinity of damaged hair follicles; necrosis (resembling caseation) was present in 11% of patients in one series
    Microscopic (histologic) images

    Images hosted on other servers:

    Dermis of a patient with rosacea

    Positive stains
    • Direct immunofluorescence occasionally shows immunoglobulins and complement at dermoepidermal junction
    Additional references

    Sarcoidosis
    Definition / general
    • Multisystem granulomatous disorder with cutaneous involvement in up to 35% of cases
    Essential features
    • Multisystem granulomatous disorder with cutaneous involvement in up to 35% of cases
    • Etiology is unknown
    • Disease development and progression may be related to disordered immune response to extrinsic antigens in genetically susceptible individuals
    • Clinically heterogeneous and can mimic other dermatological conditions
    • Histologically characterized by discrete, nonnecrotizing granulomas with minimal lymphocytic inflammation involving the dermis or subcutaneous tissue
    • Diagnosis of exclusion; particularly with respect to infection
    ICD coding
    • ICD-10: D86.3 - sarcoidosis of skin
    • ICD-11: 4B20.5 - cutaneous sarcoidosis
    Epidemiology
    Sites
    Pathophysiology
    • Granulomatous inflammation is characterized by Th1 mediated immune response
    • Antigen presenting cells recognize and internalize putative agents to form major histocompatibility complex (MHC) peptide complexes that are recognized by CD4+ T cells
    • Cellular immune response is elicited with secretion of tumor necrosis factor alpha (TNF alpha), interferon gamma (IFN gamma), IL1, IL2, IL6, IL12, IL15, IL18 and macrophage inflammatory protein 1 (J Am Acad Dermatol 2012;66:699.e1, J Eur Acad Dermatol Venereol 2010;24:747, Curr Opin Rheumatol 2000;12:71)
    Etiology
    Clinical features
    • Familial clustering suggests genetic susceptibility in some cases
    • Cutaneous manifestations are variable and can mimic other dermatological conditions
    • Clinical morphology can range from papules and plaques (sometimes annular) to subcutaneous nodules
    • Skin from any anatomic site may be affected
    • Presentations include
      • Acute and usually self limiting variant (Löfgren syndrome characterized by erythema nodosum, bilateral hilar adenopathy, fever and polyarthralgia)
      • Chronic form involving skin only (20 - 40% of cutaneous sarcoidosis cases lack systemic involvement)
      • Systemic chronic variant affecting multiple organs
    • Lupus pernio is a chronic and disfiguring variant characterized by indurated lesions of the nose, cheeks and ears
    • Scar sarcoidosis is characterized by development of lesions in surgical scars, tattoos, skin piercings and other sites of trauma
    • Erythema nodosum occurs in up to 40% of patients with sarcoidosis (Medicine (Baltimore) 2017;96:e7595)
    Diagnosis
    • Diagnosis of exclusion
    • Thorough history and physical exam (including ophthalmic evaluation) to assess for potential occupation / environmental exposures or associated systemic symptoms and signs
    • Chest Xray, pulmonary function tests and cardiac investigations
    • Reference: Cells 2021;10:766
    Laboratory
    • Skin biopsy to rule out infection and other inflammatory conditions
    • Elevated serum angiotensin converting enzyme (ACE), elevated calcium, elevated erythrocyte sedimentation rate (ESR)
    Radiology description
    • Bilateral hilar lymphadenopathy on chest radiography is common
    • Other less common findings: pulmonary infiltrates, fibrosis, bullae, cysts, emphysema
    • Occasionally, bone lesions and cysts
    • Reference: AJR Am J Roentgenol 2020;214:50
    Prognostic factors
    Case reports
    Treatment
    • First line (limited or mild disease): topical and intralesional corticosteroids
    • Systemic immunomodulators: tetracycline class antibiotics, antimalarials and other agents
    • Systemic immunosuppressive and immunomodulatory agents: prednisone, methotrexate, TNF inhibitors and other biologic therapies
    • References: Front Immunol 2020;11:545413, Eur Respir J 2021;58:2004079,
    Clinical images

    Contributed by Kerri Purdy, M.D.

    Annular hypopigmented lesions

    Erythematous eruption

    Lupus pernio

    Annular hypopigmented lesions

    Microscopic (histologic) description
    • Nodular, diffuse or angiocentric pattern of granulomatous inflammation in superficial or deep dermis with or without involvement of the subcutis
    • Discrete or confluent nonnecrotizing granulomata composed of epithelioid histiocytes with abundant eosinophilic cytoplasm
    • Variable number of Langhans or other giant cells
    • Limited lymphocytic cuff surrounding granulomata (so called naked granulomas)
    • Small foci of fibrinoid necrosis sometimes present but geographic or caseating necrosis is rare
    • Typically, normal epidermis
    • Intracytoplasmic inclusion bodies may be seen but none are specific or pathognomonic
      • Schaumann body: basophilic, calcified, laminated round structure
      • Asteroid body: intracytoplasmic eosinophilic star shaped structure
    • Foreign material is present in at least 5% of cases and does not exclude sarcoidosis (Arch Dermatol 2001;137:485, Arch Dermatol 2001;137:427, Am J Dermatopathol 1993;15:203)
    Microscopic (histologic) images

    Contributed by Thai Yen Ly, M.D., Jijgee Munkhdelger, M.D., Ph.D. and Andrey Bychkov, M.D., Ph.D.

    Superficial granulomatous inflammation

    Superficial and deep granulomatous inflammation

    Well formed granulomata

    Granulomata with lymphocytic inflammation

    Granulomata with lymphocytic cuff


    Associated foreign material

    Missing Image

    Asteroid body

    Missing Image Missing Image

    Schaumann bodies


    Schaumann body

    Missing Image

    Small, tight granulomas

    Missing Image

    Noncaseating granulomas

    Missing Image

    Extension to subcutaneous fat tssue

    Missing Image

    Langhans giant cells

    Positive stains
    Sample pathology report
    • Skin, punch biopsy:
      • Granulomatous dermatitis, sarcoidal type (see comment)
      • Comment: Special stains for fungal organisms (PAS and GMS) and mycobacteria (fite) are negative. The pattern of the granulomatous infiltrate, in the appropriate clinical context, would be consistent with sarcoidosis. Clinical pathologic correlation is recommended.
      • Note: As sarcoidosis is a diagnosis of exclusion, a descriptive diagnosis is appropriate.
    Differential diagnosis
    • Infection:
      • Concomitant neutrophilic infiltrate may be present
        • Fungal organisms on PAS or GMS stains
        • Mycobacteria on fite or Ziehl-Neelsen stains
        • Positive culture
        • Clinical features of infection
    • Foreign body reaction:
      • Presence of foreign material
      • Irregularly distributed giant cells with haphazardly arranged nuclei
    • Cutaneous Crohn's disease:
      • Usually perianal / genital distribution
      • Ill defined granulomata
      • Mixed inflammation is common (lymphocytes, eosinophils, plasma cells)
      • Epidermal ulceration may be present
      • Clinical history of Crohn's disease
    • Granulomatous cheilitis (Melkersson-Rosenthal syndrome):
      • Subtle and poorly formed granulomata
      • Characteristic facial, lip and oral mucosal swelling with or without facial nerve palsy and fissured tongue
    • Granulomatous rosacea (including perioral and periorbital dermatitis):
      • Perifollicular distribution of granulomatous inflammation
      • Tuberculoid granulomata (less circumscribed, more confluence, prominent lymphoplasmacytic inflammation) rather than sarcoidal granulomata
      • Background of prominent vascular ectasia
      • Characterized by persistent erythema and recurrent episodes of flushing, edema, papules and pustules
    • Tuberculoid leprosy:
      • Elongated sausage shaped granulomata around nerves
      • PCR studies may be helpful
      • Localized lesion on trunk or limb with associated sensory impairment
    • Necrobiosis lipoidica:
      • Features altered collagen (necrobiosis), lymphoid aggregates and lymphoplasmacytic infiltrate
      • Association with diabetes mellitus
    • Granuloma annulare (sarcoidal variant):
      • Palisading or interstitial pattern of more typical granuloma annulare may also be present to suggest the diagnosis
      • Increased dermal mucin
    • Beryllium and zirconium:
      • Granulomata are indistinguishable from sarcoidosis
      • History of exposure
    Board review style question #1

    A 45 year old woman presents with fever, polyarthralgia, bilateral hilar adenopathy, erythema nodosum and an erythematous papular skin eruption on the face. Punch biopsy of a facial lesion is shown the image above. What is the most likely diagnosis?

    1. Granuloma annulare
    2. Granuloma faciale
    3. Rheumatoid nodule
    4. Sarcoidosis
    5. Xanthelasma
    Board review style answer #1
    D. Sarcoidosis. Given the microscopic findings and clinical context, the most likely diagnosis is sarcoidosis (acute variant Löfgren syndrome). The image shows well formed, nonnecrotizing and nonpalisading sarcoidal granulomata. Granuloma faciale is considered a localized form of vasculitis with an eosinophil rich infiltrate that is not granulomatous. Rheumatoid nodules feature palisading granulomatous inflammation around pink fibrinous material that can mimic necrosis. Granuloma annulare is another palisading granulomatous process associated with increased dermal mucin. Xanthelasma features loose, ill defined aggregates of foam cells in the dermis.

    Comment Here

    Reference: Sarcoidosis
    Board review style question #2
    Regarding sarcoidosis, which of the following statements is true?

    1. Associated foreign material may be present and does not exclude sarcoidosis
    2. Asteroid body inclusions are a diagnostic feature
    3. Fungal organisms are sometimes seen on special stains
    4. Geographic necrosis is a common finding
    5. Schaumann body inclusions are a diagnostic feature
    Board review style answer #2
    A. Associated foreign material may be present in some cases of sarcoidosis (e.g., scar sarcoidosis) and does not exclude the diagnosis of sarcoidosis. Schaumann and asteroid body inclusions are not specific or pathognomonic for sarcoidosis and can be seen in other conditions such as tuberculosis, tuberculoid leprosy and berylliosis. Prominent geographic necrosis is rare but small foci of fibrinoid necrosis are sometimes evident in sarcoidosis. The presence of fungal organisms confirms infection and is incompatible with sarcoidosis, which is a diagnosis of exclusion.

    Comment Here

    Reference: Sarcoidosis

    Scabies (mite)
    Definition / general
    • Scabies are 8 legged mites that infest the stratum corneum
    • Cause intense pruritus accompanied by a variety of cutaneous findings; infestation may lead to secondary impetiginization and psychological sequelae
    Essential features
    • Obligate ectoparasite
    • Burrows into the stratum corneum of acral and intertriginous sites
    • Diagnosis is through visualization of burrows on clinical exam or through identification of mites, eggs or scybala on microscopy
    Terminology
    • Itch mite, Sarcoptes scabiei var. hominis
    ICD coding
    • ICD-10: B86 - scabies
    Epidemiology
    Sites
    • Finger and toe webs
    • Volar surfaces of wrists
    • Intertriginous areas
    • Areas under occlusion (i.e. waistbands)
    • Areola (females), genitalia (males)
    • Spares head (high sebaceous areas) in adults, immune competent (Dermatol Ther 2020;33:e13746)
    • May affect head in infants, immunocompromised
    Pathophysiology
    • Skin to skin transmission or fomites
    • Classic scabies: mite load 5 - 15 mites (J Am Acad Dermatol 2020;82:533)
      • Th1 / Th2 immune response
    • Crusted scabies: 100s - 1,000s of mites
      • Th2 response
    • Mites burrow into stratum corneum
    Etiology
    Diagrams / tables

    Images hosted on other servers:
    Life cycle

    Life cycle

    Diagnostic criteria for scabies

    Diagnostic criteria for scabies

    Clinical features
    • Initial symptom onset 4 - 6 weeks post initial infection (J Am Acad Dermatol 2020;82:533)
    • Subsequent infection symptom onset within days (J Am Acad Dermatol 2020;82:533)
    • Classic presentation:
      • Extreme pruritus, nighttime worsening
      • Burrows
        • Small white serpiginous scaly linear papules on finger and toe webs
      • Erythematous scaly papules on intertriginous, areolar and genital skin and areas under occlusion
        • Excoriations common
    • Genital / scrotal erythematous nodules (IDCases 2020;22:e00947)
    • With or without pruritus in infants, immunosuppressed and patients on topical steroids (J Am Acad Dermatol 2020;82:533)
    • With or without secondary impetiginization or autoeczematization
    • Crusted scabies
      • Yellow to white hyperkeratotic plaques
    • With or without postscabetic pruritus for 4+ weeks posttreatment
      • Use topical corticosteroids, education
    Diagnosis
    • Mineral oil scraping (low sensitivity)
    • Clinical exam (burrows on finger and toe webs, genital nodules)
    • Dermoscopy (delta wing jet sign) (Tidsskr Nor Laegeforen 2020;140:1)
    • Other noninvasive techniques: videodermoscopy, confocal microscopy, optical coherence tomography (J Am Acad Dermatol 2020;82:533)
    • Skin biopsy (shave or punch)
    Laboratory
    Prognostic factors
    Case reports
    Treatment
    • Permethrin 5% cream
      • First line
      • Close contacts treated simultaneously
      • FDA approved for > 2 months age
    • Oral ivermectin
    • Less commonly used (J Am Acad Dermatol 2020;82:533):
      • Benzyl benoate 10 - 25%
      • Precipitated sulfur 2 - 10%
      • Crotamiton 10%
      • Malathion 0.5%
      • Lindane 1%
    Clinical images

    Images hosted on other servers:
    Penile nodules

    Penile nodules

    Web spaces

    Web spaces

    Genital papulonodules Genital papulonodules

    Genital papulonodules

    Areolar papules

    Areolar papules

    Burrow

    Burrow

    Microscopic (histologic) description
    Microscopic (histologic) images

    Contributed by Eric W. Hossler, M.D. and Bethany R. Rohr, M.D.
    Crusted scabies

    Crusted scabies

    Numerous mites

    Numerous mites

    Crusted scabies Crusted scabies

    Crusted scabies

    Crusted scabies Crusted scabies

    Crusted scabies


    Crusted scabies

    Crusted scabies

    Scraping of adult mite

    Scraping of adult mite

    Scraping of an egg

    Scraping of an egg

    Ova and scybala

    Ova and scybala

    Mite anatomy

    Mite anatomy

    Intracorneal mite

    Intracorneal mite


    Adult mite

    Adult mite

    Ova on skin scraping

    Ova on skin scraping

    Setae

    Setae

    Mite on skin scraping

    Mite on skin scraping

    Rudimentary front legs

    Rudimentary front legs

    Virtual slides

    Images hosted on other servers:
    Scabies

    Scabies

    Scabies mineral scraping

    Scabies mineral oil scraping

    Crusted scabies

    Crusted scabies

    Crusted scabies clinical

    Crusted scabies clinical

    Positive stains
    Videos

    Probe of scabies burrow under dermoscopy

     

    Ask a dermatologist: how do I get rid of scabies?

    Sarcoptes scabiei var. hominis

    Sample pathology report
    • Skin, anatomic location, (punch or shave) biopsy:
      • Scabies
    Differential diagnosis
    Board review style question #1
    A biopsy from a patient with crusted scabies is likely to reveal which findings compared with typical scabies?

    1. Few mites
    2. Few ova
    3. Many ova but few mites
    4. Mites present in the upper dermis
    5. Numerous mites, ova and scybala
    Board review style answer #1
    E. Numerous mites, ova and scybala. Crusted scabies occurs in immunosuppressed patients and characteristically contains hyperkeratosis with numerous mites, ova and scybala. Scabies mites reside in the epidermis only.

    Comment Here

    Reference: Scabies (mite)
    Board review style question #2

    Scabies invade into which level of the skin?

    1. Papillary dermis
    2. Reticular dermis
    3. Stratum corneum
    4. Stratum granulosum
    5. Stratum spinosum
    Board review style answer #2
    C. Stratum corneum

    Comment Here

    Reference: Scabies (mite)
    Board review style question #3
    Scabies mites avoid which anatomic region, except in infants and immunosuppressed patients?

    1. Acral sites
    2. Genitals
    3. Head (face, scalp)
    4. Intertriginous areas
    5. Volar wrists
    Board review style answer #3
    C. Head (face, scalp); scabies typically avoid the face / scalp (highly sebaceous areas) in immunocompetent adults. The other listed sites are typical areas of involvement.

    Comment Here

    Reference: Scabies (mite)

    Scleredema
    Definition / general
    • Nonpitting, woody induration of upper back skin; associated with diabetes
    • Rare primary mucinosis that presents with nonpitting indurated edema and associated dermal hardening in the absence of any clinical abnormality
    • After viral infection, especially UTI or insidious onset with no prior acute illness
    • Occasionally associated with paraproteinemia, secondary hyperparathyroidism, rheumatoid arthritis, Sjogren syndrome, HIV infection and nuchal fibroma
    Clinical features
    • Symmetrical nonpitting edema and dermal hardening, which affects the posterior and lateral aspect of the neck, face, upper trunk and upper limbs
    • Rarely affects lower abdomen and legs
    Case reports
    Microscopic (histologic) description
    • Epidermis appear slightly thinned or normal
    • Thickened reticular dermis, often at expense of subcutaneous fat; eccrine glands in upper third of mid dermis
    • Collagen fibers are broadened, abnormally separated by clear spaces, which may contain mucin
    • May be mild chronic inflammatory cell infiltrate in superficial dermis, increased mast cells but normal amount of fibroblasts
    Negative stains
    • Negative direct immunofluorescence
    Differential diagnosis
    • Scleroderma: appendages are atrophic or absent; diffuse dermal sclerosis, not the fenestrated appearance of scleredema
    Additional references

    Scleromyxedema (pending)
    [Pending]

    Scrub typhus
    Definition / general
    • Chigger-borne zoonosis of tropical Asia and western Pacific islands, caused by Orientia (formerly Rickettsia) tsutsugamushi
    Terminology
    • Formerly called Rickettsia tsutsugamushi but now distinguished from Rickettsiaceae family by differences in cell wall
    • Similar in presentation to other forms of typhus, but caused by agent in a different genus (Wikipedia: Scrub Typhus [Accessed 28 August 2018])
    • "Scrub" refers to the type of vegetation (i.e., terrain between woods and clearings) that harbors the vector but name is not entirely correct because endemic areas can be sandy and semiarid (eMedicine: Scrub Typhus [Accessed 28 August 2018])
    Epidemiology
    • Infected large number of soldiers in World War II
    • Today, a frequent cause of febrile illness leading to hospital admissions in indigenous populations in eastern Asia, the southwestern Pacific (Korea to Australia) and from Japan to India and Pakistan
    Etiology
    Clinical features
    • Travelers with imported disease often become sick before or within a few days of return from an endemic region; unlikely diagnosis if illness begins > 18 days after return (Curr Infect Dis Rep 2009;11:66)
    Treatment
    Clinical images

    Images hosted on other servers:

    Adult trombiculid mite

    Chigger

    Eschar

    Microscopic (histologic) description
    • Lymphocytic vasculitis of arterioles (not postcapillary venules) causes cutaneous findings
    • Variable thrombosis of vessels
    • Microorganisms are detected in endothelium of skin and other organs using fluorescein-labelled antiserum; also present in macrophages as fine granules on high power
    • Eschar exhibits necrotic ulceration, due to coagulative necrosis of of the epidermis and underlying dermis
    • Often only few neutrophils in the inflammatory infiltrate
    Positive stains
    Molecular / cytogenetics description
    • PCR is useful for confirmation
    Additional references

    Scurvy (pending)
    [Pending]

    Seborrheic dermatitis
    Definition / general
    • Nummular eczema (silver dollar sized patches) to generalized exfoliative dermatitis (severe atopic dermatitis) to large vesicles on palms and soles (dyshidrosis)
    • Not due to any known agents, although associated with irritant contact dermatitis
    • One of the most common cutaneous manifestations of AIDS, affecting 20 - 80%
    • Also associated with Parkinson's disease, epilepsy, congestive heart failure, obesity, chronic alcoholism, Leiner disease (exfoliative dermatitis of infancy) and zinc deficiency
    • May occur as reaction to arsenic, gold, chlorpromazine, methyldopa and cimetidine
    Clinical features
    • Erythematous scaling papules and plaques, sometimes with a greasy yellow appearance, with a characteristic distribution on scalp, ears, eyebrows, eyelid margin, and nasolabial area - the so called seborrheic areas
    Treatment
    • Keratolytic agents, some over the counter
    • Anti-inflammatory products, such as topical corticosteroids, have some effect
    Clinical images

    Contributed by Mark R. Wick, M.D.

    Breast skin



    Images hosted on other servers:

    Redness (erythema) and mild scaling

    Microscopic (histologic) description
    • Acute, subacute or chronic spongiotic dermatitis
    • In acute lesions, there is focal, usually mild, spongiosis with overlying scale crust containing a few neutrophils; the crust is often centered on a follicle; papillary dermis is mildly edematous; blood vessels in superficial vascular plexus are dilated and there is mild superficial perivascular infiltrate of lymphocytes, histiocytes and occasional neutrophils; some exocytosis of inflammatory cells but not as prominent as in nummular dermatitis
    • In subacute lesions, there is also psoriasiform hyperplasia, initially slight, with mild spongiosis and the other changes already mentioned; numerous yeast-like organisms can usually be found in the surface keratin
    • Chronic lesions show more pronounced psoriasiform hyperplasia and only minimal spongiosis; sometimes the differentiation from psoriasis can be difficult but the presence of scale crusts in a folliculocentric distribution favors seborrheic dermatitis
    Microscopic (histologic) images

    Contributed by Mark R. Wick, M.D.

    Breast skin


    Sporotrichosis (pending)

    Stasis dermatitis
    Definition / general
    • Due to chronic venous stasis, usually on lower extremities
    • May have secondary infection and ulcers
    Epidemiology
    • Common disorder of middle aged and older individuals
    Clinical features
    • In the early stages, there is edema of the lower one-third of the legs, which have a shiny and erythematous appearance
    • Subsequently, dry and scaly or crusted and weeping areas may develop
    • Sometimes the changes are most prominent above the medial malleoli
    Clinical images

    Images hosted on other servers:

    Stasis dermatitis

    Microscopic (histologic) description
    • Characterstic lobular pattern of superficial and deep dermal neovascularization with dermal fibrosis, perivascular lymphocytic infiltrates, histiocytes and variable number of plasma cells, extravasated erythrocytes and hemosiderin laden macrophages
    • Hyperplasia of endothelial cells
    • Variable acanthosis and hyperkeratosis

    Stevens-Johnson syndrome
    Definition / general
    • Historically there has been controversy as to whether Stevens-Johnson syndrome (SJS) is a distinct entity or at the center of the disease spectrum that includes erythema multiforme (EM) and toxic epidermal necrolysis (TEN)
    • Currently, there is an increasing trend for SJS and TEN as representing the ends of a spectrum of severe epidermolytic adverse cutaneous drug reactions (Orphanet J Rare Dis 2010;5:39)
    Terminology
    • SJS: keratinocyte necrosis; epidermal detachment < 10% body surface area; the beginning of the spectrum (Orphanet J Rare Dis 2010;5:39)
    • SJS / TEN: considered point of overlap of two diseases in patients with 10 - 30% body surface area epidermal detachment (N Engl J Med 1994;331:1272)
    • TEN: severe disease state and the end of a spectrum of epidermolytic adverse cutaneous drug reactions; full thickness epidermal necrosis; epidermal detachment > 30% body surface area (Orphanet J Rare Dis 2010;5:39)
    • Atypical SJS: referred to by some as severe mucositis with Mycoplasma pneumoniae infection but without skin lesions (Pediatrics 2007;119:e1002), while others argue that the classification of SJS requires cutaneous involvement (Pediatr Dermatol 2006;23:546)
    Epidemiology
    Sites
    • Lesions usually begin on trunk and spread centrifugally
    Pathophysiology
    • Mechanism unknown but appears to be CD8+ T cell mediated immune reaction (J Allergy Clin Immunol 2011;127:S74)
    • Granulysin, a mitochondrial protein, is most important factor in epidermal destruction; is found in blister fluid in increasing concentrations with increased severity of disease (Nat Med 2008;14:1343)
    Etiology
    • Cases in children are usually associated with infection, especially Mycoplasma pneumonia and herpes simplex virus

    • In adults, most cases are due to medications, including:
    • ALDEN (algorithm for assessment of drug causality in SJS and TEN) provides a structured approach to determine the responsible drug (Clin Pharmacol Ther 2010;88:60), although no identifiable cause in some cases (Orphanet J Rare Dis 2010;5:39)
    Diagrams / tables

    Images hosted on other servers:

    SJS vs. SJS / TEN overlap vs. TEN

    SCORTEN severity of illness score

    Clinical features
    • Drug associated cases typically present one to three weeks following initiation of therapy with the offending drug; occurs more rapidly with re-challenge (N Engl J Med 1994;331:1272)
    • Fever, malaise, cutaneous and mucosal eruption
    • Cutaneous and mucosal lesions are tender
    • Nikolsky sign positive for epidermal detachment with application of tangential mechanical pressure (Orphanet J Rare Dis 2010;5:39)
    • Eruption consists of either "flat atypical target" lesions or erythematous to purpuric macules, many with central epidermal necrosis or blister formation
    • Epidermal detachment < 10% body surface area
    • Variable GI and respiratory tract involvement (Orphanet J Rare Dis 2010;5:39)
    • Potentially fatal, a medical emergency
    Diagnosis
    • Based on clinical picture plus confirmatory skin biopsy showing vacuolar interface alteration, often with epidermal necrolysis (Orphanet J Rare Dis 2010;5:39)
    Prognostic factors
    Case reports
    Treatment
    Clinical images

    Images hosted on other servers:

    Cutaneous and mucosal lesions

    Widespread macules

    Gross description
    • Cutaneous and mucosal lesions
    • Eruption consists of either "flat atypical target" lesions or erythematous to purpuric macules, many with central epidermal necrosis or blister formation
    Frozen section description
    • Helpful to confirm diagnosis of Stevens-Johnson syndrome
    • Patient below had toxic epidermal necrolysis with extensive body surface and mucosal involvement
    Frozen section images

    Contributed by Hillary Rose Elwood, M.D.

    Detached and necrotic epidermis

    Microscopic (histologic) description
    • Early lesions: apoptotic keratinocytes scattered in basal epidermis
    • Later lesions: numerous necrotic keratinocytes, full thickness epidermal necrosis and subepidermal bullae
    • Epidermal changes are often accompanied by a moderate or dense lymphocyte predominant dermal infiltrate
    • Less commonly, neutrophils and eosinophils are present (Mayo Clin Proc 2010;85:131)
    • Less common findings are red blood cell extravasation, pigment incontinence, regenerating epidermis, parakeratosis and necrosis of hair follicle (Mayo Clin Proc 2010;85:131)
    • Clinical correlation is essential to distinguish erythema multiforme, SJS and TEN, as they may look nearly identical histologically
      • Cannot reliably distinguish based on full thickness epidermal necrosis / necrolysis, because EM may have it and SJS / TEN may not, depending on the site of the biopsy
    Microscopic (histologic) images

    Contributed by Hillary Rose Elwood, M.D.

    Detached epidermis with full thickness necrosis and dyskeratotic cells



    Images hosted on other servers:

    Basal cell vacuolar change

    Full thickness necrosis of epidermis

    Differential diagnosis

    Subacute cutaneous lupus erythematosus
    Definition / general
    • Lupus erythematosus is an autoimmune disease that involves a pathological spectrum ranging from a skin limited disease to a severe multisystemic illness
    • Skin lesions can be classified as specific (eruptions diagnostic of or unique to lupus erythematosus) and nonspecific (commonly seen in but not unique to lupus erythematosus) (J Am Acad Dermatol 1981;4:471)
    • Specific skin lesions include acute, subacute and chronic cutaneous lupus erythematosus
    Essential features
    • Subacute cutaneous lupus erythematosus (SCLE) presents clinically as a photosensitive, nonscarring, nonatrophy producing eruption
    • Associated anti-SSA / Ro autoantibodies (40 - 100%); anti-SSB / La antibodies are often present as well
    • Microscopic features include vacuolar interface changes, apoptotic keratinocytes and superficial (or superficial and deep) perivascular lymphocytic infiltration; dyskeratotic keratinocytes can extend into upper spinous layers (this last feature is more specific to SCLE but is a rare finding)
    • While certain microscopic features may suggest a particular subtype of cutaneous lupus erythematosus, significant overlap in histopathologic findings exist between subtypes and final diagnosis requires clinicopathologic and laboratory correlation
    ICD coding
    • ICD-10: L93.1 - subacute cutaneous lupus erythematosus
    • ICD-11: EB50 - subacute cutaneous lupus erythematosus
    Epidemiology
    • F > M (6:1)
    • More frequent in young to middle aged women
    • Neonatal forms of SCLE may occur in infants whose mothers have anti-SSA / Ro or U1RNP autoantibodies (Indian Dermatol Online J 2014;5:7)
    Sites
    Pathophysiology
    • Genetic predisposition, drug biotransformation and epigenetic dysregulation in different immune cells, NETosis and participation of the innate immune system (Curr Opin Rheumatol 2018;30:490)
    • Adaptive immune system creates autoantibodies that react inaccurately to self antigens that are present in cellular debris after apoptosis; this process results in the activation and recruitment of B and T cells and the production of immune complexes, which cause direct tissue injury (Int J Womens Dermatol 2019;5:320)
    • Photosensitivity is correlated with association to anti-SSA / Ro autoantibodies (~70%) (J Eur Acad Dermatol Venereol 2016;30:2097)
    Etiology
    • Often idiopathic
    • Sun exposure (UVA and UVB)
    • Drugs: 20 - 30% of cases are drug induced; at least 100 agents have been reported (terbinafine, TNFα inhibitors, antiepileptics and proton pump inhibitors are most frequently associated) (JAAD Case Rep 2019;5:271)
    • Transplacental passage of autoantibodies (neonatal lupus erythematosus)
    Clinical features
    • Erythematous annular variant: raised pink-red borders and central clearing
    • Papulosquamous variant: chronic psoriasiform or eczematous appearance (G Ital Dermatol Venereol 2018;153:216)
    • Nonscarring, nonatrophic; commonly resolves with hypopigmentation
    • Patient can have a mild systemic illness with musculoskeletal complaints and serological abnormalities
    • Approximately 50% of patients with SCLE meet criteria for systemic lupus erythematosus; despite this SLE categorization, it is believed that these patients with SCLE / SLE have a lower incidence of severe central nervous system disease, renal disease and severe systemic vasculitis than patients with SLE without SCLE (J Am Acad Dermatol 2016;74:862)
    • Concomitant Sjögren syndrome may occur due to shared presence of anti-SSA / Ro antibodies (Mayo Clin Proc 2017;92:406)
    Diagnosis
    • Clinicopathologic correlation is indispensable
    • Blood tests: ANA, anti-Ro / SSA, anti-La / SSB, anti-dsDNA
    • Skin biopsy
    Laboratory
    Prognostic factors
    Case reports
    Treatment
    Clinical images

    Contributed by Maria E. Mazzei, M.D.
    Papulosquamous lesions

    Papulosquamous lesions

    Annular lesions

    Annular lesions

    No scarring or atrophy

    No scarring or atrophy

    Face lesions; midface preserved

    Face lesions; midface preserved

    Microscopic (histologic) description
    • Interface dermatitis with variable degrees of basal layer vacuolization and scattered cytoid bodies (Lupus 2012;21:577)
      • Occasionally, basal layer vacuolation and keratinocyte apoptosis may be so severe that full thickness epidermal necrosis may result, generating a differential diagnosis of erythema multiforme
      • Erythema multiforme-like lesions may occur, showing exuberant interface tissue reaction (Rowell syndrome)
    • Dyskeratotic keratinocytes extending into upper spinous layers is a very characteristic but rare finding of SCLE (J Cutan Pathol 2001;28:1)
    • Hyperkeratosis / parakeratosis may be present
    • Sometimes focal hypergranulosis
    • Superficial and sometimes superficial and deep perivascular lymphocytic infiltrate (Dermatol Ther (Heidelb) 2021;11:131)
    • Periadnexal mononuclear cell infiltrate may be present (Dermatol Ther 2021;11:131)
    • Dermal mucin, which may be highlighted with Alcian blue or colloidal iron
    • Compared with idiopathic SCLE, drug induced variants are more commonly associated with leukocytoclastic vasculitis and have less mucin deposition (J Am Acad Dermatol 2019;81:403)
    • Epidermal atrophy, follicular plugging, basement membrane thickening, dermal mucin and pigment incontinence may be present but less prominent than in chronic lupus erythematosus
    Microscopic (histologic) images

    Contributed by Maria E. Mazzei, M.D.
    Epidermal thinning and vacuolar changes

    Epidermal thinning and vacuolar changes

    Dyskeratotic keratinocytes

    Dyskeratotic keratinocytes

    Vacuolar interface dermatitis Vacuolar interface dermatitis Vacuolar interface dermatitis

    Vacuolar interface dermatitis

    Immunofluorescence description
    • Immunofluorescence is not regularly used to confirm the diagnosis; however, it might help when a clinicopathological correlation is lacking
    • Direct immunofluorescence: continuous granular deposition of IgG, IgM and C3 along the dermal epidermal junction (J Invest Dermatol 1992;99:251)
    • Lupus band: 65 - 80% of lesional skin and 20% nonlesional skin (Histopathology 2022;80:233)
    Sample pathology report
    • Skin, arm, biopsy:
      • Cutaneous lupus erythematosus (see comment)
      • Comment: Epidermal thinning with hyperkeratosis. Vacuolar interface dermatitis with dyskeratotic keratinocytes that extend into upper spinous layers. Superficial perivascular lymphocytic infiltrate. Correlate with clinical and laboratory testing. These findings are more consistent with SCLE subtype.
    Differential diagnosis
    • Acute lupus erythematosus:
      • Specific lesions include central face malar violaceous erythema that resolves without scarring (butterfly rash), photosensitive maculopapular rash, blistering rash (acute toxic epidermal necrolysis-like ACLE due to extensive interface dermatitis) mainly in photo exposed areas, mucosal erosions and ulcerations
      • Histopathology shows vacuolar interface dermatitis with relatively mild lymphocytic inflammation; there may be dermal edema and microhemorrhage
    • Chronic cutaneous lupus:
      • Erythematous and scaly plaques located mainly in the face and scalp, that may lead to scarring / alopecia if not treated
      • Microscopic findings include epidermal atrophy, hyperparakeratosis, follicular keratotic plugging and basement membrane thickening with vacuolar interface changes
      • Additionally, the density of the infiltrate tends to be heaviest in chronic cutaneous lupus
    • Dermatomyositis:
      • Heliotrope rash, swelling of the eyelids, shawl sign, Gottron papules, photosensitive rash, Gottron sign, holster sign, mechanic hands and Raynaud phenomenon
      • Microscopic findings include interface dermatitis that can resemble lupus erythematosus, increased dermal mucin, sparse perivascular lymphocytic infiltrate
    • Erythema multiforme:
      • Triggered by infections (mainly herpes simplex virus), medications, vaccines
      • Round, erythematous papules, which later develop into target lesions: central area with epidermal necrosis surrounded by a lighter edematous area with a peripheral erythematous margin
      • Microscopic changes include interface changes with mild to moderate infiltrate of lymphocytes; there may also be subepidermal bullae and epidermal necrosis
    Board review style question #1


    A 35 year old woman presents with multiple nonscarring erythematous annular plaques that arise in photo exposed areas 8 weeks after introducing terbinafine for treatment of onychomycosis. A biopsy of one of the lesions was performed (shown above). What is the most likely diagnosis?

    1. Chronic lupus erythematosus
    2. Dermatomyositis
    3. Erythema multiforme
    4. Stevens-Johnson syndrome
    5. Subacute cutaneous lupus erythematosus
    Board review style answer #1
    E. Subacute cutaneous lupus erythematosus

    Comment Here

    Reference: Subacute cutaneous lupus erythematosus
    Board review style question #2

    A 50 year old man with a history of recent onset hypertension treated with enalapril presented with a 1 month history of annular and erythematous polycyclic plaques with peripheral scale that were located in sun exposed areas. No other systemic manifestations were found. Serologic test results were positive for ANA, anti-Ro / SSA and anti-La / SSB, while negative for ds-DNA, anti-histone and anti-Smith. Which histopathological finding that can be seen in this image is more characteristic of the patient's clinical type?

    1. Dyskeratotic keratinocytes extending to upper layers
    2. Epidermal atrophy
    3. Pigment incontinence
    4. Superficial perivascular lymphocytic infiltrate
    5. Vacuolar interface dermatitis
    Board review style answer #2
    A. Dyskeratotic keratinocytes extending to upper layers

    Comment Here

    Reference: Subacute cutaneous lupus erythematosus

    Subcorneal pustular dermatosis
    Definition / general
    • Rare, recurrent, noninfective, nonautoimmune pustular eruption of unknown etiology
    Terminology
    • Also called Sneddon-Wilkinson disease
    Epidemiology
    • Associated with benign or malignant IgA paraproteinemia, rheumatoid arthritis, SLE, bullous pemphigoid, squamous cell carcinoma of the bronchus and metastatic gatrinomas
    Clinical features
    • Large pustules on trunk and body folds
    Treatment
    Clinical images

    Images hosted on other servers:

    Symmetric and diffusely scattered vesicopustules on trunk and extremities

    Microscopic (histologic) description
    • Large unilocular subcorneal pustule containing a small amount of neutrophils and minimal acantholysis
    • May be psoriasiform
    • Superficial dermis has neutrophils and lymphocytes
    Microscopic (histologic) images

    Images hosted on other servers:

    Subcorneal pustule containing a few neutrophils and acantholytic cells

    Differential diagnosis
    • Impetigo
    • Pustular psoriasis
    • Staphylococcal scalded skin syndrome (SSSS)

    Subcutaneous fat necrosis of newborn
    Definition / general
    • Uncommon disorder of infants in good health with lesions on trunk, extremities, buttocks, cheeks
    • Hypercalcemia has been reported in some cases; thrombocytopenia, hypoglycemia and hypertriglyceridemia may also develop
    • Lactic acidosis and hyperferritinemia were each present in one case
    • Numerous possible causes, including obstetrical trauma, hypothermia, asphyxia, anemia, forceps delivery, macrosomia, exposure to active or passive smoking during pregnancy, prostaglandin E administration, maternal exposure to cocaine or calcium channel blockers
    Clinical features
    • Indurated areas and distinct nodules with a predilection for the cheeks, shoulders, buttocks, thighs and calves
    Case reports
    Microscopic (histologic) description
    • Basophilic fat necrosis, clusters of needle like clefts in lipocytes and histiocytes
    • Clefts may contain refractile crystals
    • Variable granulomatous inflammation and small calcium deposits
    • Normal epidermis and dermis with an underlying lobular panniculitis
    • Focal fat necrosis is present and this may lead to fat cyst formation
    • Lymphocytes, histiocytes, foreign body giant cells and sometimes a few eosinophils wedged between the fat cells
    • Many fat cells retain their outline but contain fine, eosinophilic cytoplasmic strands and granules, between which are narrow clefts radiating from a point near the periphery of the cell; clefts contain doubly refractile crystals, representing triglycerides, on frozen section
    • Similar fine, needle-like crystals can be seen in giant cells
    • Older lesions have fibrosis between fat cells
    Electron microscopy description
    • Intact and necrotic fat cells containing needle-shaped crystals arranged radially or in parallel
    • Dense granular material is also present in necrotic fat cells, which are surrounded by macrophages
    Additional references

    Syphilis
    Definition / general
    • Syphilis is primarily a sexually transmitted disease
    • Clinically, in the secondary stage, presents with a papulosquamous eruption and has many histologic mimickers, including psoriasis
    Essential features
    • Lichenoid superficial and deep perivascular and periadnexal lymphohistiocytic inflammation with admixed plasma cells
    • Up to 33% of biopsies may lack plasma cells
    • Immunohistochemistry is more sensitive than silver stains
    Terminology
    • Lues venerea, Cupid's disease
    ICD coding
    • ICD-10: A51.39 - other secondary syphilis of the skin
    Epidemiology
    • Transmitted via sexual contact
    • Can also be transmitted congenitally and rarely via blood products
    • More prevalent in men than in women, especially ages 20 - 29
    • Higher incidence in gay and bisexual men (MMWR Morb Mortal Wkly Rep 2014;63:402)
    Sites
    Pathophysiology
    Etiology
    • Treponema pallidum, highly motile coiled spirochete
    Clinical features
    • Primary syphilis: painless chancre with nontender lymphadenopathy 1 - 3 weeks after exposure
    • Secondary syphilis:
      • Papulosquamous thin papules on the trunk and extremities, palms and soles, fever and adenopathy
      • Rash may resemble a drug eruption, pityriasis rosea and psoriasis
      • May present as moth eaten alopecia on the scalp, mucous patches on tongue
    • Tertiary syphilis: may present with gummatous lesions, neurologic or cardiovascular symptoms (Infect Dis Clin North Am 2013;27:705)
    • Lymph node: solitary inguinal lymphadenopathy associated with syphilitic chancre
    Diagnosis
    • T. pallidum can be visualized using darkfield microscopy
    • Blood and cerebrospinal fluid (CSF) analysis
    • Skin biopsy for H&E, special stain and immunohistochemical analysis (Infect Dis Clin North Am 2013;27:705)
    Laboratory
    • Nontreponemal and treponemal tests are performed on blood serum
      • Nontreponemal tests: rapid plasma reagin (RPR) test or venereal disease research laboratory (VDRL) test
      • Treponemal tests: Treponema pallidum particle agglutination assay (TPPA) or fluorescent treponemal antibody absorption (FTA-ABS) test
    • Cerebrospinal fluid (CSF) analysis
      • Lymphocytic pleocytosis and elevated cerebrospinal fluid total protein
      • Cerebrospinal fluid venereal disease research laboratory test confirms diagnosis of neurosyphilis
      • Of note, cerebrospinal fluid fluorescent treponemal antibody absorption test has a high false positive rate but is more sensitive than cerebrospinal fluid venereal disease research laboratory test
    Prognostic factors
    • Favorable with early treatment
    Case reports
    Treatment
    Clinical images

    Contributed by Mark R. Wick, M.D.

    Secondary

    Gumma

    Microscopic (histologic) description
    • Primary syphilis: ulcer with lymphoplasmacytic inflammation
    • Secondary syphilis:
      • Lichenoid superficial and deep perivascular and periadnexal lymphohistiocytic inflammation with admixed plasma cells
      • Plasma cells may be absent in 33% of biopsies
      • Eosinophils can be present
      • Marked endothelial swelling and proliferation of blood vessels can be seen
      • Commonly identified are vacuolar interface change, acanthosis or lymphocyte exocytosis
      • In late secondary syphilis, granulomatous inflammation may be present
      • Microabscess formation of the hair follicle has also been seen
      • Condyloma lata (lesions on genital skin) show marked epidermal hyperplasia and inflammatory infiltrate akin to secondary syphilis
    • Tertiary syphilis: large areas of necrosis with peripheral mixed inflammatory infiltrate and Langhans giant cells 
    • Lymph node:
      • Primary syphilis:
        • Capsular and pericapsular fibrosis, follicular hyperplasia, diffuse interfollicular plasma cells, prominent endarteritis outside the capsule
        • Noncaseating granulomas and abscesses are rare
        • Spirochetes easiest to identify within blood vessel walls
      • Secondary / tertiary syphilis: follicular hyperplasia
    • References: J Cutan Pathol 2004;31:595, J Am Acad Dermatol 2015;73:1025, J Am Acad Dermatol 2020;82:156
    Microscopic (histologic) images

    Contributed by Silvija P. Gottesman, M.D., Hillary Rose Elwood, M.D., Mark R. Wick, M.D. and AFIP images

    Dense lymphohistiocytic inflammation

    Plasma cells

    Psoriasiform hyperplasia

    Secondary syphilis

    Treponemal spirochetes


    Numerous spirochetes

    Dense plasma cell infiltrate

    Steiner stain, secondary

    Luetic (syphilitic) lymphadenopathy

    Virtual slides

    Images hosted on other servers:

    Secondary syphilis on acral skin

    Positive stains
    Negative stains
    • Not relevant to the diagnosis
    Molecular / cytogenetics description
    • PCR based techniques can be used on tissue
    Sample pathology report
    • Skin, punch biopsy:
      • Psoriasiform lymphohistiocytic inflammation with plasma cells (see comment)
      • Comment: There is psoriasiform epidermal hyperplasia with superficial and deep perivascular lymphohistiocytic infiltrate. Admixed plasma cells are also noted. The infiltrate forms a band beneath the hyperplastic epidermis and envelopes some of the adnexal structures. Endothelial swelling of the superficial vascular plexus is present. A treponemal immunohistochemical stain shows numerous coiled spirochetes in the lower half of the epidermis. These findings are diagnostic of syphilis.
    Differential diagnosis
    • Pityriasis lichenoides et varioliformis acuta:
      • Differentiation on histologic grounds is difficult as some cases may also have plasma cells
      • Treponema-, silver-
    • Pityriasis rosea:
      • Similar clinically
      • Basilar spongiosis and extravasated red blood cells are more likely
      • Neutrophils in the stratum corneum are less common
    • Mycosis fungoides:
      • Papillary dermal fibrosis and complete vacuolar changes are more often seen
      • While late lesions may have plasma cells, they are rarely seen in early lesions
    • Alopecia areata:
      • Also can show peribulbar lymphocytic inflammation
      • Peribulbar plasma cells will not be present
    Board review style question #1
    A 28 year old man presents with fever, sore throat, lymphadenopathy and a skin rash. Which of the following is the most important cutaneous finding that should prompt suspicion of syphilis?

    1. Erythematous scaly papules on the palms and soles
    2. Genital mucocutaneous ulceration
    3. Maculopapular eruption on the trunk and extremities
    4. Mucosal patches on the bilateral oral commissures
    Board review style answer #1
    A. Erythematous scaly papules on the palms and soles

    Comment Here

    Reference: Syphilis
    Board review style question #2
    A punch biopsy specimen from a 26 year old woman with a rash shows a superficial and deep perivascular and periadnexal lymphocytic inflammation. The presence of which of the following should prompt the pathologist to order treponemal immunohistochemical stain?



    1. Histiocytes
    2. Mucin
    3. Neutrophils
    4. Plasma cells
    Board review style answer #2
    D. Plasma cells

    Comment Here

    Reference: Syphilis

    Syphilitic alopecia (pending)

    Tattoo
    Definition / general
    • Dye injected into dermis is phagocytosized by macrophages, which are permanent
    • Image does fade over time
    • Complications include infections introduced at time of tattooing; cutaneous diseases that localize in tattoos, often in a koebner-type phenomenon; allergic reactions to tattoo pigments and photosensitivity reactions
    Clinical features
    Lymph nodes:
    • May cause lymphadenopathy, usually resolves spontaneously
    • Pigment initially within keratinocytes, fibroblasts, macrophages and mast cells; later only within dermal fibroblasts surrounded by connective tissue; eventually tattoo ink appears in regional lymph nodes
    • Melanoma patients should be questioned regarding history of tattoos, particularly prior to sentinel node treatment (Int Semin Surg Oncol 2005;2:28)
    • Histologic confirmation of melanoma in nodes is necessary to avoid unnecessary surgery (Dermatol Surg 1996;22:92)
    Treatment
    • Laser therapy
    Microscopic (histologic) description
    • Tattoo pigments are easily visualized in tissue sections
    • After several weeks, they localize around vessels in the upper and mid-dermis in macrophages and fibroblasts
    • Extracellular deposits of pigment are also found between collagen bundles; the pigment is generally refractile, but not doubly refractile
    • No foreign body granulomatous reaction except in presence of other severe reactions
    • Hypersensitivity reactions vary from a diffuse dermal lymphohistiocytic infiltrate with plasma cells and eosinophils, to a lichenoid reaction sometimes with associated epithelial hyperplasia
    • Sarcoidal granulomas, a granuloma annulare-like reaction, vasculitis, pseudolymphomatous patterns and scarring may be present
    • Rare features are a morphea-like reaction, epidermal spongiosis and pseudoepitheliomatous hyperplasia

    Telogen effluvium (pending)
    [Pending]

    Tinea nigra (pending)

    Tinea versicolor
    Definition / general
    Microscopic (histologic) description
    • Variably pigmented macules of all sizes, with orthokeratotic hyperkeratosis, yeast spores and pseudohyphae within stratum corneum
    • Short hyphae and spores (spaghetti and meatballs) with GMS or PAS stains

    • Note: presence of fungi does not rule out coexisting inflammatory and neoplastic disorders
    Microscopic (histologic) images

    Images hosted on other servers:

    Malassezia furfur


    Traction alopecia (pending)
    [Pending]

    Transient acantholytic dermatosis
    Definition / general
    • Transient papulovesicular eruption on the trunk
    • Sporadic, reactive condition
    • Initially characterized by Grover in 1970 (Arch Dermatol 1970;101:426)
    Essential features
    • Middle aged to elderly, white, mostly male patients
    • Multiple pruritic papulovesicular eruption
    • Pathologically can mimic other vesiculobullous disease and epithelial neoplasms
    • Correlation with clinical findings is essential
    Epidemiology
    Sites
    Etiology
    Clinical features
    • Pruritic papules, papulovesicles, plaques and bullae lesions
    • Lesions distributed mainly on the trunk and proximal extremities but rarely can appear on the scalp, on the face and in the mouth (these lesions resembled aphthae)
    • No systemic symptoms
    Case reports
    Treatment
    • Treatment options include topical steroids, retinoids and vitamin D analogues and systemic steroids, retinoids and methotrexate
    • 5-aminolevulinic acid photodynamic therapy (ALA-PDT) shows rapid, dramatic improvement in symptoms, allowing for significant improvement in quality of life (Dermatol Surg 2013;39:960)
    Clinical images

    Contributed by Omar P. Sangueza, M.D. and Mark R. Wick, M.D.

    Diffuse papular erythematous eruption involving the trunk

    Erythematous, keratotic papules and plaques

    Breast skin

    Microscopic (histologic) description
    • Suprabasal acantholysis with vesicle formation is the principal microscopic finding
    • Elongation of rete ridges with focal acantholysis and eosinophils may be the only early changes in Grover disease
    • Main 4 histopathological patterns are:
      • Pemphigus vulgaris / foliaceous-like: predominantly limited to suprabasal acantholysis, with basal keratinocytes attached to basement membrane forming a characteristic tombstone appearance
      • Darier-like: suprabasal acantholysis of keratinocytes with scattered apoptotic or dyskeratotic cells within various levels of the epidermis
      • Spongiotic: edema within epidermis, causing separation of keratinocytes and prominent intracellular bridges
      • Hailey-Hailey-like: suprabasal acantholysis of all levels of the epidermis without significant dyskeratosis (dilapidated brick wall appearance)
    • More than one pattern can be seen in the same setting
    • Other newly described histopathological pattern include: (Am J Dermatopathol 2010;32:541)
      • Porokeratotic: presence of oblique columns of parakeratosis over areas of epidermis devoid of granular layer, with occasional dyskeratotic cells in the upper epidermal layers and a tendency to basal vacuolization
      • Lentiginous: elongation of the rete ridges with some basal hyperpigmentation resembling a solar lentigo or Dowling-Degos disease
      • Vesicular: intraepidermal vesicles with minimal spongiosis or acantholysis on their sides
      • Lichenoid: vacuolar damage and slight lymphocytic infiltrate at the dermoepidermal junction, with occasional dyskeratosis and barely identifiable acantholysis
      • Dysmaturative: disordered keratinocyte maturation with nuclear pleomorphism and slight hyperchromasia (Am J Dermatopathol 2018;40:642)
      • Epidermolytic hyperkeratosis-like pattern (Am J Dermatopathol 2014;36:358)
    Microscopic (histologic) images

    Contributed by Omar P. Sangueza, M.D.

    Suprabasal vesicle formation

    Spongiotic pattern

    Follicular induction

    Atypical nuclei

    Negative stains
    Immunofluorescence description
    Molecular / cytogenetics description
    Differential diagnosis
    Board review style question #1

    A white, elderly male presented with multiple pruritic papule involving the trunk. Which of the following is true regarding the diagnosis of this lesion?

    1. Associated mucosal lesion is diagnostic
    2. ATP2A2 gene abnormality is a common finding
    3. Net pattern of IgG and C3 on direct immunofluorescence
    4. Sparing of acrosyringium can be a helpful feature
    Board review style answer #1
    D. Sparing of acrosyringium can be a helpful feature. Transient acantholytic dermatosis (Grover disease) can mimic variable vesiculobullous entities, especially pemphigus vulgaris, Darier and Hailey-Hailey. Differentiation depends on clinical presentation, microscopic findings and direct immunofluorescence findings. One microscopic finding that can help to differentiate between pemphigus vulgaris and Grover disease is adnexal structure and acrosyringium involvement in the former.

    Comment here

    Reference: Transient acantholytic dermatosis
    Board review style question #2
    Which of the following case scenarios is typical for transient acantholytic dermatosis (Grover disease)?

    1. 17 year old man with crusted popular lesions involving head and neck
    2. 20 year old woman with flaccid blisters involving the axillae and inframammary region
    3. 40 year old woman with oral blisters and flaccid bullae involving the trunk
    4. 60 year old man with erythematous papule involving the trunk
    Board review style answer #2
    D. 60 year old man with erythematous papule involving the trunk. Transient acantholytic dermatosis (Grover disease) is most common in middle aged to elderly, Caucasian men. It most commonly manifest as a vesicular eruption of the trunk.

    Comment here

    Reference: Transient acantholytic dermatosis

    Trichosporon
    Definition / general
    • Fungi common in soil that also colonize human skin and GI tract
    • Cause superficial infections, such as white piedra (distal infection of hair shaft) and disseminated yeast infections (eMedicine)
    Case reports
    Clinical images

    Images hosted on other servers:

    64 year old man with
    posttransplant,
    disseminated
    infection

    White piedra - nodules on scalp hair

    Microscopic (histologic) images

    Images hosted on other servers:

    H&E and GMS of skin biopsy

    Additional references

    Trichotillomania (pending)
    [Pending]

    Tuberculosis
    Definition / general
    • Cutaneous disease is uncommon in U.S
    Epidemiology
    • Primary (inoculation) tuberculosis of the skin is the cutaneous analogue of the pulmonary ghon focus
    • Primary cutaneous tuberculosis may be associated with tattooing, ritual circumcision, nose piercing, inoculation of homeopathic solutions or injury by contaminated objects to laboratory workers, surgeons or prosectors; there may be no obvious source of infection

    • Erythema induratum of Bazin: also called nodular vasculitis if no coexisting tuberculosis; rare, more common in past; presents as recurrent tender subcutaneous nodules on calves of women with tuberculin hypersensitivity
    • Lichen scrofulosorum: lichenoid eruption of minute papules in children and adolescents with tuberculosis (Indian J Dermatol Venereol Leprol 2010;76:494)
    • Lupus vulgaris: most common form of re-infection tuberculosis, occurring predominantly in young adults; refers to painful cutaneous TB skin lesions with a nodular appearance, usually near the nose, eyelids, lips, cheeks and ears; untreated lesions may develop into disfiguring skin ulcers ("lupus" historically meant an ulcerative skin disorder), and are associated with squamous cell carcinoma
    • Tuberculids: heterogeneous group of cutaneous lesions due to TB infection elsewhere in the body
    Clinical features
    • Red indurated papule appears 1 - 3 weeks after penetrating injury; papule ulcerates and forms a so-called "tuberculoid chancer"
    Case reports
    Clinical images

    Images hosted on other servers:

    Adult woman with
    erythrocyanotic
    circulation and
    ulcerated lesions

    Microscopic (histologic) description
    • Early lesions have a mixed dermal infiltrate of neutrophils, lymphocytes and plasma cells; followed by superficial necrosis and ulceration
    • Weeks later, tuberculoid granulomas form, with variable caseation necrosis
    • Acid-fast bacilli are usually easy to find in the early lesions, but are rare once granulomas develop
    • Erythema induratum of Bazin: granulomatous vasculitis affecting subcutaneous large vessels (may need multiple serial sections to identify); also granulomatous inflammation of lobules of subcutis; lesions may contain acid-fast bacilli by PCR
    Microscopic (histologic) images

    Contributed by Angel Fernandez-Flores, M.D., Ph.D.

    Positive stains

    Tunga penetrans
    Definition / general
    • Sand flea that burrows into human skin and skin of mammals, causing the disease tungiasis
    • Parasite is common in tropical locations (South America, Caribbean, Africa, Pakistan, west coast of India)
    • Single flea bite causes local discomfort
    • Multiple bites cause debilitation and secondary infections (tetanus, gas gangrene)
    Clinical features
    • Nodular lesion
    Microscopic (histologic) description
    • Skin biopsies show exoskeleton, hypodermal layer, trachea, digestive tract and developing eggs
    • Striated muscle in 47%, posterior end in 43%
    • Head usually absent
    • The dermis contains mixed inflammatory cell infiltrate of lymphocytes, plasma cells and eosinophils

    Urticaria (hives)
    Definition / general
    • Cutaneous reaction pattern characterized clinically by transient, often pruritic, edematous, erythematous papules or wheals due to localized mast cell degranulation and dermal microvascular hyperpermeability
    • Individual lesions fade within 24 hours, although episodes may last days to months (Allergy 2018;73:1393, J Eur Acad Dermatol Venereol 2018;32:282)
    • Chronic urticaria is defined as recurrent whealing that lasts for more than 6 weeks
    Essential features
    • Urticaria is characterized by transient skin or mucosal swellings due to plasma leakage
    • Superficial dermal swellings are termed wheals and deep swellings of the skin or mucosa are termed angioedema
    • Wheals are characteristically pruritic and pink or pale in the center, whereas angioedema is often painful, less well defined and shows no color change
    Terminology
    • Urticaria / hives
      • Superficial dermal swellings are termed wheals: characteristically pruritic and pink or pale in the center
      • Deep swellings of the skin or mucosa are termed angioedema: often painful, less well defined and showing no color change
    ICD coding
    • ICD-10: L50.9 - urticaria, unspecified
    Epidemiology
    • 15% of population on at least 1 occasion in their lifetime (Allergy 2011;66:317)
    • F:M = 2:1
    • Children and young adults more likely to have acute urticaria
    • Middle aged adults for chronic urticaria
    Sites
    • Acute: disseminated
    • Chronic:
      • Spontaneous: disseminated
      • Inducible: areas of exposure (sun exposed sites, colder areas, physical pressure, etc.)
    Pathophysiology
    • Stimuli → mast cell degranulation → histamine and other proinflammatory mediators release → vasodilation and increased permeability of postcapillary venules to large plasma proteins, including albumin and immunoglobulins → transient skin or mucosal swellings due to plasma leakage
    Etiology
    • Approximately 75% of cases are idiopathic
    • Infections (typically viral, including SARS-CoV2)
    • Collagen vascular disorders (SLE, autoinflammatory disorders, etc.)
    • Malignancy (most common: Hodgkin lymphoma)
    • IgE mediated allergies or IgE independent allergies (contrast dye, opiates, aspirin, ACE inhibitors - suppresses prostaglandins)
    • Hereditary angioedema (deficiency of C1 esterase inhibitor causes uncontrolled complement component activation)
    • Physical stimuli (heat, cold, pressure, vibration, etc.)
    • Reference: Allergy 2018;73:1393
    Diagrams / tables

    Contributed by Pedro Rodríguez-Jiménez, M.D.
    Distribution of clinical presentation of patients with urticaria

    Distribution of clinical presentation of urticaria patients

    Clinical features
    • Acute spontaneous urticaria can have an allergic basis but chronic spontaneous urticaria usually does not
    • While urticaria often develops in the setting of anaphylaxis, the vast majority of patients who present with wheals rarely progress to anaphylaxis
    Laboratory
    Prognostic factors
    • None reliable to date
    Case reports
    Treatment
    • Acute:
      • Second generation antihistamines
      • Oral corticoisteroids (only if angioedema or highly symptomatic)
    • Chronic:
      • Second generation antihistamines
      • Oral corticoisteroids (only if angioedema or highly symptomatic)
      • Omalizumab (anti IgE antibody)
    • Reference: Allergy 2018;73:1393
    Clinical images

    Contributed by Pedro Rodríguez-Jiménez, M.D.
    Angioedema

    Angioedema

    Wheals Wheals Wheals Wheals

    Wheals

    Microscopic (histologic) description
    • Age of lesion and nature of evoking stimulus may both influence type and intensity of inflammatory response
    • Changes most marked in upper dermis but involvement of the deep dermis may be present
    • Mild dermal edema (separation of the collagen bundles)
    • Dilatation of small blood vessels, lymphatics and often swelling of their endothelium
    • Cellular infiltrate usually mild and perivascular; lymphocytes and few eosinophils in most cases
    • Neutrophils are often noted in early lesions but relatively sparse; more prominent in physical urticarias
    • Neutrophils and sometimes eosinophils in the lumen of small vessels are seen in early urticarias in the upper dermis
    • No fibrinoid changes, hemorrhage or leukocytoclasis
    • In neutrophilic urticaria, there is more diffuse interstitial dermal neutrophilia, usually mild in intensity
    • In papular urticaria, inflammatory cell infiltrate is usually heavier than in other chronic urticarias
    • Reference: Patterson: Weedon's Skin Pathology, 4th Edition, 2015
    Microscopic (histologic) images

    Contributed by Patricia Muñoz-Hernández, M.D.
    Dermal edema Dermal edema

    Dermal edema

    Perivascular lymphocytes and eosinophils

    Perivascular lymphocytes and eosinophils

    Perivascular infiltrates

    Perivascular infiltrates

    Perivascular and interstitial infiltrate

    Perivascular and interstitial infiltrate


    Interstitial infiltrate of eosinophils

    Interstitial infiltrate of eosinophils

    Perivascular infiltrate of lymphocytes

    Perivascular infiltrate of lymphocytes

    Perivascular and interstitial infiltrate

    Perivascular and interstitial infiltrate

    Perivascular neutrophils and eosinophils

    Perivascular neutrophils and eosinophils


    Interstitial neutrophils Interstitial neutrophils

    Interstitial neutrophils

    Perivascular infiltrate

    Perivascular infiltrate

    Intravascular neutrophils Intravascular neutrophils

    Intravascular neutrophils

    Virtual slides

    Images hosted on other servers:
    Neutrophilic urticaria

    Neutrophilic urticaria

    Perivascular infiltrate, interstititial eosinophils

    Perivascular infiltrate, interstitial eosinophils

    Sample pathology report
    • Left arm, biopsy:
      • Superficial perivascular dermatitis with dermal edema and eosinophils; the histologic features are consistent with urticaria (see comment)
      • Comment: The upper dermis shows interstitial edema with mild perivascular infiltrate of lymphocytes and relatively numerous eosinophils. Neutrophils are seen within vessels lumens without vasculitis. Clinical correlation is recommended.
    Differential diagnosis
    • Histology of urticaria is nonspecific, clinicopathological correlation is needed
    • Polymorphous light eruption:
      • More edema in upper or papillary dermis
    • Drug induced exanthem:
      • Mild spongiosis and discrete basal layer vacuolization
    • Arthropod reaction:
      • Epidermal changes, wedge shaped superficial and deep mixed inflammatory infiltrate
    • Viral exanthem:
      • Can have very similar appearance to urticaria; clinical correlation is required
    • Urticarial vasculitis:
      • Endothelial swelling, leukocytoclasis, extravasated erythrocytes and fibrin deposition
      • The main differential feature in urticarial vasculitis is the presence of fibrinoid necrosis of the dermal capillaries in contrast to the absence of vascular wall damage in urticaria
    Board review style question #1

    Urticaria is shown in the image above. Which is true of this lesion?

    1. Individual lesions fade within 100 hours, although episodes may last days to months
    2. This disease is characterized by transient skin or mucosal swellings due to plasma leakage
    3. Vasculitis with fibrin deposition is characteristically seen
    4. Typical lesions known as wheals are not usually pruritic
    5. The biopsy commonly shows epidermal changes, such as parakeratosis, acanthosis, spongiosis and vacuolar basal degeneration
    Board review style answer #1
    B. This disease (urticaria) is characterized by transient skin or mucosal swellings due to plasma leakage. Due to a stimulus, there is mast cell degranulation with release of histamine and other proinflammatory mediators, which results in vasodilation and increased permeability of postcapillary venules to large plasma proteins, including albumin and inmmunoglobulins. Option A is incorrect since individual lesions fade within 24 hours, although episodes may last days to months. Option C is incorrect since there is no vasculitis in urticaria. No fibrinoid changes, hemorrhage or leukocytoclasis are seen. Option D is incorrect because wheals are characteristically pruritic and pink or pale in the center, whereas angioedema is often painful, less well defined and shows no color change. Option E is incorrect since epidermal changes are not usually seen. In urticaria, typical changes include dermal edema and lymphocytes perivascular infiltrate with few eosinophils.

    Comment Here

    Reference: Urticaria (hives)
    Board review style question #2
    Which of the following is true about the pathology of urticaria?

    1. Neutrophils are never found on biopsies
    2. Perivascular infiltrate is due only to eosinophils
    3. When neutrophils are found, they are perivascular
    4. Neutrophils can be found interstitially
    5. Lymphocytes appear only rarely in biopsies of urticaria
    Board review style answer #2
    D. Neutrophils can be found interstitially. Option A is incorrect since lymphocytes, neutrophils and eosinophils can all appear on biopsies. Option B is incorrect since lymphocytes, neutrophils and eosinophils can all appear in the perivascular infiltration. Option C is incorrect because neutrophils can be perivascular or interstitial. Option E is incorrect since lymphocytes, neutrophils and eosinophils can all appear frequently.

    Comment Here

    Reference: Urticaria (hives)

    Verruga peruana
    Definition / general
    Clinical images

    Images hosted on other servers:

    Angiomatous lesions caused by Bartonella bacilliformis

    Gross description
    • Numerous dark red papules
    Microscopic (histologic) description
    • Vascular proliferative process with Rocha-Lima inclusions (large cytoplasmic inclusions of endothelial cells)
    Microscopic (histologic) images

    Images hosted on other servers:

    Peripheral blood smear

    Positive stains
    Moved to Skin nontumor - infections, per Dr. Ho, by Nat on 10Mar22

    Vesiculobullous and acantholytic reaction patterns-general
    Definition / general
    • Heterogeneous group of disorders affecting the skin or mucous membranes
    • Blisters are fluid filled cavities within or beneath the epidermis
    • Vesicles are 0.5 cm or less; bullae are greater than 0.5 cm
    • Definitive diagnosis requires clinical information
    • Large intraepidermal bullae without acantholysis may represent healed subepidermal bullae (re-epitheliazation phenomemon)
    • Blisters can be a result of spongiotic or lichenoid inflammatory reaction patterns, infection, autoimmune mediated processes, inherited / genetic mutations, paraneoplastic, drug and physical injury / external alterations
    • Blisters are categorized broadly into subcorneal, intraepidermal and subepidermal, based on location of the dermal / epidermal split
    ICD coding
    • ICD-10: L10-L14 - Bullous disorders
    Epidemiology
    • Any age, gender and race
    • Wide variety of clinical settings including autoimmune disorders, drug reactions, infections, genetic disorders and physical injury
    • Specific types of blistering disorders have predilection for certain population groups
    • Any site can be affected
    Pathophysiology
    Diagrams / tables

    Images hosted on other servers:

    Layers of skin

    Diagnosis
    • Clinical:
      • Lesion distribution (including presence or absence of mucosal lesions)
      • History and demographic information
      • Tense vs flaccid blisters can help signify at what level of skin the blister is occurring
      • See clinical algorithm for diagnosing blistering diseases
      • Nikolsky sign: elicitation of blistering by gentle mechanical pressure on the skin (J Am Acad Dermatol 2006;54:411); generally signifies a superficial blistering process (i.e. pemphigus)

    • Skin biopsy:
      • Edge of the lesion should be biopsied and submitted in formalin for H&E evaluation (Cutis 2015;95:237)
      • Second biopsy should be taken from perilesional skin that is clinically normal and submitted in Michel's solution if there is concern for an autoimmune mediated blistering disorder
      • Histologic evaluation must include:
        • Plane of separation (where is the blister ocurring?)
        • Presence or absence of acantholysis
        • Characterization of any associated inflammatory infiltrate
        • Immunofluorescent pattern

    • Direct immunofluorescence (DIF):
      • Useful in the distinction of immune mediated vs. non immune mediated blistering disorders
      • Requires submission in Michel's solution (preserves protein antigenicity without fixing the tissue)
      • Currently almost always performed on frozen tissue sections

    • Reactants and patterns of deposition:
      • IgG and C3 (pemphigus foliaceus, pemphigus vulgaris, pemphigus vegetans, paraneoplastic pemphigus, epidermolysis bullosa acquisita; bullous pemphigoid, Indian J Dermatol 2016;61:288)
        • Linear deposition along basement membrane zone: epidermolysis bullosa acquisita, bullous pemphigoid
        • Intercellular deposition: pemphigus family of diseases
      • IgA (linear IgA, dermatitis herpetiformis, Am J Dermatopathol 2016;38:283)
        • Linear deposition along basement membrane zone: linear IgA bullous dermatosis
        • Granular deposition along basement membrane zone: dermatitis herpetiformis
      • IgM and C3 (porphyria cutanea tarda, erythema multiforme)
        • Perivascular deposition: porphyria cutanea tarda
        • Globular deposits in papillary dermis: erythema multiforme
      • DIF negative (Sneddon-Wilkinson, AGEP, SSSS, Hailey-Hailey, Darier's, Grover's, TEN / SJS, infections such as herpes, impetigo)

    • Laboratory:
      • Indirect immunofluorescence: utilization of skin substrate (usually non-human) incubated with patient serum to determine pattern of autoantibody deposition
        • Provides a titer of autoantibodies, which (especially in pemphigus) correlates with disease activity
      • Antigen specific serologic testing

    • Basement membrane zone salt-split skin technique:
      • Test of choice to differentiate between subtypes of autoimmune bullous diseases with autoantibodies directed against proteins of the dermoepidermal basement membrane zone (Dermatol Clin 2011;29:365)
      • Indirect test performed by incubating normal skin substrate in hypertonic saline (which induces a subepidermal blister) followed by application of patient serum to see where autoantibodies bind (roof = bullous pemphigoid, floor = epidermolysis bullosa acquisita)
    Case reports
    Treatment
    • Antibiotics or antivirals for infectious etiologies
    • Topical corticosteroids
    • Systemic glucocorticoids
    • Immunosuppressive therapy
    • Anti-inflammatory therapy
    • Intravenous immunoglobulin
    • Reference: Dermatol Clin 2016;34:251
    Microscopic (histologic) description
    • May or may not contain a blister (separation of the layers of the skin)
    • If blister present, may be subcorneal, intraepidermal (suprabasilar) or subepidermal
    • Level and content of inflammation varies
    Microscopic (histologic) images

    Contributed by J. Taube, M.D.

    Bullous pemphigoid; eosinophils within blister



    Images hosted on other servers:

    Hailey-Hailey

    Pemphigus foliaceous



    For more images, please see condition specific pages
    Electron microscopy description
    • Electron microscopy may be useful for determining the exact level of blister split, particularly in cases of inherited blistering disorders
      • May distinguish between epidermolysis bullosa simplex vs junctional epidermolysis bullosa vs dystrophic epidermolysis bullosa vs other variants
    Molecular / cytogenetics description
    • Generally not applicable

    Virus associated trichodysplasia spinulosa
    Definition / general
    • Very rare (< 10 cases reported) cutaneous eruption of spiny papules due to polyoma virus infection
    Clinical features
    • Usually affects face and ear, less often trunk and extremities; may cause alopecia
    • Associated with kidney transplant, acute lymphocytic leukemia
    • Similar to cyclosporine-induced follicular dystrophy and pilomatrix dysplasia
    Microscopic (histologic) description
    • Abnormally maturing anagen hair follicles (no papillae, ectatic infundibula) with excessive inner root sheath differentiation, and hyperkeratotic infundibula (Am J Surg Pathol 2005;29:241)
    • Hair follicles are dilated and contained hyperkeratotic and parakeratotic debris in place of hair shafts
    • Hyperplastic areas of differentiation into hair matrix with cellular disorganization and loss of nuclear polarity
    Electron microscopy description
    • Intranuclear polyoma type virus particles
    Differential diagnosis
    • Keratosis pilaris: usually arms and legs, small ecstatic infundibula with keratin plugs
    • Lichen spinulosis: children and adolescents, extensor arms, lateral thighs, neck and buttock, minute filiform horny spines grossly due to follicular infundibular hyperkeratosis
    • Drug induced folliculitis

    Vitiligo
    Definition / general
    • Partial or complete loss of pigment producing melanocytes within the epidermis (Wikipedia, eMedicine)
    Epidemiology
    • Affects 1% of world’s population; more noticeable in dark skinned individuals
    • Usually hands / wrists, axilla, perioral, periorbital, anogenital skin
    Sites
    • Focal: only a few areas
    • Segmented: one side of the body only
    • Generalized: most common, both sides of body
    • Trichrome: patient has three shades of skin color
    Clinical features
    • Asymptomatic, flat, well-demarcated zones of pigment loss
    • Autoimmune disorder associated with pernicious anemia, Addison’s disease, Hashimoto’s thyroiditis
    • Perilesional skin up to 5 cm from vitiligo spot is still lighter than normal (Photodermatol Photoimmunol Photomed 2008;24:314)
    • Associated with polymorphisms in COX2 gene (J Dermatol Sci 2009;53:176), mutations of autoimmune regulator gene (Br J Dermatol 2008;159:591)
    • May cause severe psychological distress
    • May worsen with local trauma (cuts, scrapes, burns, Koebner phenomenon)
    • Decreased risk for melanoma and nonmelanoma skin cancer (Br J Dermatol 2013;168:162)
    Treatment
    Clinical images

    Images hosted on other servers:

    Various images

    Marginal inflammatory

    Segmental


    Nonsegmental

    Hands

    Autologous epidermal graft using suction blister

    Microscopic (histologic) description
    • Difficult to diagnose by histology; decreased melanocytes (use S100 or MelanA and control biopsy from adjacent normal skin)(Am J Dermatopathol 2008;30:112)
    • At advancing border, melanocytes may be increased in size with an increased number of dendrites; occasionally lymphocytes are present in this region, particularly if an inflammatory border is present; epidermotropic lymphocytes may form small Pautrier-like collections in the basal layer, with an associated perivascular infiltrate of mononuclear cells involving the superficial plexus and some superficial edema
    • Focal spongiosis may be present in marginal areas
    • Degenerative changes have also been reported in nerves and sweat glands
    • Langerhans cells are usually increased
    • Melanocytes are always reduced more in vitiligo than they are in nevus depigmentosus
    Electron microscopy description
    Differential diagnosis
    • Leukoderma: chemical, melanoma related, scleroderma related; acquired condition with localized loss of skin pigmentation associated with inflammatory skin conditions, burns, intralesional steroid injections, postdermabrasion (Wikipedia)
    • Oculocutaneous albinism: melanocytes present, but no melanin due to defect in tyrosinase enzyme or melanogenesis

    Warts (verrucae)
    Definition / general
    Essential features
    Terminology
    • Verruca vulgaris: common wart
    • Condyloma acuminata: genital warts
    • Verruca palmaris / verruca plantaris: palmar / plantar warts
    • Myrmecia: special type of palmoplantar wart
    • Verrucae planae: Plane warts
    ICD coding
    • ICD-10:
      • B07 - viral warts
      • B07.9 - viral wart, unspecified
    • ICD-11:
      • 1E80 - common warts
      • 1E81 - plane warts
      • 1E82 - warts of lips or oral cavity
      • 1E83 - wart virus proliferation in immune deficient states
      • 1E8Z - viral warts, not elsewhere classified
      • 1A95 - anogenital warts
    Epidemiology
    Sites
    Pathophysiology
    • Spread via direct contact (sexual, skin to skin / fomite)
    • HPV invades the epidermal basal cells through microabrasions; induces hyperplasia and hyperkeratosis
    • Most infections are cleared by immune response
    • Virus binding to the receptor → virus is internalized into the cell by endocytosis → viral genome enters the nucleus
    • E6 and E7 HPV proteins hijack the checkpoint mechanisms of the cell cycle → uncontrolled proliferation
    • Viral genome replication with increased levels of the E1, E2, E4 and E5 proteins
    • In the terminally differentiated layer of epithelium L1 and L2 capsid proteins are expressed and viral particles are assembled
    • The virions are sloughed off with the dead squamous cells of the host epithelium for further transmission (Front Biosci (Landmark Ed) 2018;23:1587)
    Etiology
    Clinical features
    Diagnosis
    Prognostic factors
    Case reports
    Treatment
    Clinical images

    Contributed by Heba Ahmed Abdelkader, M.D.

    Verrucae vulgaris



    Images hosted on other servers:

    Tattoo associated flat warts

    Epidermodysplasia verruciformis

    Verruca plantaris

    Gross description
    • Flesh colored or brown papules or plaques with rough surface
    • Single or multiple
    • See also: clinical features
    Gross images
    Microscopic (histologic) description
    • Common warts (verruca vulgaris) (Arch Pathol Lab Med 2018;142:700):
      • Hyperkeratosis, papillomatosis, hypergranulosis
      • Columns of parakeratosis, especially over projecting dermal papillae
      • Intracorneal hemorrhage
      • Vacuolated superficial keratinocytes with pyknotic raisin-like nuclei (koilocytes)
      • Koilocytes may not be seen in older lesions
      • Koilocytes are not needed for the diagnosis
      • Inward bending of rete ridges at borders of lesion (toeing in)
      • Dilated capillaries in dermal papillae
      • Projects above the plane of the epidermis
    • Palmar / plantar warts (verruca palmaris and verruca plantaris) (Arch Pathol Lab Med 2018;142:700):
      • Similar to verruca vulgaris (clinical distinction based on location)
      • Endophytic growth - greater proportion of the lesion lies beneath the plane of the epidermis
    • Myrmecia warts (J Cutan Pathol 2012;39:936):
      • Marked acanthosis
      • Large eosinophilic intracytoplasmic inclusion bodies
    • Plane warts (verrucae planae) (Arch Pathol Lab Med 2018;142:700):
      • Acanthosis and hyperkeratosis
      • No papillomatosis or parakeratosis
      • Vacuolization of the cells of the upper stratum spinosum and stratum granulosum with margination of keratohyalin granules
      • Often have dyskeratotic keratinocytes
    • Condyloma acuminatum (venereal / genital warts) (Head Neck Pathol 2019;13:80):
      • More massive acanthosis with bulbous rete ridges (condyloma means fist or knuckle)
      • Koilocytes in the upper spinous layer
      • Parakeratosis ofen in the valleys of the epithelium (crypt parakeratosis) (see condyloma)
    • Epidermodysplasia verruciformis (Arch Pathol Lab Med 2018;142:700):
      • Hyperkeratosis and hypergranulosis
      • Acanthosis with small nests of large cells with pale blue-gray cytoplasm, clear nuclei and perinuclear halos
      • May evolve to squamous cell carcinoma
    Microscopic (histologic) images

    Contributed by Heba Ahmed Abdelkader, M.D.

    Verrucae vulgaris

    Koilocytes

    Myrmecia wart

    Myrmecia inclusion bodies



    Contributed by Hillary Rose Elwood, M.D. and AFIP images
    Verruca vulgaris Verruca vulgaris Verruca vulgaris

    Verruca vulgaris

    Condyloma accuminatum



    Contributed by @MirunaPopescu13 on Twitter
    Contributed by @MirunaPopescu13 on Twitter (see original post here)"> Warts (verrucae)Contributed by @MirunaPopescu13 on Twitter (see original post here)"> Warts (verrucae)

    Warts (verrucae)

    Positive stains
    Electron microscopy description
    Molecular / cytogenetics description
    • HPV: in situ hybridization
    Molecular / cytogenetics images

    Images hosted on other servers:

    HPV16 integration patterns

    Videos

    Myrmecia

    Verruca vulgaris

    Epidermodysplasia verruciformis (EDV)

    Respiratory papillomatosis (squamous papilloma)

    Sample pathology report
    • Skin papule, dorsal left foot, punch biopsy:
      • Histopathological features are consistent with verruca vulgaris (see comment)
      • Comment: The epidermis showed hyperkeratosis, papillomatosis, hypergranulosis and columns of parakeratosis over projecting dermal papillae. Dilated capillaries were seen in the dermal papillae. Rete ridges showed inward bending at the borders of the lesion. Vacuolated superficial keratinocytes with pyknotic raisin-like nuclei (koilocytes) were seen.
    Differential diagnosis
    • Verruca vulgaris:
      • Seborrheic keratosis:
        • Variable combinations of hyperkeratosis, papillomatosis, acanthosis
        • Horn pseudocysts
        • No hypergranulosis, koilocytes or tiers of parakeratosis
      • Epidermal nevus:
        • Hyperkeratosis, papillomatosis, acanthosis
        • No koilocytes or tiers of parakeratosis
      • Acrochordon:
        • Pedunculated papule, epidermis often extends almost completely around a fibrovascular core
        • Papillomatosis and acanthosis
        • No koilocytes or tiers of parakeratosis
      • Acanthosis nigricans:
        • Lesser degree of hyperkeratosis and papillomatosis
        • Acanthosis minimal or absent
        • Basal layer hyperpigmentation
        • No koilocytes or tiers of parakeratosis
    • Myrmecia:
      • Molluscum contagiosum:
        • Crater filled with eosinophilic to basophilic intracytoplasmic inclusions (Henderson-Patterson bodies) that push the nucleus and numerous keratohyaline granules aside
    • Condyloma acuminata:
      • Bowenoid papulosis:
        • Atypical keratinocytes throughout the full thickness of the epidermis with several mitoses in metaphase
      • Verrucous carcinoma:
        • Well differentiated SCC with pushing invasive border
        • More mitoses and mild cytologic atypia
      • Condyloma lata:
        • More moist than condylomata acuminata and may be ulcerated.
        • Acanthosis with neutrophils in the epidermis
        • Perivascular plasma cell infiltrate
        • Spirochetes on dark field examination (and IHC) and reactive syphilis serology
    Board review style question #1

    A 9 year old boy presents with multiple flesh colored skin papules on his hand. A skin biopsy of one of the lesions is shown. Which of the following is the most likely cause of his condition?

    1. Cytomegalovirus (CMV)
    2. Human herpesvirus type 1 (HHV1)
    3. Human papillomavirus type 1 (HPV1)
    4. Molluscum contagiosum virus type 1 (MCV1)
    Board review style answer #1
    C. Human papillomavirus type 1 (HPV1). HPV is the cause of verrucae vulgaris which presents as flesh colored papules, especially on the hands and feet. A is incorrect because CMV infection presents with fever, pneumonitis, hepatitis, encephalitis, chorioretinitis, gastroenteritis and a wide variety of skin manifestations, including morbilliform eruption, blueberry muffin rash in babies and ulcers. B is incorrect because 1 (Herpes simplex virus 1) presents as painful grouped vesicles on an erythematous base. D is incorrect, as MCV presents with umbilicated papules.

    Comment Here

    Reference: Warts
    Board review style question #2
    Which of the following histopathologic features is most specific for a diagnosis of verruca vulgaris?

    1. Acanthosis
    2. Basal layer vacuolation
    3. Hyperkeratosis
    4. Koilocytic change
    Board review style answer #2
    D. Koilocytic change. Koilocytes represent the characteristic viral cytopathic change seen in verruca vulgaris. While their presence is not required for diagnosis, when identified, they definitively implicate HPV. They are vacuolated keratinocytes with pyknotic raisin-like nuclei in the superficial layers of the epidermis. A and C are nonspecific findings that can occur in many other diseases. B is incorrect, as basal layer vaculoation is not seen in warts.

    Comment Here

    Reference: Warts

    Weber-Christian disease
    Definition / general
    • Also called relapsing febrile nodular panniculitis
    • Recurring tender erythematous subcutaneous nodules of legs and trunk, usually in women, with malaise and fever
    • Some cases are associated with other disorders
    • No known risk factors
    Epidemiology
    • Females 30 - 60 years of age, although can occur in both sexes, in all ages, and rarely in infants
    Microscopic (histologic) description
    • Lobular acute and subacute inflammation of subcutis with fat cell necrosis; later fibrosis and macrophages
    Differential diagnosis
    Additional references

    Zygomycosis
    Clinical features
    • Infections caused by fungi in class Zygomycetes, including Absidia, Mucor, Rhizomucor, Rhizopus, Apophysomyces elegans (rare human pathogen)
    • Most zygomycosis occur in immunocompromised patients (due to leukemia, lymphoma, diabetes, transplantation), with rhinocerebral or pulmonary infection, then dissemination
    • Other causes include iron overload, major trauma, chronic corticosteroids, intravenous drug use / injection abscess (Indian J Med Res 2010;131:765), neonatal prematurity, malnourishment
    • A. elegans causes progressive necrosis of wound in previously healthy patients after trauma or invasive procedures (Arch Pathol Lab Med 1999;123:386)
    Diagnosis
    • PCR or culture
    • A. elegans - grows as a rapidly growing mold with sporangiophores having dark brown, funnel shaped apophyses and pyriform sporangia
    Treatment
    Clinical images

    Images hosted on other servers:

    Rhizomucor culture has
    "cotton candy"-like texture,
    grows rapidly, maturation
    within 4 days

    Rhizomucor culture demonstrates sparsely septate broad hyphae, irregularly branched sporangiophores, brown round sporangia, rhizoids located on stolons between sporangiophores

    Microscopic (histologic) description
    • Extensive coagulative necrosis due to fungi with broad, sparsely septate, thin-walled hyphae
    • Angioinvasion with thrombosis
    Microscopic (histologic) images

    Images hosted on other servers:

    Aseptate hyphae

    Additional references
    Back to top
    Recent Skin nontumor Pathology books

    Bhawan: 2022

    Biswas: 2017

    Bolognia: 2017

    Brenn: 2017

    Busam: 2015

    Calonje: 2019

    Dadzie: 2016

    Desman: 2016

    Elder: 2020

    Elder: 2022

    Elston: 2018

    Gardner: 2019

    IARC: 2018

    Johnston: 2023

    Murrell: 2015

    Norman: 2015

    Patterson: 2020

    Rapini: 2021

    Subtil: 2019



    Find related Pathology books: dermatopathology, hematopathology
    Image 01 Image 02